Consti 1 2nd Batch of Cases

You might also like

Download as docx, pdf, or txt
Download as docx, pdf, or txt
You are on page 1of 172

EN BANC

[G.R. No. 183591. October 14, 2008.]


THE PROVINCE OF NORTH COTABATO, duly represented by GOVERNOR JESUS SACDALAN and/or VICE-GOVERNOR
EMMANUEL PIÑOL, for and in his own behalf, petitioners, vs. THE GOVERNMENT OF THE REPUBLIC OF THE PHILIPPINES
PEACE PANEL ON ANCESTRAL DOMAIN (GRP), represented by SEC. RODOLFO GARCIA, ATTY. LEAH ARMAMENTO, ATTY.
SEDFREY CANDELARIA, MARK RYAN SULLIVAN and/or GEN. HERMOGENES ESPERON, JR., the latter in his capacity as the
present and duly-appointed Presidential Adviser on the Peace Process (OPAPP) or the so-called Office of the Presidential
Adviser on the Peace Process, respondents.
DECISION
CARPIO-MORALES, J p:
Subject of these consolidated cases is the extent of the powers of the President in pursuing the peace process. While the facts surrounding this
controversy center on the armed conflict in Mindanao between the government and the Moro Islamic Liberation Front (MILF), the legal issue
involved has a bearing on all areas in the country where there has been a long-standing armed conflict. Yet again, the Court is tasked to perform a
delicate balancing act. It must uncompromisingly delineate the bounds within which the President may lawfully exercise her discretion, but it must
do so in strict adherence to the Constitution, lest its ruling unduly restricts the freedom of action vested by that same Constitution in the Chief
Executive precisely to enable her to pursue the peace process effectively.
I.FACTUAL ANTECEDENTS OF THE PETITIONS
On August 5, 2008, the Government of the Republic of the Philippines (GRP) and the MILF, through the Chairpersons of their respective peace
negotiating panels, were scheduled to sign a Memorandum of Agreement on the Ancestral Domain (MOA-AD) Aspect of the GRP-MILF Tripoli
Agreement on Peace of 2001 in Kuala Lumpur, Malaysia.
The MILF is a rebel group which was established in March 1984 when, under the leadership of the late Salamat Hashim, it splintered from the Moro
National Liberation Front (MNLF) then headed by Nur Misuari, on the ground, among others, of what Salamat perceived to be the manipulation of
the MNLF away from an Islamic basis towards Marxist-Maoist orientations. 1
The signing of the MOA-AD between the GRP and the MILF was not to materialize, however, for upon motion of petitioners, specifically those who
filed their cases before the scheduled signing of the MOA-AD, this Court issued a Temporary Restraining Order enjoining the GRP from signing the
same. IECcaA
The MOA-AD was preceded by a long process of negotiation and the concluding of several prior agreements between the two parties beginning in
1996, when the GRP-MILF peace negotiations began. On July 18, 1997, the GRP and MILF Peace Panels signed the Agreement on General Cessation of
Hostilities. The following year, they signed the General Framework of Agreement of Intent on August 27, 1998.
The Solicitor General, who represents respondents, summarizes the MOA-AD by stating that the same contained, among others, the commitment of
the parties to pursue peace negotiations, protect and respect human rights, negotiate with sincerity in the resolution and pacific settlement of the
conflict, and refrain from the use of threat or force to attain undue advantage while the peace negotiations on the substantive agenda are on-
going. 2
Early on, however, it was evident that there was not going to be any smooth sailing in the GRP-MILF peace process. Towards the end of 1999 up to
early 2000, the MILF attacked a number of municipalities in Central Mindanao and, in March 2000, it took control of the town hall of Kauswagan,
Lanao del Norte. 3 In response, then President Joseph Estrada declared and carried out an "all-out-war" against the MILF. cHDaEI
When President Gloria Macapagal-Arroyo assumed office, the military offensive against the MILF was suspended and the government sought a
resumption of the peace talks. The MILF, according to a leading MILF member, initially responded with deep reservation, but when President Arroyo
asked the Government of Malaysia through Prime Minister Mahathir Mohammad to help convince the MILF to return to the negotiating table, the
MILF convened its Central Committee to seriously discuss the matter and, eventually, decided to meet with the GRP. 4
The parties met in Kuala Lumpur on March 24, 2001, with the talks being facilitated by the Malaysian government, the parties signing on the same
date the Agreement on the General Framework for the Resumption of Peace Talks Between the GRP and the MILF. The MILF thereafter suspended all
its military actions. 5
Formal peace talks between the parties were held in Tripoli, Libya from June 20-22, 2001, the outcome of which was the GRP-MILF Tripoli Agreement
on Peace (Tripoli Agreement 2001) containing the basic principles and agenda on the following aspects of the
negotiation: Security Aspect, Rehabilitation Aspect, and Ancestral Domain Aspect. With regard to the Ancestral Domain Aspect, the parties in Tripoli
Agreement 2001 simply agreed "that the same be discussed further by the Parties in their next meeting".
A second round of peace talks was held in Cyberjaya, Malaysia on August 5-7, 2001 which ended with the signing of the Implementing Guidelines on
the Security Aspect of the Tripoli Agreement 2001 leading to a ceasefire status between the parties. This was followed by the Implementing
Guidelines on the Humanitarian Rehabilitation and Development Aspects of the Tripoli Agreement 2001, which was signed on May 7, 2002 at
Putrajaya, Malaysia. Nonetheless, there were many incidence of violence between government forces and the MILF from 2002 to 2003.
Meanwhile, then MILF Chairman Salamat Hashim passed away on July 13, 2003 and he was replaced by Al Haj Murad, who was then the chief peace
negotiator of the MILF. Murad's position as chief peace negotiator was taken over by Mohagher Iqbal. 6 ISCHET
In 2005, several exploratory talks were held between the parties in Kuala Lumpur, eventually leading to the crafting of the draft MOA-AD in its final
form, which, as mentioned, was set to be signed last August 5, 2008.
II.STATEMENT OF THE PROCEEDINGS
Before the Court is what is perhaps the most contentious "consensus" ever embodied in an instrument — the MOA-AD which is assailed principally
by the present petitions bearing docket numbers 183591, 183752, 183893, 183951 and 183962.
Commonly impleaded as respondents are the GRP Peace Panel on Ancestral Domain. 7 and the Presidential Adviser on the Peace Process (PAPP)
Hermogenes Esperon, Jr.
On July 23, 2008, the Province of North Cotabato 8 and Vice-Governor Emmanuel Piñol filed a petition, docketed as G.R. No. 183591,
for Mandamus and Prohibition with Prayer for the Issuance of Writ of Preliminary Injunction and Temporary Restraining Order. 9 Invoking the right
to information on matters of public concern, petitioners seek to compel respondents to disclose and furnish them the complete and official copies of
the MOA-AD including its attachments, and to prohibit the slated signing of the MOA-AD, pending the disclosure of the contents of the MOA-AD and
the holding of a public consultation thereon. Supplementarily, petitioners pray that the MOA-AD be declared unconstitutional. 10
This initial petition was followed by another one, docketed as G.R. No. 183752, also for Mandamus and Prohibition. 11 filed by the City of
Zamboanga, 12 Mayor Celso Lobregat, Rep. Ma. Isabelle Climaco and Rep. Erico Basilio Fabian who likewise pray for similar injunctive reliefs.
Petitioners herein moreover pray that the City of Zamboanga be excluded from the Bangsamoro Homeland and/or Bangsamoro Juridical Entity and,
in the alternative, that the MOA-AD be declared null and void.
By Resolution of August 4, 2008, the Court issued a Temporary Restraining Order commanding and directing public respondents and their agents to
cease and desist from formally signing the MOA-AD. 13 The Court also required the Solicitor General to submit to the Court and petitioners the
official copy of the final draft of the MOA-AD, 14 to which she complied. 15
Meanwhile, the City of Iligan 16 filed a petition for Injunction and/or Declaratory Relief, docketed as G.R. No. 183893, praying that respondents be
enjoined from signing the MOA-AD or, if the same had already been signed, from implementing the same, and that the MOA-AD be declared
unconstitutional. Petitioners herein additionally implead Executive Secretary Eduardo Ermita as respondent. HSDCTA
The Province of Zamboanga del Norte, 17 Governor Rolando Yebes, Vice-Governor Francis Olvis, Rep. Cecilia Jalosjos-Carreon, Rep. Cesar Jalosjos,
and the members 18 of the Sangguniang Panlalawigan of Zamboanga del Norte filed on August 15, 2008 a petition for Certiorari, Mandamus and
Prohibition, 19 docketed as G.R. No. 183951. They pray, inter alia, that the MOA-AD be declared null and void and without operative effect, and that
respondents be enjoined from executing the MOA-AD.
On August 19, 2008, Ernesto Maceda, Jejomar Binay, and Aquilino Pimentel III filed a petition for Prohibition, 20 docketed as G.R. No. 183962,
praying for a judgment prohibiting and permanently enjoining respondents from formally signing and executing the MOA-AD and or any other
agreement derived therefrom or similar thereto, and nullifying the MOA-AD for being unconstitutional and illegal. Petitioners herein additionally
implead as respondent the MILF Peace Negotiating Panel represented by its Chairman Mohagher Iqbal.
Various parties moved to intervene and were granted leave of court to file their petitions-/comments-in-intervention. Petitioners-in-intervention
include Senator Manuel A. Roxas, former Senate President Franklin Drilon and Atty. Adel Tamano, the City of Isabela 21 and Mayor Cherrylyn Santos-
Akbar, the Province of Sultan Kudarat 22 and Gov. Suharto Mangudadatu, the Municipality of Linamon in Lanao del Norte, 23 Ruy Elias Lopez of
Davao City and of the Bagobo tribe, Sangguniang Panlungsod member Marino Ridao and businessman Kisin Buxani, both of Cotabato City; and
lawyers Carlo Gomez, Gerardo Dilig, Nesario Awat, Joselito Alisuag, Richalex Jagmis, all of Palawan City. The Muslim Legal Assistance Foundation, Inc.
(Muslaf) and the Muslim Multi-Sectoral Movement for Peace and Development (MMMPD) filed their respective Comments-in-Intervention. aIHSEc
By subsequent Resolutions, the Court ordered the consolidation of the petitions. Respondents filed Comments on the petitions, while some of
petitioners submitted their respective Replies.
Respondents, by Manifestation and Motion of August 19, 2008, stated that the Executive Department shall thoroughly review the MOA-AD and
pursue further negotiations to address the issues hurled against it, and thus moved to dismiss the cases. In the succeeding exchange of pleadings,
respondents' motion was met with vigorous opposition from petitioners.
The cases were heard on oral argument on August 15, 22 and 29, 2008 that tackled the following principal issues:
1.Whether the petitions have become moot and academic
(i)insofar as the mandamus aspect is concerned, in view of the disclosure of official copies of the final draft of the
Memorandum of Agreement (MOA); and
(ii)insofar as the prohibition aspect involving the Local Government Units is concerned, if it is considered that
consultation has become fait accompli with the finalization of the draft;
2.Whether the constitutionality and the legality of the MOA is ripe for adjudication;
3.Whether respondent Government of the Republic of the Philippines Peace Panel committed grave abuse of discretion
amounting to lack or excess of jurisdiction when it negotiated and initiated the MOA vis-Ã -vis ISSUES Nos. 4 and 5;
4.Whether there is a violation of the people's right to information on matters of public concern (1987 Constitution, Article III,
Sec. 7) under a state policy of full disclosure of all its transactions involving public interest (1987 Constitution, Article
II, Sec. 28) including public consultation under Republic Act No. 7160 (LOCAL GOVERNMENT CODE OF 1991)[;]
If it is in the affirmative, whether prohibition under Rule 65 of the 1997 Rules of Civil Procedure is an appropriate remedy;
5.Whether by signing the MOA, the Government of the Republic of the Philippines would be BINDING itself ACcISa
a)to create and recognize the Bangsamoro Juridical Entity (BJE) as a separate state, or a juridical, territorial or
political subdivision not recognized by law;
b)to revise or amend the Constitution and existing laws to conform to the MOA;
c)to concede to or recognize the claim of the Moro Islamic Liberation Front for ancestral domain in violation of
Republic Act No. 8371 (THE INDIGENOUS PEOPLES RIGHTS ACT OF 1997), particularly Section 3(g) &
Chapter VII (DELINEATION, RECOGNITION OF ANCESTRAL DOMAINS)[;]
If in the affirmative, whether the Executive Branch has the authority to so bind the Government of the Republic of the
Philippines;
6.Whether the inclusion/exclusion of the Province of North Cotabato, Cities of Zamboanga, Iligan and Isabela, and the
Municipality of Linamon, Lanao del Norte in/from the areas covered by the projected Bangsamoro Homeland is a
justiciable question; and
7.Whether desistance from signing the MOA derogates any prior valid commitments of the Government of the Republic of the
Philippines. 24
The Court, thereafter, ordered the parties to submit their respective Memoranda. Most of the parties submitted their memoranda on time.
III.OVERVIEW OF THE MOA-AD
As a necessary backdrop to the consideration of the objections raised in the subject five petitions and six petitions-in-intervention against the MOA-
AD, as well as the two comments-in-intervention in favor of the MOA-AD, the Court takes an overview of the MOA.
The MOA-AD identifies the Parties to it as the GRP and the MILF.
Under the heading "Terms of Reference" (TOR), the MOA-AD includes not only four earlier agreements between the GRP and MILF, but also two
agreements between the GRP and the MNLF: the 1976 Tripoli Agreement, and the Final Peace Agreement on the Implementation of the 1976 Tripoli
Agreement, signed on September 2, 1996 during the administration of President Fidel Ramos. TICAcD
The MOA-AD also identifies as TOR two local statutes — the organic act for the Autonomous Region in Muslim Mindanao (ARMM) 25 and the
Indigenous Peoples Rights Act (IPRA), 26and several international law instruments — the ILO Convention No. 169 Concerning Indigenous and Tribal
Peoples in Independent Countries in relation to the UN Declaration on the Rights of the Indigenous Peoples, and the UN Charter, among others.
The MOA-AD includes as a final TOR the generic category of "compact rights entrenchment emanating from the regime of dar-ul-mua'hada (or
territory under compact) and dar-ul-sulh (or territory under peace agreement) that partakes the nature of a treaty device".
During the height of the Muslim Empire, early Muslim jurists tended to see the world through a simple dichotomy: there was the dar-ul-Islam (the
Abode of Islam) and dar-ul-harb (the Abode of War). The first referred to those lands where Islamic laws held sway, while the second denoted those
lands where Muslims were persecuted or where Muslim laws were outlawed or ineffective. 27 This way of viewing the world, however, became
more complex through the centuries as the Islamic world became part of the international community of nations.
As Muslim States entered into treaties with their neighbors, even with distant States and inter-governmental organizations, the classical division of
the world into dar-ul-Islamand dar-ul-harb eventually lost its meaning. New terms were drawn up to describe novel ways of perceiving non-Muslim
territories. For instance, areas like dar-ul-mua'hada (land of compact) and dar-ul-sulh (land of treaty) referred to countries which, though under a
secular regime, maintained peaceful and cooperative relations with Muslim States, having been bound to each other by treaty or agreement. Dar-ul-
aman (land of order), on the other hand, referred to countries which, though not bound by treaty with Muslim States, maintained freedom of
religion for Muslims. 28
It thus appears that the "compact rights entrenchment" emanating from the regime of dar-ul-mua'hada and dar-ul-sulh simply refers to all other
agreements between the MILF and the Philippine government — the Philippines being the land of compact and peace agreement — that partake of
the nature of a treaty device, "treaty" being broadly defined as "any solemn agreement in writing that sets out understandings, obligations, and
benefits for both parties which provides for a framework that elaborates the principles declared in the [MOA-AD]". 29

The MOA-AD states that the Parties "HAVE AGREED AND ACKNOWLEDGED AS FOLLOWS", and starts with its main body.
The main body of the MOA-AD is divided into four strands, namely, Concepts and Principles, Territory, Resources, and Governance.
A.CONCEPTS AND PRINCIPLES
This strand begins with the statement that it is "the birthright of all Moros and all Indigenous peoples of Mindanao to identify themselves and be
accepted as 'Bangsamoros'". It defines "Bangsamoro people" as the natives or original inhabitants of Mindanao and its adjacent islands including
Palawan and the Sulu archipelago at the time of conquest or colonization, and their descendants whether mixed or of full blood, including their
spouses. 30
Thus, the concept of "Bangsamoro", as defined in this strand of the MOA-AD, includes not only "Moros" as traditionally understood even by
Muslims, 31 but all indigenous peoplesof Mindanao and its adjacent islands. The MOA-AD adds that the freedom of choice of indigenous peoples
shall be respected. What this freedom of choice consists in has not been specifically defined. ICTaEH
The MOA-AD proceeds to refer to the "Bangsamoro homeland", the ownership of which is vested exclusively in the Bangsamoro people by virtue of
their prior rights of occupation. 32 Both parties to the MOA-AD acknowledge that ancestral domain does not form part of the public domain. 33
The Bangsamoro people are acknowledged as having the right to self-governance, which right is said to be rooted on ancestral territoriality exercised
originally under the suzerain authority of their sultanates and the Pat a Pangampong ku Ranaw. The sultanates were described as states
or "karajaan/kadatuan" resembling a body politic endowed with all the elements of a nation-state in the modern sense. 34
The MOA-AD thus grounds the right to self-governance of the Bangsamoro people on the past suzerain authority of the sultanates. As gathered, the
territory defined as the Bangsamoro homeland was ruled by several sultanates and, specifically in the case of the Maranao, by the Pat a
Pangampong ku Ranaw, a confederation of independent principalities (pangampong) each ruled by datus and sultans, none of whom was supreme
over the others. 35
The MOA-AD goes on to describe the Bangsamoro people as "the 'First Nation' with defined territory and with a system of government having
entered into treaties of amity and commerce with foreign nations".
The term "First Nation" is of Canadian origin referring to the indigenous peoples of that territory, particularly those known as Indians. In Canada,
each of these indigenous peoples is equally entitled to be called "First Nation", hence, all of them are usually described collectively by the plural
"First Nations". 36 To that extent, the MOA-AD, by identifying the Bangsamoro people as "the First Nation" — suggesting its exclusive entitlement to
that designation — departs from the Canadian usage of the term.
The MOA-AD then mentions for the first time the "Bangsamoro Juridical Entity" (BJE) to which it grants the authority and jurisdiction over the
Ancestral Domain and Ancestral Lands of the Bangsamoro. 37 ADaECI
B.TERRITORY
The territory of the Bangsamoro homeland is described as the land mass as well as the maritime, terrestrial, fluvial and alluvial domains, including
the aerial domain and the atmospheric space above it, embracing the Mindanao-Sulu-Palawan geographic region. 38
More specifically, the core of the BJE is defined as the present geographic area of the ARMM — thus constituting the following areas: Lanao del Sur,
Maguindanao, Sulu, Tawi-Tawi, Basilan, and Marawi City. Significantly, this core also includes certain municipalities of Lanao del Norte that voted for
inclusion in the ARMM in the 2001 plebiscite. 39
Outside of this core, the BJE is to cover other provinces, cities, municipalities and barangays, which are grouped into two categories, Category A and
Category B. Each of these areas is to be subjected to a plebiscite to be held on different dates, years apart from each other. Thus, Category A areas
are to be subjected to a plebiscite not later than twelve (12) months following the signing of the MOA-AD. 40 Category B areas, also called "Special
Intervention Areas", on the other hand, are to be subjected to a plebiscite twenty-five (25) years from the signing of a separate agreement — the
Comprehensive Compact. 41
The Parties to the MOA-AD stipulate that the BJE shall have jurisdiction over all natural resources within its "internal waters", defined as extending
fifteen (15) kilometers from the coastline of the BJE area; 42 that the BJE shall also have "territorial waters", which shall stretch beyond the BJE
internal waters up to the baselines of the Republic of the Philippines (RP) south east and south west of mainland Mindanao; and that within
these territorial waters, the BJE and the "Central Government" (used interchangeably with RP) shall exercise joint jurisdiction, authority and
management over all natural resources. 43 Notably, the jurisdiction over the internal waters is not similarly described as "joint".
The MOA-AD further provides for the sharing of minerals on the territorial waters between the Central Government and the BJE, in favor of the
latter, through production sharing and economic cooperation agreement. 44 The activities which the Parties are allowed to conduct on
the territorial waters are enumerated, among which are the exploration and utilization of natural resources, regulation of shipping and fishing
activities, and the enforcement of police and safety measures. 45 There is no similar provision on the sharing of minerals and allowed activities with
respect to the internal waters of the BJE.
C.RESOURCES
The MOA-AD states that the BJE is free to enter into any economic cooperation and trade relations with foreign countries and shall have the option
to establish trade missions in those countries. Such relationships and understandings, however, are not to include aggression against the GRP. The
BJE may also enter into environmental cooperation agreements. 46 ACcaET
The external defense of the BJE is to remain the duty and obligation of the Central Government. The Central Government is also bound to "take
necessary steps to ensure the BJE's participation in international meetings and events" like those of the ASEAN and the specialized agencies of the
UN. The BJE is to be entitled to participate in Philippine official missions and delegations for the negotiation of border agreements or protocols for
environmental protection and equitable sharing of incomes and revenues involving the bodies of water adjacent to or between the islands forming
part of the ancestral domain. 47
With regard to the right of exploring for, producing, and obtaining all potential sources of energy, petroleum, fossil fuel, mineral oil and natural gas,
the jurisdiction and control thereon is to be vested in the BJE "as the party having control within its territorial jurisdiction". This right carries
the proviso that, "in times of national emergency, when public interest so requires", the Central Government may, for a fixed period and under
reasonable terms as may be agreed upon by both Parties, assume or direct the operation of such resources. 48
The sharing between the Central Government and the BJE of total production pertaining to natural resources is to be 75:25 in favor of the BJE. 49
The MOA-AD provides that legitimate grievances of the Bangsamoro people arising from any unjust dispossession of their territorial and proprietary
rights, customary land tenures, or their marginalization shall be acknowledged. Whenever restoration is no longer possible, reparation is to be in
such form as mutually determined by the Parties. 50 STDEcA
The BJE may modify or cancel the forest concessions, timber licenses, contracts or agreements, mining concessions, Mineral Production and Sharing
Agreements (MPSA), Industrial Forest Management Agreements (IFMA), and other land tenure instruments granted by the Philippine Government,
including those issued by the present ARMM. 51
D.GOVERNANCE
The MOA-AD binds the Parties to invite a multinational third-party to observe and monitor the implementation of the Comprehensive Compact. This
compact is to embody the "details for the effective enforcement" and "the mechanisms and modalities for the actual implementation" of the MOA-
AD. The MOA-AD explicitly provides that the participation of the third party shall not in any way affect the status of the relationship between the
Central Government and the BJE. 52
The "associative" relationship
between the Central Government
and the BJE
The MOA-AD describes the relationship of the Central Government and the BJE as "associative", characterized by shared authority and responsibility.
And it states that the structure of governance is to be based on executive, legislative, judicial, and administrative institutions with defined powers
and functions in the Comprehensive Compact.
The MOA-AD provides that its provisions requiring "amendments to the existing legal framework" shall take effect upon signing of the
Comprehensive Compact and upon effecting the aforesaid amendments, with due regard to the non-derogation of prior agreements and within the
stipulated timeframe to be contained in the Comprehensive Compact. As will be discussed later, much of the present controversy hangs on the
legality of this provision.
The BJE is granted the power to build, develop and maintain its own institutions inclusive of civil service, electoral, financial and banking, education,
legislation, legal, economic, police and internal security force, judicial system and correctional institutions, the details of which shall be discussed in
the negotiation of the comprehensive compact. AIaDcH
As stated early on, the MOA-AD was set to be signed on August 5, 2008 by Rodolfo Garcia and Mohagher Iqbal, Chairpersons of the Peace
Negotiating Panels of the GRP and the MILF, respectively. Notably, the penultimate paragraph of the MOA-AD identifies the signatories as "the
representatives of the Parties", meaning the GRP and MILF themselves, and not merely of the negotiating panels. 53 In addition, the signature page of
the MOA-AD states that it is "WITNESSED BY" Datuk Othman Bin Abd Razak, Special Adviser to the Prime Minister of Malaysia, "ENDORSED BY"
Ambassador Sayed Elmasry, Adviser to Organization of the Islamic Conference (OIC) Secretary General and Special Envoy for Peace Process in
Southern Philippines, and SIGNED "IN THE PRESENCE OF" Dr. Albert G. Romulo, Secretary of Foreign Affairs of RP and Dato' Seri Utama Dr. Rais Bin
Yatim, Minister of Foreign Affairs, Malaysia, all of whom were scheduled to sign the Agreement last August 5, 2008.

Annexed to the MOA-AD are two documents containing the respective lists cum maps of the provinces, municipalities, and barangays under
Categories A and B earlier mentioned in the discussion on the strand on TERRITORY.
IV.PROCEDURAL ISSUES
A.RIPENESS
The power of judicial review is limited to actual cases or controversies. 54 Courts decline to issue advisory opinions or to resolve hypothetical or
feigned problems, or mere academic questions. 55 The limitation of the power of judicial review to actual cases and controversies defines the role
assigned to the judiciary in a tripartite allocation of power, to assure that the courts will not intrude into areas committed to the other branches of
government. 56
An actual case or controversy involves a conflict of legal rights, an assertion of opposite legal claims, susceptible of judicial resolution as distinguished
from a hypothetical or abstract difference or dispute. There must be a contrariety of legal rights that can be interpreted and enforced on the basis of
existing law and jurisprudence. 57 The Court can decide the constitutionality of an act or treaty only when a proper case between opposing parties is
submitted for judicial determination. 58 CEHcSI
Related to the requirement of an actual case or controversy is the requirement of ripeness. A question is ripe for adjudication when the act being
challenged has had a direct adverse effect on the individual challenging it. 59 For a case to be considered ripe for adjudication, it is a prerequisite that
something had then been accomplished or performed by either branch before a court may come into the picture, 60 and the petitioner must allege
the existence of an immediate or threatened injury to itself as a result of the challenged action. 61 He must show that he has sustained or is
immediately in danger of sustaining some direct injury as a result of the act complained of. 62
The Solicitor General argues that there is no justiciable controversy that is ripe for judicial review in the present petitions, reasoning that:
The unsigned MOA-AD is simply a list of consensus points subject to further negotiations and legislative enactments as well as
constitutional processes aimed at attaining a final peaceful agreement. Simply put, the MOA-AD remains to be a proposal that
does not automatically create legally demandable rights and obligations until the list of operative acts required have been duly
complied with. . . .
xxx xxx xxx
In the cases at bar, it is respectfully submitted that this Honorable Court has no authority to pass upon issues based on
hypothetical or feigned constitutional problems or interests with no concrete bases. Considering the preliminary character of
the MOA-AD, there are no concrete acts that could possibly violate petitioners' and intervenors' rights since the acts
complained of are mere contemplated steps toward the formulation of a final peace agreement. Plainly, petitioners and
intervenors' perceived injury, if at all, is merely imaginary and illusory apart from being unfounded and based on mere
conjectures. (Underscoring supplied)
The Solicitor General cites 63 the following provisions of the MOA-AD:
TERRITORY
xxx xxx xxx
2.Toward this end, the Parties enter into the following stipulations:
xxx xxx xxx
d.Without derogating from the requirements of prior agreements, the Government stipulates to conduct and deliver,
using all possible legal measures, within twelve (12) months following the signing of the MOA-AD, a
plebiscite covering the areas as enumerated in the list and depicted in the map as Category A attached
herein (the "Annex"). The Annex constitutes an integral part of this framework agreement. Toward this
end, the Parties shall endeavor to complete the negotiations and resolve all outstanding issues on the
Comprehensive Compact within fifteen (15) months from the signing of the MOA-AD. caTESD
xxx xxx xxx
GOVERNANCE
xxx xxx xxx
7.The Parties agree that mechanisms and modalities for the actual implementation of this MOA-AD shall be spelt out in the
Comprehensive Compact to mutually take such steps to enable it to occur effectively. CASTDI
Any provisions of the MOA-AD requiring amendments to the existing legal framework shall come into force upon the signing of
a Comprehensive Compact and upon effecting the necessary changes to the legal framework with due regard to non-
derogation of prior agreements and within the stipulated timeframe to be contained in the Comprehensive
Compact. 64 (Underscoring supplied)
The Solicitor General's arguments fail to persuade.
Concrete acts under the MOA-AD are not necessary to render the present controversy ripe. In Pimentel, Jr. v. Aguirre, 65 this Court held:
. . . [B]y the mere enactment of the questioned law or the approval of the challenged action, the dispute is said to have ripened
into a judicial controversy even without any other overt act. Indeed, even a singular violation of the Constitution and/or the
law is enough to awaken judicial duty. DEaCSA
xxx xxx xxx
By the same token, when an act of the President, who in our constitutional scheme is a coequal of Congress, is seriously
alleged to have infringed the Constitution and the laws . . . settling the dispute becomes the duty and the responsibility of the
courts. 66
In Santa Fe Independent School District v. Doe, 67 the United States Supreme Court held that the challenge to the constitutionality of the school's
policy allowing student-led prayers and speeches before games was ripe for adjudication, even if no public prayer had yet been led under the policy,
because the policy was being challenged as unconstitutional on its face. 68
That the law or act in question is not yet effective does not negate ripeness. For example, in New York v. United States, 69 decided in 1992, the
United States Supreme Court held that the action by the State of New York challenging the provisions of the Low-Level Radioactive Waste Policy Act
was ripe for adjudication even if the questioned provision was not to take effect until January 1, 1996, because the parties agreed that New York had
to take immediate action to avoid the provision's consequences. 70
The present petitions pray for Certiorari, 71 Prohibition, and Mandamus. Certiorari and Prohibition are remedies granted by law when any tribunal,
board or officer has acted, in the case ofcertiorari, or is proceeding, in the case of prohibition, without or in excess of its jurisdiction or with grave
abuse of discretion amounting to lack or excess of jurisdiction. 72 Mandamus is a remedy granted by law when any tribunal, corporation, board,
officer or person unlawfully neglects the performance of an act which the law specifically enjoins as a duty resulting from an office, trust, or station,
or unlawfully excludes another from the use or enjoyment of a right or office to which such other is entitled. 73 Certiorari, Mandamus and
Prohibition are appropriate remedies to raise constitutional issues and to review and/or prohibit/nullify, when proper, acts of legislative and
executive officials. 74
The authority of the GRP Negotiating Panel is defined by Executive Order No. 3 (E.O. No. 3), issued on February 28, 2001. 75 The said executive order
requires that "[t]he government's policy framework for peace, including the systematic approach and the administrative structure for carrying out
the comprehensive peace process . . . be governed by this Executive Order". 76
The present petitions allege that respondents GRP Panel and PAPP Esperon drafted the terms of the MOA-AD without consulting the local
government units or communities affected, nor informing them of the proceedings. As will be discussed in greater detail later, such omission, by
itself, constitutes a departure by respondents from their mandate under E.O. No. 3.
Furthermore, the petitions allege that the provisions of the MOA-AD violate the Constitution. The MOA-AD provides that "any provisions of the
MOA-AD requiring amendments to the existing legal framework shall come into force upon the signing of a Comprehensive Compact and upon
effecting the necessary changes to the legal framework", implying an amendment of the Constitution to accommodate the MOA-AD. This stipulation,
in effect, guaranteed to the MILF the amendment of the Constitution. Such act constitutes another violation of its authority. Again, these points will
be discussed in more detail later.
As the petitions allege acts or omissions on the part of respondent that exceed their authority, by violating their duties under E.O. No. 3 and the
provisions of the Constitution and statutes, the petitions make a prima facie case for Certiorari, Prohibition, and Mandamus, and an actual case or
controversy ripe for adjudication exists. When an act of a branch of government is seriously alleged to have infringed the Constitution, it becomes
not only the right but in fact the duty of the judiciary to settle the dispute. 77HSIDTE
B.LOCUS STANDI
For a party to have locus standi, one must allege "such a personal stake in the outcome of the controversy as to assure that concrete adverseness
which sharpens the presentation of issues upon which the court so largely depends for illumination of difficult constitutional questions". 78
Because constitutional cases are often public actions in which the relief sought is likely to affect other persons, a preliminary question frequently
arises as to this interest in the constitutional question raised. 79
When suing as a citizen, the person complaining must allege that he has been or is about to be denied some right or privilege to which he is lawfully
entitled or that he is about to be subjected to some burdens or penalties by reason of the statute or act complained of. 80 When the issue concerns a
public right, it is sufficient that the petitioner is a citizen and has an interest in the execution of the laws. 81 EaHcDS

For a taxpayer, one is allowed to sue where there is an assertion that public funds are illegally disbursed or deflected to an illegal purpose, or that
there is a wastage of public funds through the enforcement of an invalid or unconstitutional law. 82 The Court retains discretion whether or not to
allow a taxpayer's suit. 83
In the case of a legislator or member of Congress, an act of the Executive that injures the institution of Congress causes a derivative but nonetheless
substantial injury that can be questioned by legislators. A member of the House of Representatives has standing to maintain inviolate the
prerogatives, powers and privileges vested by the Constitution in his office. 84
An organization may be granted standing to assert the rights of its members, 85 but the mere invocation by the Integrated Bar of the Philippines or
any member of the legal profession of the duty to preserve the rule of law does not suffice to clothe it with standing. 86
As regards a local government unit (LGU), it can seek relief in order to protect or vindicate an interest of its own, and of the other LGUs. 87
Intervenors, meanwhile, may be given legal standing upon showing of facts that satisfy the requirements of the law authorizing
intervention, . 88 such as a legal interest in the matter in litigation, or in the success of either of the parties.
In any case, the Court has discretion to relax the procedural technicality on locus standi, given the liberal attitude it has exercised, highlighted in the
case of David v. Macapagal-Arroyo, 89 where technicalities of procedure were brushed aside, the constitutional issues raised being of paramount
public interest or of transcendental importance deserving the attention of the Court in view of their seriousness, novelty and weight as
precedents. 90 The Court's forbearing stance on locus standi on issues involving constitutional issues has for its purpose the protection of
fundamental rights.
In not a few cases, the Court, in keeping with its duty under the Constitution to determine whether the other branches of government have kept
themselves within the limits of the Constitution and the laws and have not abused the discretion given them, has brushed aside technical rules of
procedure. 91 ScHADI
In the petitions at bar, petitioners Province of North Cotabato (G.R. No. 183591) Province of Zamboanga del Norte (G.R. No. 183951), City of
Iligan (G.R. No. 183893) and City of Zamboanga (G.R. No. 183752) and petitioners-in-intervention Province of Sultan Kudarat, City of
Isabela and Municipality of Linamon have locus standi in view of the direct and substantial injury that they, as LGUs, would suffer as their territories,
whether in whole or in part, are to be included in the intended domain of the BJE. These petitioners allege that they did not vote for their inclusion in
the ARMM which would be expanded to form the BJE territory. Petitioners' legal standing is thus beyond doubt.
In G.R. No. 183962, petitioners Ernesto Maceda, Jejomar Binay and Aquilino Pimentel III would have no standing as citizens and taxpayers for their
failure to specify that they would be denied some right or privilege or there would be wastage of public funds. The fact that they are a former
Senator, an incumbent mayor of Makati City, and a resident of Cagayan de Oro, respectively, is of no consequence. Considering their invocation of
the transcendental importance of the issues at hand, however, the Court grants them standing.
Intervenors Franklin Drilon and Adel Tamano, in alleging their standing as taxpayers, assert that government funds would be expended for the
conduct of an illegal and unconstitutional plebiscite to delineate the BJE territory. On that score alone, they can be given legal standing. Their
allegation that the issues involved in these petitions are of "undeniable transcendental importance" clothes them with added basis for their
personality to intervene in these petitions.
With regard to Senator Manuel Roxas, his standing is premised on his being a member of the Senate and a citizen to enforce compliance by
respondents of the public's constitutional right to be informed of the MOA-AD, as well as on a genuine legal interest in the matter in litigation, or in
the success or failure of either of the parties. He thus possesses the requisite standing as an intervenor.
With respect to Intervenors Ruy Elias Lopez, as a former congressman of the 3rd district of Davao City, a taxpayer and a member of the Bagobo
tribe; Carlo B. Gomez, et al., as members of the IBP Palawan chapter, citizens and taxpayers; Marino Ridao, as taxpayer, resident and member of
the Sangguniang Panlungsod of Cotabato City; and Kisin Buxani, as taxpayer, they failed to allege any proper legal interest in the present petitions.
Just the same, the Court exercises its discretion to relax the procedural technicality onlocus standi given the paramount public interest in the issues
at hand. ECaAHS
Intervening respondents Muslim Multi-Sectoral Movement for Peace and Development, an advocacy group for justice and the attainment of peace
and prosperity in Muslim Mindanao; and Muslim Legal Assistance Foundation, Inc., a non-government organization of Muslim lawyers, allege that
they stand to be benefited or prejudiced, as the case may be, in the resolution of the petitions concerning the MOA-AD, and prays for the denial of
the petitions on the grounds therein stated. Such legal interest suffices to clothe them with standing.
B.MOOTNESS
Respondents insist that the present petitions have been rendered moot with the satisfaction of all the reliefs prayed for by petitioners and the
subsequent pronouncement of the Executive Secretary that "[n]o matter what the Supreme Court ultimately decides[,] the government will not sign
the MOA". 92
In lending credence to this policy decision, the Solicitor General points out that the President had already disbanded the GRP Peace Panel. 93
In David v. Macapagal-Arroyo, 94 this Court held that the "moot and academic" principle not being a magical formula that automatically dissuades
courts in resolving a case, it will decide cases, otherwise moot and academic, if it finds that (a) there is a grave violation of the Constitution; 95 (b) the
situation is of exceptional character and paramount public interest is involved; 96(c) the constitutional issue raised requires formulation of
controlling principles to guide the bench, the bar, and the public; 97 and (d) the case is capable of repetition yet evading review. 98
Another exclusionary circumstance that may be considered is where there is a voluntary cessation of the activity complained of by the defendant or
doer. Thus, once a suit is filed and the doer voluntarily ceases the challenged conduct, it does not automatically deprive the tribunal of power to hear
and determine the case and does not render the case moot especially when the plaintiff seeks damages or prays for injunctive relief against the
possible recurrence of the violation. 93 SICDAa
The present petitions fall squarely into these exceptions to thus thrust them into the domain of judicial review. The grounds cited above in David are
just as applicable in the present cases as they were, not only in David, but also in Province of Batangas v. Romulo 100 and Manalo v.
Calderon 101 where the Court similarly decided them on the merits, supervening events that would ordinarily have rendered the same moot
notwithstanding.
Petitions not mooted
Contrary then to the asseverations of respondents, the non-signing of the MOA-AD and the eventual dissolution of the GRP Peace Panel did not moot
the present petitions. It bears emphasis that the signing of the MOA-AD did not push through due to the Court's issuance of a Temporary Restraining
Order.
Contrary too to respondents' position, the MOA-AD cannot be considered a mere "list of consensus points", especially given its nomenclature,
the need to have it signed or initialed by all the parties concerned on August 5, 2008, and the far-reaching Constitutional implications of these
"consensus points", foremost of which is the creation of the BJE.
In fact, as what will, in the main, be discussed, there is a commitment on the part of respondents to amend and effect necessary changes to the
existing legal framework for certain provisions of the MOA-AD to take effect. Consequently, the present petitions are not confined to the terms and
provisions of the MOA-AD, but to other on-goingand future negotiations and agreements necessary for its realization. The petitions have not,
therefore, been rendered moot and academic simply by the public disclosure of the MOA-AD, 101 the manifestation that it will not be signed as well
as the disbanding of the GRP Panel notwithstanding.
Petitions are imbued with paramount public interest
There is no gainsaying that the petitions are imbued with paramount public interest, involving a significant part of the country's territory and the
wide-ranging political modifications of affected LGUs. The assertion that the MOA-AD is subject to further legal enactments including possible
Constitutional amendments more than ever provides impetus for the Court to formulate controlling principles to guide the bench, the bar, the
public and, in this case, the government and its negotiating entity.
Respondents cite Suplico v. NEDA, et al. 103 where the Court did not "pontificat[e] on issues which no longer legitimately constitute an actual case or
controversy [as this] will do more harm than good to the nation as a whole".
The present petitions must be differentiated from Suplico. Primarily, in Suplico, what was assailed and eventually cancelled was a stand-alone
government procurement contract for a national broadband network involving a one-time contractual relation between two parties — the
government and a private foreign corporation. As the issues therein involved specific government procurement policies and standard principles on
contracts, the majority opinion in Suplico found nothing exceptional therein, the factual circumstances being peculiar only to the transactions and
parties involved in the controversy. ATcaID

The MOA-AD is part of a series of agreements


In the present controversy, the MOA-AD is a significant part of a series of agreements necessary to carry out the Tripoli Agreement 2001. The MOA-
AD which dwells on theAncestral Domain Aspect of said Tripoli Agreement is the third such component to be undertaken following the
implementation of the Security Aspect in August 2001 and theHumanitarian, Rehabilitation and Development Aspect in May 2002.
Accordingly, even if the Executive Secretary, in his Memorandum of August 28, 2008 to the Solicitor General, has stated that "no matter what the
Supreme Court ultimately decides[,] the government will not sign the MOA[-AD]", mootness will not set in in light of the terms of the Tripoli
Agreement 2001.
Need to formulate principles-guidelines
Surely, the present MOA-AD can be renegotiated or another one will be drawn up to carry out the Ancestral Domain Aspect of the Tripoli
Agreement 2001, in another or in any form, which could contain similar or significantly drastic provisions. While the Court notes the word of the
Executive Secretary that the government "is committed to securing an agreement that is both constitutional and equitable because that is the only
way that long-lasting peace can be assured", it is minded to render a decision on the merits in the present petitions to formulate controlling
principles to guide the bench, the bar, the public and, most especially, the government in negotiating with the MILF regarding Ancestral Domain.
Respondents invite the Court's attention to the separate opinion of then Chief Justice Artemio Panganiban in Sanlakas v. Reyes 104 in which he
stated that the doctrine of "capable of repetition yet evading review" can override mootness, "provided the party raising it in a proper case has been
and/or continue to be prejudiced or damaged as a direct result of their issuance". They contend that the Court must have jurisdiction over the
subject matter for the doctrine to be invoked. DHSEcI
The present petitions all contain prayers for Prohibition over which this Court exercises original jurisdiction. While G.R. No. 183893 (City of Iligan v.
GRP) is a petition for Injunction and Declaratory Relief, the Court will treat it as one for Prohibition as it has far reaching implications and raises
questions that need to be resolved. 105 At all events, the Court has jurisdiction over most if not the rest of the petitions.
Indeed, the present petitions afford a proper venue for the Court to again apply the doctrine immediately referred to as what it had done in a
number of landmark cases. 106There is a reasonable expectation that petitioners, particularly the Provinces of North Cotabato, Zamboanga del Norte
and Sultan Kudarat, the Cities of Zamboanga, Iligan and Isabela, and the Municipality of Linamon, will again be subjected to the same problem in the
future as respondents' actions are capable of repetition, in another or any form.
It is with respect to the prayers for Mandamus that the petitions have become moot, respondents having, by Compliance of August 7, 2008, provided
this Court and petitioners with official copies of the final draft of the MOA-AD and its annexes. Too, intervenors have been furnished, or have
procured for themselves, copies of the MOA-AD.
V.SUBSTANTIVE ISSUES
As culled from the Petitions and Petitions-in-Intervention, there are basically two SUBSTANTIVE issues to be resolved, one relating to the manner in
which the MOA-AD was negotiated and finalized, the other relating to its provisions, viz.:
1.Did respondents violate constitutional and statutory provisions on public consultation and the right to information when they
negotiated and later initialed the MOA-AD?
2.Do the contents of the MOA-AD violate the Constitution and the laws?
ON THE FIRST SUBSTANTIVE ISSUE
Petitioners invoke their constitutional right to information on matters of public concern, as provided in Section 7, Article III on the Bill of Rights:
Sec. 7.The right of the people to information on matters of public concern shall be recognized. Access to official records, and to
documents, and papers pertaining to official acts, transactions, or decisions, as well as to government research data used as
basis for policy development, shall be afforded the citizen, subject to such limitations as may be provided by law. 107
As early as 1948, in Subido v. Ozaeta, 108 Court has recognized the statutory right to examine and inspect public records, a right which was
eventually accorded constitutional status. SaHTCE
The right of access to public documents, as enshrined in both the 1973 Constitution and the 1987 Constitution, has been recognized as a self-
executory constitutional right. 109
In the 1976 case of Baldoza v. Hon. Judge Dimaano, 110 the Court ruled that access to public records is predicated on the right of the people to
acquire information on matters of public concern since, undoubtedly, in a democracy, the public has a legitimate interest in matters of social and
political significance.
. . . The incorporation of this right in the Constitution is a recognition of the fundamental role of free exchange of information
in a democracy. There can be no realistic perception by the public of the nation's problems, nor a meaningful democratic
decision-making if they are denied access to information of general interest. Information is needed to enable the members of
society to cope with the exigencies of the times. As has been aptly observed: "Maintaining the flow of such information
depends on protection for both its acquisition and its dissemination since, if either process is interrupted, the flow inevitably
ceases." . . . 111
In the same way that free discussion enables members of society to cope with the exigencies of their time, access to information of general interest
aids the people in democratic decision-making by giving them a better perspective of the vital issues confronting the nation 112 so that they may be
able to criticize and participate in the affairs of the government in a responsible, reasonable and effective manner. It is by ensuring an unfettered and
uninhibited exchange of ideas among a well-informed public that a government remains responsive to the changes desired by the people. 113
The MOA-AD is a matter of public concern
That the subject of the information sought in the present cases is a matter of public concern 114 faces no serious challenge. In fact, respondents
admit that the MOA-AD is indeed of public concern. 115 In previous cases, the Court found that the regularity of real estate transactions entered in
the Register of Deeds, 116 the need for adequate notice to the public of the various laws, 117 the civil service eligibility of a public employee, 118 the
proper management of GSIS funds allegedly used to grant loans to public officials, 119 the recovery of the Marcoses' alleged ill-gotten
wealth, 120 and the identity of party-list nominees, 121 among others, are matters of public concern. Undoubtedly, the MOA-AD subject of the
present cases is of public concern, involving as it does the sovereignty and territorial integrity of the State, which directly affects the lives of the
public at large. ACcHIa
Matters of public concern covered by the right to information include steps and negotiations leading to the consummation of the contract. In not
distinguishing as to the executory nature or commercial character of agreements, the Court has categorically ruled:
. . . [T]he right to information "contemplates inclusion of negotiations leading to the consummation of the transaction".
Certainly, a consummated contract is not a requirement for the exercise of the right to information. Otherwise, the people can
never exercise the right if no contract is consummated, and if one is consummated, it may be too late for the public to expose
its defects.
Requiring a consummated contract will keep the public in the dark until the contract, which may be grossly disadvantageous to
the government or even illegal, becomes fait accompli. This negates the State policy of full transparency on matters of public
concern, a situation which the framers of the Constitution could not have intended. Such a requirement will prevent the
citizenry from participating in the public discussion of any proposed contract, effectively truncating a basic right enshrined in
the Bill of Rights. We can allow neither an emasculation of a constitutional right, nor a retreat by the State of its avowed
"policy of full disclosure of all its transactions involving public interest." 122(Emphasis and italics in the original)
Intended as a "splendid symmetry" 123 to the right to information under the Bill of Rights is the policy of public disclosure under Section 28, Article II
of the Constitution reading:
Sec. 28.Subject to reasonable conditions prescribed by law, the State adopts and implements a policy of full public disclosure of
all its transactions involving public interest. 124
The policy of full public disclosure enunciated in above-quoted Section 28 complements the right of access to information on matters of public
concern found in the Bill of Rights. The right to information guarantees the right of the people to demand information, while Section 28 recognizes
the duty of officialdom to give information even if nobody demands. 125
The policy of public disclosure establishes a concrete ethical principle for the conduct of public affairs in a genuinely open democracy, with the
people's right to know as the centerpiece. It is a mandate of the State to be accountable by following such policy. 126 These provisions are vital to
the exercise of the freedom of expression and essential to hold public officials at all times accountable to the people. 127 DcaCSE
Whether Section 28 is self-executory, the records of the deliberations of the Constitutional Commission so disclose:
MR. SUAREZ.
And since this is not self-executory, this policy will not be enunciated or will not be in force and effect until after Congress shall
have provided it.

MR. OPLE.
I expect it to influence the climate of public ethics immediately but, of course, the implementing law will have to be enacted by
Congress, Mr. Presiding Officer. 128
The following discourse, after Commissioner Hilario Davide, Jr., sought clarification on the issue, is enlightening.
MR. DAVIDE.
I would like to get some clarifications on this. Mr. Presiding Officer, did I get the Gentleman correctly as having said that this is
not a self-executing provision? It would require a legislation by Congress to implement?
MR. OPLE.
Yes. Originally, it was going to be self-executing, but I accepted an amendment from Commissioner Regalado, so that the
safeguards on national interest are modified by the clause "as may be provided by law".
MR. DAVIDE.
But as worded, does it not mean that this will immediately take effect and Congress may provide for reasonable
safeguards on the sole ground national interest?
MR. OPLE.
Yes. I think so, Mr. Presiding Officer, I said earlier that it should immediately influence the climate of the conduct of public
affairs but, of course, Congress here may no longer pass a law revoking it, or if this is approved, revoking this
principle, which is inconsistent with this policy. 129 (Emphasis supplied)
Indubitably, the effectivity of the policy of public disclosure need not await the passing of a statute. As Congress cannot revoke this principle, it is
merely directed to provide for "reasonable safeguards". The complete and effective exercise of the right to information necessitates that its
complementary provision on public disclosure derive the same self-executory nature. Since both provisions go hand-in-hand, it is absurd to say that
the broader 130 right to information on matters of public concern is already enforceable while the correlative duty of the State to disclose its
transactions involving public interest is not enforceable until there is an enabling law. Respondents cannot thus point to the absence of an
implementing legislation as an excuse in not effecting such policy. SDHAEC
An essential element of these freedoms is to keep open a continuing dialogue or process of communication between the government and the
people. It is in the interest of the State that the channels for free political discussion be maintained to the end that the government may perceive and
be responsive to the people's will. 131 Envisioned to becorollary to the twin rights to information and disclosure is the design for feedback
mechanisms.
MS. ROSARIO BRAID.
Yes. And lastly, Mr. Presiding Officer, will the people be able to participate? Will the government provide feedback
mechanisms so that the people can participate and can react where the existing media facilities are not able to
provide full feedback mechanisms to the government? I suppose this will be part of the government implementing
operational mechanisms.
MR. OPLE.
Yes. I think through their elected representatives and that is how these courses take place. There is a message and a feedback,
both ways.
xxx xxx xxx
MS. ROSARIO BRAID.
Mr. Presiding Officer, may I just make one last sentence?
I think when we talk about the feedback network, we are not talking about public officials but also network of private
business o[r] community-based organizations that will be reacting. As a matter of fact, we will put more credence
or credibility on the private network of volunteers and voluntary community-based organizations. So I do not think
we are afraid that there will be another OMA in the making. 132 (Emphasis supplied) ScCEIA
The imperative of a public consultation, as a species of the right to information, is evident in the "marching orders" to respondents. The mechanics
for the duty to disclose information and to conduct public consultation regarding the peace agenda and process is manifestly provided by E.O. No.
3. 133 The preambulatory clause of E.O. No. 3 declares that there is a need to further enhance the contribution of civil society to the comprehensive
peace process by institutionalizing the people's participation.
One of the three underlying principles of the comprehensive peace process is that it "should be community-based, reflecting the sentiments, values
and principles important to all Filipinos" and "shall be defined not by the government alone, nor by the different contending groups only, but by all
Filipinos as one community". 134 Included as a component of the comprehensive peace process is consensus-building and empowerment for peace,
which includes "continuing consultations on both national and local levels to build consensus for a peace agenda and process, and the mobilization
and facilitation of people's participation in the peace process". 135
Clearly, E.O. No. 3 contemplates not just the conduct of a plebiscite to effectuate "continuing" consultations, contrary to respondents' position
that plebiscite is "more than sufficient consultation". 136
Further, E.O. No. 3 enumerates the functions and responsibilities of the PAPP, one of which is to "[c]onduct regular dialogues with the National Peace
Forum (NPF) and other peace partners to seek relevant information, comments, recommendations as well as to render appropriate and timely
reports on the progress of the comprehensive peace process". 137 E.O. No. 3 mandates the establishment of the NPF to be "the principal forum for
the PAPP to consult with and seek advi[c]e from the peace advocates, peace partners and concerned sectors of society on both national and local
levels, on the implementation of the comprehensive peace process, as well as for government[-]civil society dialogue and consensus-building on
peace agenda and initiatives". 138 aTADcH
In fine, E.O. No. 3 establishes petitioners' right to be consulted on the peace agenda, as a corollary to the constitutional right to information and
disclosure.
PAPP Esperon committed grave
abuse of discretion
The PAPP committed grave abuse of discretion when he failed to carry out the pertinent consultation. The furtive process by which the MOA-AD
was designed and craftedruns contrary to and in excess of the legal authority, and amounts to a whimsical, capricious, oppressive, arbitrary and
despotic exercise thereof.
The Court may not, of course, require the PAPP to conduct the consultation in a particular way or manner. It may, however, require him to comply
with the law and discharge the functions within the authority granted by the President. 139
Petitioners are not claiming a seat at the negotiating table, contrary to respondents' retort in justifying the denial of petitioners' right to be
consulted. Respondents' stance manifests the manner by which they treat the salient provisions of E.O. No. 3 on people's participation. Such
disregard of the express mandate of the President is not much different from superficial conduct toward token provisos that border on classic lip
service. 140 It illustrates a gross evasion of positive duty and a virtual refusal to perform the duty enjoined.
As for respondents' invocation of the doctrine of executive privilege, it is not tenable under the premises. The argument defies sound reason when
contrasted with E.O. No. 3's explicit provisions on continuing consultation and dialogue on both national and local levels. The executive order even
recognizes the exercise of the public's right even before the GRP makes its official recommendations or before the government proffers its definite
propositions. 141 It bears emphasis that E.O. No. 3 seeks to elicit relevant advice, information, comments and recommendations from the people
through dialogue.
AT ALL EVENTS, respondents effectively waived the defense of executive privilege in view of their unqualified disclosure of the official copies of the
final draft of the MOA-AD. By unconditionally complying with the Court's August 4, 2008 Resolution, without a prayer for the document's
disclosure in camera, or without a manifestation that it was complying therewith ex abundante ad cautelam. ESTDcC
Petitioners' assertion that the Local Government Code (LGC) of 1991 declares it a State policy to "require all national agencies and offices to conduct
periodic consultations with appropriate local government units, non-governmental and people's organizations, and other concerned sectors of the
community before any project or program is implemented in their respective jurisdictions" 142 is well-taken. The LGC chapter on intergovernmental
relations puts flesh into this avowed policy:
Prior Consultations Required. — No project or program shall be implemented by government authorities unless the
consultations mentioned in Sections 2 (c) and 26 hereof are complied with, and prior approval of the sanggunian concerned is
obtained: Provided, That occupants in areas where such projects are to be implemented shall not be evicted unless
appropriate relocation sites have been provided, in accordance with the provisions of the Constitution. 143 (Italics and
underscoring supplied)
In Lina, Jr. v. Hon. Paño, 144 the Court held that the above-stated policy and above-quoted provision of the LGU apply only to national programs or
projects which are to be implemented in a particular local community. Among the programs and projects covered are those that are critical to the
environment and human ecology including those that may call for the eviction of a particular group of people residing in the locality where these will
be implemented. 145 The MOA-AD is one peculiar program that unequivocally and unilaterally vests ownership of a vast territory to the
Bangsamoro people, 146 which could pervasively and drastically result to the diaspora or displacement of a great number of inhabitants from
their total environment.
With respect to the indigenous cultural communities/indigenous peoples (ICCs/IPs), whose interests are represented herein by petitioner Lopez and
are adversely affected by the MOA-AD, the ICCs/IPs have, under the IPRA, the right to participate fully at all levels of decision-making in matters
which may affect their rights, lives and destinies. 147 The MOA-AD, an instrument recognizing ancestral domain, failed to justify its non-compliance
with the clear-cut mechanisms ordained in said Act, 148 which entails, among other things, the observance of the free and prior informed consent of
the ICCs/IPs.

Notably, the IPRA does not grant the Executive Department or any government agency the power to delineate and recognize an ancestral domain
claim by mere agreement or compromise. The recognition of the ancestral domain is the raison d'etre of the MOA-AD, without which all other
stipulations or "consensus points" necessarily must fail. In proceeding to make a sweeping declaration on ancestral domain, without complying with
the IPRA, which is cited as one of the TOR of the MOA-AD, respondents clearly transcended the boundaries of their authority. As it seems, even the
heart of the MOA-AD is still subject to necessary changes to the legal framework. While paragraph 7 on Governance suspends the effectivity of all
provisions requiring changes to the legal framework, such clause is itself invalid, as will be discussed in the following section. TcSHaD
Indeed, ours is an open society, with all the acts of the government subject to public scrutiny and available always to public cognizance. This has to
be so if the country is to remain democratic, with sovereignty residing in the people and all government authority emanating from them. 149
ON THE SECOND SUBSTANTIVE ISSUE
With regard to the provisions of the MOA-AD, there can be no question that they cannot all be accommodated under the present
Constitution and laws. Respondents have admitted as much in the oral arguments before this Court, and the MOA-AD itself recognizes the need
to amend the existing legal framework to render effective at least some of its provisions. Respondents, nonetheless, counter that the MOA-AD is
free of any legal infirmity because any provisions therein which are inconsistent with the present legal framework will not be effective until the
necessary changes to that framework are made. The validity of this argument will be considered later. For now, the Court shall pass upon how
The MOA-AD is inconsistent with
the Constitution and laws as
presently worded.
In general, the objections against the MOA-AD center on the extent of the powers conceded therein to the BJE. Petitioners assert that the powers
granted to the BJE exceed those granted to any local government under present laws, and even go beyond those of the present ARMM. Before
assessing some of the specific powers that would have been vested in the BJE, however, it would be useful to turn first to a general idea that serves
as a unifying link to the different provisions of the MOA-AD, namely, the international lawconcept of association. Significantly, the MOA-AD explicitly
alludes to this concept, indicating that the Parties actually framed its provisions with it in mind.
Association is referred to in paragraph 3 on TERRITORY, paragraph 11 on RESOURCES, and paragraph 4 on GOVERNANCE. It is in the last mentioned
provision, however, that the MOA-AD most clearly uses it to describe the envisioned relationship between the BJE and the Central
Government. aTCAcI
4.The relationship between the Central Government and the Bangsamoro juridical entity shall be associative characterized
by shared authority and responsibilitywith a structure of governance based on executive, legislative, judicial and
administrative institutions with defined powers and functions in the comprehensive compact. A period of transition
shall be established in a comprehensive peace compact specifying the relationship between the Central Government
and the BJE. (Emphasis and underscoring supplied)
The nature of the "associative" relationship may have been intended to be defined more precisely in the still to be forged Comprehensive Compact.
Nonetheless, given that there is a concept of "association" in international law, and the MOA-AD — by its inclusion of international law instruments
in its TOR — placed itself in an international legal context, that concept of association may be brought to bear in understanding the use of the
term "associative" in the MOA-AD. cdrep
Keitner and Reisman state that
[a]n association is formed when two states of unequal power voluntarily establish durable links. In the basic model, one state,
the associate, delegates certain responsibilities to the other, the principal, while maintaining its international status as a
state. Free associations represent a middle ground between integration and independence. . . . 150 (Emphasis and
underscoring supplied)
For purposes of illustration, the Republic of the Marshall Islands and the Federated States of Micronesia (FSM), formerly part of the U.S.-
administered Trust Territory of the Pacific Islands, 151 are associated states of the U.S. pursuant to a Compact of Free Association. The currency in
these countries is the U.S. dollar, indicating their very close ties with the U.S., yet they issue their own travel documents, which is a mark of their
statehood. Their international legal status as states was confirmed by the UN Security Council and by their admission to UN membership.
According to their compacts of free association, the Marshall Islands and the FSM generally have the capacity to conduct foreign affairs in their own
name and right, such capacity extending to matters such as the law of the sea, marine resources, trade, banking, postal, civil aviation, and cultural
relations. The U.S. government, when conducting its foreign affairs, is obligated to consult with the governments of the Marshall Islands or the FSM
on matters which it (U.S. government) regards as relating to or affecting either government.
In the event of attacks or threats against the Marshall Islands or the FSM, the U.S. government has the authority and obligation to defend them as if
they were part of U.S. territory. The U.S. government, moreover, has the option of establishing and using military areas and facilities within these
associated states and has the right to bar the military personnel of any third country from having access to these territories for military purposes.
It bears noting that in U.S. constitutional and international practice, free association is understood as an international association between
sovereigns. The Compact of Free Association is a treaty which is subordinate to the associated nation's national constitution, and each party may
terminate the association consistent with the right of independence. It has been said that, with the admission of the U.S.-associated states to the UN
in 1990, the UN recognized that the American model of free association is actually based on an underlying status of independence. 152 ACTISE
In international practice, the "associated state" arrangement has usually been used as a transitional device of former colonies on their way to full
independence. Examples of states that have passed through the status of associated states as a transitional phase are Antigua, St. Kitts-Nevis-
Anguilla, Dominica, St. Lucia, St. Vincent and Grenada. All have since become independent states. 153
Back to the MOA-AD, it contains many provisions which are consistent with the international legal concept of association, specifically the following:
the BJE's capacity to enter into economic and trade relations with foreign countries, the commitment of the Central Government to ensure the BJE's
participation in meetings and events in the ASEAN and the specialized UN agencies, and the continuing responsibility of the Central Government over
external defense. Moreover, the BJE's right to participate in Philippine official missions bearing on negotiation of border agreements, environmental
protection, and sharing of revenues pertaining to the bodies of water adjacent to or between the islands forming part of the ancestral
domain, resembles the right of the governments of FSM and the Marshall Islands to be consulted by the U.S. government on any foreign affairs
matter affecting them.
These provisions of the MOA indicate, among other things, that the Parties aimed to vest in the BJE the status of an associated state or, at any rate,
a status closely approximating it.
The concept of association is not
recognized under the present
Constitution
No province, city, or municipality, not even the ARMM, is recognized under our laws as having an "associative" relationship with the national
government. Indeed, the concept implies powers that go beyond anything ever granted by the Constitution to any local or regional government. It
also implies the recognition of the associated entity as a state. The Constitution, however, does not contemplate any state in this jurisdiction other
than the Philippine State, much less does it provide for a transitory status that aims to prepare any part of Philippine territory for
independence. caTIDE
Even the mere concept animating many of the MOA-AD's provisions, therefore, already requires for its validity the amendment of constitutional
provisions, specifically the following provisions of Article X:
SEC. 1.The territorial and political subdivisions of the Republic of the Philippines are the provinces, cities, municipalities, and
barangays. There shall be autonomous regionsin Muslim Mindanao and the Cordilleras as hereinafter provided.
SEC. 15.There shall be created autonomous regions in Muslim Mindanao and in the Cordilleras consisting of provinces, cities,
municipalities, and geographical areas sharing common and distinctive historical and cultural heritage, economic and social
structures, and other relevant characteristics within the framework of this Constitution and the national sovereignty as well
as territorial integrity of the Republic of the Philippines.
The BJE is a far more powerful
entity than the autonomous region
recognized in the Constitution
It is not merely an expanded version of the ARMM, the status of its relationship with the national government being fundamentally different from
that of the ARMM. Indeed, BJE is a state in all but name as it meets the criteria of a state laid down in the Montevideo Convention, 154 namely,
a permanent population, a defined territory, a government, and a capacity to enter into relations with other states. AEITDH

Even assuming arguendo that the MOA-AD would not necessarily sever any portion of Philippine territory, the spirit animating it — which has
betrayed itself by its use of the concept of association — runs counter to the national sovereignty and territorial integrity of the Republic.
The defining concept underlying the relationship between the national government and the BJE being itself contrary to the present Constitution,
it is not surprising that many of the specific provisions of the MOA-AD on the formation and powers of the BJE are in conflict with the Constitution
and the laws.
Article X, Section 18 of the Constitution provides that "[t]he creation of the autonomous region shall be effective when approved by a majority of the
votes cast by the constituent units in a plebiscite called for the purpose, provided that only provinces, cities, and geographic areas voting favorably
in such plebiscite shall be included in the autonomous region". (Emphasis supplied)
As reflected above, the BJE is more of a state than an autonomous region. But even assuming that it is covered by the term "autonomous region" in
the constitutional provision just quoted, the MOA-AD would still be in conflict with it. Under paragraph 2 (c) on TERRITORY in relation to 2 (d) and 2
(e), the present geographic area of the ARMM and, in addition, the municipalities of Lanao del Norte which voted for inclusion in the ARMM during
the 2001 plebiscite — Baloi, Munai, Nunungan, Pantar, Tagoloan and Tangkal — are automatically part of the BJE without need of another
plebiscite, in contrast to the areas under Categories A and B mentioned earlier in the overview. That the present components of the ARMM and the
above-mentioned municipalities voted for inclusion therein in 2001, however, does not render another plebiscite unnecessary under the
Constitution, precisely because what these areas voted for then was their inclusion in the ARMM, not the BJE. ScCIaA
The MOA-AD, moreover, would
not comply with Article X, Section
20 of the Constitution
since that provision defines the powers of autonomous regions as follows:
SEC. 20.Within its territorial jurisdiction and subject to the provisions of this Constitution and national laws, the organic act of
autonomous regions shall provide for legislative powers over:
(1)Administrative organization;
(2)Creation of sources of revenues;
(3)Ancestral domain and natural resources;
(4)Personal, family, and property relations;
(5)Regional urban and rural planning development;
(6)Economic, social, and tourism development;
(7)Educational policies;
(8)Preservation and development of the cultural heritage; and
(9)Such other matters as may be authorized by law for the promotion of the general welfare of the people of the
region. (Underscoring supplied)
Again on the premise that the BJE may be regarded as an autonomous region, the MOA-AD would require an amendment that would expand the
above-quoted provision. The mere passage of new legislation pursuant to sub-paragraph No. 9 of said constitutional provision would not suffice,
since any new law that might vest in the BJE the powers found in the MOA-AD must, itself, comply with other provisions of the Constitution. It would
not do, for instance, to merely pass legislation vesting the BJE with treaty-making power in order to accommodate paragraph 4 of the strand on
RESOURCES which states: "The BJE is free to enter into any economic cooperation and trade relations with foreign countries: provided, however, that
such relationships and understandings do not include aggression against the Government of the Republic of the Philippines . . . ." Under our
constitutional system, it is only the President who has that power. Pimentel v. Executive Secretary 155 instructs:
In our system of government, the President, being the head of state, is regarded as the sole organ and authority in external
relations and is the country's sole representative with foreign nations. As the chief architect of foreign policy, the President
acts as the country's mouthpiece with respect to international affairs. Hence, the President is vested with the authority
to deal with foreign states and governments, extend or withhold recognition, maintain diplomatic relations, enter into
treaties, and otherwise transact the business of foreign relations. In the realm of treaty-making, the President has the sole
authority to negotiate with other states. (Emphasis and underscoring supplied) CIHAED
Article II, Section 22 of the Constitution must also be amended if the scheme envisioned in the MOA-AD is to be effected. That constitutional
provision states: "The State recognizes and promotes the rights of indigenous cultural communities within the framework of national unity and
development." (Underscoring supplied) An associativearrangement does not uphold national unity. While there may be a semblance of unity
because of the associative ties between the BJE and the national government, the act of placing a portion of Philippine territory in a status which, in
international practice, has generally been a preparation for independence, is certainly not conducive to nationalunity.
Besides being irreconcilable with the
Constitution, the MOA-AD is also
inconsistent with prevailing
statutory law, among which are
R.A. No. 9054 156 or the Organic
|Act of the ARMM, and the IPRA. 157
Article X, Section 3 of the Organic Act of the ARMM is a bar to the adoption of the definition of "Bangsamoro people" used in the MOA-AD.
Paragraph 1 on CONCEPTS AND PRINCIPLES states:
1.It is the birthright of all Moros and all Indigenous peoples of Mindanao to identify themselves and be accepted as
"Bangsamoros". The Bangsamoro people refers to those who are natives or original inhabitants of Mindanao and its adjacent
islands including Palawan and the Sulu archipelago at the time of conquest or colonization of its descendants whether mixed
or of full blood. Spouses and their descendants are classified as Bangsamoro. The freedom of choice of the Indigenous people
shall be respected. (Emphasis and underscoring supplied) cHAaCE
This use of the term Bangsamoro sharply contrasts with that found in the Article X, Section 3 of the Organic Act, which, rather than lumping together
the identities of the Bangsamoro and other indigenous peoples living in Mindanao, clearly distinguishes between Bangsamoro people and Tribal
peoples, as follows:
"As used in this Organic Act, the phrase "indigenous cultural community" refers to Filipino citizens residing in the autonomous
region who are:
(a)Tribal peoples. These are citizens whose social, cultural and economic conditions distinguish them from other sectors of the
national community; and
(b)Bangsa Moro people. These are citizens who are believers in Islam and who have retained some or all of their own social,
economic, cultural, and political institutions."
Respecting the IPRA, it lays down the prevailing procedure for the delineation and recognition of ancestral domains. The MOA-AD's manner of
delineating the ancestral domain of the Bangsamoro people is a clear departure from that procedure. By paragraph 1 of TERRITORY, the Parties
simply agree that, subject to the delimitations in the agreed Schedules, "[t]he Bangsamoro homeland and historic territory refer to the land mass as
well as the maritime, terrestrial, fluvial and alluvial domains, and the aerial domain, the atmospheric space above it, embracing the Mindanao-Sulu-
Palawan geographic region".
Chapter VIII of the IPRA, on the other hand, lays down a detailed procedure, as illustrated in the following provisions thereof:
SEC. 52.Delineation Process. — The identification and delineation of ancestral domains shall be done in accordance with the
following procedures:
xxx xxx xxx
b)Petition for Delineation. — The process of delineating a specific perimeter may be initiated by the NCIP with the consent of
the ICC/IP concerned, or through a Petition for Delineation filed with the NCIP, by a majority of the members of the
ICCs/IPs; CSaIAc
c)Delineation Proper. — The official delineation of ancestral domain boundaries including census of all community members
therein, shall be immediately undertaken by the Ancestral Domains Office upon filing of the application by the
ICCs/IPs concerned. Delineation will be done in coordination with the community concerned and shall at all times
include genuine involvement and participation by the members of the communities concerned;
d)Proof Required. — Proof of Ancestral Domain Claims shall include the testimony of elders or community under oath, and
other documents directly or indirectly attesting to the possession or occupation of the area since time immemorial
by such ICCs/IPs in the concept of owners which shall be any one (1) of the following authentic documents:
1)Written accounts of the ICCs/IPs customs and traditions;
2)Written accounts of the ICCs/IPs political structure and institution; ISaCTE
3)Pictures showing long term occupation such as those of old improvements, burial grounds, sacred places and old
villages;
4)Historical accounts, including pacts and agreements concerning boundaries entered into by the ICCs/IPs concerned
with other ICCs/IPs;
5)Survey plans and sketch maps;
6)Anthropological data;
7)Genealogical surveys;
8)Pictures and descriptive histories of traditional communal forests and hunting grounds;
9)Pictures and descriptive histories of traditional landmarks such as mountains, rivers, creeks, ridges, hills, terraces
and the like; and
10)Write-ups of names and places derived from the native dialect of the community.
e)Preparation of Maps. — On the basis of such investigation and the findings of fact based thereon, the Ancestral Domains
Office of the NCIP shall prepare a perimeter map, complete with technical descriptions, and a description of the
natural features and landmarks embraced therein;
f)Report of Investigation and Other Documents. — A complete copy of the preliminary census and a report of investigation,
shall be prepared by the Ancestral Domains Office of the NCIP;

g)Notice and Publication. — A copy of each document, including a translation in the native language of the ICCs/IPs concerned
shall be posted in a prominent place therein for at least fifteen (15) days. A copy of the document shall also be
posted at the local, provincial and regional offices of the NCIP, and shall be published in a newspaper of general
circulation once a week for two (2) consecutive weeks to allow other claimants to file opposition thereto within
fifteen (15) days from date of such publication: Provided, That in areas where no such newspaper exists,
broadcasting in a radio station will be a valid substitute: Provided, further, That mere posting shall be deemed
sufficient if both newspaper and radio station are not available; aCTADI
h)Endorsement to NCIP. — Within fifteen (15) days from publication, and of the inspection process, the Ancestral Domains
Office shall prepare a report to the NCIP endorsing a favorable action upon a claim that is deemed to have sufficient
proof. However, if the proof is deemed insufficient, the Ancestral Domains Office shall require the submission of
additional evidence: Provided, That the Ancestral Domains Office shall reject any claim that is deemed patently false
or fraudulent after inspection and verification: Provided, further, That in case of rejection, the Ancestral Domains
Office shall give the applicant due notice, copy furnished all concerned, containing the grounds for denial. The denial
shall be appealable to the NCIP: Provided, furthermore, That in cases where there are conflicting claims among
ICCs/IPs on the boundaries of ancestral domain claims, the Ancestral Domains Office shall cause the contending
parties to meet and assist them in coming up with a preliminary resolution of the conflict, without prejudice to its full
adjudication according to the section below.
xxx xxx xxx
To remove all doubts about the irreconcilability of the MOA-AD with the present legal system, a discussion of not only the Constitution
and domestic statutes, but also of international law is in order, for
Article II, Section 2 of the
Constitution states that the
Philippines "adopts the generally
accepted principles of international
law as part of the law of the land".
Applying this provision of the Constitution, the Court, in Mejoff v. Director of Prisons, 158 held that the Universal Declaration of Human Rights is
part of the law of the land on account of which it ordered the release on bail of a detained alien of Russian descent whose deportation order
had not been executed even after two years. Similarly, the Court in Agustin v. Edu 159applied the aforesaid constitutional provision to the 1968
Vienna Convention on Road Signs and Signals.
International law has long recognized the right to self-determination of "peoples", understood not merely as the entire population of a State but also
a portion thereof. In considering the question of whether the people of Quebec had a right to unilaterally secede from Canada, the Canadian
Supreme Court in REFERENCE RE SECESSION OF QUEBEC160 had occasion to acknowledge that "the right of a people to self-determination is now so
widely recognized in international conventions that the principle has acquired a status beyond 'convention' and is considered a general principle of
international law".
Among the conventions referred to are the International Covenant on Civil and Political Rights 161 and the International Covenant on Economic,
Social and Cultural Rights 162 which state, in Article 1 of both covenants, that all peoples, by virtue of the right of self-determination, "freely
determine their political status and freely pursue their economic, social, and cultural development". DICSaH
The people's right to self-determination should not, however, be understood as extending to a unilateral right of secession. A distinction should be
made between the right of internal and external self-determination. REFERENCE RE SECESSION OF QUEBEC is again instructive:
"(ii)Scope of the Right to Self-determination
126.The recognized sources of international law establish that the right to self-determination of a people is normally fulfilled
through internal self-determination — a people's pursuit of its political, economic, social and cultural development within
the framework of an existing state. A right to external self-determination (which in this case potentially takes the form of
the assertion of a right to unilateral secession) arises in only the most extreme of cases and, even then, under carefully
defined circumstances. . . .
External self-determination can be defined as in the following statement from the Declaration on Friendly Relations,
supra, as
The establishment of a sovereign and independent State, the free association or integration with an independent State or
the emergence into any other political status freely determined by a people constitute modes of implementing the right of
self-determination by that people. (Emphasis added) HDAaIS
127.The international law principle of self-determination has evolved within a framework of respect for the territorial
integrity of existing states. The various international documents that support the existence of a people's right to self-
determination also contain parallel statements supportive of the conclusion that the exercise of such a right must be
sufficiently limited to prevent threats to an existing state's territorial integrity or the stability of relations between sovereign
states.
xxx xxx xxx (Emphasis, italics and underscoring supplied)
The Canadian Court went on to discuss the exceptional cases in which the right to external self-determination can arise, namely, where a people is
under colonial rule, is subject to foreign domination or exploitation outside a colonial context, and — less definitely but asserted by a number of
commentators — is blocked from the meaningful exercise of its right to internal self-determination. The Court ultimately held that the population of
Quebec had no right to secession, as the same is not under colonial rule or foreign domination, nor is it being deprived of the freedom to make
political choices and pursue economic, social and cultural development, citing that Quebec is equitably represented in legislative, executive and
judicial institutions within Canada, even occupying prominent positions therein.
The exceptional nature of the right of secession is further exemplified in the REPORT OF THE INTERNATIONAL COMMITTEE OF JURISTS ON THE LEGAL
ASPECTS OF THE AALAND ISLANDS QUESTION. 163 There, Sweden presented to the Council of the League of Nations the question of whether the
inhabitants of the Aaland Islands should be authorized to determine by plebiscite if the archipelago should remain under Finnish sovereignty or be
incorporated in the kingdom of Sweden. The Council, before resolving the question, appointed an International Committee composed of three jurists
to submit an opinion on the preliminary issue of whether the dispute should, based on international law, be entirely left to the domestic jurisdiction
of Finland. The Committee stated the rule as follows:
. . . [I]n the absence of express provisions in international treaties, the right of disposing of national territory is essentially an
attribute of the sovereignty of every State. Positive International Law does not recognize the right of national groups, as
such, to separate themselves from the State of which they form part by the simple expression of a wish, any more than it
recognizes the right of other States to claim such a separation. Generally speaking, the grant or refusal of the right to a
portion of its population of determining its own political fate by plebiscite or by some other method, is, exclusively, an
attribute of the sovereignty of every State which is definitively constituted. A dispute between two States concerning such a
question, under normal conditions therefore, bears upon a question which International Law leaves entirely to the domestic
jurisdiction of one of the States concerned. Any other solution would amount to an infringement of sovereign rights of a State
and would involve the risk of creating difficulties and a lack of stability which would not only be contrary to the very idea
embodied in term "State", but would also endanger the interests of the international community. If this right is not possessed
by a large or small section of a nation, neither can it be held by the State to which the national group wishes to be attached,
nor by any other State. (Emphasis and underscoring supplied) DHTCaI
The Committee held that the dispute concerning the Aaland Islands did not refer to a question which is left by international law to the domestic
jurisdiction of Finland, thereby applying the exception rather than the rule elucidated above. Its ground for departing from the general rule,
however, was a very narrow one, namely, the Aaland Islands agitation originated at a time when Finland was undergoing drastic political
transformation. The internal situation of Finland was, according to the Committee, so abnormal that, for a considerable time, the conditions
required for the formation of a sovereign State did not exist. In the midst of revolution, anarchy, and civil war, the legitimacy of the Finnish
national government was disputed by a large section of the people, and it had, in fact, been chased from the capital and forcibly prevented from
carrying out its duties. The armed camps and the police were divided into two opposing forces. In light of these circumstances, Finland was not,
during the relevant time period, a "definitively constituted" sovereign state. The Committee, therefore, found that Finland did not possess the
right to withhold from a portion of its population the option to separate itself — a right which sovereign nations generally have with respect to
their own populations.

Turning now to the more specific category of indigenous peoples, this term has been used, in scholarship as well as international, regional, and state
practices, to refer to groups with distinct cultures, histories, and connections to land (spiritual and otherwise) that have been forcibly incorporated
into a larger governing society. These groups are regarded as "indigenous" since they are the living descendants of pre-invasion inhabitants of lands
now dominated by others. Otherwise stated, indigenous peoples, nations, or communities are culturally distinctive groups that find themselves
engulfed by settler societies born of the forces of empire and conquest. 164 Examples of groups who have been regarded as indigenous peoples are
the Maori of New Zealand and the aboriginal peoples of Canada.
As with the broader category of "peoples", indigenous peoples situated within states do not have a general right to independence or secession from
those states under international law, 165 but they do have rights amounting to what was discussed above as the right to internal self-determination.
In a historic development last September 13, 2007, the UN General Assembly adopted the United Nations Declaration on the Rights of Indigenous
Peoples (UN DRIP) through General Assembly Resolution 61/295. The vote was 143 to 4, the Philippines being included among those in favor, and the
four voting against being Australia, Canada, New Zealand, and the U.S. The Declaration clearly recognized the right of indigenous peoples to self-
determination, encompassing the right to autonomy or self-government, to wit:
Article 3
Indigenous peoples have the right to self-determination. By virtue of that right they freely determine their political status and
freely pursue their economic, social and cultural development. TAHCEc
Article 4
Indigenous peoples, in exercising their right to self-determination, have the right to autonomy or self-government in matters
relating to their internal and local affairs, as well as ways and means for financing their autonomous functions.
Article 5
Indigenous peoples have the right to maintain and strengthen their distinct political, legal, economic, social and cultural
institutions, while retaining their right to participate fully, if they so choose, in the political, economic, social and cultural life of
the State.
Self-government, as used in international legal discourse pertaining to indigenous peoples, has been understood as equivalent to "internal self-
determination". 166 The extent of self-determination provided for in the UN DRIP is more particularly defined in its subsequent articles, some of
which are quoted hereunder:
Article 8
1.Indigenous peoples and individuals have the right not to be subjected to forced assimilation or destruction of their culture.
2.States shall provide effective mechanisms for prevention of, and redress for:
(a)Any action which has the aim or effect of depriving them of their integrity as distinct peoples, or of their cultural
values or ethnic identities;
(b)Any action which has the aim or effect of dispossessing them of their lands, territories or resources;
(c)Any form of forced population transfer which has the aim or effect of violating or undermining any of their
rights;
(d)Any form of forced assimilation or integration;
(e)Any form of propaganda designed to promote or incite racial or ethnic discrimination directed against them.
Article 21
1.Indigenous peoples have the right, without discrimination, to the improvement of their economic and social conditions,
including, inter alia, in the areas of education, employment, vocational training and retraining, housing, sanitation,
health and social security. SHCaDA
2.States shall take effective measures and, where appropriate, special measures to ensure continuing improvement of their
economic and social conditions. Particular attention shall be paid to the rights and special needs of indigenous elders,
women, youth, children and persons with disabilities.
Article 26
1.Indigenous peoples have the right to the lands, territories and resources which they have traditionally owned, occupied or
otherwise used or acquired.
2.Indigenous peoples have the right to own, use, develop and control the lands, territories and resources that they possess by
reason of traditional ownership or other traditional occupation or use, as well as those which they have otherwise
acquired.
3.States shall give legal recognition and protection to these lands, territories and resources. Such recognition shall be
conducted with due respect to the customs, traditions and land tenure systems of the indigenous peoples
concerned. ScCIaA
Article 30
1.Military activities shall not take place in the lands or territories of indigenous peoples, unless justified by a relevant public
interest or otherwise freely agreed with or requested by the indigenous peoples concerned.
2.States shall undertake effective consultations with the indigenous peoples concerned, through appropriate procedures and
in particular through their representative institutions, prior to using their lands or territories for military activities.
Article 32
1.Indigenous peoples have the right to determine and develop priorities and strategies for the development or use of their
lands or territories and other resources.
2.States shall consult and cooperate in good faith with the indigenous peoples concerned through their own representative
institutions in order to obtain their free and informed consent prior to the approval of any project affecting their
lands or territories and other resources, particularly in connection with the development, utilization or exploitation
of mineral, water or other resources. CEHcSI
3.States shall provide effective mechanisms for just and fair redress for any such activities, and appropriate measures shall be
taken to mitigate adverse environmental, economic, social, cultural or spiritual impact.
Article 37
1.Indigenous peoples have the right to the recognition, observance and enforcement of treaties, agreements and other
constructive arrangements concluded with States or their successors and to have States honour and respect such
treaties, agreements and other constructive arrangements.
2.Nothing in this Declaration may be interpreted as diminishing or eliminating the rights of indigenous peoples contained in
treaties, agreements and other constructive arrangements.
Article 38
States in consultation and cooperation with indigenous peoples, shall take the appropriate measures, including legislative
measures, to achieve the ends of this Declaration. EScIAa
Assuming that the UN DRIP, like the Universal Declaration on Human Rights, must now be regarded as embodying customary international law — a
question which the Court need not definitively resolve here — the obligations enumerated therein do not strictly require the Republic to grant the
Bangsamoro people, through the instrumentality of the BJE, the particular rights and powers provided for in the MOA-AD. Even the more specific
provisions of the UN DRIP are general in scope, allowing for flexibility in its application by the different States.
There is, for instance, no requirement in the UN DRIP that States now guarantee indigenous peoples their own police and internal security force.
Indeed, Article 8 presupposes that it is the State which will provide protection for indigenous peoples against acts like the forced dispossession of
their lands — a function that is normally performed by police officers. If the protection of a right so essential to indigenous people's identity is
acknowledged to be the responsibility of the State, then surely the protection of rights less significant to them as such peoples would also be the
duty of States. Nor is there in the UN DRIP an acknowledgement of the right of indigenous peoples to the aerial domain and atmospheric space.
What it upholds, in Article 26 thereof, is the right of indigenous peoples to the lands, territories and resources which they have traditionally owned,
occupied or otherwise used or acquired. IDTSEH
Moreover, the UN DRIP, while upholding the right of indigenous peoples to autonomy, does not obligate States to grant indigenous peoples the near-
independent status of an associated state. All the rights recognized in that document are qualified in Article 46 as follows:
1.Nothing in this Declaration may be interpreted as implying for any State, people, group or person any right to engage in any
activity or to perform any act contrary to the Charter of the United Nations or construed as authorizing or
encouraging any action which would dismember or impair, totally or in part, the territorial integrity or political
unity of sovereign and independent States.
Even if the UN DRIP were considered as part of the law of the land pursuant to Article II, Section 2 of the Constitution, it would not suffice to uphold
the validity of the MOA-AD so as to render its compliance with other laws unnecessary.
It is, therefore, clear that the MOA-AD contains numerous provisions that cannot be reconciled with the Constitution and the laws as presently
worded. Respondents proffer, however, that the signing of the MOA-AD alone would not have entailed any violation of law or grave abuse of
discretion on their part, precisely because it stipulates that the provisions thereof inconsistent with the laws shall not take effect until these laws are
amended. They cite paragraph 7 of the MOA-AD strand on GOVERNANCE quoted earlier, but which is reproduced below for convenience: AcSIDE
7.The Parties agree that the mechanisms and modalities for the actual implementation of this MOA-AD shall be spelt out in the
Comprehensive Compact to mutually take such steps to enable it to occur effectively.
Any provisions of the MOA-AD requiring amendments to the existing legal framework shall come into force upon signing of a
Comprehensive Compact and upon effecting the necessary changes to the legal framework with due regard to non derogation
of prior agreements and within the stipulated timeframe to be contained in the Comprehensive Compact.
Indeed, the foregoing stipulation keeps many controversial provisions of the MOA-AD from coming into force until the necessary changes to the legal
framework are effected.While the word "Constitution" is not mentioned in the provision now under consideration or anywhere else in the MOA-
AD, the term "legal framework" is certainly broad enough to include the Constitution.
Notwithstanding the suspensive clause, however, respondents, by their mere act of incorporating in the MOA-AD the provisions thereof regarding
the associative relationship between the BJE and the Central Government, have already violated the Memorandum of Instructions From The
President dated March 1, 2001, which states that the "negotiations shall be conducted in accordance with . . . the principles of the sovereignty
and territorial integrity of the Republic of the Philippines". (Emphasis supplied) Establishing an associative relationship between the BJE and the
Central Government is, for the reasons already discussed, a preparation for independence, or worse, an implicit acknowledgment of an independent
status already prevailing. HDAaIS
Even apart from the above-mentioned Memorandum, however, the MOA-AD is defective because the suspensive clause is invalid, as discussed
below.
The authority of the GRP Peace Negotiating Panel to negotiate with the MILF is founded on E.O. No. 3, Section 5 (c), which states that there shall be
established Government Peace Negotiating Panels for negotiations with different rebel groups to be "appointed by the President as her official
emissaries to conduct negotiations, dialogues, and face-to-face discussions with rebel groups". These negotiating panels are to report to the
President, through the PAPP on the conduct and progress of the negotiations.
It bears noting that the GRP Peace Panel, in exploring lasting solutions to the Moro Problem through its negotiations with the MILF, was not
restricted by E.O. No. 3 only to those options available under the laws as they presently stand. One of the components of a comprehensive peace
process, which E.O. No. 3 collectively refers to as the "Paths to Peace", is the pursuit of social, economic, and political reforms which may require
new legislation or even constitutional amendments. Sec. 4 (a) of E.O. No. 3, which reiterates Section 3 (a), of E.O. No. 125, 167 states: HSEcTC
SEC. 4.The Six Paths to Peace. — The components of the comprehensive peace process comprise the processes known as the
"Paths to Peace". These component processes are interrelated and not mutually exclusive, and must therefore be pursued
simultaneously in a coordinated and integrated fashion. They shall include, but may not be limited to, the following:
a.PURSUIT OF SOCIAL, ECONOMIC AND POLITICAL REFORMS. This component involves the vigorous implementation
of various policies, reforms, programs and projects aimed at addressing the root causes of internal armed
conflicts and social unrest. This may require administrative action, new legislation or even constitutional
amendments.
xxx xxx xxx (Emphasis supplied)
The MOA-AD, therefore, may reasonably be perceived as an attempt of respondents to address, pursuant to this provision of E.O. No. 3, the root
causes of the armed conflict in Mindanao. The E.O. authorized them to "think outside the box", so to speak. Hence, they negotiated and were set on
signing the MOA-AD that included various social, economic, and political reforms which cannot, however, all be accommodated within the present
legal framework, and which thus would require new legislation and constitutional amendments.
The inquiry on the legality of the "suspensive clause", however, cannot stop here, because it must be asked AIcaDC
whether the President herself may
exercise the power delegated to the
GRP Peace Panel under E.O. No. 3,
Sec. 4 (a).
The President cannot delegate a power that she herself does not possess. May the President, in the course of peace negotiations, agree to pursue
reforms that would require new legislation and constitutional amendments, or should the reforms be restricted only to those solutions which the
present laws allow? The answer to this question requires a discussion of HIEAcC
the extent of the President's power
to conduct peace negotiations.
That the authority of the President to conduct peace negotiations with rebel groups is not explicitly mentioned in the Constitution does not mean
that she has no such authority. In Sanlakas v. Executive Secretary, 168 in issue was the authority of the President to declare a state of rebellion — an
authority which is not expressly provided for in the Constitution. The Court held thus:
"In her ponencia in Marcos v. Manglapus, Justice Cortes put her thesis into jurisprudence. There, the Court, by a slim 8-7
margin, upheld the President's power to forbid the return of her exiled predecessor. The rationale for the majority's ruling
rested on the President's
. . . unstated residual powers which are implied from the grant of executive power and which are necessary for her
to comply with her duties under the Constitution. The powers of the President are not limited to what are
expressly enumerated in the article on the Executive Department and in scattered provisions of the Constitution.
This is so, notwithstanding the avowed intent of the members of the Constitutional Commission of 1986 to limit the
powers of the President as a reaction to the abuses under the regime of Mr. Marcos, for the result was a limitation of
specific powers of the President, particularly those relating to the commander-in-chief clause, but not a diminution
of the general grant of executive power. cEITCA
Thus, the President's authority to declare a state of rebellion springs in the main from her powers as chief executive and, at
the same time, draws strength from her Commander-in-Chief powers. . . . (Emphasis and underscoring supplied)
Similarly, the President's power to conduct peace negotiations is implicitly included in her powers as Chief Executive and Commander-in-Chief. As
Chief Executive, the President has the general responsibility to promote public peace, and as Commander-in-Chief, she has the more specific duty to
prevent and suppress rebellion and lawless violence. 169
As the experience of nations which have similarly gone through internal armed conflict will show, however, peace is rarely attained by simply
pursuing a military solution. Oftentimes, changes as far-reaching as a fundamental reconfiguration of the nation's constitutional structure is required.
The observations of Dr. Kirsti Samuels are enlightening, to wit:
. . . [T]he fact remains that a successful political and governance transition must form the core of any post-conflict peace-
building mission. As we have observed in Liberia and Haiti over the last ten years, conflict cessation without modification of the
political environment, even where state-building is undertaken through technical electoral assistance and institution- or
capacity-building, is unlikely to succeed. On average, more than 50 percent of states emerging from conflict return to conflict.
Moreover, a substantial proportion of transitions have resulted in weak or limited democracies.
The design of a constitution and its constitution-making process can play an important role in the political and governance
transition. Constitution-making after conflict is an opportunity to create a common vision of the future of a state and a road
map on how to get there. The constitution can be partly a peace agreement and partly a framework setting up the rules by
which the new democracy will operate. 170
In the same vein, Professor Christine Bell, in her article on the nature and legal status of peace agreements, observed that the typical way that peace
agreements establish or confirm mechanisms for demilitarization and demobilization is by linking them to new constitutional structures addressing
governance, elections, and legal and human rights institutions. 171
In the Philippine experience, the link between peace agreements and constitution-making has been recognized by no less than the framers of the
Constitution. Behind the provisions of the Constitution on autonomous regions 172 is the framers' intention to implement a particular peace
agreement, namely, the Tripoli Agreement of 1976 between the GRP and the MNLF, signed by then Undersecretary of National Defense Carmelo Z.
Barbero and then MNLF Chairman Nur Misuari. aTIEcA
MR. ROMULO.
There are other speakers; so, although I have some more questions, I will reserve my right to ask them if they are not covered
by the other speakers. I have only two questions.
I heard one of the Commissioners say that local autonomy already exists in the Muslim region; it is working very well; it has,
in fact, diminished a great deal of the problems. So, my question is: since that already exists, why do we have to go
into something new?
MR. OPLE.
May I answer that on behalf of Chairman Nolledo. Commissioner Yusup Abubakar is right that certain definite steps have been
taken to implement the provisions of the Tripoli Agreement with respect to an autonomous region in Mindanao.
This is a good first step, but there is no question that this is merely a partial response to the Tripoli Agreement
itself and to the fuller standard of regional autonomy contemplated in that agreement, and now by state
policy. 173 (Emphasis supplied)
The constitutional provisions on autonomy and the statutes enacted pursuant to them have, to the credit of their drafters, been partly successful.
Nonetheless, the Filipino people are still faced with the reality of an on-going conflict between the Government and the MILF. If the President is to be
expected to find means for bringing this conflict to an end and to achieve lasting peace in Mindanao, then she must be given the leeway to explore,
in the course of peace negotiations, solutions that may require changes to the Constitution for their implementation. Being uniquely vested with the
power to conduct peace negotiations with rebel groups, the President is in a singular position to know the precise nature of their grievances which, if
resolved, may bring an end to hostilities. DAaIEc

The President may not, of course, unilaterally implement the solutions that she considers viable, but she may not be prevented from submitting
them as recommendations to Congress, which could then, if it is minded, act upon them pursuant to the legal procedures for constitutional
amendment and revision. In particular, Congress would have the option, pursuant to Article XVII, Sections 1 and 3 of the Constitution, to propose the
recommended amendments or revision to the people, call a constitutional convention, or submit to the electorate the question of calling such a
convention.
While the President does not possess constituent powers — as those powers may be exercised only by Congress, a Constitutional Convention, or the
people through initiative and referendum — she may submit proposals for constitutional change to Congress in a manner that does not involve the
arrogation of constituent powers.
In Sanidad v. COMELEC, 174 in issue was the legality of then President Marcos' act of directly submitting proposals for constitutional amendments to
a referendum, bypassing the interim National Assembly which was the body vested by the 1973 Constitution with the power to propose such
amendments. President Marcos, it will be recalled, never convened the interim National Assembly. The majority upheld the President's act, holding
that "the urges of absolute necessity" compelled the President as the agent of the people to act as he did, there being no interim National Assembly
to propose constitutional amendments. Against this ruling, Justices Teehankee and Muñoz Palma vigorously dissented. The Court's concern at
present, however, is not with regard to the point on which it was then divided in that controversial case, but on that which was not disputed by
either side.
Justice Teehankee's dissent, 175 in particular, bears noting. While he disagreed that the President may directly submit proposed constitutional
amendments to a referendum, implicit in his opinion is a recognition that he would have upheld the President's action along with the majority had
the President convened the interim National Assembly and coursed his proposals through it. Thus Justice Teehankee opined:
"Since the Constitution provides for the organization of the essential departments of government, defines and delimits the
powers of each and prescribes the manner of the exercise of such powers, and the constituent power has not been granted to
but has been withheld from the President or Prime Minister, it follows that the President's questioned decrees proposing and
submitting constitutional amendments directly to the people (without the intervention of the interim National Assembly in
whom the power is expressly vested) are devoid of constitutional and legal basis." 176 (Emphasis supplied)
From the foregoing discussion, the principle may be inferred that the President — in the course of conducting peace negotiations — may validly
consider implementing even those policies that require changes to the Constitution, but she may not unilaterally implement them without the
intervention of Congress, or act in any way as if the assent of that body were assumed as a certainty. TEAaDC
Since, under the present Constitution, the people also have the power to directly propose amendments through initiative and referendum, the
President may also submit her recommendations to the people, not as a formal proposal to be voted on in a plebiscite similar to what President
Marcos did in Sanidad, but for their independent consideration of whether these recommendations merit being formally proposed through initiative.
These recommendations, however, may amount to nothing more than the President's suggestions to the people, for any further involvement in the
process of initiative by the Chief Executive may vitiate its character as a genuine "people's initiative". The only initiative recognized by the
Constitution is that which truly proceeds from the people. As the Court stated in Lambino v. COMELEC: 177
"The Lambino Group claims that their initiative is the 'people's voice'. However, the Lambino Group unabashedly states in
ULAP Resolution No. 2006-02, in the verification of their petition with the COMELEC, that 'ULAP maintains its unqualified
support to the agenda of Her Excellency President Gloria Macapagal-Arroyo for constitutional reforms'. The Lambino Group
thus admits that their 'people's' initiative is an 'unqualified support to the agenda' of the incumbent President to change the
Constitution. This forewarns the Court to be wary of incantations of 'people's voice' or 'sovereign will' in the present
initiative." SEcITC
It will be observed that the President has authority, as stated in her oath of office, 178 only to preserve and defend the Constitution. Such
presidential power does not, however, extend to allowing her to change the Constitution, but simply to recommend proposed amendments or
revision. As long as she limits herself to recommending these changes and submits to the proper procedure for constitutional amendments and
revision, her mere recommendation need not be construed as an unconstitutional act.
The foregoing discussion focused on the President's authority to propose constitutional amendments, since her authority to propose
new legislation is not in controversy. It has been an accepted practice for Presidents in this jurisdiction to propose new legislation. One of the more
prominent instances the practice is usually done is in the yearly State of the Nation Address of the President to Congress. Moreover, the annual
general appropriations bill has always been based on the budget prepared by the President, which — for all intents and purposes — is a proposal for
new legislation coming from the President. 179
The "suspensive clause" in the
MOA-AD viewed in light of the
above-discussed standards
Given the limited nature of the President's authority to propose constitutional amendments, she cannot guarantee to any third party that the
required amendments will eventually be put in place, nor even be submitted to a plebiscite. The most she could do is submit these proposals as
recommendations either to Congress or the people, in whom constituent powers are vested.
Paragraph 7 on Governance of the MOA-AD states, however, that all provisions thereof which cannot be reconciled with the present Constitution
and laws "shall come into force upon signing of a Comprehensive Compact and upon effecting the necessary changes to the legal framework". This
stipulation does not bear the marks of a suspensive condition — defined in civil law as a future and uncertain event — but of a term. It is not a
question of whether the necessary changes to the legal framework will be effected, but when. That there is no uncertainty being contemplated is
plain from what follows, for the paragraph goes on to state that the contemplated changes shall be "with due regard to non derogation of prior
agreements and within the stipulated timeframe to be contained in the Comprehensive Compact".
Pursuant to this stipulation, therefore, it is mandatory for the GRP to effect the changes to the legal framework contemplated in the MOA-AD —
which changes would include constitutional amendments, as discussed earlier. It bears noting that, TSHcIa
By the time these changes are put in
place, the MOA-AD itself would be
counted among the "prior
agreements" from which there
could be no derogation.
What remains for discussion in the Comprehensive Compact would merely be the implementing details for these "consensus points" and, notably,
the deadline for effecting the contemplated changes to the legal framework.
Plainly, stipulation-paragraph 7 on GOVERNANCE is inconsistent with the limits of the President's authority to propose constitutional amendments,
it being a virtual guarantee that the Constitution and the laws of the Republic of the Philippines will certainly be adjusted to conform to all the
"consensus points" found in the MOA-AD. Hence, it must be struck down as unconstitutional.
A comparison between the "suspensive clause" of the MOA-AD with a similar provision appearing in the 1996 final peace agreement between the
MNLF and the GRP is most instructive.
As a backdrop, the parties to the 1996 Agreement stipulated that it would be implemented in two phases. Phase I covered a three-year transitional
period involving the putting up of new administrative structures through Executive Order, such as the Special Zone of Peace and Development
(SZOPAD) and the Southern Philippines Council for Peace and Development (SPCPD), while Phase II covered the establishment of the new regional
autonomous government through amendment or repeal of R.A. No. 6734, which was then the Organic Act of the ARMM.
The stipulations on Phase II consisted of specific agreements on the structure of the expanded autonomous region envisioned by the parties. To that
extent, they are similar to the provisions of the MOA-AD. There is, however, a crucial difference between the two agreements. While the MOA-
AD virtually guarantees that the "necessary changes to the legal framework" will be put in place, the GRP-MNLF final peace agreement states thus:
"Accordingly, these provisions [on Phase II] shall be recommended by the GRP to Congress for incorporation in the amendatory or repealing
law". cHECAS
Concerns have been raised that the MOA-AD would have given rise to a binding international law obligation on the part of the Philippines to change
its Constitution in conformity thereto, on the ground that it may be considered either as a binding agreement under international law, or a unilateral
declaration of the Philippine government to the international community that it would grant to the Bangsamoro people all the concessions therein
stated. Neither ground finds sufficient support in international law, however.
The MOA-AD, as earlier mentioned in the overview thereof, would have included foreign dignitaries as signatories. In addition, representatives of
other nations were invited to witness its signing in Kuala Lumpur. These circumstances readily lead one to surmise that the MOA-AD would have had
the status of a binding international agreement had it been signed. An examination of the prevailing principles in international law, however, leads to
the contrary conclusion.

The Decision on CHALLENGE TO JURISDICTION: LOMÉ ACCORD AMNESTY 180 (the Lomé Accord case) of the Special Court of Sierra Leone is
enlightening. The Lomé Accord was a peace agreement signed on July 7, 1999 between the Government of Sierra Leone and the Revolutionary
United Front (RUF), a rebel group with which the Sierra Leone Government had been in armed conflict for around eight years at the time of signing.
There were non-contracting signatories to the agreement, among which were the Government of the Togolese Republic, the Economic Community
of West African States, and the UN.
On January 16, 2002, after a successful negotiation between the UN Secretary-General and the Sierra Leone Government, another agreement was
entered into by the UN and that Government whereby the Special Court of Sierra Leone was established. The sole purpose of the Special Court, an
international court, was to try persons who bore the greatest responsibility for serious violations of international humanitarian law and Sierra
Leonean law committed in the territory of Sierra Leone since November 30, 1996. AETcSa
Among the stipulations of the Lomé Accord was a provision for the full pardon of the members of the RUF with respect to anything done by them in
pursuit of their objectives as members of that organization since the conflict began.
In the Lomé Accord case, the Defence argued that the Accord created an internationally binding obligation not to prosecute the beneficiaries of the
amnesty provided therein,citing, among other things, the participation of foreign dignitaries and international organizations in the finalization of that
agreement. The Special Court, however, rejected this argument, ruling that the Lome Accord is not a treaty and that it can only create binding
obligations and rights between the parties in municipal law, not in international law. Hence, the Special Court held, it is ineffective in depriving an
international court like it of jurisdiction.
"37.In regard to the nature of a negotiated settlement of an internal armed conflict it is easy to assume and to argue with
some degree of plausibility, as Defence counsel for the defendants seem to have done, that the mere fact that in
addition to the parties to the conflict, the document formalizing the settlement is signed by foreign heads of state
or their representatives and representatives of international organizations, means the agreement of the parties is
internationalized so as to create obligations in international law. cHITCS
xxx xxx xxx
40.Almost every conflict resolution will involve the parties to the conflict and the mediator or facilitator of the settlement, or
persons or bodies under whose auspices the settlement took place but who are not at all parties to the conflict, are
not contracting parties and who do not claim any obligation from the contracting parties or incur any obligation from
the settlement.
41.In this case, the parties to the conflict are the lawful authority of the State and the RUF which has no status of statehood
and is to all intents and purposes a faction within the state. The non-contracting signatories of the Lomé
Agreement were moral guarantors of the principle that, in the terms of Article XXXIV of the Agreement, "this
peace agreement is implemented with integrity and in good faith by both parties". The moral guarantors assumed
no legal obligation. It is recalled that the UN by its representative appended, presumably for avoidance of doubt, an
understanding of the extent of the agreement to be implemented as not including certain international crimes.
42.An international agreement in the nature of a treaty must create rights and obligations regulated by international law so
that a breach of its terms will be a breach determined under international law which will also provide principle
means of enforcement. The Lomé Agreement created neither rights nor obligations capable of being regulated by
international law. An agreement such as the Lomé Agreement which brings to an end an internal armed conflict no
doubt creates a factual situation of restoration of peace that the international community acting through the
Security Council may take note of. That, however, will not convert it to an international agreement which creates
an obligation enforceable in international, as distinguished from municipal, law.A breach of the terms of such a
peace agreement resulting in resumption of internal armed conflict or creating a threat to peace in the
determination of the Security Council may indicate a reversal of the factual situation of peace to be visited with
possible legal consequences arising from the new situation of conflict created. Such consequences such as action by
the Security Council pursuant to Chapter VII arise from the situation and not from the agreement, nor from the
obligation imposed by it. Such action cannot be regarded as a remedy for the breach. A peace agreement which
settles an internal armed conflict cannot be ascribed the same status as one which settles an international armed
conflict which, essentially, must be between two or more warring States. The Lomé Agreement cannot be
characterised as an international instrument. . . ." (Emphasis, italics and underscoring supplied) AHSaTI
Similarly, that the MOA-AD would have been signed by representatives of States and international organizations not parties to the Agreement would
not have sufficed to vest in it a binding character under international law.
In another vein, concern has been raised that the MOA-AD would amount to a unilateral declaration of the Philippine State, binding under
international law, that it would comply with all the stipulations stated therein, with the result that it would have to amend its Constitution
accordingly regardless of the true will of the people. Cited as authority for this view is Australia v. France, 181 also known as the Nuclear Tests Case,
decided by the International Court of Justice (ICJ). EcIaTA
In the Nuclear Tests Case, Australia challenged before the ICJ the legality of France's nuclear tests in the South Pacific. France refused to appear in
the case, but public statements from its President, and similar statements from other French officials including its Minister of Defence, that its 1974
series of atmospheric tests would be its last, persuaded the ICJ to dismiss the case. 182 Those statements, the ICJ held, amounted to a legal
undertaking addressed to the international community, which required no acceptance from other States for it to become effective.
Essential to the ICJ ruling is its finding that the French government intended to be bound to the international community in issuing its public
statements, viz.:
43.It is well recognized that declarations made by way of unilateral acts, concerning legal or factual situations, may have the
effect of creating legal obligations. Declarations of this kind may be, and often are, very specific. When it is the
intention of the State making the declaration that it should become bound according to its terms, that intention
confers on the declaration the character of a legal undertaking, the State being thenceforth legally required to
follow a course of conduct consistent with the declaration. An undertaking of this kind, if given publicly, and with an
intent to be bound, even though not made within the context of international negotiations, is binding. In these
circumstances, nothing in the nature of a quid pro quo nor any subsequent acceptance of the declaration, nor even
any reply or reaction from other States, is required for the declaration to take effect, since such a requirement would
be inconsistent with the strictly unilateral nature of the juridical act by which the pronouncement by the State was
made.
44.Of course, not all unilateral acts imply obligation; but a State may choose to take up a certain position in relation to a
particular matter with the intention of being bound — the intention is to be ascertained by interpretation of the
act. When States make statements by which their freedom of action is to be limited, a restrictive interpretation is
called for. cCSTHA
xxx xxx xxx
51.In announcing that the 1974 series of atmospheric tests would be the last, the French Government conveyed to the world
at large, including the Applicant, its intention effectively to terminate these tests. It was bound to assume that
other States might take note of these statements and rely on their being effective. The validity of these statements
and their legal consequences must be considered within the general framework of the security of international
intercourse,and the confidence and trust which are so essential in the relations among States. It is from the actual
substance of these statements, and from the circumstances attending their making, that the legal implications of
the unilateral act must be deduced. The objects of these statements are clear and they were addressed to the
international community as a whole, and the Court holds that they constitute an undertaking possessing legal
effect. The Court considers *270 that the President of the Republic, in deciding upon the effective cessation of
atmospheric tests, gave an undertaking to the international community to which his words were addressed. . . .
(Emphasis and underscoring supplied)
As gathered from the above-quoted ruling of the ICJ, public statements of a state representative may be construed as a unilateral declaration only
when the following conditions are present: the statements were clearly addressed to the international community, the state intended to be bound to
that community by its statements, and that not to give legal effect to those statements would be detrimental to the security of international
intercourse. Plainly, unilateral declarations arise only in peculiar circumstances.
The limited applicability of the Nuclear Tests Case ruling was recognized in a later case decided by the ICJ entitled Burkina Faso v. Mali, 183 also
known as the Case Concerning the Frontier Dispute. The public declaration subject of that case was a statement made by the President of Mali, in an
interview by a foreign press agency, that Mali would abide by the decision to be issued by a commission of the Organization of African Unity on a
frontier dispute then pending between Mali and Burkina Faso. CaASIc

Unlike in the Nuclear Tests Case, the ICJ held that the statement of Mali's President was not a unilateral act with legal implications. It clarified that its
ruling in the Nuclear Tests case rested on the peculiar circumstances surrounding the French declaration subject thereof, to wit:
40.In order to assess the intentions of the author of a unilateral act, account must be taken of all the factual circumstances in
which the act occurred. For example, in the Nuclear Tests cases, the Court took the view that since the applicant
States were not the only ones concerned at the possible continuance of atmospheric testing by the French
Government, that Government's unilateral declarations had 'conveyed to the world at large, including the
Applicant, its intention effectively to terminate these tests' (I.C.J. Reports 1974, p. 269, para. 51; p. 474, para.
53). In the particular circumstances of those cases, the French Government could not express an intention to be
bound otherwise than by unilateral declarations. It is difficult to see how it could have accepted the terms of a
negotiated solution with each of the applicants without thereby jeopardizing its contention that its conduct was
lawful. The circumstances of the present case are radically different. Here, there was nothing to hinder the Parties
from manifesting an intention to accept the binding character of the conclusions of the Organization of African
Unity Mediation Commission by the normal method: a formal agreement on the basis of reciprocity. Since no
agreement of this kind was concluded between the Parties, the Chamber finds that there are no grounds to interpret
the declaration made by Mali's head of State on 11 April 1975 as a unilateral act with legal implications in regard to
the present case. (Emphasis and underscoring supplied)
Assessing the MOA-AD in light of the above criteria, it would not have amounted to a unilateral declaration on the part of the Philippine State to the
international community. The Philippine panel did not draft the same with the clear intention of being bound thereby to the international community
as a whole or to any State, but only to the MILF. While there were States and international organizations involved, one way or another, in the
negotiation and projected signing of the MOA-AD, they participated merely as witnesses or, in the case of Malaysia, as facilitator. As held in the Lomé
Accord case, the mere fact that in addition to the parties to the conflict, the peace settlement is signed by representatives of states and international
organizations does not mean that the agreement is internationalized so as to create obligations in international law. HaAIES
Since the commitments in the MOA-AD were not addressed to States, not to give legal effect to such commitments would not be detrimental to the
security of international intercourse — to the trust and confidence essential in the relations among States.
In one important respect, the circumstances surrounding the MOA-AD are closer to that of Burkina Faso wherein, as already discussed, the Mali
President's statement was not held to be a binding unilateral declaration by the ICJ. As in that case, there was also nothing to hinder the Philippine
panel, had it really been its intention to be bound to other States, to manifest that intention by formal agreement. Here, that formal agreement
would have come about by the inclusion in the MOA-AD of a clear commitment to be legally bound to the international community, not just the
MILF, and by an equally clear indication that the signatures of the participating states-representatives would constitute an acceptance of that
commitment. Entering into such a formal agreement would not have resulted in a loss of face for the Philippine government before the international
community, which was one of the difficulties that prevented the French Government from entering into a formal agreement with other countries.
That the Philippine panel did not enter into such a formal agreement suggests that it had no intention to be bound to the international community.
On that ground, the MOA-AD may not be considered a unilateral declaration under international law.
The MOA-AD not being a document that can bind the Philippines under international law notwithstanding, respondents' almost consummated act
of guaranteeing amendments to the legal framework is, by itself, sufficient to constitute grave abuse of discretion. The grave abuse lies not in the
fact that they considered, as a solution to the Moro Problem, the creation of a state within a state, but in their brazen willingness to guarantee that
Congress and the sovereign Filipino people would give their imprimatur to their solution. Upholding such an act would amount to authorizing a
usurpation of the constituent powers vested only in Congress, a Constitutional Convention, or the people themselves through the process of
initiative, for the only way that the Executive can ensure the outcome of the amendment process is through an undue influence or interference with
that process. aTIAES
The sovereign people may, if it so desired, go to the extent of giving up a portion of its own territory to the Moros for the sake of peace, for it can
change the Constitution in any it wants, so long as the change is not inconsistent with what, in international law, is known as Jus
Cogens. 184 Respondents, however, may not preempt it in that decision.
SUMMARY
The petitions are ripe for adjudication. The failure of respondents to consult the local government units or communities affected constitutes a
departure by respondents from their mandate under E.O. No. 3. Moreover, respondents exceeded their authority by the mere act of guaranteeing
amendments to the Constitution. Any alleged violation of the Constitution by any branch of government is a proper matter for judicial review.
As the petitions involve constitutional issues which are of paramount public interest or of transcendental importance, the Court grants the
petitioners, petitioners-in-intervention and intervening respondents the requisite locus standi in keeping with the liberal stance adopted in David v.
Macapagal-Arroyo.
Contrary to the assertion of respondents that the non-signing of the MOA-AD and the eventual dissolution of the GRP Peace Panel mooted the
present petitions, the Court finds that the present petitions provide an exception to the "moot and academic" principle in view of (a) the grave
violation of the Constitution involved; (b) the exceptional character of the situation and paramount public interest; (c) the need to formulate
controlling principles to guide the bench, the bar, and the public; and (d) the fact that the case is capable of repetition yet evading review. EDcICT
The MOA-AD is a significant part of a series of agreements necessary to carry out the GRP-MILF Tripoli Agreement on Peace signed by the
government and the MILF back in June 2001. Hence, the present MOA-AD can be renegotiated or another one drawn up that could contain similar or
significantly dissimilar provisions compared to the original.
The Court, however, finds that the prayers for mandamus have been rendered moot in view of the respondents' action in providing the Court and
the petitioners with the official copy of the final draft of the MOA-AD and its annexes.
The people's right to information on matters of public concern under Sec. 7, Article III of the Constitution is in splendid symmetry with the state
policy of full public disclosure of all its transactions involving public interest under Sec. 28, Article II of the Constitution. The right to information
guarantees the right of the people to demand information, while Section 28 recognizes the duty of officialdom to give information even if nobody
demands. The complete and effective exercise of the right to information necessitates that its complementary provision on public disclosure derive
the same self-executory nature, subject only to reasonable safeguards or limitations as may be provided by law. HcaATE
The contents of the MOA-AD is a matter of paramount public concern involving public interest in the highest order. In declaring that the right to
information contemplates steps and negotiations leading to the consummation of the contract, jurisprudence finds no distinction as to the executory
nature or commercial character of the agreement.
An essential element of these twin freedoms is to keep a continuing dialogue or process of communication between the government and the people.
Corollary to these twin rights is the design for feedback mechanisms. The right to public consultation was envisioned to be a species of these public
rights.
At least three pertinent laws animate these constitutional imperatives and justify the exercise of the people's right to be consulted on relevant
matters relating to the peace agenda.
One, E.O. No. 3 itself is replete with mechanics for continuing consultations on both national and local levels and for a principal forum for consensus-
building. In fact, it is the duty of the Presidential Adviser on the Peace Process to conduct regular dialogues to seek relevant information, comments,
advice, and recommendations from peace partners and concerned sectors of society. AIHECa
Two, Republic Act No. 7160 or the Local Government Code of 1991 requires all national offices to conduct consultations before any project or
program critical to the environment and human ecology including those that may call for the eviction of a particular group of people residing in such
locality, is implemented therein. The MOA-AD is one peculiar program that unequivocally and unilaterally vests ownership of a vast territory to the
Bangsamoro people, which could pervasively and drastically result to the diaspora or displacement of a great number of inhabitants from their total
environment.
Three, Republic Act No. 8371 or the Indigenous Peoples Rights Act of 1997 provides for clear-cut procedure for the recognition and delineation of
ancestral domain, which entails, among other things, the observance of the free and prior informed consent of the Indigenous Cultural
Communities/Indigenous Peoples. Notably, the statute does not grant the Executive Department or any government agency the power to delineate
and recognize an ancestral domain claim by mere agreement or compromise.

The invocation of the doctrine of executive privilege as a defense to the general right to information or the specific right to consultation is untenable.
The various explicit legal provisions fly in the face of executive secrecy. In any event, respondents effectively waived such defense after it
unconditionally disclosed the official copies of the final draft of the MOA-AD, for judicial compliance and public scrutiny. ISCDEA
IN SUM, the Presidential Adviser on the Peace Process committed grave abuse of discretion when he failed to carry out the pertinent consultation
process, as mandated by E.O. No. 3, Republic Act No. 7160, and Republic Act No. 8371. The furtive process by which the MOA-AD was designed and
crafted runs contrary to and in excess of the legal authority, and amounts to a whimsical, capricious, oppressive, arbitrary and despotic exercise
thereof. It illustrates a gross evasion of positive duty and a virtual refusal to perform the duty enjoined.
The MOA-AD cannot be reconciled with the present Constitution and laws. Not only its specific provisions but the very concept underlying them,
namely, the associative relationship envisioned between the GRP and the BJE, are unconstitutional, for the concept presupposes that the associated
entity is a state and implies that the same is on its way to independence.
While there is a clause in the MOA-AD stating that the provisions thereof inconsistent with the present legal framework will not be effective until
that framework is amended, the same does not cure its defect. The inclusion of provisions in the MOA-AD establishing an associative relationship
between the BJE and the Central Government is, itself, a violation of the Memorandum of Instructions From The President dated March 1, 2001,
addressed to the government peace panel. Moreover, as the clause is worded, it virtually guarantees that the necessary amendments to the
Constitution and the laws will eventually be put in place. Neither the GRP Peace Panel nor the President herself is authorized to make such a
guarantee. Upholding such an act would amount to authorizing a usurpation of the constituent powers vested only in Congress, a Constitutional
Convention, or the people themselves through the process of initiative, for the only way that the Executive can ensure the outcome of the
amendment process is through an undue influence or interference with that process. ScaEIT
While the MOA-AD would not amount to an international agreement or unilateral declaration binding on the Philippines under international law,
respondents' act of guaranteeing amendments is, by itself, already a constitutional violation that renders the MOA-AD fatally defective.
WHEREFORE, respondents' motion to dismiss is DENIED. The main and intervening petitions are GIVEN DUE COURSE and hereby GRANTED.
The Memorandum of Agreement on the Ancestral Domain Aspect of the GRP-MILF Tripoli Agreement on Peace of 2001 is declared CONTRARY TO
LAW AND THE CONSTITUTION.DTcASE
SO ORDERED.
Quisumbing, J., concurs.
Puno, C.J., please see separate concurring opinion.
Ynares-Santiago, J., see separate concurring opinion; I concur with separate opinion of C.J. Puno.
Carpio, J., see concurring opinion.
Austria-Martinez, J., also concurs with C.J.'s separate opinion.
Corona, J., shares the dissent of Mr. Justice Tinga.
Azcuna, J., concurs in a separate opinion.
Tinga, J., dissents from the result. See separate opinion.
Chico-Nazario, Velasco, Jr., Nachura and Brion, JJ., please see dissenting opinion.
Reyes, J., certifies that J. Reyes filed a Separate Opinion concurring with the majority. — C.J., Puno (RSP).
Leonardo-de Castro, J., please see concurring and dissenting opinion.
||| (Province of North Cotabato v. Government of the Republic of the Philippines Peace Panel on Ancestral Domain, G.R. No. 183591, 183752, 183893,
183951, 183962, [October 14, 2008], 589 PHIL 387-732)
FIRST DIVISION
[G.R. No. 128195. October 3, 2001.]
ELIZABETH LEE and PACITA YU LEE, HON. JUDGE JOSE D. ALOVERA, * Presiding Judge, Regional Trial Court, Branch 17, Roxas
City, THE REGISTER OF DEEDS OF ROXAS CITY, petitioners, vs. REPUBLIC OF THE PHILIPPINES, represented by THE DIRECTOR
OF LANDS AND THE ADMINISTRATOR, LAND REGISTRATION AUTHORITY and THE HON. COURT OF APPEALS, respondents.
Patrocinio S. Palanog for petitioners.
The Solicitor General for respondents.
SYNOPSIS
Petitioners, heirs of Lee Liong, sought reconstitution of title of subject parcel of land in the name of Lee Liong whose transfer
certificate of title was lost or destroyed during the war. At the trial, they presented documents proving the sale of the land from the Dinglasans
to Lee Liong and the latter's subsequent possession of the property in the concept of owner. The trial court ordered the reconstitution of title in
the name of Lee Liong. The Dinglasans, however, claimed that Lee Liong, a Chinese citizen, was disqualified to acquire the land in question. The
Court of Appeals nullified the trial court's order.
On appeal, the Supreme Court held that ownership of the land cannot revert to the Dinglasans (original sellers), because of the
doctrine of pari delicto. The Solicitor General may file an action for reversion or escheat of the land to the State, but in this case, petitioners'
acquisition of Philippine citizenship cured the flaw in the transaction and transfer of subject property to them can no longer be impugned. Proof
of the transfer of ownership of subject property, however, must be threshed out in a separate proceeding. Reconstitution of title cannot also be
ordered because it must be based on an owner's duplicate, secondary evidence thereof, or other valid sources of the title to be reconstituted,
none of which was presented in this case. cCaDSA
SYLLABUS
1. CONSTITUTIONAL LAW; PROSCRIPTION ON SALE OF REAL ESTATE TO ALIENS; DOCTRINE OF PARI DELICTO APPLICABLE TO VENDOR AND VENDEE
THEREIN; CASE AT BAR. — "In sales of real estate to aliens incapable of holding title thereto by virtue of the provisions of the Constitution both the
vendor and the vendee are deemed to have committed the constitutional violation and being thus in pari delicto the courts will not afford protection
to either party." The proper party to assail the sale is the Solicitor General. This was what was done in this case when the Solicitor General initiated
an action for annulment of judgment of reconstitution of title. While it took the Republic more than sixty years to assert itself, it is not barred from
initiating such action. Prescription never lies against the State. Although ownership of the land cannot revert to the original sellers, because of the
doctrine of pari delicto, the Solicitor General may initiate an action for reversion or escheat of the land to the State, subject to other defenses, as
hereafter set forth. In this case, subsequent circumstances militate against escheat proceedings because the land is now in the hands of Filipinos. The
original vendee, Lee Liong, has since died and the land has been inherited by his heirs and subsequently their heirs, petitioners herein. Petitioners are
Filipino citizens, a fact the Solicitor General does not dispute.
2. ID.; ID.; ID.; EFFECT OF SUBSEQUENT ACQUISITION OF PHILIPPINE CITIZENSHIP BY TRANSFEREE; CASE AT BAR. — The constitutional proscription on
alien ownership of lands of the public or private domain was intended to protect lands from falling in the hands of non-Filipinos. In this case,
however, there would be no more public policy violated since the land is in the hands of Filipinos qualified to acquire and own such land. "If land is
invalidly transferred to an alien who subsequently becomes a citizen or transfers it to a citizen, the flaw in the original transaction is considered cured
and the title of the transferee is rendered valid." Thus, the subsequent transfer of the property to qualified Filipinos may no longer be impugned on
the basis of the invalidity of the initial transfer. The objective of the constitutional provision to keep our lands in Filipino hands has been
achieved. ASCTac
3. CIVIL LAW; LAND REGISTRATION; RECONSTITUTION OF TITLE; VALID SOURCES OF THE TITLE TO BE RECONSTITUTED; CASE AT BAR. — Incidentally,
it must be mentioned that reconstitution of the original certificate of title must be based on an owner's duplicate, secondary evidence thereof, or
other valid sources of the title to be reconstituted. In this case, reconstitution was based on the plan and technical description approved by the Land
Registration Authority. This renders the order of reconstitution void for lack of factual support. A judgment with absolutely nothing to support it is
void.
DECISION
PARDO, J p:
The case under consideration is a petition for review on certiorari of the decision 1 of the Court of Appeals nullifying that of the Regional Trial Court,
Roxas City, in Reconstitution Case No. R-1928, 2 pertaining to Lot 398, Capiz Cadastre, covered by Original Certificate of Title No. 3389.
Sometime in March 1936, Rafael, Carmen, Francisco, Jr., Ramon, Lourdes, Mercedes, Concepcion, Mariano, Jose, Loreto, Manuel, Rizal and Jimmy, all
surnamed Dinglasan sold to Lee Liong, a Chinese citizen, a parcel of land with an approximate area of 1,631 square meters, designated as Lot 398 and
covered by Original Certificate of Title No. 3389, situated at the corner of Roxas Avenue and Pavia Street, Roxas City. 3
However, in 1948, the former owners filed with the Court of First Instance, Capiz an action against the heirs of Lee Liong for annulment of sale and
recovery of land. 4 The plaintiffs assailed the validity of the sale because of the constitutional prohibition against aliens acquiring ownership of
private agricultural land, including residential, commercial or industrial land. Rebuffed in the trial court and the Court of Appeals, plaintiffs appealed
to the Supreme Court. On June 27, 1956, the Supreme Court ruled thus:
". . . granting the sale to be null and void and can not give title to the vendee, it does not necessarily follow therefrom that the
title remained in the vendor, who had also violated the constitutional prohibition, or that he (vendor) has the right to recover
the title of which he has divested himself by his act in ignoring the prohibition. In such contingency another principle of law
sets in to bar the equally guilty vendor from recovering the title which he had voluntarily conveyed for a consideration, that
of pari delicto." 5
On July 1, 1968, the same former owners Rafael A. Dinglasan, together with Francisco, Carmen, Ramon, Lourdes, Mercedes, Concepcion, Mariano,
Jose, Loreto, Rizal, Jimmy, and Jesse Dinglasan filed with the Court of First Instance, Capiz an action for recovery of the same parcel of land. 6 Citing
the case of Philippine Banking Corporation v. Lui She, 7 they submitted that the sale to Lee Liong was null and void for being violative of
the Constitution. On September 23, 1968, the heirs of Lee Liong filed with the trial court a motion to dismiss the case on the ground of res
judicata. 8 On October 10, 1968, and November 9, 1968, the trial court denied the motion. 9 The heirs of Lee Liong elevated the case to the Supreme
Court by petition for certiorari. On April 22, 1977, the Supreme Court annulled the orders of the trial court and directed it to dismiss the case, holding
that the suit was barred by res judicata. 10 CTIEac
On September 7, 1993, Elizabeth Manuel-Lee and Pacita Yu Lee filed with the Regional Trial Court, Roxas City a petition for reconstitution of title of
Lot No. 398 of the Capiz Cadastre, formerly covered by Original Certificate of Title No. 3389 of the Register of Deeds of Roxas City. 11 Petitioners
alleged that they were the widows of the deceased Lee Bing Hoo and Lee Bun Ting, who were the heirs of Lee Liong, the owner of the lot. Lee Liong
died intestate in February 1944. On June 30, 1947, Lee Liong's widow, Ang Chia, and his two sons, Lee Bun Ting and Lee Bing Ho, executed an extra-
judicial settlement of the estate of Lee Liong, adjudicating to themselves the subject parcel of land. 12 Petitioner Elizabeth Lee acquired her share in
Lot No. 398 through an extra-judicial settlement and donation executed in her favor by her deceased husband Lee Bing Hoo. Petitioner Pacita Yu Lee
acquired her share in the same lot by succession from her deceased husband Lee Bun Ting, as evidenced by a deed of extra-judicial settlement. 13
Previously, on December 9, 1948, the Register of Deeds Capiz, Salvador Villaluz, issued a certification that a transfer certificate of title over the
property was issued in the name of Lee Liong. 14 However, the records of the Register of Deeds, Roxas City were burned during the war. Thus, as
heretofore stated, on September 7, 1968, petitioners filed a petition for reconstitution of title.
On June 10, 1994, the Regional Trial Court, Roxas City, Branch 17, ordered the reconstitution of the lost or destroyed certificate of title in the name of
Lee Liong on the basis of an approved plan and technical description. 15 The dispositive portion of the trial court's decision reads thus:
"WHEREFORE, in reiteration, the Register of Deeds for the City of Roxas is ordered to reconstitute the lost or destroyed
certificate of title in the name of Lee Liong, deceased, of Roxas City, with all the conditions stated in paragraph 2 of this
decision. This decision shall become final after the lapse of thirty (30) days from receipt by the Register of Deeds and by the
Commissioner of LRA of a notice of such judgment without any appeal having been filed by any of such officials.
"SO ORDERED.
"Given at Roxas City, Philippines,
"June 10, 1994.
"JOSE O. ALOVERA
"Judge" 16
On August 18, 1994, the Clerk of Court, Regional Trial Court, Roxas City, Branch 17 issued an Entry of Judgment. 17

On January 25, 1995, the Solicitor General filed with the Court of Appeals a petition for annulment of judgment in Reconstitution Case No. 1928,
alleging that the Regional Trial Court, Roxas City had no jurisdiction over the case. 18 The Solicitor General contended that the petitioners were not
the proper parties in the reconstitution of title, since their predecessor-in-interest Lee Liong did not acquire title to the lot because he was a Chinese
citizen and was constitutionally not qualified to own the subject land.
On April 30, 1996, the Court of Appeals promulgated its decision declaring the judgment of reconstitution void. 19
On May 24, 1996, Elizabeth Manuel-Lee and Pacita Yu Lee filed with the Court of Appeals a motion for reconsideration of the decision. 20 On
February 18, 1997, the Court of Appeals denied the motion. 21
Hence, this petition. 22
Petitioners submitted that the Solicitor General was estopped from seeking annulment of the judgment of reconstitution after failing to object during
the reconstitution proceedings before the trial court, despite due notice. Petitioners alleged that the Solicitor General merely acted on the request of
private and politically powerful individuals who wished to capitalize on the prime location of the subject land.
Petitioners emphasized that the ownership of the land had been settled in two previous cases of the Supreme Court, where the Court ruled in favor
of their predecessor-in-interest, Lee Liong. Petitioners also pointed out that they acquired ownership of the land through actual possession of the lot
and their consistent payment of taxes over the land for more than sixty years.
On the other hand, the Solicitor General submitted that the decision in the reconstitution case was void; otherwise, it would amount to
circumventing the constitutional proscription against aliens acquiring ownership of private or public agricultural lands.
We grant the petition.
The reconstitution of a certificate of title denotes restoration in the original form and condition of a lost or destroyed instrument attesting the title of
a person to a piece of land.23 The purpose of the reconstitution of title is to have, after observing the procedures prescribed by law, the title
reproduced in exactly the same way it has been when the loss or destruction occurred. 24
In this case, petitioners sought a reconstitution of title in the name of Lee Liong, alleging that the transfer certificate of title issued to him was lost or
destroyed during World War II. All the documents recorded and issued by the Register of Deeds, Capiz, which include the transfer certificate of title
issued in the name of Lee Liong, were all destroyed during the war. The fact that the original of the transfer certificate of title was not in the files of
the Office of the Register of Deeds did not imply that a transfer certificate of title had not been issued. 25 In the trial court proceedings, petitioners
presented evidence proving the sale of the land from the Dinglasans to Lee Liong and the latter's subsequent possession of the property in the
concept of owner. Thus, the trial court, after examining all the evidence before it, ordered the reconstitution of title in the name of Lee Liong.
However, there is a question as to whether Lee Liong has the qualification to own land in the Philippines.
The sale of the land in question was consummated sometime in March 1936, during the effectivity of the 1935 Constitution. Under the 1935
Constitution, 26 aliens could not acquire private agricultural lands, save in cases of hereditary succession. 27 Thus, Lee Liong, a Chinese citizen, was
disqualified to acquire the land in question. 28
The fact that the Court did not annul the sale of the land to an alien did not validate the transaction, for it was still contrary to the constitutional
proscription against aliens acquiring lands of the public or private domain. However, the proper party to assail the illegality of the transaction was
not the parties to the transaction. 29 "In sales of real estate to aliens incapable of holding title thereto by virtue of the provisions of
the Constitution both the vendor and the vendee are deemed to have committed the constitutional violation and being thus in pari delicto the courts
will not afford protection to either party." 30 The proper party to assail the sale is the Solicitor General. This was what was done in this case when
the Solicitor General initiated an action for annulment of judgment of reconstitution of title. While it took the Republic more than sixty years to
assert itself, it is not barred from initiating such action. Prescription never lies against the State. 31
Although ownership of the land cannot revert to the original sellers, because of the doctrine of pari delicto, the Solicitor General may initiate an
action for reversion or escheat of the land to the State, subject to other defenses, as hereafter set forth. 32
In this case, subsequent circumstances militate against escheat proceedings because the land is now in the hands of Filipinos. The original vendee,
Lee Liong, has since died and the land has been inherited by his heirs and subsequently their heirs, petitioners herein. Petitioners are Filipino citizens,
a fact the Solicitor General does not dispute. DSHTaC
The constitutional proscription on alien ownership of lands of the public or private domain was intended to protect lands from falling in the hands of
non-Filipinos. In this case, however, there would be no more public policy violated since the land is in the hands of Filipinos qualified to acquire and
own such land. "If land is invalidly transferred to an alien who subsequently becomes a citizen or transfers it to a citizen, the flaw in the original
transaction is considered cured and the title of the transferee is rendered valid." 33Thus, the subsequent transfer of the property to qualified
Filipinos may no longer be impugned on the basis of the invalidity of the initial transfer. 34 The objective of the constitutional provision to keep our
lands in Filipino hands has been achieved.
Incidentally, it must be mentioned that reconstitution of the original certificate of title must be based on an owner's duplicate, secondary evidence
thereof, or other valid sources of the title to be reconstituted. 35 In this case, reconstitution was based on the plan and technical description
approved by the Land Registration Authority. 36 This renders the order of reconstitution void for lack of factual support. 37 A judgment with
absolutely nothing to support it is void. 38
As earlier mentioned, a reconstitution of title is the re-issuance of a new certificate of title lost or destroyed in its original form and condition. 39 It
does not pass upon the ownership of the land covered by the lost or destroyed title. 40 Any change in the ownership of the property must be the
subject of a separate suit. 41 Thus, although petitioners are in possession of the land, a separate proceeding is necessary to thresh out the issue of
ownership of the land.
WHEREFORE, the Court REVERSES and SETS ASIDE the decision of the Court of Appeals in CA-G.R. SP No. 36274. In lieu thereof, the Court sets aside
the order of reconstitution of title in Reconstitution Case No. R-1928, Regional Trial Court, Roxas City, and dismisses the petition, without prejudice.
No costs.
SO ORDERED.
Davide, Jr., C.J., Puno and Ynares-Santiago, JJ., concur.
Kapunan, J., is on official leave.
||| (Lee v. Republic, G.R. No. 128195, [October 3, 2001], 418 PHIL 793-803)
EN BANC
[G.R. No. 161434. March 3, 2004.]
MARIA JEANETTE C. TECSON and FELIX B. DESIDERIO, JR., petitioners, vs. The COMMISSION ON ELECTIONS, RONALD ALLAN
KELLY POE (a.k.a. FERNANDO POE, JR.) and VICTORINO X. FORNIER, respondents.
[G.R. No. 161634. March 3, 2004.]
ZOILO ANTONIO VELEZ, petitioner, vs. RONALD ALLAN KELLEY POE, a.k.a. FERNANDO POE, JR., respondent.
[G.R. No. 161824. March 3, 2004.]
VICTORINO X. FORNIER, petitioner, vs. HON. COMMISSION ON ELECTIONS and RONALD ALLAN KELLEY POE, ALSO KNOWN AS
FERNANDO POE JR.,respondents.
DECISION
VITUG, J p:
Citizenship is a treasured right conferred on those whom the state believes are deserving of the privilege. It is a "precious heritage, as well as an
inestimable acquisition," 1 that cannot be taken lightly by anyone — either by those who enjoy it or by those who dispute it.
Before the Court are three consolidated cases, all of which raise a single question of profound importance to the nation. The issue of citizenship is
brought up to challenge the qualifications of a presidential candidate to hold the highest office of the land. Our people are waiting for the judgment
of the Court with bated breath. Is Fernando Poe, Jr., the hero of silver screen, and now one of the main contenders for the presidency, a natural-born
Filipino or is he not?
The moment of introspection takes us face to face with Spanish and American colonial roots and reminds us of the rich heritage of civil law and
common law traditions, the fusion resulting in a hybrid of laws and jurisprudence that could be no less than distinctly Filipino.
Antecedent Case Settings
On 31 December 2003, respondent Ronald Allan Kelly Poe, also known as Fernando Poe, Jr. (hereinafter "FPJ"), filed his certificate of candidacy for
the position of President of the Republic of the Philippines under the Koalisyon ng Nagkakaisang Pilipino (KNP) Party, in the forthcoming national
elections. In his certificate of candidacy, FPJ, representing himself to be a natural-born citizen of the Philippines, stated his name to be "Fernando
Jr.," or "Ronald Allan" Poe, his date of birth to be 20 August 1939 and his place of birth to be Manila.
Victorino X. Fornier, petitioner in G.R. No. 161824, entitled "Victorino X. Fornier, Petitioner, versus Hon. Commission on Elections and Ronald Allan
Kelley Poe, also known as Fernando Poe, Jr., Respondents," initiated, on 09 January 2004, a petition docketed SPA No. 04-003 before the Commission
on Elections ("COMELEC") to disqualify FPJ and to deny due course or to cancel his certificate of candidacy upon the thesis that FPJ made a material
misrepresentation in his certificate of candidacy by claiming to be a natural- born Filipino citizen when in truth, according to Fornier, his parents were
foreigners; his mother, Bessie Kelley Poe, was an American, and his father, Allan Poe, was a Spanish national, being the son of Lorenzo Pou, a Spanish
subject. Granting, petitioner asseverated, that Allan F. Poe was a Filipino citizen, he could not have transmitted his Filipino citizenship to FPJ, the
latter being an illegitimate child of an alien mother. Petitioner based the allegation of the illegitimate birth of respondent on two assertions — first,
Allan F. Poe contracted a prior marriage to a certain Paulita Gomez before his marriage to Bessie Kelley and, second, even if no such prior marriage
had existed, Allan F. Poe, married Bessie Kelly only a year after the birth of respondent.
In the hearing before the Third Division of the COMELEC on 19 January 2004, petitioner, in support of his claim, presented several documentary
exhibits — 1) a copy of the certificate of birth of FPJ, 2) a certified photocopy of an affidavit executed in Spanish by Paulita Poe y Gomez attesting to
her having filed a case for bigamy and concubinage against the father of respondent, Allan F. Poe, after discovering his bigamous relationship with
Bessie Kelley, 3) an English translation of the affidavit aforesaid, 4) a certified photocopy of the certificate of birth of Allan F. Poe, 5) a certification
issued by the Director of the Records Management and Archives Office, attesting to the fact that there was no record in the National Archives that a
Lorenzo Poe or Lorenzo Pou resided or entered the Philippines before 1907, and 6) a certification from the Officer-In-Charge of the Archives Division
of the National Archives to the effect that no available information could be found in the files of the National Archives regarding the birth of Allan F.
Poe.
On his part, respondent, presented twenty-two documentary pieces of evidence, the more significant ones being — a) a certification issued by
Estrella M. Domingo of the Archives Division of the National Archives that there appeared to be no available information regarding the birth of Allan
F. Poe in the registry of births for San Carlos, Pangasinan, b) a certification issued by the Officer-In-Charge of the Archives Division of the National
Archives that no available information about the marriage of Allan F. Poe and Paulita Gomez could be found, c) a certificate of birth of Ronald Allan
Poe, d) Original Certificate of Title No. P-2247 of the Registry of Deeds for the Province of Pangasinan, in the name of Lorenzo Pou, e) copies of Tax
Declaration No. 20844, No. 20643, No. 23477 and No. 23478 in the name of Lorenzo Pou, f) a copy of the certificate of death of Lorenzo Pou, g) a
copy of the purported marriage contract between Fernando Pou and Bessie Kelley, and h) a certification issued by the City Civil Registrar of San
Carlos City, Pangasinan, stating that the records of birth in the said office during the period of from 1900 until May 1946 were totally destroyed
during World War II.
On 23 January 2004, the COMELEC dismissed SPA No. 04-003 for lack of merit. Three days later, or on 26 January 2004, Fornier filed his motion for
reconsideration. The motion was denied on 06 February 2004 by the COMELEC en banc. On 10 February 2004, petitioner assailed the decision of the
COMELEC before this Court conformably with Rule 64, in relation to Rule 65, of the Revised Rules of Civil Procedure. The petition, docketed G.R. No.
161824, likewise prayed for a temporary restraining order, a writ of preliminary injunction or any other resolution that would stay the finality and/or
execution of the COMELEC resolutions.
The other petitions, later consolidated with G.R. No. 161824, would include G.R. No. 161434, entitled "Maria Jeanette C. Tecson, and Felix B.
Desiderio, Jr., vs. The Commission on Elections, Ronald Allan Kelley Poe (a.k.a. 'Fernando Poe, Jr.'), and Victorino X. Fornier," and the other, docketed
G.R. No. 161634, entitled "Zoilo Antonio G. Velez, vs. Ronald Allan Kelley Poe, a.k.a. Fernando Poe, Jr.," both challenging the jurisdiction of the
COMELEC and asserting that, under Article VII, Section 4, paragraph 7, of the 1987 Constitution, only the Supreme Court had original and exclusive
jurisdiction to resolve the basic issue on the case.
Jurisdiction of the Court
In G.R. No. 161824
In seeking the disqualification of the candidacy of FPJ and to have the COMELEC deny due course or to cancel FPJ's certificate of candidacy for alleged
misrepresentation of a material fact (i.e., that FPJ was a natural-born citizen) before the COMELEC, petitioner Fornier invoked Section 78 of
the Omnibus Election Code —
"Section 78.Petition to deny due course or to cancel a certificate of candidacy. — A verified petition seeking to deny due course
or to cancel a certificate of candidacy may be filed by any person exclusively on the ground that any material representation
contained therein as required under Section 74 hereof is false" —
in consonance with the general powers of COMELEC expressed in Section 52 of the Omnibus Election Code —
"Section 52.Powers and functions of the Commission on Elections. In addition to the powers and functions conferred upon it
by the Constitution, the Commission shall have exclusive charge of the enforcement and administration of all laws relative to
the conduct of elections for the purpose of ensuring free, orderly and honest elections" —
and in relation to Article 69 of the Omnibus Election Code which would authorize "any interested party" to file a verified petition to deny or
cancel the certificate of candidacy of any nuisance candidate.
Decisions of the COMELEC on disqualification cases may be reviewed by the Supreme Court per Rule 64 2 in an action for certiorari under Rule
65 3 of the Revised Rules of Civil Procedure. Section 7, Article IX, of the 1987 Constitution also reads —
"Each Commission shall decide by a majority vote of all its Members any case or matter brought before it within sixty days
from the date of its submission for decision or resolution. A case or matter is deemed submitted for decision or resolution
upon the filing of the last pleading, brief, or memorandum, required by the rules of the Commission or by the Commission
itself. Unless otherwise provided by this Constitution or by law, any decision, order, or ruling of each Commission may be
brought to the Supreme Court on certiorari by the aggrieved party within thirty days from receipt of a copy thereof."
Additionally, Section 1, Article VIII, of the same Constitution provides that judicial power is vested in one Supreme Court and in such lower courts as
may be established by law which power "includes the duty of the courts of justice to settle actual controversies involving rights which are legally
demandable and enforceable, and to determine whether or not there has been a grave abuse of discretion amounting to lack or excess of jurisdiction
on the part of any branch or instrumentality of the Government."
It is sufficiently clear that the petition brought up in G.R. No. 161824 was aptly elevated to, and could well be taken cognizance of, by this Court. A
contrary view could be a gross denial to our people of their fundamental right to be fully informed, and to make a proper choice, on who could or
should be elected to occupy the highest government post in the land.

In G.R. No. 161434 and G.R. No. 161634


Petitioners Tecson, et al., in G.R. No. 161434, and Velez, in G.R. No. 161634, invoke the provisions of Article VII, Section 4, paragraph 7, of the 1987
Constitution in assailing the jurisdiction of the COMELEC when it took cognizance of SPA No. 04-003 and in urging the Supreme Court to instead take
on the petitions they directly instituted before it. The Constitutional provision cited reads:
"The Supreme Court, sitting en banc, shall be the sole judge of all contests relating to the election, returns, and qualifications of
the President or Vice-President, and may promulgate its rules for the purpose."
The provision is an innovation of the 1987 Constitution. The omission in the 1935 and the 1973 Constitution to designate any tribunal to be the sole
judge of presidential and vice-presidential contests, has constrained this Court to declare, in Lopez vs. Roxas, 4 as "not (being) justiciable"
controversies or disputes involving contests on the elections, returns and qualifications of the President or Vice-President. The constitutional lapse
prompted Congress, on 21 June 1957, to enact Republic Act No. 1793, "An Act Constituting an Independent Presidential Electoral Tribunal to Try,
Hear and Decide Protests Contesting the Election of the President-Elect and the Vice-President-Elect of the Philippines and Providing for the Manner of
Hearing the Same." Republic Act 1793 designated the Chief Justice and the Associate Justices of the Supreme Court to be the members of the
tribunal. Although the subsequent adoption of the parliamentary form of government under the 1973 Constitution might have implicitly
affected Republic Act No. 1793, the statutory set-up, nonetheless, would now be deemed revived under the present Section 4, paragraph 7, of
the 1987 Constitution.
Ordinary usage would characterize a "contest" in reference to a post-election scenario. Election contests consist of either an election protest or a quo
warranto which, although two distinct remedies, would have one objective in view, i.e., to dislodge the winning candidate from office. A perusal of
the phraseology in Rule 12, Rule 13, and Rule 14 of the "Rules of the Presidential Electoral Tribunal," promulgated by the Supreme Court en banc on
18 April 1992, would support this premise —
"Rule 12.Jurisdiction. — The Tribunal shall be the sole judge of all contests relating to the election, returns, and qualifications of
the President or Vice-President of the Philippines.
"Rule 13.How Initiated. — An election contest is initiated by the filing of an election protest or a petition for quo
warranto against the President or Vice-President. An election protest shall not include a petition for quo warranto. A petition
for quo warranto shall not include an election protest.
"Rule 14.Election Protest. — Only the registered candidate for President or for Vice-President of the Philippines who
received the second or third highest number of votes may contest the election of the President or the Vice-President, as the
case may be, by filing a verified petition with the Clerk of the Presidential Electoral Tribunal within thirty (30) days after
the proclamation of the winner."
The rules categorically speak of the jurisdiction of the tribunal over contests relating to the election, returns and qualifications of the "President" or
"Vice-President", of the Philippines, and not of "candidates" for President or Vice-President. A quo warranto proceeding is generally defined as being
an action against a person who usurps, intrudes into, or unlawfully holds or exercises a public office. 5 In such context, the election contest can only
contemplate a post-election scenario. In Rule 14, only a registered candidate who would have received either the second or third highest number of
votes could file an election protest. This rule again presupposes a post-election scenario.
It is fair to conclude that the jurisdiction of the Supreme Court, defined by Section 4, paragraph 7, of the 1987 Constitution, would not include cases
directly brought before it, questioning the qualifications of a candidate for the presidency or vice-presidency before the elections are held.
Accordingly, G.R. No. 161434, entitled "Maria Jeanette C. Tecson, et al., vs. Commission on Elections, et al.," and G.R. No. 161634, entitled "Zoilo
Antonio Velez vs. Ronald Allan Kelley Poe a.k.a. Fernando Poe, Jr." would have to be dismissed for want of jurisdiction.
The Citizenship Issue
Now, to the basic issue; it should be helpful to first give a brief historical background on the concept of citizenship.
Perhaps, the earliest understanding of citizenship was that given by Aristotle, who, sometime in 384 to 322 B.C., described the "citizen" to refer to a
man who shared in the administration of justice and in the holding of an office. 6 Aristotle saw its significance if only to determine the constituency
of the "State," which he described as being composed of such persons who would be adequate in number to achieve a self-sufficient existence. 7 The
concept grew to include one who would both govern and be governed, for which qualifications like autonomy, judgment and loyalty could be
expected. Citizenship was seen to deal with rights and entitlements, on the one hand, and with concomitant obligations, on the other. 8 In its ideal
setting, a citizen was active in public life and fundamentally willing to submit his private interests to the general interest of society.
The concept of citizenship had undergone changes over the centuries. In the 18th century, the concept was limited, by and large, to civil citizenship,
which established the rights necessary for individual freedom, such as rights to property, personal liberty and justice. 9 Its meaning expanded during
the 19th century to include political citizenship, which encompassed the right to participate in the exercise of political power. 10 The 20th century
saw the next stage of the development of social citizenship, which laid emphasis on the right of the citizen to economic well-being and social
security. 11 The idea of citizenship has gained expression in the modern welfare state as it so developed in Western Europe. An ongoing and final
stage of development, in keeping with the rapidly shrinking global village, might well be the internationalization of citizenship. 12
The Local Setting — from Spanish Time to the Present
There was no such term as "Philippine citizens" during the Spanish regime but "subjects of Spain" or "Spanish subjects." 13 In church records, the
natives were called 'indios', denoting a low regard for the inhabitants of the archipelago. Spanish laws on citizenship became highly codified during
the 19th century but their sheer number made it difficult to point to one comprehensive law. Not all of these citizenship laws of Spain however, were
made to apply to the Philippine Islands except for those explicitly extended by Royal Decrees. 14
Spanish laws on citizenship were traced back to the Novisima Recopilacion, promulgated in Spain on 16 July 1805 but as to whether the law was
extended to the Philippines remained to be the subject of differing views among experts; 15 however, three royal decrees were undisputably made
applicable to Spaniards in the Philippines — the Order de la Regencia of 14 August 1841, 16 the Royal Decree of 23 August 1868 specifically defining
the political status of children born in the Philippine Islands, 17 and finally, the Ley Extranjera de Ultramar of 04 July 1870, which was expressly made
applicable to the Philippines by the Royal Decree of 13 July 1870. 18
The Spanish Constitution of 1876 was never extended to the Philippine Islands because of the express mandate of its Article 89, according to which
the provisions of theUltramar among which this country was included, would be governed by special laws. 19
It was only the Civil Code of Spain, made effective in this jurisdiction on 18 December 1889, which came out with the first categorical enumeration of
who were Spanish citizens. —
"(a)Persons born in Spanish territory,
"(b)Children of a Spanish father or mother, even if they were born outside of Spain,
"(c)Foreigners who have obtained naturalization papers,
"(d)Those who, without such papers, may have become domiciled inhabitants of any town of the Monarchy." 20
The year 1898 was another turning point in Philippine history. Already in the state of decline as a superpower, Spain was forced to so cede her sole
colony in the East to an upcoming world power, the United States. An accepted principle of international law dictated that a change in sovereignty,
while resulting in an abrogation of all political laws then in force, would have no effect on civil laws, which would remain virtually intact.
The Treaty of Paris was entered into on 10 December 1898 between Spain and the United States. 21 Under Article IX of the treaty, the civil rights and
political status of the native inhabitants of the territories ceded to the United States would be determined by its Congress —
"Spanish subjects, natives of the Peninsula, residing in the territory over which Spain by the present treaty relinquishes or
cedes her sovereignty may remain in such territory or may remove therefrom, retaining in either event all their rights of
property, including the right to sell or dispose of such property or of its proceeds; and they shall also have the right to carry on
their industry, commerce, and professions, being subject in respect thereof to such laws as are applicable to foreigners. In case
they remain in the territory they may preserve their allegiance to the Crown of Spain by making, before a court of record,
within a year from the date of the exchange of ratifications of this treaty, a declaration of their decision to preserve such
allegiance; in default of which declaration they shall be held to have renounced it and to have adopted the nationality of the
territory in which they reside.

Thus —
"The civil rights and political status of the native inhabitants of the territories hereby ceded to the United States shall be
determined by the Congress." 22
Upon the ratification of the treaty, and pending legislation by the United States Congress on the subject, the native inhabitants of the
Philippines ceased to be Spanish subjects. Although they did not become American citizens, they, however, also ceased to be "aliens" under
American laws and were thus issued passports describing them to be citizens of the Philippines entitled to the protection of the United
States. LibLex
The term "citizens of the Philippine Islands" appeared for the first time in the Philippine Bill of 1902, also commonly referred to as the Philippine
Organic Act of 1902, the first comprehensive legislation of the Congress of the United States on the Philippines —
". . . that all inhabitants of the Philippine Islands continuing to reside therein, who were Spanish subjects on the 11th day of
April, 1891, and then resided in said Islands, and their children born subsequent thereto, shall be deemed and held to be citizens
of the Philippine Islands and as such entitled to the protection of the United States, except such as shall have elected to
preserve their allegiance to the Crown of Spain in accordance with the provisions of the treaty of peace between the United
States and Spain, signed at Paris, December tenth eighteen hundred and ninety eight." 23
Under the organic act, a "citizen of the Philippines" was one who was an inhabitant of the Philippines, and a Spanish subject on the 11th day of
April 1899. The term "inhabitant" was taken to include 1) a native-born inhabitant, 2) an inhabitant who was a native of Peninsular Spain, and 3)
an inhabitant who obtained Spanish papers on or before 11 April 1899. 24
Controversy arose on to the status of children born in the Philippines from 11 April 1899 to 01 July 1902, during which period no citizenship law was
extant in the Philippines. Weight was given to the view, articulated in jurisprudential writing at the time, that the common law principle of jus soli,
otherwise also known as the principle of territoriality, operative in the United States and England, governed those born in the Philippine Archipelago
within that period. 25 More about this later.
In 23 March 1912, the Congress of the United States made the following amendment to the Philippine Bill of 1902 —
"Provided, That the Philippine Legislature is hereby authorized to provide by law for the acquisition of Philippine citizenship by
those natives of the Philippine Islands who do not come within the foregoing provisions, the natives of other insular possession
of the United States, and such other persons residing in the Philippine Islands who would become citizens of the United States,
under the laws of the United States, if residing therein." 26
With the adoption of the Philippine Bill of 1902, the concept of "Philippine citizens" had for the first time crystallized. The word "Filipino" was
used by William H. Taft, the first Civil Governor General in the Philippines when he initially made mention of it in his slogan, "The Philippines for
the Filipinos." In 1916, the Philippine Autonomy Act, also known as the Jones Law restated virtually the provisions of the Philippine Bill of 1902,
as so amended by the Act of Congress in 1912 —
"That all inhabitants of the Philippine Islands who were Spanish subjects on the eleventh day of April, eighteen hundred and
ninety-nine, and then resided in said Islands, and their children born subsequently thereto, shall be deemed and held to be
citizens of the Philippine Islands, except such as shall have elected to preserve their allegiance to the Crown of Spain in
accordance with the provisions of the treaty of peace between the United States and Spain, signed at Paris December tenth,
eighteen hundred and ninety-eight and except such others as have since become citizens of some other country; Provided,
That the Philippine Legislature, herein provided for, is hereby authorized to provide for the acquisition of Philippine citizenship
by those natives of the Philippine Islands who do not come within the foregoing provisions, the natives of the insular
possessions of the United States, and such other persons residing in the Philippine Islands who are citizens of the United
States, or who could become citizens of the United States under the laws of the United States, if residing therein."
Under the Jones Law, a native-born inhabitant of the Philippines was deemed to be a citizen of the Philippines as of 11 April 1899 if he was 1) a
subject of Spain on 11 April 1899, 2) residing in the Philippines on said date, and, 3) since that date, not a citizen of some other country.
While there was, at one brief time, divergent views on whether or not jus soli was a mode of acquiring citizenship, the 1935 Constitution brought to
an end to any such link with common law, by adopting, once and for all, jus sanguinis or blood relationship as being the basis of Filipino citizenship —
"Section 1, Article III, 1935 Constitution. The following are citizens of the Philippines —
"(1)Those who are citizens of the Philippine Islands at the time of the adoption of this Constitution
"(2)Those born in the Philippines Islands of foreign parents who, before the adoption of this Constitution,had been elected to
public office in the Philippine Islands.
"(3)Those whose fathers or mothers are citizens of the Philippines.
"(4)Those whose mothers are citizens of the Philippines and upon reaching the age of majority, elect Philippine citizenship.
"(5)Those who are naturalized in accordance with law."
Subsection (4), Article III, of the 1935 Constitution, taken together with existing civil law provisions at the time, which provided that women would
automatically lose their Filipino citizenship and acquire that of their foreign husbands, resulted in discriminatory situations that effectively
incapacitated the women from transmitting their Filipino citizenship to their legitimate children and required illegitimate children of Filipino mothers
to still elect Filipino citizenship upon reaching the age of majority. Seeking to correct this anomaly, as well as fully cognizant of the newly found status
of Filipino women as equals to men, the framers of the 1973 Constitution crafted the provisions of the new Constitution on citizenship to reflect such
concerns —
"Section 1, Article III, 1973 Constitution — The following are citizens of the Philippines:
"(1)Those who are citizens of the Philippines at the time of the adoption of this Constitution.
"(2)Those whose fathers or mothers are citizens of the Philippines.
"(3)Those who elect Philippine citizenship pursuant to the provisions of the Constitution of nineteen hundred and thirty-five.
"(4)Those who are naturalized in accordance with law."
For good measure, Section 2 of the same article also further provided that —
"A female citizen of the Philippines who marries an alien retains her Philippine citizenship, unless by her act or omission she is
deemed, under the law to have renounced her citizenship."
The 1987 Constitution generally adopted the provisions of the 1973 Constitution, except for subsection (3) thereof that aimed to correct the irregular
situation generated by the questionable proviso in the 1935 Constitution.
Section 1, Article IV, 1987 Constitution now provides:
"The following are citizens of the Philippines:
"(1)Those who are citizens of the Philippines at the time of the adoption of this Constitution.
"(2)Those whose fathers or mothers are citizens of the Philippines.
"(3)Those born before January 17, 1973 of Filipino mothers, who elect Philippine citizenship upon reaching the age of majority;
and
"(4)Those who are naturalized in accordance with law."
The Case Of FPJ
Section 2, Article VII, of the 1987 Constitution expresses:
"No person may be elected President unless he is a natural-born citizen of the Philippines, a registered voter, able to read and
write, at least forty years of age on the day of the election, and a resident of the Philippines for at least ten years immediately
preceding such election."
The term "natural-born citizens," is defined to include "those who are citizens of the Philippines from birth without having to perform any act to
acquire or perfect their Philippine citizenship." 27
The date, month and year of birth of FPJ appeared to be 20 August 1939 during the regime of the 1935 Constitution. Through its history, four modes
of acquiring citizenship — naturalization, jus soli, res judicata and jus sanguinis 28 — had been in vogue. Only two, i.e., jus soli and jus
sanguinis, could qualify a person to being a "natural-born" citizen of the Philippines. Jus soli, per Roa vs. Collector of Customs 29 (1912), did not last
long. With the adoption of the 1935 Constitution and the reversal of Roa in Tan Chong vs. Secretary of Labor 30 (1947), jus sanguinis or blood
relationship would now become the primary basis of citizenship by birth.
Documentary evidence adduced by petitioner would tend to indicate that the earliest established direct ascendant of FPJ was his paternal
grandfather Lorenzo Pou, married to Marta Reyes, the father of Allan F. Poe. While the record of birth of Lorenzo Pou had not been presented in
evidence, his death certificate, however, identified him to be a Filipino, a resident of San Carlos, Pangasinan, and 84 years old at the time of his death
on 11 September 1954. The certificate of birth of the father of FPJ, Allan F. Poe, showed that he was born on 17 May 1915 to an Español father,
Lorenzo Pou, and a mestiza Español mother, Marta Reyes. Introduced by petitioner was an "uncertified" copy of a supposed certificate of the alleged
marriage of Allan F. Poe and Paulita Gomez on 05 July 1936. The marriage certificate of Allan F. Poe and Bessie Kelley reflected the date of their
marriage to be on 16 September 1940. In the same certificate, Allan F. Poe was stated to be twenty-five years old, unmarried, and a Filipino citizen,
and Bessie Kelley to be twenty-two years old, unmarried, and an American citizen. The birth certificate of FPJ, would disclose that he was born on 20
August 1939 to Allan F. Poe, a Filipino, twenty-four years old, married to Bessie Kelly, an American citizen, twenty-one years old and married.

Considering the reservations made by the parties on the veracity of some of the entries on the birth certificate of respondent and the marriage
certificate of his parents, the only conclusions that could be drawn with some degree of certainty from the documents would be that —
1.The parents of FPJ were Allan F. Poe and Bessie Kelley;
2.FPJ was born to them on 20 August 1939;
3.Allan F. Poe and Bessie Kelley were married to each other on 16 September, 1940;
4.The father of Allan F. Poe was Lorenzo Poe; and
5.At the time of his death on 11 September 1954, Lorenzo Poe was 84 years old.
Would the above facts be sufficient or insufficient to establish the fact that FPJ is a natural-born Filipino citizen? The marriage certificate of Allan F.
Poe and Bessie Kelley, the birth certificate of FPJ, and the death certificate of Lorenzo Pou are documents of public record in the custody of a public
officer. The documents have been submitted in evidence by both contending parties during the proceedings before the COMELEC.
The birth certificate of FPJ was marked Exhibit "A" for petitioner and Exhibit "3" for respondent. The marriage certificate of Allan F. Poe to Bessie
Kelley was submitted as Exhibit "21" for respondent. The death certificate of Lorenzo Pou was submitted by respondent as his Exhibit "5." While the
last two documents were submitted in evidence for respondent, the admissibility thereof, particularly in reference to the facts which they purported
to show, i.e., the marriage certificate in relation to the date of marriage of Allan F. Poe to Bessie Kelley and the death certificate relative to the death
of Lorenzo Pou on 11 September 1954 in San Carlos, Pangasinan, were all admitted by petitioner, who had utilized those material statements in his
argument. All three documents were certified true copies of the originals.
Section 3, Rule 130, Rules of Court states that —
"Original document must be produced; exceptions. — When the subject of inquiry is the contents of a document, no evidence
shall be admissible other than the original document itself, except in the following cases:
xxx xxx xxx
"(d)When the original is a public record in the custody of a public office or is recorded in a public office."
Being public documents, the death certificate of Lorenzo Pou, the marriage certificate of Allan F. Poe and Bessie Kelly, and the birth certificate of
FPJ, constitute prima facieproof of their contents. Section 44, Rule 130, of the Rules of Court provides:
"Entries in official records. Entries in official records made in the performance of his duty by a public officer of the Philippines,
or by a person in the performance of a duty specially enjoined by law, are prima facie evidence of the facts therein stated."
The trustworthiness of public documents and the value given to the entries made therein could be grounded on 1) the sense of official duty in the
preparation of the statement made, 2) the penalty which is usually affixed to a breach of that duty, 3) the routine and disinterested origin of most
such statements, and 4) the publicity of record which makes more likely the prior exposure of such errors as might have occurred. 31
The death certificate of Lorenzo Pou would indicate that he died on 11 September 1954, at the age of 84 years, in San Carlos, Pangasinan. It could
thus be assumed that Lorenzo Pou was born sometime in the year 1870 when the Philippines was still a colony of Spain. Petitioner would argue that
Lorenzo Pou was not in the Philippines during the crucial period of from 1898 to 1902 considering that there was no existing record about such fact
in the Records Management and Archives Office. Petitioner, however, likewise failed to show that Lorenzo Pou was at any other place during the
same period. In his death certificate, the residence of Lorenzo Pou was stated to be San Carlos, Pangasinan. In the absence of any evidence to the
contrary, it should be sound to conclude, or at least to presume, that the place of residence of a person at the time of his death was also his
residence before death. It would be extremely doubtful if the Records Management and Archives Office would have had complete records of all
residents of the Philippines from 1898 to 1902.
Proof of Paternity and Filiation Under Civil Law.
Petitioner submits, in any case, that in establishing filiation (relationship or civil status of the child to the father [or mother]) or paternity (relationship
or civil status of the father to the child) of an illegitimate child, FPJ evidently being an illegitimate son according to petitioner, the mandatory rules
under civil law must be used.
Under the Civil Code of Spain, which was in force in the Philippines from 08 December 1889 up until the day prior to 30 August 1950 when the Civil
Code of the Philippines took effect, acknowledgment was required to establish filiation or paternity. Acknowledgment was either judicial
(compulsory) or voluntary. Judicial or compulsory acknowledgment was possible only if done during the lifetime of the putative parent; voluntary
acknowledgment could only be had in a record of birth, a will, or a public document. 32Complementary to the new code was Act No. 3753 or the Civil
Registry Law expressing in Section 5 thereof, that —
"In case of an illegitimate child, the birth certificate shall be signed and sworn to jointly by the parents of the infant or only by
the mother if the father refuses. In the latter case, it shall not be permissible to state or reveal in the document the name of
the father who refuses to acknowledge the child, or to give therein any information by which such father could be identified."
In order that the birth certificate could then be utilized to prove voluntary acknowledgment of filiation or paternity, the certificate was required
to be signed or sworn to by the father. The failure of such requirement rendered the same useless as being an authoritative document of
recognition. 33 In Mendoza vs. Mella, 34 the Court ruled —
"Since Rodolfo was born in 1935, after the registry law was enacted, the question here really is whether or not his birth
certificate (Exhibit 1), which is merely a certified copy of the registry record, may be relied upon as sufficient proof of his
having been voluntarily recognized. No such reliance, in our judgment, may be placed upon it. While it contains the names of
both parents, there is no showing that they signed the original, let alone swore to its contents as required in Section 5 of Act
No. 3753. For all that might have happened, it was not even they or either of them who furnished the data to be entered in the
civil register. Petitioners say that in any event the birth certificate is in the nature of a public document wherein voluntary
recognition of a natural child may also be made, according to the same Article 131. True enough, but in such a case, there must
be a clear statement in the document that the parent recognizes the child as his or her own."
In the birth certificate of respondent FPJ, presented by both parties, nowhere in the document was the signature of Allan F. Poe found. There being
no will apparently executed, or at least shown to have been executed, by decedent Allan F. Poe, the only other proof of voluntary recognition
remained to be "some other public document." In Pareja vs.Pareja, 35 this Court defined what could constitute such a document as proof of
voluntary acknowledgment:
"Under the Spanish Civil Code there are two classes of public documents, those executed by private individuals which must be
authenticated by notaries, and those issued by competent public officials by reason of their office. The public document
pointed out in Article 131 as one of the means by which recognition may be made belongs to the first class."
Let us leave it at that for the moment.
The 1950 Civil Code categorized the acknowledgment or recognition of illegitimate children into voluntary, legal or compulsory. Voluntary
recognition was required to be expressedly made in a record of birth, a will, a statement before a court of record or in any authentic writing. Legal
acknowledgment took place in favor of full blood brothers and sisters of an illegitimate child who was recognized or judicially declared as natural.
Compulsory acknowledgment could be demanded generally in cases when the child had in his favor any evidence to prove filiation. Unlike an action
to claim legitimacy which would last during the lifetime of the child, and might pass exceptionally to the heirs of the child, an action to claim
acknowledgment, however, could only be brought during the lifetime of the presumed parent.
Amicus Curiae Ruben F. Balane defined, during the oral argument, "authentic writing," so as to be an authentic writing for purposes of voluntary
recognition, simply as being a genuine or indubitable writing of the father. The term would include a public instrument (one duly acknowledged
before a notary public or other competent official) or a private writing admitted by the father to be his.
The Family Code has further liberalized the rules; Article 172, Article 173, and Article 175 provide:
"Art. 172.The filiation of legitimate children is established by any of the following:
"(1)The record of birth appearing in the civil register or a final judgment; or
"(2)An admission of legitimate filiation in a public document or a private handwritten instrument and signed by the parent
concerned.
"In the absence of the foregoing evidence, the legitimate filiation shall be proved by:
"(1)The open and continuous possession of the status of a legitimate child; or
"(2)Any other means allowed by the Rules of Court and special laws.
"Art. 173.The action to claim legitimacy may be brought by the child during his or her lifetime and shall be transmitted to the
heirs should the child die during minority or in a state of insanity. In these cases, the heirs shall have a period of five years
within which to institute the action.
"The action already commenced by the child shall survive notwithstanding the death of either or both the parties.
"xxx xxx xxx
"Art. 175.Illegitimate children may establish their illegitimate filiation in the same way and on the same evidence as legitimate
children.
"The action must be brought within the same period specified in Article 173, except when the action is based on the second
paragraph of Article 172, in which case the action may be brought during the lifetime of the alleged parent."
The provisions of the Family Code are retroactively applied; Article 256 of the code reads:
"Art. 256.This Code shall have retroactive effect insofar as it does not prejudice or impair vested or acquired rights in
accordance with the Civil Code or other laws."
Thus, in Vda. De SyQuia vs. Court of Appeals, 36 the Court has ruled:
"We hold that whether Jose was a voluntarily recognized natural child should be decided under Article 278 of the Civil Code of
the Philippines. Article 2260 of that Code provides that 'the voluntary recognition of a natural child shall take place according
to this Code, even if the child was born before the effectivity of this body of laws' or before August 30, 1950. Hence, Article 278
may be given retroactive effect."
It should be apparent that the growing trend to liberalize the acknowledgment of recognition of illegitimate children is an attempt to break away
from the traditional idea of keeping well apart legitimate and non-legitimate relationships within the family in favor of the greater interest and
welfare of the child. The provisions are intended to merely govern the private and personal affairs of the family. There is little, if any, to indicate that
the legitimate or illegitimate civil status of the individual would also affect his political rights or, in general, his relationship to the State. While,
indeed, provisions on "citizenship" could be found in the Civil Code,such provisions must be taken in the context or private relations, the domain of
civil law; particularly —
"Civil Law is that branch of law which has for its double purpose the organization of the family and the regulation of property.
It has thus [been] defined as the mass of precepts which determine and regulate the relations of assistance, authority and
obedience among member of a family, and those which exist among members of a society for the protection of private
interests." 37
In Yañez de Barnuevo vs. Fuster, 38 the Court has held:
"In accordance with Article 9 of the Civil Code of Spain, . . . the laws relating to family rights and duties, or to the status,
condition and legal capacity of persons, govern Spaniards although they reside in a foreign country; that, in consequence, 'all
questions of a civil nature, such as those dealing with the validity or nullity of the matrimonial bond, the domicile of the
husband and wife, their support, as between them, the separation of their properties, the rules governing property, marital
authority, division of conjugal property, the classification of their property, legal causes for divorce, the extent of the latter, the
authority to decree it, and, in general, the civil effects of marriage and divorce upon the persons and properties of the spouses,
are questions that are governed exclusively by the national law of the husband and wife."
The relevance of "citizenship" or "nationality" to Civil Law is best exemplified in Article 15 of the Civil Code,stating that —
"Laws relating to family rights and duties, or to the status, condition and legal capacity of persons are binding upon citizens of
the Philippines, even though living abroad" —
that explains the need to incorporate in the code a reiteration of the Constitutional provisions on citizenship. Similarly, citizenship is significant
in civil relationships found in different parts of the Civil Code,39 such as on successional rights and family relations. 40 In adoption, for instance,
an adopted child would be considered the child of his adoptive parents and accorded the same rights as their legitimate child but such legal
fiction extended only to define his rights under civil law 41 and not his political status.
Civil law provisions point to an obvious bias against illegitimacy. This discriminatory attitude may be traced to the Spanish family and property laws,
which, while defining proprietary and successional rights of members of the family, provided distinctions in the rights of legitimate and illegitimate
children. In the monarchial set-up of old Spain, the distribution and inheritance of titles and wealth were strictly according to bloodlines and the
concern to keep these bloodlines uncontaminated by foreign blood was paramount.
These distinctions between legitimacy and illegitimacy were codified in the Spanish Civil Code, and the invidious discrimination survived when
the Spanish Civil Code became the primary source of our own Civil Code. Such distinction, however, remains and should remain only in the sphere of
civil law and not unduly impede or impinge on the domain of political law.
The proof of filiation or paternity for purposes of determining his citizenship status should thus be deemed independent from and not inextricably
tied up with that prescribed for civil law purposes. The Civil Code or Family Code provisions on proof of filiation or paternity, although good law, do
not have preclusive effects on matters alien to personal and family relations. The ordinary rules on evidence could well and should govern. For
instance, the matter about pedigree is not necessarily precluded from being applicable by the Civil Code or Family Code provisions.
Section 39, Rule 130, of the Rules of Court provides —
"Act or Declaration about pedigree. The act or declaration of a person deceased, or unable to testify, in respect to the pedigree
of another person related to him by birth or marriage, may be received in evidence where it occurred before the controversy,
and the relationship between the two persons is shown by evidence other than such act or declaration. The word 'pedigree'
includes relationship, family genealogy, birth, marriage, death, the dates when and the places where these facts occurred, and
the names of the relatives. It embraces also facts of family history intimately connected with pedigree."
For the above rule to apply, it would be necessary that (a) the declarant is already dead or unable to testify, (b) the pedigree of a person must
be at issue, (c) the declarant must be a relative of the person whose pedigree is in question, (d) declaration must be made before the
controversy has occurred, and (e) the relationship between the declarant and the person whose pedigree is in question must be shown by
evidence other than such act or declaration.
Thus, the duly notarized declaration made by Ruby Kelley Mangahas, sister of Bessie Kelley Poe submitted as Exhibit 20 before the COMELEC, might
be accepted to prove the facts of Allan F. Poe, recognizing his own paternal relationship with FPJ, i.e., living together with Bessie Kelly and his
children (including respondent FPJ) in one house, and as one family —
"I, Ruby Kelly Mangahas, of legal age and sound mind, presently residing in Stockton, California, U.S.A., after being sworn in
accordance with law do hereby declare that:
"1.I am the sister of the late Bessie Kelly Poe.
"2.Bessie Kelley Poe was the wife of Fernando Poe, Sr.
"3.Fernando and Bessie Poe had a son by the name of Ronald Allan Poe, more popularly known in the Philippines as 'Fernando
Poe, Jr., or FPJ'.
"4.Ronald Allan Poe 'FPJ' was born on August 20, 1939 at St. Luke's Hospital, Magdalena Street, Manila.
"xxx xxx xxx
"7.Fernando Poe Sr., and my sister Bessie, met and became engaged while they were students at the University of the
Philippines in 1936. I was also introduced to Fernando Poe Sr., by my sister that same year.
"8.Fernando Poe, Sr., and my sister Bessie had their first child in 1938.
"9.Fernando Poe, Sr., my sister Bessie and their first three children, Elizabeth, Ronald, Allan and Fernando II, and myself lived
together with our mother at our family's house on Dakota St. (now Jorge Bocobo St.), Malate until the liberation of
Manila in 1945, except for some months between 1943-1944.
"10.Fernando Poe, Sr., and my sister, Bessie, were blessed with four (4) more children after Ronald Allan Poe.
"xxx xxx xxx
"18.I am executing this Declaration to attest to the fact that my nephew, Ronald Allan Poe is a natural born Filipino, and that he
is the legitimate child of Fernando Poe, Sr.
"Done in City of Stockton, California, U.S.A., this 12th day of January 2004.
Ruby Kelly Mangahas
Declarant
DNA Testing
In case proof of filiation or paternity would be unlikely to satisfactory establish or would be difficult to obtain, DNA testing, which examines genetic
codes obtained from body cells of the illegitimate child and any physical residue of the long dead parent could be resorted to. A positive match would
clear up filiation or paternity. In Tijing vs. Court of Appeals, 42 this Court has acknowledged the strong weight of DNA testing —
"Parentage will still be resolved using conventional methods unless we adopt the modern and scientific ways available.
Fortunately, we have now the facility and expertise in using DNA test for identification and parentage testing. The University of
the Philippines Natural Science Research Institute (UP-NSRI) DNA Analysis Laboratory has now the capability to conduct DNA
typing using short tandem repeat (STR) analysis. The analysis is based on the fact that the DNA of a child/person has two (2)
copies, one copy from the mother and the other from the father. The DNA from the mother, the alleged father and the child
are analyzed to establish parentage. Of course, being a novel scientific technique, the use of DNA test as evidence is still open
to challenge. Eventually, as the appropriate case comes, courts should not hesitate to rule on the admissibility of DNA
evidence. For it was said, that courts should apply the results of science when completely obtained in aid of situations
presented, since to reject said result is to deny progress."

Petitioner's Argument For Jurisprudential Conclusiveness


Petitioner would have it that even if Allan F. Poe were a Filipino citizen, he could not have transmitted his citizenship to respondent FPJ, the latter
being an illegitimate child. According to petitioner, prior to his marriage to Bessie Kelly, Allan F. Poe, on July 5, 1936, contracted marriage with a
certain Paulita Gomez, making his subsequent marriage to Bessie Kelly bigamous and respondent FPJ an illegitimate child. The veracity of the
supposed certificate of marriage between Allan F. Poe and Paulita Gomez could be most doubtful at best. But the documentary evidence introduced
by no less than respondent himself, consisting of a birth certificate of respondent and a marriage certificate of his parents showed that FPJ was born
on 20 August 1939 to Filipino father and an American mother who were married to each other a year later, or on 16 September 1940. Birth to
unmarried parents would make FPJ an illegitimate child. Petitioner contended that as an illegitimate child, FPJ so followed the citizenship of his
mother, Bessie Kelly, an American citizen, basing his stand on the ruling of this Court in Morano vs. Vivo, 43 citing Chiongbian vs. de
Leon 44 and Serra vs. Republic. 45
On the above score, the disqualification made by amicus curiae Joaquin G. Bernas, SJ, is most convincing; he states —
"We must analyze these cases and ask what the lis mota was in each of them. If the procurement of the Court on jus
sanguinis was on the lis mota, the pronouncement would be a decision constituting doctrine under the rule of stare decisis. But
if the pronouncement was irrelevant to the lis mota, the pronouncement would not be a decision but a mereobiter
dictum which did not establish doctrine. I therefore invite the Court to look closely into these cases.
"First, Morano vs. Vivio. The case was not about an illegitimate child of a Filipino father. It was about a stepson of a Filipino, a
stepson who was the child of a Chinese mother and a Chinese father. The issue was whether the stepson followed the
naturalization of the stepfather. Nothing about jus sanguinis there. The stepson did not have blood of the naturalized
stepfather.
"Second, Chiongbian vs. de Leon. This case was not about the illegitimate son of a Filipino father. It was about a legitimate son
of a father who had become Filipino by election to public office before the 1935 Constitution pursuant to Article IV, Section
1(2) of the 1935 Constitution. No one was illegitimate here.
"Third, Serra vs. Republic. The case was not about the illegitimate son of a Filipino father. Serra was an illegitimate child of a
Chinese father and a Filipino mother. The issue was whether one who was already a Filipino because of his mother who still
needed to be naturalized. There is nothing there about invidious jus sanguinis.
"Finally, Paa vs. Chan. 46 This is more complicated case. The case was about the citizenship of Quintin Chan who was the son of
Leoncio Chan. Quintin Chan claimed that his father, Leoncio, was the illegitimate son of a Chinese father and a Filipino mother.
Quintin therefore argued that he got his citizenship from Leoncio, his father. But the Supreme Court said that there was no
valid proof that Leoncio was in fact the son of a Filipina mother. The Court therefore concluded that Leoncio was not Filipino. If
Leoncio was not Filipino, neither was his son Quintin. Quintin therefore was not only not a natural-born Filipino but was not
even a Filipino.
"The Court should have stopped there. But instead it followed with an obiter dictum. The Court said obiterthat even if Leoncio,
Quintin's father, were Filipino, Quintin would not be Filipino because Quintin was illegitimate. This statement about Quintin,
based on a contrary to fact assumption, was absolutely necessary for the case. . . . It was obiter dictum, pure and simple, simply
repeating the obiter dictum in Morano vs. Vivo.
"xxx xxx xxx
"Aside from the fact that such a pronouncement would have no textual foundation in the Constitution, it would also violate the
equal protection clause of the Constitution not once but twice. First, it would make an illegitimate distinction between a
legitimate child and an illegitimate child, and second, it would make an illegitimate distinction between the illegitimate child of
a Filipino father and the illegitimate child of a Filipino mother.
"The doctrine on constitutionality allowable distinctions was established long ago by People vs. Cayat. 47 I would grant that the
distinction between legitimate children and illegitimate children rests on real differences. . . . But real differences alone do not
justify invidious distinction. Real differences may justify distinction for one purpose but not for another purpose.
". . . What is the relevance of legitimacy to elective public service? What possible state interest can there be for disqualifying an
illegitimate child from becoming a public officer. It was not the fault of the child that his parents had illicit liaison. Why deprive
the child of the fullness of political rights for no fault of his own? To disqualify an illegitimate child from holding an important
public office is to punish him for the indiscretion of his parents. There is neither justice nor rationality in that. And if there is
neither justice nor rationality in the distinction, then the distinction transgresses the equal protection clause and must be
reprobated."
The other amici curiae, Mr. Justice Vicente Mendoza (a former member of this Court), Professor Ruben Balane and Dean Martin Magallona, at
bottom, have expressed similar views. The thesis of petitioner, unfortunately hinging solely on pure obiter dicta, should indeed fail.
Where jurisprudence regarded an illegitimate child as taking after the citizenship of its mother, it did so for the benefit of the child. It was to ensure a
Filipino nationality for the illegitimate child of an alien father in line with the assumption that the mother had custody, would exercise parental
authority and had the duty to support her illegitimate child. It was to help the child, not to prejudice or discriminate against him.
The fact of the matter — perhaps the most significant consideration — is that the 1935 Constitution, the fundamental law prevailing on the day,
month and year of birth of respondent FPJ, can never be more explicit than it is. Providing neither conditions nor distinctions, the Constitution states
that among the citizens of the Philippines are "those whose fathers are citizens of the Philippines." There utterly is no cogent justification to
prescribe conditions or distinctions where there are clearly none provided.
In Sum —
(1)The Court, in the exercise of its power of judicial review, possesses jurisdiction over the petition in G.R. No. 161824, filed under Rule 64, in relation
to Rule 65, of the Revised Rules of Civil Procedure. G.R. No. 161824 assails the resolution of the COMELEC for alleged grave abuse of discretion in
dismissing, for lack of merit, the petition in SPA No. 04-003 which has prayed for the disqualification of respondent FPJ from running for the position
of President in the 10th May 2004 national elections on the contention that FPJ has committed material representation in his certificate of candidacy
by representing himself to be a natural-born citizen of the Philippines.
(2)The Court must dismiss, for lack of jurisdiction and prematurity, the petitions in G.R. No. 161434 and No. 161634 both having been directly
elevated to this Court in the latter's capacity as the only tribunal to resolve a presidential and vice-presidential election contest under the
Constitution. Evidently, the primary jurisdiction of the Court can directly be invoked only after, not before, the elections are held.
(3)In ascertaining, in G.R. No. 161824, whether grave abuse of discretion has been committed by the COMELEC, it is necessary to take on the matter
of whether or not respondent FPJ is a natural-born citizen, which, in turn, depended on whether or not the father of respondent, Allan F. Poe, would
have himself been a Filipino citizen and, in the affirmative, whether or not the alleged illegitimacy of respondent prevents him from taking after the
Filipino citizenship of his putative father. Any conclusion on the Filipino citizenship of Lorenzo Pou could only be drawn from the presumption that
having died in 1954 at 84 years old, Lorenzo would have been born sometime in the year 1870, when the Philippines was under Spanish rule, and
that San Carlos, Pangasinan, his place of residence upon his death in 1954, in the absence of any other evidence, could have well been his place of
residence before death, such that Lorenzo Pou would have benefited from the "en masse Filipinization" that the Philippine bill had effected in 1902.
That citizenship (of Lorenzo Pou), if acquired, would thereby extend to his son, Allan F. Poe, father of respondent FPJ. The 1935 Constitution, during
which regime respondent FPJ has seen first light, confers citizenship to all persons whose fathers are Filipino citizens regardless of whether such
children are legitimate or illegitimate.
(4)But while the totality of the evidence may not establish conclusively that respondent FPJ is a natural-born citizen of the Philippines, the evidence
on hand still would preponderate in his favor enough to hold that he cannot be held guilty of having made a material misrepresentation in his
certificate of candidacy in violation of Section 78, in relation to Section 74, of the Omnibus Election Code. Petitioner has utterly failed to substantiate
his case before the Court, notwithstanding the ample opportunity given to the parties to present their position and evidence, and to prove whether
or not there has been material misrepresentation, which, as so ruled in Romualdez-Marcos vs. COMELEC, 48must not only be material, but also
deliberate and willful.

WHEREFORE, the Court RESOLVES to DISMISS —


1.G.R. No. 161434, entitled "Maria Jeanette C. Tecson and Felix B. Desiderio, Jr., Petitioners, versus Commission on Elections, Ronald Allan Kelley Poe
(a.k.a. "Fernando Poe, Jr.,) and Victorino X. Fornier, Respondents," and G.R. No. 161634, entitled "Zoilo Antonio Velez, Petitioner, versus Ronald Allan
Kelley Poe, a.k.a. Fernando Poe, Jr., Respondent," for want of jurisdiction.
2.G.R. No. 161824, entitled "Victorino X. Fornier, Petitioner, versus Hon. Commission on Elections and Ronald Allan Kelley Poe, also known as
Fernando Poe, Jr.," for failure to show grave abuse of discretion on the part of respondent Commission on Elections in dismissing the petition in SPA
No. 04-003.
No Costs. ASCTac
SO ORDERED.
Davide, Jr., C.J., see separate opinion.
Puno, J., is on leave but was allowed to vote; see separate opinion.
Panganiban, J., is on official leave; allowed to vote but did not send his vote.
Quisumbing, J., joins the dissent of Justices Tinga and Morales; case should have been REMANDED.
Ynares-Santiago, J., concurs and also with J. Puno's separate opinion.
Sandoval-Gutierrez, J., concurs, please see separate opinion.
Carpio, J., see separate opinion.
Austria-Martinez, J., concurs, please see separate opinion.
Corona, J., joins the dissenting opinion of Justice Morales.
Carpio Morales, J., see dissenting opinion.
Callejo, Sr., J ., see concurring opinion.
Azcuna, J., concurs in a separate opinion.
Tinga, J., dissents as per his separate opinion.
||| (Tecson v. Commission on Elections, G.R. No. 161434, 161634, 161824, [March 3, 2004], 468 PHIL 421-755)
EN BANC
[G.R. No. 137000. August 9, 2000.]
CIRILO R. VALLES, petitioner, vs. COMMISSION ON ELECTIONS and ROSALIND YBASCO LOPEZ, respondents.
Ifurung & Marquinez for petitioner.
The Solicitor General for respondents.
SYNOPSIS
This is a petition for certiorari assailing the Resolutions of the COMELEC, dismissing the petition for disqualification filed by petitioner against private
respondent Rosalind Ybasco Lopez, in the May 1998 elections for governor of Davao Oriental. EHTCAa
Petitioner maintained that private respondent is an Australian citizen, not qualified to run for elective office, because: she is a holder of an Australian
passport; and she expressly renounced her Filipino citizenship when she declared under oath in her application for alien certificate of registration and
immigrant certificate of residence that she was a citizen or subject of Australia.
In dismissing the petition, the Supreme Court held that the mere fact that private respondent was a holder of an Australian passport and had an alien
certificate of registration are not acts constituting an effective renunciation of Filipino citizenship. Renunciation must be express, to effectively result
in the loss of Filipino citizenship. At most, private respondent had dual citizenship — she was an Australian and a Filipino, as well. Dual citizenship as
a disqualification refers to citizens with dual allegiance. Her filing of a certificate of candidacy, where she declared that she is a Filipino citizen and
that she will support and defend the Philippine Constitution and will maintain true faith and allegiance thereto, sufficed to renounce her foreign
citizenship, effectively removing any disqualification as a dual citizen.
SYLLABUS
1. CONSTITUTIONAL LAW; CITIZENSHIP; RENUNCIATION MUST BE EXPRESS; APPLYING FOR AN ALIEN CERTIFICATE OF REGISTRATION AND HOLDING A
FOREIGN PASSPORT, NOT A CASE OF; CASE AT BAR. — In order that citizenship may be lost by renunciation, such renunciation must be express.
Petitioner's contention that the application of private respondent for an alien certificate of registration, and her Australian passport, is bereft of
merit. This issue was put to rest in the case of Aznar vs. COMELEC and in the more recent case of Mercado vs. Manzano and COMELEC. In the case of
Aznar, the Court ruled that the mere fact that respondent Osmena was a holder of a certificate stating that he is an American did not mean that he is
no longer a Filipino, and that an application for an alien certificate of registration was not tantamount to renunciation of his Philippine citizenship.
And, in Mercado vs. Manzano and COMELEC, it was held that the fact that respondent Manzano was registered as an American citizen in the Bureau
of Immigration and Deportation and was holding an American passport on April 22, 1997, only a year before he filed a certificate of candidacy for
vice-mayor of Makati, were just assertions of his American nationality before the termination of his American citizenship. Thus, the mere fact that
private respondent Rosalind Ybasco Lopez was a holder of an Australian passport and had an alien certificate of registration are not acts constituting
an effective renunciation of citizenship and do not militate against her claim of Filipino citizenship. For renunciation to effectively result in the loss of
citizenship, the same must be express. EDATSI
2. ID.; ID.; DUAL CITIZENSHIP; AS A DISQUALIFICATION FROM RUNNING FOR PUBLIC OFFICE REFERS TO CITIZENS WITH DUAL ALLEGIANCE; CASE AT
BAR. — Petitioner maintains that even on the assumption that the private respondent had dual citizenship, still, she is disqualified to run for
governor of Davao Oriental; citing Section 40 ofRepublic Act 7160 otherwise known as the Local Government Code of 1991, . . . In the aforecited case
of Mercado vs. Manzano, the Court clarified "dual citizenship" as used in the Local Government Code and reconciled the same with Article IV, Section
5 of the 1987 Constitution on dual allegiance. Recognizing situations in which a Filipino citizen may, without performing any act, and as an
involuntary consequence of the conflicting laws of different countries, be also a citizen of another state, the Court explained that dual citizenship as a
disqualification must refer to citizens with dual allegiance. . . Thus, the fact that the private respondent had dual citizenship did not automatically
disqualify her from running for a public office.
3. ID.; ID.; ID.; RENUNCIATION OF FOREIGN CITIZENSHIP EFFECTIVELY REMOVES ANY DISQUALIFICATION AS A DUAL CITIZEN; CASE AT BAR. — It was
ruled that for candidates with dual citizenship, it is enough that they elect Philippine citizenship upon the filing of their certificate of candidacy, to
terminate their status as persons with dual citizenship. The filing of a certificate of candidacy sufficed to renounce foreign citizenship, effectively
removing any disqualification as a dual citizen. This is so because in the certificate of candidacy, one declares that he/she is a Filipino citizen and that
he/she will support and defend the Constitution of the Philippines and will maintain true faith and allegiance thereto. Such declaration, which is
under oath, operates as an effective renunciation of foreign citizenship. Therefore, when the herein private respondent filed her certificate of
candidacy in 1992, such fact alone terminated her Australian citizenship. Then, too, it is significant to note that on January 15, 1992, private
respondent executed a Declaration of Renunciation of Australian Citizenship, duly registered in the Department of Immigration and Ethnic Affairs of
Australia on May 12, 1992. And, as a result, on February 11, 1992, the Australian passport of private respondent was cancelled, as certified to by
Second Secretary Richard F. Munro of the Embassy of Australia in Manila. As aptly appreciated by the COMELEC, the aforesaid acts were enough to
settle the issue of the alleged dual citizenship of Rosalind Ybasco Lopez.
4. REMEDIAL LAW; CIVIL PROCEDURE; JUDGMENT; PRINCIPLE THEREOF GENERALLY DOES NOT APPLY IN CASES OF CITIZENSHIP; EXCEPTION; CASE AT
BAR. — Petitioner is correct insofar as the general rule is concerned, i.e. the principle of res judicata generally does not apply in cases hinging on the
issue of citizenship. However, in the case ofBurca vs. Republic, an exception to this general rule was recognized. The Court ruled in that case that in
order that the doctrine of res judicata may be applied in cases of citizenship, the following must be present: 1) a person's citizenship be raised as a
material issue in a controversy where said person is a party; 2) the Solicitor General or his authorized representative took active part in the resolution
thereof; and 3) the finding on citizenship is affirmed by this Court. Although the general rule was set forth in the case of Moy Ya Lim Yao, the case did
not foreclose the weight of prior rulings on citizenship. It elucidated that reliance may somehow be placed on these antecedent official findings,
though not really binding, to make the effort easier or simpler. Indeed, there appears sufficient basis to rely on the prior rulings of the Commission
on Elections in SPA No. 95-066 and EPC 92-54 which resolved the issue of citizenship in favor of the herein private respondent. The evidence adduced
by petitioner is substantially the same evidence presented in these two prior cases. Petitioner failed to show any new evidence or supervening event
to warrant a reversal of such prior resolutions. TCaEAD
DECISION
PURISIMA, J p:
This is a petition for certiorari under Rule 65, pursuant to Section 2, Rule 64 of the 1997 Rules of Civil Procedure, assailing Resolutions dated July 17,
1998 and January 15, 1999, respectively, of the Commission on Elections in SPA No. 98-336, dismissing the petition for disqualification filed by the
herein petitioner, Cirilo R. Valles, against private respondent Rosalind Ybasco Lopez, in the May 1998 elections for governor of Davao Oriental.
Rosalind Ybasco Lopez was born on May 16, 1934 in Napier Terrace, Broome, Western Australia, to the spouses, Telesforo Ybasco, a Filipino citizen
and native of Daet, Camarines Norte, and Theresa Marquez, an Australian. In 1949, at the age of fifteen, she left Australia and came to settle in the
Philippines.
On June 27, 1952, she was married to Leopoldo Lopez, a Filipino citizen, at the Malate Catholic Church in Manila. Since then, she has continuously
participated in the electoral process not only as a voter but as a candidate, as well. She served as Provincial Board Member of the Sangguniang
Panlalawigan of Davao Oriental. In 1992, she ran for and was elected governor of Davao Oriental. Her election was contested by her opponent, Gil
Taojo, Jr., in a petition for quo warranto, docketed as EPC No. 92-54, alleging as ground therefor her alleged Australian citizenship. However, finding
no sufficient proof that respondent had renounced her Philippine citizenship, the Commission on Elections en bancdismissed the petition,
ratiocinating thus:
"A cursory reading of the records of this case vis-a-vis the impugned resolution shows that respondent was able to produce
documentary proofs of the Filipino citizenship of her late father . . . and consequently, prove her own citizenship and filiation
by virtue of the Principle of Jus Sanguinis, the perorations of the petitioner to the contrary notwithstanding. ETIDaH
On the other hand, except for the three (3) alleged important documents . . . no other evidence substantial in nature surfaced
to confirm the allegations of petitioner that respondent is an Australian citizen and not a Filipino. Express renunciation of
citizenship as a mode of losing citizenship under Commonwealth Act No. 63 is an equivocal and deliberate act with full
awareness of its significance and consequence. The evidence adduced by petitioner are inadequate, nay meager, to prove that
respondent contemplated renunciation of her Filipino citizenship". 1

In the 1995 local elections, respondent Rosalind Ybasco Lopez ran for re-election as governor of Davao Oriental. Her opponent, Francisco Rabat, filed
a petition for disqualification, docketed as SPA No. 95-066 before the COMELEC, First Division, contesting her Filipino citizenship but the said petition
was likewise dismissed by the COMELEC, reiterating substantially its decision in EPC 92-54.
The citizenship of private respondent was once again raised as an issue when she ran for re-election as governor of Davao Oriental in the May 11,
1998 elections. Her candidacy was questioned by the herein petitioner, Cirilo Valles, in SPA No. 98-336.
On July 17, 1998, the COMELEC's First Division came out with a Resolution dismissing the petition, and disposing as follows:
"Assuming arguendo that res judicata does not apply and We are to dispose the instant case on the merits trying it de novo,
the above table definitely shows that petitioner herein has presented no new evidence to disturb the Resolution of this
Commission in SPA No. 95-066. The present petition merely restates the same matters and incidents already passed upon by
this Commission not just in 1995 Resolution but likewise in the Resolution of EPC No. 92-54. Not having put forth any new
evidence and matter substantial in nature, persuasive in character or sufficiently provocative to compel reversal of such
Resolutions, the dismissal of the present petition follows as a matter of course.
xxx xxx xxx
"WHEREFORE, premises considered and there being no new matters and issues tendered, We find no convincing reason or
impressive explanation to disturb and reverse the Resolutions promulgated by this Commission in EPC 92-54 and SPA 95-066.
This Commission RESOLVES as it hereby RESOLVES to DISMISS the present petition.
SO ORDERED." 2
Petitioner interposed a motion for reconsideration of the aforesaid Resolution but to no avail. The same was denied by the COMELEC in
its en banc Resolution of January 15, 1999.
Undaunted, petitioner found his way to this Court via the present petition; questioning the citizenship of private respondent Rosalind Ybasco Lopez.
The Commission on Elections ruled that private respondent Rosalind Ybasco Lopez is a Filipino citizen and therefore, qualified to run for a public
office because (1) her father, Telesforo Ybasco, is a Filipino citizen, and by virtue of the principle of jus sanguinis she was a Filipino citizen under
the 1987 Philippine Constitution; (2) she was married to a Filipino, thereby making her also a Filipino citizen ipso jure under Section 4
of Commonwealth Act 473; (3) and that, she renounced her Australian citizenship on January 15, 1992 before the Department of Immigration and
Ethnic Affairs of Australia and her Australian passport was accordingly cancelled as certified to by the Australian Embassy in Manila; and (4)
furthermore, there are the COMELEC Resolutions in EPC No. 92-54 and SPA Case No. 95-066, declaring her a Filipino citizen duly qualified to run for
the elective position of Davao Oriental governor.
Petitioner, on the other hand, maintains that the private respondent is an Australian citizen, placing reliance on the admitted facts that:
a) In 1988, private respondent registered herself with the Bureau of Immigration as an Australian national and was issued Alien
Certificate of Registration No. 404695 dated September 19, 1988; SCEDaT
b) On even date, she applied for the issuance of an Immigrant Certificate of Residence (ICR); and
c) She was issued Australian Passport No. H700888 on March 3, 1988.
Petitioner theorizes that under the aforestated facts and circumstances, the private respondent had renounced her Filipino citizenship. He contends
that in her application for alien certificate of registration and immigrant certificate of residence, private respondent expressly declared under oath
that she was a citizen or subject of Australia; and said declaration forfeited her Philippine citizenship, and operated to disqualify her to run for
elective office.
As regards the COMELEC's finding that private respondent had renounced her Australian citizenship on January 15, 1992 before the Department of
Immigration and Ethnic Affairs of Australia and had her Australian passport cancelled on February 11, 1992, as certified to by the Australian Embassy
here in Manila, petitioner argues that the said acts did not automatically restore the status of private respondent as a Filipino citizen. According to
petitioner, for the private respondent to reacquire Philippine citizenship she must comply with the mandatory requirements for repatriation
under Republic Act 8171; and the election of private respondent to public office did not mean the restoration of her Filipino citizenship since the
private respondent was not legally repatriated. Coupled with her alleged renunciation of Australian citizenship, private respondent has effectively
become a stateless person and as such, is disqualified to run for a public office in the Philippines; petitioner concluded.
Petitioner theorizes further that the Commission on Elections erred in applying the principle of res judicata to the case under consideration; citing
the ruling in Moy Ya Lim Yao vs. Commissioner of Immigration, 3 that:
". . . Everytime the citizenship of a person is material or indispensable in a judicial or administrative case, whatever the
corresponding court or administrative authority decides therein as to such citizenship is generally not considered as res
adjudicata, hence it has to be threshed out again and again as the occasion may demand. . . . "
The petition is unmeritorious.
The Philippine law on citizenship adheres to the principle of jus sanguinis. Thereunder, a child follows the nationality or citizenship of the parents
regardless of the place of his/her birth, as opposed to the doctrine of jus soli which determines nationality or citizenship on the basis of place of
birth.
Private respondent Rosalind Ybasco Lopez was born on May 16, 1934 in Napier Terrace, Broome, Western Australia, to the spouses, Telesforo
Ybasco, a Filipino citizen and native of Daet, Camarines Norte, and Theresa Marquez, an Australian. Historically, this was a year before the 1935
Constitution took into effect and at that time, what served as theConstitution of the Philippines were the principal organic acts by which the United
States governed the country. These were the Philippine Bill of July 1, 1902 and the Philippine Autonomy Act of August 29, 1916, also known as the
Jones Law.
Among others, these laws defined who were deemed to be citizens of the Philippine islands. The Philippine Bill of 1902 defined Philippine citizens as:
SEC. 4. . . . all inhabitants of the Philippine Islands continuing to reside therein who were Spanish subjects on the eleventh day of
April, eighteen hundred and ninety-nine, and then resided in the Philippine Islands, and their children born subsequent thereto;
shall be deemed and held to be citizens of the Philippine Islands and as such entitled to the protection of the United States,
except such as shall have elected to preserve their allegiance to the Crown of Spain in accordance with the provisions of the
treaty of peace between the United States and Spain signed at Paris December tenth, eighteen hundred and ninety-eight.
(italics supplied) HSIADc
The Jones Law, on the other hand, provides:
SEC. 2. That all inhabitants of the Philippine Islands who were Spanish subjects on the eleventh day of April, eighteen hundred and ninety-nine, and
then resided in said Islands, and their children born subsequent thereto, shall be deemed and held to be citizens of the Philippine Islands, except such
as shall have elected to preserve their allegiance to the Crown of Spain in accordance with the provisions of the treaty of peace between the United
States and Spain, signed at Paris December tenth, eighteen hundred and ninety-eight, and except such others as have since become citizens of some
other country: Provided, That the Philippine Legislature, herein provided for, is hereby authorized to provide by law for the acquisition of Philippine
citizenship by those natives of the Philippine Islands who cannot come within the foregoing provisions, the natives of the insular possessions of the
United States, and such other persons residing in the Philippine Islands who are citizens of the United States, or who could become citizens of the
United States under the laws of the United States if residing therein. (italics supplied)
Under both organic acts, all inhabitants of the Philippines who were Spanish subjects on April 11, 1899 and resided therein including their children
are deemed to be Philippine citizens. Private respondent's father, Telesforo Ybasco, was born on January 5, 1879 in Daet, Camarines Norte, a fact
duly evidenced by a certified true copy of an entry in the Registry of Births. Thus, under the Philippine Bill of 1902 and the Jones Law, Telesforo
Ybasco was deemed to be a Philippine citizen. By virtue of the same laws, which were the laws in force at the time of her birth, Telesforo's daughter,
herein private respondent Rosalind Ybasco Lopez, is likewise a citizen of the Philippines.
The signing into law of the 1935 Philippine Constitution has established the principle of jus sanguinis as basis for the acquisition of Philippine
citizenship, to wit:
(1) Those who are citizens of the Philippine Islands at the time of the adoption of this Constitution.
(2) Those born in the Philippine Islands of foreign parents who, before the adoption of this Constitution had been elected to
public office in the Philippine Islands.
(3) Those whose fathers are citizens of the Philippines.
(4) Those whose mothers are citizens of the Philippines and, upon reaching the age of majority, elect Philippine citizenship.
(5) Those who are naturalized in accordance with law.
So also, the principle of jus sanguinis, which confers citizenship by virtue of blood relationship, was subsequently retained under
the 1973 4 and 1987 5 Constitutions. Thus, the herein private respondent, Rosalind Ybasco Lopez, is a Filipino citizen, having been born to a Filipino
father. The fact of her being born in Australia is not tantamount to her losing her Philippine citizenship. If Australia follows the principle of jus soli,
then at most, private respondent can also claim Australian citizenship resulting to her possession of dual citizenship.

Petitioner also contends that even on the assumption that the private respondent is a Filipino citizen, she has nonetheless renounced her Philippine
citizenship. To buttress this contention, petitioner cited private respondent's application for an Alien Certificate of Registration (ACR) and Immigrant
Certificate of Residence (ICR), on September 19, 1988, and the issuance to her of an Australian passport on March 3, 1988.
Under Commonwealth Act No. 63, a Filipino citizen may lose his citizenship:
(1) By naturalization in a foreign country;
(2) By express renunciation of citizenship;
(3) By subscribing to an oath of allegiance to support the constitution or laws of a foreign county upon attaining twenty-one
years of age or more;
(4) By accepting commission in the military, naval or air service of a foreign country;
(5) By cancellation of the certificate of naturalization; SEHTIc
(6) By having been declared by competent authority, a deserter of the Philippine armed forces in time of war, unless
subsequently, a plenary pardon or amnesty has been granted; and
(7) In case of a woman, upon her marriage, to a foreigner if, by virtue of the laws in force in her husband's country, she
acquires his nationality.
In order that citizenship may be lost by renunciation, such renunciation must be express. Petitioner's contention that the application of private
respondent for an alien certificate of registration, and her Australian passport, is bereft of merit. This issue was put to rest in the case of Aznar vs.
COMELEC 6 and in the more recent case of Mercado vs. Manzano and COMELEC. 7
In the case of Aznar, the Court ruled that the mere fact that respondent Osmena was a holder of a certificate stating that he is an American did not
mean that he is no longer a Filipino, and that an application for an alien certificate of registration was not tantamount to renunciation of his
Philippine citizenship.
And, in Mercado vs. Manzano and COMELEC, it was held that the fact that respondent Manzano was registered as an American citizen in the Bureau
of Immigration and Deportation and was holding an American passport on April 22, 1997, only a year before he filed a certificate of candidacy for
vice-mayor of Makati, were just assertions of his American nationality before the termination of his American citizenship.
Thus, the mere fact that private respondent Rosalind Ybasco Lopez was a holder of an Australian passport and had an alien certificate of registration
are not acts constituting an effective renunciation of citizenship and do not militate against her claim of Filipino citizenship. For renunciation to
effectively result in the loss of citizenship, the same must be express. 8 As held by this court in the aforecited case of Aznar, an application for an
alien certificate of registration does not amount to an express renunciation or repudiation of one's citizenship. The application of the herein private
respondent for an alien certificate of registration, and her holding of an Australian passport, as in the case of Mercado vs. Manzano, were mere acts
of assertion of her Australian citizenship before she effectively renounced the same. Thus, at the most, private respondent had dual citizenship —
she was an Australian and a Filipino, as well. ISHCcT
Moreover, under Commonwealth Act 63, the fact that a child of Filipino parent/s was born in another country has not been included as a ground for
losing one's Philippine citizenship. Since private respondent did not lose or renounce her Philippine citizenship, petitioner's claim that respondent
must go through the process of repatriation does not hold water.
Petitioner also maintains that even on the assumption that the private respondent had dual citizenship, still, she is disqualified to run for governor of
Davao Oriental; citingSection 40 of Republic Act 7160 otherwise known as the Local Government Code of 1991, which states:
"SEC. 40. Disqualifications. — The following persons are disqualified from running for any elective local position:
xxx xxx xxx
(d) Those with dual citizenship;
xxx xxx xxx
Again, petitioner's contention is untenable.
In the aforecited case of Mercado vs. Manzano, the Court clarified "dual citizenship" as used in the Local Government Code and reconciled the same
with Article IV, Section 5 of the 1987 Constitution on dual allegiance. 9 Recognizing situations in which a Filipino citizen may, without performing any
act, and as an involuntary consequence of the conflicting laws of different countries, be also a citizen of another state, the Court explained that dual
citizenship as a disqualification must refer to citizens with dual allegiance. The Court succinctly pronounced:
". . . the phrase 'dual citizenship' in R.A. No. 7160, ... 40 (d) and in R.A. No. 7854, . . . 20 must be understood as referring to
'dual allegiance'. Consequently, persons with mere dual citizenship do not fall under this disqualification."
Thus, the fact that the private respondent had dual citizenship did not automatically disqualify her from running for a public office. Furthermore, it
was ruled that for candidates with dual citizenship, it is enough that they elect Philippine citizenship upon the filing of their certificate of candidacy,
to terminate their status as persons with dual citizenship. 10The filing of a certificate of candidacy sufficed to renounce foreign citizenship, effectively
removing any disqualification as a dual citizen. 11 This is so because in the certificate of candidacy, one declares that he/she is a Filipino citizen and
that he/she will support and defend the Constitution of the Philippines and will maintain true faith and allegiance thereto. Such declaration, which is
under oath, operates as an effective renunciation of foreign citizenship. Therefore, when the herein private respondent filed her certificate of
candidacy in 1992, such fact alone terminated her Australian citizenship.
Then, too, it is significant to note that on January 15, 1992, private respondent executed a Declaration of Renunciation of Australian Citizenship, duly
registered in the Department of Immigration and Ethnic Affairs of Australia on May 12, 1992. And, as a result, on February 11, 1992, the Australian
passport of private respondent was cancelled, as certified to by Second Secretary Richard F. Munro of the Embassy of Australia in Manila. As aptly
appreciated by the COMELEC, the aforesaid acts were enough to settle the issue of the alleged dual citizenship of Rosalind Ybasco Lopez. Since her
renunciation was effective, petitioner's claim that private respondent must go through the whole process of repatriation holds no water.
Petitioner maintains further that when citizenship is raised as an issue in judicial or administrative proceedings, the resolution or decision thereon is
generally not considered res judicata in any subsequent proceeding challenging the same; citing the case of Moy Ya Lim Yao vs. Commissioner of
Immigration. 12 He insists that the same issue of citizenship may be threshed out anew.
Petitioner is correct insofar as the general rule is concerned, i.e. the principle of res judicata generally does not apply in cases hinging on the issue of
citizenship. However, in the case of Burca vs. Republic, 13 an exception to this general rule was recognized. The Court ruled in that case that in order
that the doctrine of res judicata may be applied in cases of citizenship, the following must be present:
1) a person's citizenship be raised as a material issue in a controversy where said person is a party;
2) the Solicitor General or his authorized representative took active part in the resolution thereof; and
3) the finding on citizenship is affirmed by this Court.
Although the general rule was set forth in the case of Moy Ya Lim Yao, the case did not foreclose the weight of prior rulings on citizenship. It
elucidated that reliance may somehow be placed on these antecedent official findings, though not really binding, to make the effort easier or
simpler. 14 Indeed, there appears sufficient basis to rely on the prior rulings of the Commission on Elections in SPA. No. 95-066 and EPC 92-54 which
resolved the issue of citizenship in favor of the herein private respondent. The evidence adduced by petitioner is substantially the same evidence
presented in these two prior cases. Petitioner failed to show any new evidence or supervening event to warrant a reversal of such prior resolutions.
However, the procedural issue notwithstanding, considered on the merits, the petition cannot prosper.
WHEREFORE, the petition is hereby DISMISSED and the COMELEC Resolutions, dated July 17, 1998 and January 15, 1999, respectively, in SPA No. 98-
336 AFFIRMED. ATHCac
Private respondent Rosalind Ybasco Lopez is hereby adjudged qualified to run for governor of Davao Oriental. No pronouncement as to costs.
SO ORDERED.
Davide, Jr., C.J., Melo, Puno, Vitug, Kapunan, Mendoza, Panganiban, Quisumbing, Pardo, Buena, Gonzaga-Reyes, Ynares-Santiago, and De Leon, Jr.,
JJ., concur.
Bellosillo, J., is abroad, on official business.
||| (Valles v. Commission on Elections, G.R. No. 137000, [August 9, 2000], 392 PHIL 327-342)
EN BANC
[B.M. No. 914 . October 1, 1999.]
RE: APPLICATION FOR ADMISSION TO THE PHILIPPINE BAR
VICENTE D. CHING, applicant.
The Solicitor General for applicant.
SYNOPSIS
Vicente D. Ching is the legitimate son of spouses Tat Ching, a Chinese citizen and Prescila A. Dulay, a Filipino, Ching was born in Francia West, Tubao,
La Union on 11 April 1964. Since birth, he resided in the Philippines. He is also a Certified Public Accountant and a registered voter of Tubao, La
Union. In fact, he was elected as member of the Sangguniang Bayan of Tubao, La Union during the 12 May 1992 synchronized elections. On 17 July
1998, having completed a Bachelor of Laws course at the St. Louis University, Baguio City, he filed an application to take the 1998 Bar Examinations.
He was conditionally admitted to take the Bar Examinations, subject to the condition that he must submit to the Court proof of his Philippine
citizenship. On 5 April 1999, the 1998 Bar Examinations were released and Ching was one of the successful examinees. However, because of the
questionable status of his citizenship, he was not allowed to take his oath and instead, he was required to submit further proof of his
citizenship. In compliance therewith, on 27 July 1999, Ching filed a Manifestation with attached Affidavit of Election of Philippine Citizenship and
Oath of Allegiance dated 15 July 1999. DSAEIT
The Court held that Ching failed to validly elect Philippine citizenship. The span of fourteen (14) years that lapsed from the time he reached the
age of majority until he finally expressed his intention to elect Philippine citizenship was clearly way beyond the contemplation of the
requirement of electing "upon reaching the age of majority." Moreover,Ching had offered no reason why he delayed his election of Philippine
citizenship. The prescribed procedure in electing Philippine citizenship is certainly not a tedious and painstaking process. All that is required of the
elector is to execute an affidavit of election of Philippine citizenship and, thereafter, file the same with the nearest civil registry.Ching's unreasonable
and unexplained delay in making his election cannot be simply glossed over.
SYLLABUS
1. POLITICAL LAW; CONSTITUTIONAL LAW; CITIZENSHIP; CHILDREN WITH ALIEN FATHER AND FILIPINO MOTHER BORN BEFORE JANUARY 17, 1973
MUST ELECT THEIR CITIZENSHIP PURSUANT TO 1935 CONSTITUTION. — When Ching was born in 1964, the governing charter was the 1935
Constitution. Under Article IV, Section 1(3) of the 1935 Constitution, the citizenship of a legitimate child born of a Filipino mother and an alien father
followed the citizenship of the father, unless, upon reaching the age of majority, the child elected Philippine citizenship. This right to elect Philippine
citizenship was recognized in the 1973 Constitution when it provided that "(t)hose who elect Philippine citizenship pursuant to the provisions of the
Constitution of nineteen hundred and thirty-five" are citizens of the Philippines. Likewise, this recognition by the 1973 Constitution was carried over
to the 1987 Constitution which states that "(t)hose born before January 17, 1973 of Filipino mothers, who elect Philippine citizenship upon reaching
the age of majority" are Philippine citizens.
2. ID.; ID.; ID.; 1973 AND 1987 CONSTITUTIONAL PROVISIONS ON ELECTION OF PHILIPPINE CITIZENSHIP HAVE NO CURATIVE EFFECT. — It should be
noted, however, that the 1973 and 1987 Constitutional provisions on the election of Philippine citizenship should not be understood as having a
curative effect on any irregularity in the acquisition ofcitizenship for those covered by the 1935 Constitution. If the citizenship of a person was
subject to challenge under the old charter, it remains subject to challenge under the new charter even if the judicial challenge had not been
commenced before the effectivity of the new Constitution.
3. ID.; ID.; ID.; COMMONWEALTH ACT NO. 625; PRESCRIBES PROCEDURE FOR ELECTION OF CITIZENSHIP. — C.A. No. 625 which was enacted pursuant
to Section 1(3), Article IVof the 1935 Constitution, prescribes the procedure that should be followed in order to make a valid election of Philippine
citizenship. Under Section 1 thereof, legitimate children born of Filipino mothers may elect Philippine citizenship by expressing such intention "in a
statement to be signed and sworn to by the party concerned before any officer authorized to administer oaths, and shall be filed with the nearest
civil registry. The said party shall accompany the aforesaid statement with the oath of allegiance to theConstitution and the Government of the
Philippines."
4. ID.; ID.; ID.; ID.; ID.; ELECTION SHOULD BE MADE WITHIN REASONABLE TIME AFTER ATTAINING AGE OF MAJORITY. — However, the 1935
Constitution and C.A. No. 625 did not prescribe a time period within which the election of Philippine citizenship should be made. The 1935 Charter
only provides that the election should be made "upon reaching the age of majority." The age of majority then commenced upon reaching twenty-one
(21) years. In the opinions of the Secretary of Justice on cases involving the validity ofelection of Philippine citizenship, this dilemma was resolved by
basing the time period on the decisions of this Court prior to the effectivity of the 1935 Constitution. In these decisions, the proper period for
electing Philippine citizenship was, in turn, based on the pronouncements of the Department of State of the United States Government to the effect
that the election should be made within a "reasonable time" after attaining the age of majority.
5. ID.; ID.; ID.; ID.; ID.; ID.; "REASONABLE TIME"; CONSTRUED. — The phrase "reasonable time" has been interpreted to mean that the election
should be made within three (3) years from reaching the age of majority. However, we held in Cuenco vs. Secretary of Justice, that the three (3) year
period is not an inflexible rule. We said: It is true that this clause has been construed to mean a reasonable period after reaching the age of majority,
and that the Secretary of Justice has ruled that three (3) years is the reasonable time to elect Philippine citizenship under the constitutional provision
adverted to above, which period may be extended under certain circumstances, as when the person concerned has always considered himself a
Filipino. However, we cautioned in Cuenco that the extension of the option to elect Philippine citizenship is not indefinite: Regardless of the
foregoing, petitioner was born on February 16, 1923. He became of age on February 16, 1944. His election of citizenship was made on May 15, 1951,
when he was over twenty-eight (28) years of age, or over seven (7) years after he had reached the age of majority. It is clear that said election has
not been made "upon reaching the age of majority."
6. ID.; ID.; ID.; ID.; ID.; ID.; NOT COMPLIED IN CASE AT BAR. — In the present case, Ching, having been born 11 April 1964, was already thirty-five (35)
years old when he complied with the requirements of C.A. No. 625 on 15 June 1999, or over fourteen (14) years after he had reached the
age of majority. Based on the interpretation of the phrase "upon reaching the age of majority," Ching's election was clearly beyond, by any
reasonable yardstick, the allowable period within which to exercise the privilege. It should be stated, in this connection, that the special
circumstances invoked by Ching, i.e., his continuous and uninterrupted stay in the Philippines and his being a certified public accountant, a registered
voter and a former elected public official, cannot vest in him Philippine citizenship as the law specifically lays down the requirements for
acquisition ofPhilippine citizenship by election.
7. ID.; ID.; ID.; ID.; APPLICANT FAILED TO VALIDLY ELECT PHILIPPINE CITIZENSHIP; CASE AT BAR. — Consequently, we hold that Ching failed to validly
elect Philippine citizenship. The span of fourteen (14) years that lapsed from the time he reached the age of majority until he finally expressed his
intention to elect Philippine citizenship is clearly way beyond the contemplation of the requirement of electing "upon reaching the age of majority."
Moreover, Ching has offered no reason why he delayed his election ofPhilippine citizenship. The prescribed procedure in electing Philippine
citizenship is certainly not a tedious and painstaking process. All that is required of the elector is to execute an affidavit of election of Philippine
citizenship and, thereafter, file the same with the nearest civil registry. Ching's unreasonable and unexplained delay in making his election cannot be
simply glossed over. HaAISC
8. ID.; ID.; ID.; PERSON PRIVILEGED TO ELECT PHILIPPINE CITIZENSHIP HAS ONLY AN INCHOATE RIGHT TO SUCH CITIZENSHIP. — Philippine citizenship
can never be treated like a commodity that can be claimed when needed and suppressed when convenient. One who is privileged to elect Philippine
citizenship has only an inchoate right to such citizenship. As such, he should avail of the right with fervor, enthusiasm and promptitude.
RESOLUTION
KAPUNAN, J p:
Can a legitimate child born under the 1935 Constitution of a Filipino mother and an alien father validly elect Philippine citizenship fourteen (14) years
after he has reached the age of majority? This is the question sought to be resolved in the present case involving the application for admission to the
Philippine Bar of Vicente D. Ching. cdlex
The facts of this case are as follows:
Vicente D. Ching, the legitimate son of the spouses Tat Ching, a Chinese citizen, and Prescila A. Dulay, a Filipino, was born in Francia West, Tubao, La
Union on 11 April 1964. Since his birth, Ching has resided in the Philippines.
On 17 July 1998, Ching, after having completed a Bachelor of Laws course at the St. Louis University in Baguio City, filed an application to take the
1998 Bar Examinations. In a Resolution of this Court, dated 1 September 1998, he was allowed to take the Bar Examinations, subject to the condition
that he must submit to the Court proof of his Philippine citizenship.

In compliance with the above resolution, Ching submitted on 18 November 1998, the following documents:
1. Certification, dated 9 June 1986, issued by the Board of Accountancy of the Professional Regulations Commission showing
that Ching is a certified public accountant;
2. Voter Certification, dated 14 June 1997, issued by Elizabeth B. Cerezo, Election Officer of the Commission on Elections
(COMELEC) in Tubao, La Union showing that Ching is a registered voter of the said place; and
3. Certification, dated 12 October 1998, also issued by Elizabeth B. Cerezo, showing that Ching was elected as a member of the
Sangguniang Bayan of Tubao, La Union during the 12 May 1992 synchronized elections. cdphil
On 5 April 1999, the results of the 1998 Bar Examinations were released and Ching was one of the successful Bar examinees. The oath-taking of the
successful Bar examinees was scheduled on 5 May 1999. However, because of the questionable status of Ching's citizenship, he was not allowed to
take his oath. Pursuant to the resolution of this Court, dated 20 April 1999, he was required to submit further proof of his citizenship. In the same
resolution, the Office of the Solicitor General (OSG) was required to file a comment onChing's petition for admission to the bar and on the documents
evidencing his Philippine citizenship.
The OSG filed its comment on 8 July 1999, stating that Ching, being the "legitimate child of a Chinese father and a Filipino mother born under
the 1935 Constitution was a Chinese citizen and continued to be so, unless upon reaching the age of majority he elected Philippine
citizenship" 1 in strict compliance with the provisions of Commonwealth Act No. 625 entitled "An Act Providing for the Manner in which the Option
to Elect Philippine Citizenship shall be Declared by a Person Whose Mother is a Filipino Citizen." The OSG adds that "(w)hat he acquired at best was
only an inchoate Philippine citizenship which he could perfect by election upon reaching the age of majority." 2 In this regard, the OSG clarifies that
"two (2) conditions must concur in order that the election of Philippine citizenship may be effective, namely: (a) the mother of the person making the
election must be a citizen of the Philippines; and (b) said election must be made 'upon reaching the age of majority.'" 3 The OSG then explains the
meaning of the phrase "upon reaching the age of majority:"
The clause "upon reaching the age of majority" has been construed to mean a reasonable time after reaching the
age of majority which had been interpreted by the Secretary ofJustice to be three (3) years (VELAYO, supra at p. 51 citing Op.,
Sec. of Justice No. 70, s. 1940, Feb. 27, 1940). Said period may be extended under certain circumstances, as when a (sic) person
concerned has always considered himself a Filipino (ibid., citing Op. Nos. 355 and 422, s. 1955; 3, 12, 46, 86 and 97, s. 1953).
But in Cuenco, it was held that an election done after over seven (7) years was not made within a reasonable time.
In conclusion, the OSG points out that Ching has not formally elected Philippine citizenship and, if ever he does, it would already be beyond the
"reasonable time" allowed by present jurisprudence. However, due to the peculiar circumstances surrounding Ching's case, the OSG recommends
the relaxation of the standing rule on the construction of the phrase "reasonable period" and the allowance of Ching to elect Philippine
citizenship in accordance with C.A. No. 625 prior to taking his oath as a member of the Philippine Bar. llcd
On 27 July 1999, Ching filed a Manifestation, attaching therewith his Affidavit of Election of Philippine Citizenship and his Oath of Allegiance, both
dated 15 July 1999. In his Manifestation, Ching states:
1. I have always considered myself as a Filipino;
2. I was registered as a Filipino and consistently declared myself as one in my school records and other official documents;
3. I am practicing a profession (Certified Public Accountant) reserved for Filipino citizens;
4. I participated in electoral process[es] since the time I was eligible to vote;
5. I had served the people of Tubao, La Union as a member of the Sangguniang Bayan from 1992 to 1995;
6. I elected Philippine citizenship on July 15, 1999 in accordance with Commonwealth Act No. 625;
7. My election was expressed in a statement signed and sworn to by me before a notary public;
8. I accompanied my election of Philippine citizenship with the oath of allegiance to the Constitution and the
Government of the Philippines; LexLib
9. I filed my election of Philippine citizenship and my oath of allegiance to (sic) the Civil Registrar of Tubao La Union, and
10. I paid the amount of TEN PESOS (Ps. 10.00) as filing fees.
Since Ching has already elected Philippine citizenship on 15 July 1999, the question raised is whether he has elected Philippine citizenship within a
"reasonable time." In the affirmative, whether his citizenship by election retroacted to the time he took the bar examination.
When Ching was born in 1964, the governing charter was the 1935 Constitution. Under Article IV, Section 1(3) of the 1935 Constitution, the
citizenship of a legitimate child born ofa Filipino mother and an alien father followed the citizenship of the father, unless, upon reaching the
age of majority, the child elected Philippine citizenship. 4 This right to elect Philippine citizenship was recognized in the 1973 Constitution when it
provided that "(t)hose who elect Philippine citizenship pursuant to the provisions of the Constitution ofnineteen hundred and thirty-five" are
citizens of the Philippines. 5 Likewise, this recognition by the 1973 Constitution was carried over to the 1987 Constitution which states that "(t)hose
born before January 17, 1973 of Filipino mothers, who elect Philippine citizenship upon reaching the age of majority" are Philippine citizens. 6 It
should be noted, however, that the 1973 and 1987 Constitutional provisions on the election of Philippine citizenship should not be understood as
having a curative effect on any irregularity in the acquisition of citizenship for those covered by the 1935 Constitution. 7 If the citizenship of a person
was subject to challenge under the old charter, it remains subject to challenge under the new charter even if the judicial challenge had not been
commenced before the effectivity of the new Constitution. 8
C.A. No. 625 which was enacted pursuant to Section 1(3), Article IV of the 1935 Constitution, prescribes the procedure that should be
followed in order to make a valid election ofPhilippine citizenship. Under Section 1 thereof, legitimate children born of Filipino mothers may elect
Philippine citizenship by expressing such intention "in a statement to be signed and sworn to by the party concerned before any officer authorized to
administer oaths, and shall be filed with the nearest civil registry. The said party shall accompany the aforesaid statement with the oath of allegiance
to the Constitution and the Government of the Philippines." cdasia
However, the 1935 Constitution and C.A. No. 625 did not prescribe a time period within which the election of Philippine citizenship should be made.
The 1935 Charter only provides that the election should be made "upon reaching the age of majority." The age of majority then commenced upon
reaching twenty-one (21) years. 9 In the opinions ofthe Secretary of Justice on cases involving the validity of election of Philippine citizenship, this
dilemma was resolved by basing the time period on the decisions of this Court prior to the effectivity of the 1935 Constitution. In these decisions, the
proper period for electing Philippine citizenship was, in turn, based on the pronouncements of the Departmentof State of the United States
Government to the effect that the election should be made within a "reasonable time" after attaining the age of majority. 10 The phrase "reasonable
time" has been interpreted to mean that the election should be made within three (3) years from reaching the age of majority. 11 However, we
held in Cuenco vs. Secretary ofJustice, 12 that the three (3) year period is not an inflexible rule. We said:
It is true that this clause has been construed to mean a reasonable period after reaching the age of majority, and that the
Secretary of Justice has ruled that three (3) years is the reasonable time to elect Philippine citizenship under the constitutional
provision adverted to above, which period may be extended under certain circumstances, as when the person concerned has
always considered himself a Filipino. 13
However, we cautioned in Cuenco that the extension of the option to elect Philippine citizenship is not indefinite:
Regardless of the foregoing, petitioner was born on February 16, 1923. He became of age on February 16, 1944. His
election of citizenship was made on May 15, 1951, when he was over twenty-eight (28) years of age, or over seven (7) years
after he had reached the age of majority. It is clear that said election has not been made "upon reaching the
age ofmajority." 14
In the present case, Ching, having been born on 11 April 1964, was already thirty-five (35) years old when he complied with the requirements of C.A.
No. 625 on 15 June 1999, or over fourteen (14) years after he had reached the age of majority. Based on the interpretation of the phrase "upon
reaching the age of majority," Ching's election was clearly beyond, by any reasonable yardstick, the allowable period within which to exercise the
privilege. It should be stated, in this connection, that the special circumstances invoked byChing, i.e., his continuous and uninterrupted stay in the
Philippines and his being a certified public accountant, a registered voter and a former elected public official, cannot vestin him Philippine citizenship
as the law specifically lays down the requirements for acquisition of Philippine citizenship by election. cda

Definitely, the so-called special circumstances cannot constitute what Ching erroneously labels as informal election of citizenship. Ching cannot find a
refuge in the case of In re:Florencio Mallare, 15 the pertinent portion of which reads:
And even assuming arguendo that Ana Mallare were (sic) legally married to an alien, Esteban's exercise of the right of suffrage
when he came of age, constitutes a positive act ofelection of Philippine citizenship. It has been established that Esteban
Mallare was a registered voter as of April 14, 1928, and that as early as 1925 (when he was about 22 years old), Esteban was
already participating in the elections and campaigning for certain candidate[s]. These acts are sufficient to show his preference
for Philippine citizenship. 16
Ching's reliance on Mallare is misplaced. The facts and circumstances obtaining therein are very different from those in the present case, thus,
negating its applicability. First, Esteban Mallare was born before the effectivity of the 1935 Constitution and the enactment of C.A. No. 625. Hence,
the requirements and procedures prescribed under the 1935 Constitution and C.A. No. 625 for electing Philippine citizenship would not be applicable
to him. Second, the ruling in Mallare was an obiter since, as correctly pointed out by the OSG, it was not necessary for Esteban Mallare to elect
Philippine citizenship because he was already a Filipino, he being a natural child of a Filipino mother. In this regard, the Court stated:
Esteban Mallare, natural child of Ana Mallare, a Filipina, is therefore himself a Filipino, and no other act would be necessary to
confer on him all the rights and privileges attached to Philippine citizenship (U.S. vs. Ong Tianse, 29 Phil. 332; Santos Co vs.
Government of the Philippine Islands, 42 Phil. 543, Serra vs. Republic, L-4223, May 12, 1952, Sy Quimsuan vs. Republic, L-4693,
Feb. 16, 1953; Pitallano vs. Republic, L-5111, June 28, 1954). Neither could any act be taken on the erroneous belief that he is a
non-Filipino divest him of the citizenship privileges to which he is rightfully entitled. 17
The ruling in Mallare was reiterated and further elaborated in Co vs. Electoral Tribunal of the House of Representatives, 18 where we held:
We have jurisprudence that defines 'election' as both a formal and an informal process.
In the case of In re: Florencio Mallare (59 SCRA 45 [1974]), the Court held that the exercise of the right of suffrage and the
participation in election exercises constitute a positive act of election of Philippine citizenship. In the exact
pronouncement of the Court, we held:
"Esteban's exercise of the right of suffrage when he came of age, constitutes a positive act of Philippine citizenship". (p. 52;
emphasis supplied)"
The private respondent did more than merely exercise his right of suffrage. He has established his life here in the Philippines.
For those in the peculiar situation of the respondent who cannot be expected to have elected Philippine citizenship as they
were already citizens, we apply the In Re Mallare rule.
xxx xxx xxx
The filing of sworn statement or formal declaration is a requirement for those who still have to elect citizenship. For those
already Filipinos when the time to elect came up, there are acts of deliberate choice which cannot be less binding. Entering a
profession open only to Filipinos, serving in public office where citizenship is a qualification, voting during election time,
running for public office, and other categorical acts of similar nature are themselves formal manifestations for these persons.
An election of Philippine citizenship presupposes that the person electing is an alien. Or his status is doubtful because he is a
national of two countries. There is no doubt in this case about Mr. Ong's being a Filipino when he turned twenty-one (21). cdpr
We repeat that any election of Philippine citizenship on the part of the private respondent would not only have been
superfluous but it would also have resulted in an absurdity. How can a Filipino citizen elect Philippine citizenship? 19
The Court, like the OSG, is sympathetic with the plight of Ching. However, even if we consider the special circumstances in the life of Ching like his
having lived in the Philippines all his life and his consistent belief that he is a Filipino, controlling statutes and jurisprudence constrain us to disagree
with the recommendation of the OSG. Consequently, we hold that Ching failed to validly elect Philippine citizenship. The span of fourteen (14) years
that lapsed from the time he reached the age of majority until he finally expressed his intention to elect Philippine citizenship is clearly way beyond
the contemplation of the requirement of electing "upon reaching the age of majority." Moreover, Ching has offered no reason why he delayed his
election of Philippine citizenship. The prescribed procedure in electing Philippine citizenship is certainly not a tedious and painstaking process. All
that is required of the elector is to execute an affidavit of election of Philippine citizenship and, thereafter, file the same with the nearest civil
registry. Ching's unreasonable and unexplained delay in making his election cannot be simply glossed over.
Philippine citizenship can never be treated like a commodity that can be claimed when needed and suppressed when convenient. 20 One who is
privileged to elect Philippine citizenship has only an inchoate right to such citizenship. As such, he should avail of the right with fervor, enthusiasm
and promptitude. Sadly, in this case, Ching slept on his opportunity to elect Philippine citizenship and, as a result, this golden privilege slipped away
from his grasp. LLjur
IN VIEW OF THE FOREGOING, the Court Resolves to DENY Vicente D. Ching's application for admission to the Philippine Bar.
SO ORDERED.
Davide, Jr., C.J., Bellosillo, Melo, Puno, Vitug, Mendoza, Panganiban, Quisumbing, Purisima, Pardo, Buena, Gonzaga-Reyes and Ynares-Santiago,
JJ., concur.
||| (Re: Vicente D. Ching, B.M. No. 914 (Resolution), [October 1, 1999], 374 PHIL 342-355)
EN BANC
[G.R. No. 142840. May 7, 2001.]
ANTONIO BENGSON III, petitioner, vs. HOUSE OF REPRESENTATIVES ELECTORAL TRIBUNAL and TEODORO C.
CRUZ, respondents.
DECISION
KAPUNAN, J p:
The citizenship of respondent Teodoro C. Cruz is at issue in this case, in view of the constitutional requirement that "no person shall be a Member of
the House of Representatives unless he is a natural-born citizen." 1
Respondent Cruz was a natural-born citizen of the Philippines. He was born in San Clemente, Tarlac, on April 27, 1960, of Filipino parents. The
fundamental law then applicable was the 1935 Constitution. 2
On November 5, 1985, however, respondent Cruz enlisted in the United States Marine Corps and, without the consent of the Republic of the
Philippines, took an oath of allegiance to the United States. As a consequence, he lost his Filipino citizenship for under Commonwealth Act No. 63,
Section 1(4), a Filipino citizen may lose his citizenship by, among others, "rendering service to or accepting commission in the armed forces of a
foreign country." Said provision of law reads:
SECTION 1. How citizenship may be lost. — A Filipino citizen may lose his citizenship in any of the following ways and/or
events:
xxx xxx xxx
(4) By rendering services to, or accepting commission in, the armed forces of a foreign country: Provided, That the rendering of
service to, or the acceptance of such commission in, the armed forces of a foreign country, and the taking of an oath of
allegiance incident thereto, with the consent of the Republic of the Philippines, shall not divest a Filipino of his Philippine
citizenship if either of the following circumstances is present:
(a) The Republic of the Philippines has a defensive and/or offensive pact of alliance with said foreign country; or
(b) The said foreign country maintains armed forces on Philippine territory with the consent of the Republic of the
Philippines: Provided, That the Filipino citizen concerned, at the time of rendering said service, or acceptance of said
commission, and taking the oath of allegiance incident thereto, states that he does so only in connection with his service to
said foreign country; And provided, finally, That any Filipino citizen who is rendering service to, or is commissioned in, the
armed forces of a foreign country under any of the circumstances mentioned in paragraph (a) or (b), shall not be permitted to
participate nor vote in any election of the Republic of the Philippines during the period of his service to, or commission in, the
armed forces of said country. Upon his discharge from the service of the said foreign country, he shall be automatically entitled
to the full enjoyment of his civil and political rights as a Filipino citizen . . . .
Whatever doubt that remained regarding his loss of Philippine citizenship was erased by his naturalization as a U.S. citizen on June 5, 1990, in
connection with his service in the U.S. Marine Corps.
On March 17, 1994, respondent Cruz reacquired his Philippine citizenship through repatriation under Republic Act No. 2630. 3 He ran for and was
elected as the Representative of the Second District of Pangasinan in the May 11, 1998 elections. He won by a convincing margin of 26,671 votes
over petitioner Antonio Bengson III, who was then running for reelection.
Subsequently, petitioner filed a case for Quo Warranto Ad Cautelam with respondent House of Representatives Electoral Tribunal (HRET) claiming
that respondent Cruz was not qualified to become a member of the House of Representatives since he is not a natural-born citizen as required under
Article VI, Section 6 of the Constitution. 4
On March 2, 2000, the HRET rendered its decision 5 dismissing the petition, for quo warranto and declaring respondent Cruz the duly elected
Representative of the Second District of Pangasinan in the May 1998 elections. The HRET likewise denied petitioner's motion for reconsideration of
the decision in its resolution dated April 27, 2000. 6
Petitioner thus filed the present petition for certiorari assailing the HRET's decision on the following grounds:
1. The HRET committed serious errors and grave abuse of discretion, amounting to excess of jurisdiction, when it ruled that
private respondent is a natural-born citizen of the Philippines despite the fact that he had ceased being such in view of the loss
and renunciation of such citizenship on his part. SDEHCc
2. The HRET committed serious errors and grave abuse of discretion, amounting to excess of jurisdiction, when it considered
private respondent as a citizen of the Philippinesdespite the fact that he did not validly acquire his Philippine citizenship.
3. Assuming that private respondent's acquisition of Philippine citizenship was invalid, the HRET committed serious errors and
grave abuse of discretion, amounting to excess of jurisdiction, when it dismissed the petition despite the fact that such
reacquisition could not legally and constitutionally restore his natural-born status. 7
The issue now before us is whether respondent Cruz, a natural-born Filipino who became an American citizen, can still be considered a natural-born
Filipino upon his reacquisition of Philippine citizenship.
Petitioner asserts that respondent Cruz may no longer be considered a natural-born Filipino since he lost his Philippine citizenship when he swore
allegiance to the United States in 1995, and had to reacquire the same by repatriation. He insists that Article IV, Section 2 of the
Constitution expressly states that natural-born citizens are those who are citizens from birth without having to perform any act to acquire or perfect
such citizenship.
Respondent on the other hand contends that he reacquired his status as a natural-born citizen when he was repatriated since the phrase "from
birth" in Article IV, Section 2 refers to the innate, inherent and inborn characteristic of being a natural-born citizen.
The petition is without merit.
The 1987 Constitution enumerates who are Filipino citizens as follows:
(1) Those who are citizens of the Philippines at the time of the adoption of this Constitution;
(2) Those whose fathers or mothers are citizens of the Philippines;
(3) Those born before January 17, 1973 of Filipino mothers, who elect Philippine citizenship upon reaching the age of majority,
and
(4) Those who are naturalized in accordance with law. 8
There are two ways of acquiring citizenship: (1) by birth, and (2) by naturalization. These ways of acquiring citizenship correspond to the two kinds of
citizens: the natural-born citizen, and the naturalized citizen. A person who at the time of his birth is a citizen of a particular country, is a natural-born
citizen thereof. 9
As defined in the same Constitution,natural-born citizens "are those citizens of the Philippines from birth without having to perform any act to
acquire or perfect his Philippine citizenship." 10
On the other hand, naturalized citizens are those who have become Filipino citizens through naturalization, generally under Commonwealth Act No.
473, otherwise known as theRevised Naturalization Law, which repealed the former Naturalization Law (Act No. 2927), and by Republic Act No.
530. 11 To be naturalized, an applicant has to prove that he possesses all the qualifications 12 and none of the disqualifications 13 provided by law to
become a Filipino citizen. The decision granting Philippine citizenship becomes executory only after two (2) years from its promulgation when the
court is satisfied that during the intervening period, the applicant has (1) not left the Philippines; (2) has dedicated himself to a lawful calling or
profession; (3) has not been convicted of any offense or violation of Government promulgated rules; or (4) committed any act prejudicial to the
interest of the nation or contrary to any Government announced policies. 14
Filipino citizens who have lost their citizenship may however reacquire the same in the manner provided by law. Commonwealth Act No. 63 (CA No.
63), enumerates the three modes by which Philippine citizenship may be reacquired by a former citizen: (1) by naturalization, (2) by repatriation, and
(3) by direct act of Congress. 15
Naturalization is a mode for both acquisition and reacquisition of Philippine citizenship. As a mode of initially acquiring Philippine citizenship,
naturalization is governed byCommonwealth Act No. 473, as amended. On the other hand, naturalization as a mode for reacquiring Philippine
citizenship is governed by Commonwealth Act No. 63. 16 Under this law, a former Filipino citizen who wishes to reacquire Philippine citizenship must
possess certain qualifications 17 and none of the disqualifications mentioned in Section 4 ofC.A. 473. 18
Repatriation, on the other hand, may be had under various statutes by those who lost their citizenship due to: (1) desertion of the armed
forces; 19 (2) service in the armed forces of the allied forces in World War II; 20 (3) service in the Armed Forces of the United States at any other
time; 21 (4) marriage of a Filipino woman to an alien; 22 and (5) political and economic necessity. 23
As distinguished from the lengthy process of naturalization, repatriation simply consists of the taking of an oath of allegiance to the Republic of the
Philippines and registering said oath in the Local Civil Registry of the place where the person concerned resides or last resided.
In Angara v. Republic, 24 we held:
. . . . Parenthetically, under these statutes [referring to RA Nos. 965 and 2630], the person desiring to reacquire Philippine
citizenship would not even be required to file a petition in court, and all that he had to do was to take an oath of allegiance to
the Republic of the Philippines and to register that fact with the civil registry in the place of his residence or where he had last
resided in the Philippines. [Emphasis in the original.] 25

Moreover, repatriation results in the recovery of the original nationality. 26 This means that a naturalized Filipino who lost his citizenship will be
restored to his prior status as a naturalized Filipino citizen. On the other hand, if he was originally a natural-born citizen before he lost his Philippine
citizenship, he will be restored to his former status as a natural-born Filipino.
In respondent Cruz's case, he lost his Filipino citizenship when he rendered service in the Armed Forces of the United States. However, he
subsequently reacquired Philippine citizenship under R.A. No. 2630, which provides:
SECTION 1. Any person who had lost his Philippine citizenship by rendering service to, or accepting commission in, the Armed
Forces of the United States, or after separation from the Armed Forces of the United States, acquired United States citizenship,
may reacquire Philippine citizenship by taking an oath of allegiance to the Republic of the Philippines and registering the same
with Local Civil Registry in the place where he resides or last resided in the Philippines. The said oath of allegiance shall contain
a renunciation of any other citizenship.
Having thus taken the required oath of allegiance to the Republic and having registered the same in the Civil Registry of Magantarem, Pangasinan in
accordance with the aforecited provision, respondent Cruz is deemed to have recovered his original status as a natural-born citizen, a status which
he acquired at birth as the son of a Filipino father.27 It bears stressing that the act of repatriation allows him to recover, or return to, his original
status before he lost his Philippine citizenship.
Petitioner's contention that respondent Cruz is no longer a natural-born citizen since he had to perform an act to regain his citizenship is
untenable. As correctly explained by the HRET in its decision, the term "natural-born citizen" was first defined in Article III, Section 4 of the 1973
Constitution as follows:
SECTION 4. A natural-born citizen is one who is a citizen of the Philippines from birth without having to perform any act to
acquire or perfect his Philippine citizenship.
Two requisites must concur for a person to be considered as such: (1) a person must be a Filipino citizen from birth and (2) he does not have to
perform any act to obtain or perfect his Philippine citizenship.
Under the 1973 Constitution definition, there were two categories of Filipino citizens which were not considered natural-born: (1) those who were
naturalized and (2) those born before January 17, 1973, 28 of Filipino mothers who, upon reaching the age of majority, elected Philippine citizenship.
Those "naturalized citizens" were not considered natural-born obviously because they were not Filipinos at birth and had to perform an act to
acquire Philippine citizenship. Those born of Filipino mothers before the effectivity of the1973 Constitution were likewise not considered natural-
born because they also had to perform an act to perfect their Philippine citizenship.
The present Constitution, however, now considers those born of Filipino mothers before the effectivity of the 1973 Constitution and who elected
Philippine citizenship upon reaching the majority age as natural-born. After defining who are natural-born citizens, Section 2 of Article IV adds a
sentence: "Those who elect Philippine citizenship in accordance with paragraph (3), Section 1 hereof shall be deemed natural-born citizens."
Consequently, only naturalized Filipinos are considered not natural-born citizens. It is apparent from the enumeration of who are citizens under
the present Constitution that there are only two classes of citizens: (1) those who are natural-born and (2) those who are naturalized in accordance
with law. A citizen who is not a naturalized Filipino, i.e., did not have to undergo the process of naturalization to obtain Philippine citizenship,
necessarily is a natural-born Filipino. Noteworthy is the absence in said enumeration of a separate category for persons who, after losing Philippine
citizenship, subsequently reacquire it. The reason therefor is clear: as to such persons, they would either be natural-born or naturalized depending
on the reasons for the loss of their citizenship and the mode prescribed by the applicable law for the reacquisition thereof. As respondent Cruz was
not required by law to go through naturalization proceedings in order to reacquire his citizenship, he is perforce a natural-born Filipino. As such, he
possessed all the necessary qualifications to be elected as member of the House of Representatives.
A final point. The HRET has been empowered by the Constitution to be the "sole judge" of all contests relating to the election, returns, and
qualifications of the members of the House. 29 The Court's jurisdiction over the HRET is merely to check "whether or not there has been a grave
abuse of discretion amounting to lack or excess of jurisdiction" on the part of the latter. 30 In the absence thereof, there is no occasion for the Court
to exercise its corrective power and annul the decision of the HRET nor to substitute the Court's judgment for that of the latter for the simple reason
that it is not the office of a petition for certiorari to inquire into the correctness of the assailed decision. 31 There is no such showing of grave abuse
of discretion in this case.
WHEREFORE, the petition is hereby DISMISSED.
Davide, Jr., C.J., Bellosillo and Puno, JJ., concur.
Melo, J., took no part. Chairman of the HRET which rendered the decision under review.
Vitug, J.,took no part. A member of the HRET which rendered the appealed judgment.
Mendoza, J., took no part, being ponente of decision under review.
Panganiban, J., please see concurring opinion.
Quisumbing, Buena and De Leon, Jr., JJ., are on leave.
Pardo, J., concurs on this and the concurring opinion of J. Panganiban.
Gonzaga-Reyes, J., also joins concurring opinion of J. Panganiban.
Ynares-Santiago, J., hereby certifies that J. Santiago joins with the majority opinion of J. Kapunan.
Gutierrez, J., Please see dissenting opinion.
||| (Bengson III v. House of Representatives Electoral Tribunal, G.R. No. 142840, [May 7, 2001], 409 PHIL 633-672)
EN BANC
[G.R. No. 135083. May 26, 1999.]
ERNESTO S. MERCADO, petitioner, vs. EDUARDO BARRIOS MANZANO and the COMMISSION ON ELECTIONS, respondents.
Balase, Tamase, Alampay Law Office for petitioner.
Siguion Reyna, Montecillo & Ongsiako for private respondent.
SYNOPSIS
Petitioner Mercado and private respondent Manzano were candidates for vice mayor of the City of Makati in the May 11, 1998 elections. The
proclamation of private respondent was suspended in view of a pending petition for disqualification filed by a certain Ernesto Mamaril who alleged
that private respondent was not a citizen of the Philippines but of the United States. The Second Division of the COMELEC granted the petition of
Mamaril and ordered the cancellation of the certificate of candidacy of private respondent on the ground that he is a dual citizen and under Sec. 40
of the Local Government Code, persons with dual citizenship are disqualified from running for any elective position. Private respondent filed a
motion for reconsideration. The motion remained pending until after the election. The board of canvassers tabulated the votes but suspended the
proclamation of the winner. Petitioner sought to intervene in the case for disqualification. COMELEC en banc reversed the decision and declared
private respondent qualified to run for the position. Pursuant to the ruling of the COMELEC en banc, the board of canvassers proclaimed private
respondent as vice mayor. This petition sought the reversal of the resolution of the COMELEC en banc and to declare the private respondent
disqualified to hold the office of the vice mayor of Makati. cdasia
On the issue of whether the petitioner has personality to bring this suit considering that he was not the original party in the disqualification case, the
Supreme Court ruled that under Sec. 6 of R.A. No. 6646, otherwise known as the Electoral Reforms Law of 1987, intervention may be allowed in
proceedings for disqualification even after election if there has yet been no final judgment rendered. As regards the issue of citizenship, the Court
ruled that by filing a certificate of candidacy when he ran for his present post, private respondent elected Philippine citizenship and in effect
renounced his American citizenship.
SYLLABUS
1. POLITICAL LAW; ELECTORAL REFORMS LAW OF 1987 (R.A. No. 6646); INTERVENTION, ALLOWED IN PROCEEDINGS FOR DISQUALIFICATION EVEN
AFTER ELECTION IF THERE HAS BEEN NO FINAL JUDGMENT RENDERED; CASE AT BAR. — Private respondent argues that petitioner has neither legal
interest in the matter in litigation nor an interest to protect because he is "a defeated candidate for the vice-mayoralty post of Makati City [who]
cannot be proclaimed as the Vice-Mayor of Makati City even if the private respondent be ultimately disqualified by final and executory judgment."
The flaw in this argument is it assumes that, at the time petitioner sought to intervene in the proceedings before the COMELEC, there had already
been a proclamation of the results of the election for the vice mayoralty contest for Makati City, on the basis of which petitioner came out only
second to private respondent. The fact, however, is that there had been no proclamation at that time. Certainly, petitioner had, and still has, an
interest in ousting private respondent from the race at the time he sought to intervene. The rule in Labo vs. COMELEC, reiterated in several cases,
only applies to cases in which the election of the respondent is contested, and the question is whether one who placed second to the disqualified
candidate may be declared the winner. In the present case, at the time petitioner filed a "Motion for leave to File Intervention" on May 20, 1998,
there had been no proclamation of the winner, and petitioner's purpose was precisely to have private respondent disqualified "from running for [an]
elective local position" under Section 40(d) of R.A. No. 7160. If Ernesto Mamaril (who originally instituted the disqualification proceedings), a
registered voter of Makati City, was competent to bring the action, so was petitioner since the latter was a rival candidate for vice mayor of Makati
City. Nor is petitioner's interest in the matter in litigation any less because he filed a motion for intervention only on May 20, 1998, after private
respondent had been shown to have garnered the highest number of votes among the candidates for vice mayor. That petitioner had a right to
intervene at that stage of the proceedings for the disqualification against private respondent is clear from Section 6 of R.A. No. 6646, otherwise
known as the Electoral Reforms Law of 1987, which provides: Any candidate who has been declared by final judgment to be disqualified shall not be
voted for, and the votes cast for him shall not be counted. If for any reason a candidate is not declared by final judgment before an election to be
disqualified and he is voted for and receives the winning number of votes in such election, the Court or Commission shall continue with the trial and
hearing of the action, inquiry, or protest and, upon motion of the complainant or any intervenor, may during the pendency thereof order the
suspension of the proclamation of such candidate whenever the evidence of guilt is strong. Under this provision, intervention may be allowed in
proceedings for disqualification even after election if there has yet been no final judgment rendered.
2. ID.; CITIZENSHIP; DUAL CITIZENSHIP; DISTINGUISHED FROM DUAL ALLEGIANCE. — Dual citizenship is different from dual allegiance. The former
arises when, as a result of the concurrent application of the different laws of two or more states, a person is simultaneously considered a national by
the said states. For instance, such a situation may arise when a person whose parents are citizens of a state which adheres to the principle of jus
sanguinis is born in a state which follows the doctrine of jus soli. Such a person,ipso facto and without any voluntary act on his part, is concurrently
considered a citizen of both states. Considering the citizenship clause (Art. IV) of our Constitution,it is possible for the following classes of citizens of
the Philippines to posses dual citizenship: (1) Those born of Filipino fathers and/or mothers in foreign countries which follow the principle of jus soli;
(2) Those born in the Philippines of Filipino mothers and alien fathers if by the laws of their fathers' country such children are citizens of that country;
(3) Those who marry aliens if by the laws of the latter's country the former are considered citizens, unless by their act or omission they are deemed
to have renounced Philippine citizenship. There may be other situations in which a citizen of the Philippines may, without performing any act, be also
a citizen of another state; but the above cases are possible given the constitutional provisions on citizenship. Dual allegiance, on the other hand,
refers to the situation in which a person simultaneously owes, by some positive act, loyalty to two or more states. While dual citizenship is
involuntary, dual allegiance is the result of an individual's volition. With respect to dual allegiance, Article IV, Section 5 of the Constitutionprovides:
"Dual allegiance of citizens is inimical to the national interest and shall be dealt with by law."
3. ID.; ID.; ID.; ID.; RATIONALE. — In including Section 5 in Article IV on citizenship, the concern of the Constitutional Commission was not with dual
citizens per se but with naturalized citizens who maintain their allegiance to their countries of origin even after their naturalization. Hence, the
phrase "dual citizenship" in R.A. No. 7160, Section 40(d) and in R.A. No. 7854, Section 20 must be understood as referring to "dual allegiance."
Consequently, persons with mere dual citizenship do not fall under this disqualification. Unlike those with dual allegiance, who must, therefore, be
subject to strict process with respect to the termination of their status, for candidates with dual citizenship, it should suffice if, upon the filing of their
certificates of candidacy, they elect Philippine citizenship to terminate their status as persons with dual citizenship considering that their condition is
the unavoidable consequence of conflicting laws of different states. As Joaquin G. Bernas, one of the most perceptive members of the Constitutional
Commission, pointed out: "[D]ual citizenship is just a reality imposed on us because we have no control of the laws on citizenship of other countries.
We recognize a child of a Filipino mother. But whether or not she is considered a citizen of another country is something completely beyond our
control." By electing Philippine citizenship, such candidates at the same time forswear allegiance to the other country of which they are also citizens
and thereby terminate their status as dual citizens. It may be that, from the point of view of the foreign state and of its laws, such an individual has
not effectively renounced his foreign citizenship.
4. ID.; ID.; FILING OF THE CERTIFICATE OF CANDIDACY SUFFICED TO RENOUNCE AMERICAN CITIZENSHIP; CASE AT BAR. — By filing a certificate of
candidacy when he ran for his present post, private respondent elected Philippine citizenship and in effect renounced his American citizenship. The
filing of such certificate of candidacy sufficed to renounce his American citizenship, effectively removing any disqualification he might have as a dual
citizen. Thus, in Frivaldo vs. COMELEC it was held: It is not disputed that on January 20, 1983 Frivaldo became an American. Would the retroactivity of
his repatriation not effectively give him dual citizenship, which under Sec. 40 of the Local Government Code would disqualify him "from running for
any elective local position?" We answer this question in the negative, as there is cogent reason to hold that Frivaldo was really STATELESS at the time
he took said oath of allegiance and even before that, when he ran for governor in 1988. In his Comment, Frivaldo wrote that he "had long renounced
and had long abandoned his American citizenship — long before May 8, 1995. At best, Frivaldo was stateless in the interim — when he abandoned
and renounced his US citizenship but before he was repatriated to his Filipino citizenship." On this point, we quote from the assailed Resolution
dated December 19, 1995: "By the laws of the United States, petitioner Frivaldo lost his American citizenship when he took his oath of allegiance to
the Philippine Government when he ran for Governor in 1988, in 1992, and in 1995. Every certificate of candidacy contains an oath of allegiance to
the Philippine Government." These factual findings that Frivaldo has lost his foreign nationality long before the elections of 1995 have not been
effectively rebutted by Lee. Furthermore, it is basic that such findings of the Commission are conclusive upon this Court, absent any showing of
capriciousness or arbitrariness or abuse. Until the filing of his certificate of candidacy on March 21, 1998, private respondent had dual citizenship.
The acts attributed to him can be considered simply as the assertion of his American nationality before the termination of his American citizenship.
What this Court said in Aznar vs. COMELEC applies mutatis mutandis to private respondent in the case at bar: . . . Considering the fact that admittedly
Osmeña was both a Filipino and an American, the mere fact that he has a Certificate stating he is an American does not mean that he is not still a
Filipino. . . [T]he Certification that he is an American does not mean that he is not still a Filipino, possessed as he is, of both nationalities or
citizenships. Indeed, there is no express renunciation here of Philippine citizenship; truth to tell, there is even no implied renunciation of said
citizenship. When We consider that the renunciation needed to lose Philippine citizenship must be "express," it stands to reason that there can be no
such loss of Philippine citizenship when there is no renunciation, either "express" or "implied." To recapitulate, by declaring in his certificate of
candidacy that he is a Filipino citizen; that he is not a permanent resident or immigrant of another country; that he will defend and support
the Constitution of the Philippines and bear true faith and allegiance thereto and that he does so without mental reservation, private respondent
has, as far as the laws of this country are concerned, effectively repudiated his American citizenship and anything which he may have said before as a
dual citizen. On the other hand, private respondent's oath of allegiance to the Philippines, when considered with the fact that he has spent his youth
and adulthood, received his education, practiced his profession as an artist, and taken part in past elections in this country, leaves no doubt of his
election of Philippine citizenship. acCITS

DECISION
MENDOZA, J p:
Petitioner Ernesto S. Mercado and private respondent Eduardo B. Manzano were candidates for vice mayor of the City of Makati in the
May 11, 1998 elections. The other one was Gabriel V. Daza III. The results of the election were as follows:
Eduardo B. Manzano 103,853
Ernesto S. Mercado 100,894
Gabriel V. Daza III 54,275 1
The proclamation of private respondent was suspended in view of a pending petition for disqualification filed by a certain Ernesto
Mamaril who alleged that private respondent was not a citizen of the Philippines but of the United States.
In its resolution, dated May 7, 1998, 2 the Second Division of the COMELEC granted the petition of Mamaril and ordered the
cancellation of the certificate of candidacy of private respondent on the ground that he is a dual citizen and, under §40(d) of the Local
Government Code, persons with dual citizenship are disqualified from running for any elective position. The COMELEC's Second Division said:
What is presented before the Commission is a petition for disqualification of Eduardo Barrios Manzano as candidate
for the office of Vice-Mayor of Makati City in the May 11, 1998 elections. The petition is based on the ground that the
respondent is an American citizen based on the record of the Bureau of Immigration and misrepresented himself as a natural-
born Filipino citizen.
In his answer to the petition filed on April 27, 1998, the respondent admitted that he is registered as a foreigner with
the Bureau of Immigration under Alien Certificate of Registration No. B-31632 and alleged that he is a Filipino citizen because
he was born in 1955 of a Filipino father and a Filipino mother. He was born in the United States, San Francisco, California, on
September 14, 1955, and is considered an American citizen under US Laws. But notwithstanding his registration as an American
citizen, he did not lose his Filipino citizenship.
Judging from the foregoing facts, it would appear that respondent Manzano is both a Filipino and a US citizen. In
other words, he holds dual citizenship.
The question presented is whether under our laws, he is disqualified from the position for which he filed his certificate of
candidacy. Is he eligible for the office he seeks to be elected?
Under Section 40(d) of the Local Government Code, those holding dual citizenship are disqualified from running for any
elective local position.
WHEREFORE, the Commission hereby declares the respondent Eduardo Barrios Manzano DISQUALIFIED as candidate for Vice-
Mayor of Makati City.
On May 8, 1998, private respondent filed a motion for reconsideration. 3 The motion remained pending even until after the election held on May 11,
1998.
Accordingly, pursuant to Omnibus Resolution No. 3044, dated May 10, 1998, of the COMELEC, the board of canvassers tabulated the votes cast for
vice mayor of Makati City but suspended the proclamation of the winner.
On May 19, 1998, petitioner sought to intervene in the case for disqualification. 4 Petitioner's motion was opposed by private respondent.
The motion was not resolved. Instead, on August 31, 1998, the COMELEC en banc rendered its resolution. Voting 4 to 1, with one commissioner
abstaining, the COMELEC en bancreversed the ruling of its Second Division and declared private respondent qualified to run for vice mayor of the City
of Makati in the May 11, 1998 elections. 5 The pertinent portions of the resolution of the COMELEC en banc read:
As aforesaid, respondent Eduardo Barrios Manzano was born in San Francisco, California, U.S.A. He acquired US citizenship by
operation of the United States Constitution andlaws under the principle of jus soli.
He was also a natural born Filipino citizen by operation of the 1935 Philippine Constitution, as his father and mother were
Filipinos at the time of his birth. At the age of six (6), his parents brought him to the Philippines using an American passport as
travel document. His parents also registered him as an alien with the Philippine Bureau of Immigration. He was issued an alien
certificate of registration. This, however, did not result in the loss of his Philippine citizenship, as he did not renounce
Philippine citizenship and did not take an oath of allegiance to the United States.
It is an undisputed fact that when respondent attained the age of majority, he registered himself as a voter, and voted in the
elections of 1992, 1995 and 1998, which effectively renounced his citizenship under American law. Under Philippine law, he no
longer had U.S. citizenship.
At the time of the May 11, 1998 elections, the resolution of the Second Division, adopted on May 7, 1998, was not yet final.
Respondent Manzano obtained the highest number of votes among the candidates for vice-mayor of Makati City, garnering
one hundred three thousand eight hundred fifty-three (103,853) votes over his closest rival, Ernesto S. Mercado, who obtained
one hundred thousand eight hundred ninety-four (100,894) votes, or a margin of two thousand nine hundred fifty-nine (2,959)
votes. Gabriel Daza III obtained third place with fifty four thousand two hundred seventy-five (54,275) votes. In applying
election laws, it would be far better to err in favor of the popular choice than be embroiled in complex legal issues involving
private international law which may well be settled before the highest court (Cf. Frivaldo vs. Commission on Elections, 257
SCRA 727).
WHEREFORE, the Commission en banc hereby REVERSES the resolution of the Second Division, adopted on May 7, 1998,
ordering the cancellation of the respondent's certificate of candidacy.
We declare respondent Eduardo Luis Barrios Manzano to be QUALIFIED as a candidate for the position of vice-mayor of Makati
City in the May 11, 1998, elections.
ACCORDINGLY, the Commission directs the Makati City Board of Canvassers, upon proper notice to the parties, to reconvene
and proclaim the respondent Eduardo Luis Barrios Manzano as the winning candidate for vice-mayor of Makati City.
Pursuant to the resolution of the COMELEC en banc, the board of canvassers, on the evening of August 31, 1998, proclaimed private
respondent as vice mayor of the City of Makati. cdasia
This is a petition for certiorari seeking to set aside the aforesaid resolution of the COMELEC en banc and to declare private respondent
disqualified to hold the office of vice mayor of Makati City. Petitioner contends that —
[T]he COMELEC en banc ERRED in holding that:
A. Under Philippine law, Manzano was no longer a U.S. citizen when he:
1. He renounced his U.S. citizenship when he attained the age of majority when he was already 37 years old; and,
2. He renounced his U.S. citizenship when he (merely) registered himself as a voter and voted in the elections of
1992, 1995 and 1998.
B. Manzano is qualified to run for and or hold the elective office of Vice-Mayor of the City of Makati;
C. At the time of the May 11, 1998 elections, the resolution of the Second Division adopted on 7 May 1998 was not
yet final so that, effectively, petitioner may not be declared the winner even assuming that Manzano is disqualified to run for
and hold the elective office of Vice-Mayor of the City of Makati.
We first consider the threshold procedural issue raised by private respondent Manzano — whether petitioner Mercado has
personality to bring this suit considering that he was not an original party in the case for disqualification filed by Ernesto Mamaril nor was
petitioner's motion for leave to intervene granted.
I. PETITIONER'S RIGHT TO BRING THIS SUIT
Private respondent cites the following provisions of Rule 8 of the Rules of Procedure of the COMELEC in support of his claim that
petitioner has no right to intervene and, therefore, cannot bring this suit to set aside the ruling denying his motion for intervention:
SECTION 1. When proper and when may be permitted to intervene. — Any person allowed to initiate an action or
proceeding may, before or during the trial of an action or proceeding, be permitted by the Commission, in its discretion to
intervene in such action or proceeding, if he has legal interest in the matter in litigation, or in the success of either of the
parties, or an interest against both, or when he is so situated as to be adversely affected by such action or proceeding.
xxx xxx xxx
SECTION 3. Discretion of Commission. — In allowing or disallowing a motion for intervention, the Commission or the
Division, in the exercise of its discretion, shall consider whether or not the intervention will unduly delay or prejudice the
adjudication of the rights of the original parties and whether or not the intervenor's rights may be fully protected in a separate
action or proceeding.
Private respondent argues that petitioner has neither legal interest in the matter in litigation nor an interest to protect because he is "a
defeated candidate for the vice-mayoralty post of Makati City [who] cannot be proclaimed as the Vice-Mayor of Makati City even if the private
respondent be ultimately disqualified by final and executory judgment."
The flaw in this argument is it assumes that, at the time petitioner sought to intervene in the proceedings before the COMELEC, there
had already been a proclamation of the results of the election for the vice mayoralty contest for Makati City, on the basis of which petitioner
came out only second to private respondent. The fact, however, is that there had been no proclamation at that time. Certainly, petitioner had,
and still has, an interest in ousting private respondent from the race at the time he sought to intervene. The rule in Labo
v. COMELEC, 6 reiterated in several cases, 7 only applies to cases in which the election of the respondent is contested, and the question is
whether one who placed second to the disqualified candidate may be declared the winner. In the present case, at the time petitioner filed a
"Motion for Leave to File Intervention" on May 20, 1998, there had been no proclamation of the winner, and petitioner's purpose was precisely
to have private respondent disqualified "from running for [an] elective local position" under §40(d) of R.A. No. 7160. If Ernesto Mamaril (who
originally instituted the disqualification proceedings), a registered voter of Makati City, was competent to bring the action, so was petitioner
since the latter was a rival candidate for vice mayor of Makati City.

Nor is petitioner's interest in the matter in litigation any less because he filed a motion for intervention only on May 20, 1998, after
private respondent had been shown to have garnered the highest number of votes among the candidates for vice mayor. That petitioner had a
right to intervene at that stage of the proceedings for the disqualification against private respondent is clear from §6 of R.A. No. 6646, otherwise
known as the Electoral Reforms Law of 1987, which provides:
Any candidate who has been declared by final judgment to be disqualified shall not be voted for, and the votes cast
for him shall not be counted. If for any reason a candidate is not declared by final judgment before an election to be
disqualified and he is voted for and receives the winning number of votes in such election, the Court or Commission shall
continue with the trial and hearing of the action, inquiry, or protest and, upon motion of the complainant or any intervenor,
may during the pendency thereof order the suspension of the proclamation of such candidate whenever the evidence of guilt
is strong.
Under this provision, intervention may be allowed in proceedings for disqualification even after election if there has yet been no final
judgment rendered.
The failure of the COMELEC en banc to resolve petitioner's motion for intervention was tantamount to a denial of the motion,
justifying petitioner in filing the instant petition for certiorari. As the COMELEC en banc instead decided the merits of the case, the present
petition properly deals not only with the denial of petitioner's motion for intervention but also with the substantive issues respecting private
respondent's alleged disqualification on the ground of dual citizenship.
This brings us to the next question, namely, whether private respondent Manzano possesses dual citizenship and, if so, whether he is
disqualified from being a candidate for vice mayor of Makati City.
II. DUAL CITIZENSHIP AS A GROUND FOR DISQUALIFICATION
The disqualification of private respondent Manzano is being sought under §40 of the Local Government Code of 1991 (R.A. No. 7160),
which declares as "disqualified from running for any elective local position: . . . (d) Those with dual citizenship." This provision is incorporated in
the Charter of the City of Makati. 8
Invoking the maxim dura lex sed lex, petitioner, as well as the Solicitor General, who sides with him in this case, contends that through
§40(d) of the Local Government Code, Congress has "command[ed] in explicit terms the ineligibility of persons possessing dual allegiance to
hold local elective office."
To begin with, dual citizenship is different from dual allegiance. The former arises when, as a result of the concurrent application of
the different laws of two or more states, a person is simultaneously considered a national by the said states. 9 For instance, such a situation may
arise when a person whose parents are citizens of a state which adheres to the principle of jus sanguinis is born in a state which follows the
doctrine of jus soli. Such a person, ipso facto and without any voluntary act on his part, is concurrently considered a citizen of both
states. Considering the citizenship clause (Art. IV) of our Constitution,it is possible for the following classes of citizens of the Philippines to
possess dual citizenship:
(1) Those born of Filipino fathers and/or mothers in foreign countries which follow the principle of jus soli;
(2) Those born in the Philippines of Filipino mothers and alien fathers if by the laws of their fathers' country such children are
citizens of that country;
(3) Those who marry aliens if by the laws of the latter's country the former are considered citizens, unless by their act or
omission they are deemed to have renounced Philippine citizenship.
There may be other situations in which a citizen of the Philippines may, without performing any act, be also a citizen of another state;
but the above cases are clearly possible given the constitutional provisions on citizenship.
Dual allegiance, on the other hand, refers to the situation in which a person simultaneously owes, by some positive act, loyalty to two
or more states. While dual citizenship is involuntary, dual allegiance is the result of an individual's volition.
With respect to dual allegiance, Article IV, §5 of the Constitution provides: "Dual allegiance of citizens is inimical to the national
interest and shall be dealt with by law." This provision was included in the 1987 Constitution at the instance of Commissioner Blas F. Ople who
explained its necessity as follows: 10
. . . I want to draw attention to the fact that dual allegiance is not dual citizenship. I have circulated a memorandum
to the Bernas Committee according to which a dual allegiance — and I reiterate a dual allegiance — is larger and more
threatening than that of mere double citizenship which is seldom intentional and, perhaps, never insidious. That is often a
function of the accident of mixed marriages or of birth on foreign soil. And so, I do not question double citizenship at all.
What we would like the Committee to consider is to take constitutional cognizance of the problem of dual allegiance.
For example, we all know what happens in the triennial elections of the Federation of Filipino-Chinese Chambers of Commerce
which consists of about 600 chapters all over the country. There is a Peking ticket, as well as a Taipei ticket. Not widely known
is the fact that the Filipino-Chinese community is represented in the Legislative Yuan of the Republic of China in Taiwan. And
until recently, the sponsor might recall, in Mainland China in the People's Republic of China, they have the Associated
Legislative Council for overseas Chinese wherein all of Southeast Asia including some European and Latin countries were
represented, which was dissolved after several years because of diplomatic friction. At that time, the Filipino-Chinese were also
represented in that Overseas Council.
When I speak of double allegiance, therefore, I speak of this unsettled kind of allegiance of Filipinos, of citizens who
are already Filipinos but who, by their acts, may be said to be bound by a second allegiance, either to Peking or Taiwan. I also
took close note of the concern expressed by some Commissioners yesterday, including Commissioner Villacorta, who were
concerned about the lack of guarantees of thorough assimilation, and especially Commissioner Concepcion who has always
been worried about minority claims on our natural resources.
Dual allegiance can actually siphon scarce national capital to Taiwan, Singapore, China or Malaysia, and this is already
happening. Some of the great commercial places in downtown Taipei are Filipino-owned, owned by Filipino-Chinese — it is of
common knowledge in Manila. It can mean a tragic capital outflow when we have to endure a capital famine which also means
economic stagnation, worsening unemployment and social unrest.
And so, this is exactly what we ask — that the Committee kindly consider incorporating a new section, probably
Section 5, in the article on Citizenship which will read as follows: DUAL ALLEGIANCE IS INIMICAL TO CITIZENSHIP AND SHALL BE
DEALT WITH ACCORDING TO LAW.
In another session of the Commission, Ople spoke on the problem of these citizens with dual allegiance, thus: 11
. . . A significant number of Commissioners expressed their concern about dual citizenship in the sense that it implies
a double allegiance under a double sovereignty which some of us who spoke then in a freewheeling debate thought would be
repugnant to the sovereignty which pervades the Constitution and to citizenship itself which implies a uniqueness and which
elsewhere in the Constitution is defined in terms of rights and obligations exclusive to that citizenship including, of course, the
obligation to rise to the defense of the State when it is threatened, and back of this, Commissioner Bernas, is, of course, the
concern for national security. In the course of those debates, I think some noted the fact that as a result of the wave of
naturalizations since the decision to establish diplomatic relations with the People's Republic of China was made in 1975, a
good number of these naturalized Filipinos still routinely go to Taipei every October 10; and it is asserted that some of them do
renew their oath of allegiance to a foreign government maybe just to enter into the spirit of the occasion when the anniversary
of the Sun Yat-Sen Republic is commemorated. And so, I have detected a genuine and deep concern about double citizenship,
with its attendant risk of double allegiance which is repugnant to our sovereignty and national security. I appreciate what the
Committee said that this could be left to the determination of a future legislature. But considering the scale of the problem,
the real impact on the security of this country, arising from, let us say, potentially great numbers of double citizens professing
double allegiance, will the Committee entertain a proposed amendment at the proper time that will prohibit, in effect, or
regulate double citizenship?
Clearly, in including §5 in Article IV on citizenship, the concern of the Constitutional Commission was not with dual citizens per se but
with naturalized citizens who maintain their allegiance to their countries of origin even after their naturalization. Hence, the phrase "dual
citizenship" in R.A. No. 7160, §40(d) and in R.A. No. 7854, §20 must be understood as referring to "dual allegiance." Consequently, persons with
mere dual citizenship do not fall under this disqualification. Unlike those with dual allegiance, who must, therefore, be subject to strict process
with respect to the termination of their status, for candidates with dual citizenship, it should suffice if, upon the filing of their certificates of
candidacy, they elect Philippine citizenship to terminate their status as persons with dual citizenship considering that their condition is the
unavoidable consequence of conflicting laws of different states. As Joaquin G. Bernas, one of the most perceptive members of the Constitutional
Commission, pointed out: "[D]ual citizenship is just a reality imposed on us because we have no control of the laws on citizenship of other
countries. We recognize a child of a Filipino mother. But whether or not she is considered a citizen of another country is something completely
beyond our control." 12

By electing Philippine citizenship, such candidates at the same time forswear allegiance to the other country of which they are also
citizens and thereby terminate their status as dual citizens. It may be that, from the point of view of the foreign state and of its laws, such an
individual has not effectively renounced his foreign citizenship. That is of no moment as the following discussion on §40(d) between Senators
Enrile and Pimentel clearly shows: 13
SENATOR ENRILE. Mr. President, I would like to ask clarification of line 41, page 17: "Any person with dual
citizenship" is disqualified to run for any elective local position. Under the present Constitution, Mr. President, someone whose
mother is a citizen of the Philippines but his father is a foreigner is a natural-born citizen of the Republic. There is no
requirement that such a natural born citizen, upon reaching the age of majority, must elect or give up Philippine citizenship.
On the assumption that this person would carry two passports, one belonging to the country of his or her father and
one belonging to the Republic of the Philippines, may such a situation disqualify the person to run for a local government
position?
SENATOR PIMENTEL. To my mind, Mr. President, it only means that at the moment when he would want to run for
public office, he has to repudiate one of his citizenships.
SENATOR ENRILE. Suppose he carries only a Philippine passport but the country of origin or the country of the father
claims that person, nevertheless, as a citizen? No one can renounce. There are such countries in the world.
SENATOR PIMENTEL. Well, the very fact that he is running for public office would, in effect, be an election for him of
his desire to be considered as a Filipino citizen.
SENATOR ENRILE. But, precisely, Mr. President, the Constitution does not require an election. Under the
Constitution, a person whose mother is a citizen of the Philippines is, at birth, a citizen without any overt act to claim the
citizenship.
SENATOR PIMENTEL. Yes. What we are saying, Mr. President, is: Under the Gentleman's example, if he does not
renounce his other citizenship, then he is opening himself to question. So, if he is really interested to run, the first thing he
should do is to say in the Certificate of Candidacy that: "I am a Filipino citizen, and I have only one citizenship."
SENATOR ENRILE. But we are talking from the viewpoint of Philippine law, Mr. President. He will always have one
citizenship, and that is the citizenship invested upon him or her in the Constitution of the Republic.
SENATOR PIMENTEL. That is true, Mr. President. But if he exercises acts that will prove that he also acknowledges
other citizenships, then he will probably fall under this disqualification.
This is similar to the requirement that an applicant for naturalization must renounce "all allegiance and fidelity to any foreign prince,
potentate, state, or sovereignty" 14of which at the time he is a subject or citizen before he can be issued a certificate of naturalization as a
citizen of the Philippines. In Parado v. Republic, 15 it was held:
[W]hen a person applying for citizenship by naturalization takes an oath that he renounces his loyalty to any other country or
government and solemnly declares that he owes his allegiance to the Republic of the Philippines, the condition imposed by law
is satisfied and complied with. The determination whether such renunciation is valid or fully complies with the provisions of
our Naturalization Law lies within the province and is an exclusive prerogative of our courts. The latter should apply the law
duly enacted by the legislative department of the Republic. No foreign law may or should interfere with its operation and
application. If the requirement of the Chinese Law of Nationality were to be read into our Naturalization Law, we would be
applying not what our legislative department has deemed it wise to require, but what a foreign government has thought or
intended to exact. That, of course, is absurd. It must be resisted by all means and at all cost. It would be a brazen
encroachment upon the sovereign will and power of the people of this Republic.
III. PETITIONER'S ELECTION OF PHILIPPINE CITIZENSHIP
The record shows that private respondent was born in San Francisco, California on September 4, 1955, of Filipino parents. Since the
Philippines adheres to the principle of jus sanguinis, while the United States follows the doctrine of jus soli, the parties agree that, at birth at
least, he was a national both of the Philippines and of the United States. However, the COMELEC en banc held that, by participating in Philippine
elections in 1992, 1995, and 1998, private respondent "effectively renounced his U.S. citizenship under American law," so that now he is solely a
Philippine national.
Petitioner challenges this ruling. He argues that merely taking part in Philippine elections is not sufficient evidence of renunciation and
that, in any event, as the alleged renunciation was made when private respondent was already 37 years old, it was ineffective as it should have
been made when he reached the age of majority.
In holding that by voting in Philippine elections private respondent renounced his American citizenship, the COMELEC must have in
mind §349 of the Immigration and Nationality Act of the United States, which provided that "A person who is a national of the United States,
whether by birth or naturalization, shall lose his nationality by: . . . (e) Voting in a political election in a foreign state or participating in an
election or plebiscite to determine the sovereignty over foreign territory." To be sure this provision was declared unconstitutional by the U.S.
Supreme Court in Afroyim v. Rusk 16 as beyond the power given to the U.S. Congress to regulate foreign relations. However, by filing a
certificate of candidacy when he ran for his present post, private respondent elected Philippine citizenship and in effect renounced his American
citizenship. Private respondent's certificate of candidacy, filed on March 27, 1998, contained the following statements made under oath:
6. I AM A FILIPINO CITIZEN (STATE IF "NATURAL-BORN" OR "NATURALIZED") NATURAL-BORN
xxx xxx xxx
10. I AM A REGISTERED VOTER OF PRECINCT NO. 747-A, BARANGAY SAN LORENZO, CITY/MUNICIPALITY OF MAKATI, PROVINCE
OF NCR.
11. I AM NOT A PERMANENT RESIDENT OF, OR IMMIGRANT TO, A FOREIGN COUNTRY.
12. I AM ELIGIBLE FOR THE OFFICE I SEEK TO BE ELECTED. I WILL SUPPORT AND DEFEND THE CONSTITUTION OF THE
PHILIPPINES AND WILL MAINTAIN TRUE FAITH AND ALLEGIANCE THERETO; THAT I WILL OBEY THE LAWS, LEGAL
ORDERS AND DECREES PROMULGATED BY THE DULY CONSTITUTED AUTHORITIES OF THE REPUBLIC OF THE
PHILIPPINES; AND THAT I IMPOSE THIS OBLIGATION UPON MYSELF VOLUNTARILY, WITHOUT MENTAL RESERVATION
OR PURPOSE OF EVASION. I HEREBY CERTIFY THAT THE FACTS STATED HEREIN ARE TRUE AND CORRECT OF MY OWN
PERSONAL KNOWLEDGE.
The filing of such certificate of candidacy sufficed to renounce his American citizenship, effectively removing any disqualification he
might have as a dual citizen. Thus, inFrivaldo v. COMELEC it was held: 17
It is not disputed that on January 20, 1983 Frivaldo became an American. Would the retroactivity of his repatriation
not effectively give him dual citizenship, which under Sec. 40 of the Local Government Code would disqualify him "from
running for any elective local position?" We answer this question in the negative, as there is cogent reason to hold
that Frivaldo was really STATELESS at the time he took said oath of allegiance and even before that, when he ran for governor
in 1988. In his Comment, Frivaldo wrote that he "had long renounced and had long abandoned his American citizenship-long
before May 8, 1995. At best, Frivaldo was stateless in the interim-when he abandoned and renounced his US citizenship but
before he was repatriated to his Filipino citizenship."
On this point, we quote from the assailed Resolution dated December 19, 1995:
"By the laws of the United States, petitioner Frivaldo lost his American citizenship when he took his oath of allegiance
to the Philippine Government when he ran for Governor in 1988, in 1992, and in 1995. Every certificate of candidacy
contains an oath of allegiance to the Philippine Government."
These factual findings that Frivaldo has lost his foreign nationality long before the elections of 1995 have not been
effectively rebutted by Lee. Furthermore, it is basic that such findings of the Commission are conclusive upon this Court, absent
any showing of capriciousness or arbitrariness or abuse.
There is, therefore, no merit in petitioner's contention that the oath of allegiance contained in private respondent's certificate of
candidacy is insufficient to constitute renunciation of his American citizenship. Equally without merit is petitioner's contention that, to be
effective, such renunciation should have been made upon private respondent reaching the age of majority since no law requires the election of
Philippine citizenship to be made upon majority age.
Finally, much is made of the fact that private respondent admitted that he is registered as an American citizen in the Bureau of
Immigration and Deportation and that he holds an American passport which he used in his last travel to the United States on April 22, 1997.
There is no merit in this. Until the filing of his certificate of candidacy on March 21, 1998, he had dual citizenship. The acts attributed to him can
be considered simply as the assertion of his American nationality before the termination of his American citizenship. What this Court said
in Aznar vs. COMELEC 18 applies mutatis mutandis to private respondent in the case at bar:
. . . Considering the fact that admittedly Osmeña was both a Filipino and an American, the mere fact that he has a
Certificate stating he is an American does not mean that he is not still a Filipino. . . . [T]he Certification that he is an American
does not mean that he is not still a Filipino, possessed as he is, of both nationalities or citizenships. Indeed, there is no express
renunciation here of Philippine citizenship; truth to tell, there is even no implied renunciation of said citizenship. When We
consider that the renunciation needed to lose Philippine citizenship must be "express," it stands to reason that there can be no
such loss of Philippine citizenship when there is no renunciation, either "express" or "implied."

To recapitulate, by declaring in his certificate of candidacy that he is a Filipino citizen; that he is not a permanent resident or
immigrant of another country; that he will defend and support the Constitution of the Philippines and bear true faith and allegiance thereto and
that he does so without mental reservation, private respondent has, as far as the laws of this country are concerned, effectively repudiated his
American citizenship and anything which he may have said before as a dual citizen.
On the other hand, private respondent's oath of allegiance to the Philippines, when considered with the fact that he has spent his
youth and adulthood, received his education, practiced his profession as an artist, and taken part in past elections in this country, leaves no
doubt of his election of Philippine citizenship.
His declarations will be taken upon the faith that he will fulfill his undertaking made under oath. Should he betray that trust, there are
enough sanctions for declaring the loss of his Philippine citizenship through expatriation in appropriate proceedings. In Yu v. Defensor-
Santiago, 19 we sustained the denial of entry into the country of petitioner on the ground that, after taking his oath as a naturalized citizen, he
applied for the renewal of his Portuguese passport and declared in commercial documents executed abroad that he was a Portuguese national.
A similar sanction can be taken against any one who, in electing Philippine citizenship, renounces his foreign nationality, but subsequently does
some act constituting renunciation of his Philippine citizenship. cdasia
WHEREFORE, the petition for certiorari is DISMISSED for lack of merit.
SO ORDERED.
Davide, Jr., C.J., Romero, Bellosillo, Melo, Puno, Vitug, Kapunan, Quisumbing, Buena, Gonzaga-Reyes, and Ynares-Santiago, JJ., concur.
Panganiban and Purisima, JJ., are on leave.
Pardo, J., took no part.

||| (Mercado v. Manzano, G.R. No. 135083, [May 26, 1999], 367 PHIL 132-153)
EN BANC
[G.R. Nos. 92191-92. July 30, 1991.]
ANTONIO Y. CO, petitioner, vs. ELECTORAL TRIBUNAL OF THE HOUSE OF REPRESENTATIVES and JOSE ONG, JR., respondents.
[G.R. Nos. 92202-03. July 30, 1991.]
SIXTO T. BALANQUIT, JR., petitioner, vs. ELECTORAL TRIBUNAL OF THE HOUSE OF REPRESENTATIVES and JOSE ONG,
JR., respondents.
Hechanova & Associates for petitioner Co.
Brillantes, Nachura, Navarro and Arcilla Law Offices for respondent Ong, Jr.
SYLLABUS
1. CONSTITUTIONAL LAW; ELECTORAL TRIBUNAL OF THE HOUSE OF REPRESENTATIVES AND HOUSE OF SENATE; SOLE JUDGES OF ALL CONTESTS
RELATING TO ELECTION, RETURNS AND QUALIFICATIONS OF THEIR RESPECTIVE MEMBERS. — The Constitution explicitly provides that the House of
Representatives Electoral Tribunal (HRET) and the Senate Electoral Tribunal (SET) shall be the sole judges of all contests relating to the election,
returns, and qualifications of their respective members (See Article VI, Section 17,Constitution). The authority conferred upon the Electoral Tribunal
is full, clear and complete. The use of the word sole emphasizes the exclusivity of the jurisdiction of these Tribunals. The Supreme Court in the case
of Lazatin vs. HRET (168 SCRA 391 [1988]) stated that under the 1987 Constitution, the jurisdiction of the Electoral Tribunal is original and exclusive.
And that, " . . . so long as the Constitution grants the HRET the power to be the sole judge of all contests relating to election, returns and
qualifications of members of the House of Representatives, any final action taken by the HRET on a matter within its jurisdiction shall, as a rule, not
be reviewed by this Court . . . the power granted to the Electoral Tribunal is full, clear and complete and excludes the exercise of any authority on the
part of this Court that would in any wise restrict it or curtail it or even affect the same."
2. ID.; ID.; JUDGMENTS THEREOF AS A RULE BEYOND JUDICIAL INTERFERENCE; EXCEPTION; ARBITRARY AND IMPROVIDENT USE OF POWER
RESULTING TO DENIAL OF DUE PROCESS. — In the case of Robles vs. HRET (181 SCRA 780 [1980]) the Supreme Court stated that the judgments of the
Tribunal are beyond judicial interference save only "in the exercise of this Court's so-called extraordinary jurisdiction, . . . upon a determination that
the Tribunal's decision or resolution was rendered without or in excess of its jurisdiction, or with grave abuse of discretion or paraphrasing Morrero,
upon a clear showing of such arbitrary and improvident use by the Tribunal of its power as constitutes a denial of due process of law, or upon a
demonstration of a very clear unmitigated ERROR, manifestly constituting such GRAVE ABUSE OF DISCRETION that there has to be a remedy for such
abuse." In the leading case of Morrero vs. Bocar (66 Phil. 429 [1938]) the Court ruled that the power of the Electoral Commission "is beyond judicial
interference except, in any event, upon a clear showing of such arbitrary and improvident use of power as will constitute a denial of due process."
The Court does not venture into the perilous area of trying to correct perceived errors of independent branches of the Government. It comes in only
when it has to vindicate a denial of due process or correct an abuse of discretion so grave or glaring that no less than the Constitution calls for
remedial action.
3. ID.; ID.; ID.; APPLIED IN CASE AT BAR. — In the absence of a showing that the HRET has committed grave abuse of discretion amounting to lack of
jurisdiction, there is no occasion for the Court to exercise its corrective power; it will not decide a matter which by its nature is for the HRET alone to
decide (See Marcos vs. Manglapus, 177 SCRA 668 [1989]). It has no power to look into what it thinks is apparent error. As constitutional creations
invested with necessary power, the Electoral Tribunals, although not powers in the tripartite scheme of the government, are, in the exercise of their
functions independent organs — independent of Congress and the Supreme Court. The power granted to HRET by the Constitution is intended to be
as complete and unimpaired as if it had remained originally in the legislature (Angara vs. Electoral Commission, 63 Phil. 139 [1936]). In passing upon
petitions, the Court with its traditional and careful regard for the balance of powers, must permit this exclusive privilege of the Tribunals to remain
where the Sovereign authority has placed it (See Veloso vs. Boards of Canvassers of Leyte and Samar, 39 Phil. 886 [1919]).
4. ID.; SUPREME COURT; EXPANDED JURISDICTION UNDER 1987 CONSTITUTION. — The Supreme Court under the 1987 Constitution, has been given
an expanded jurisdiction, so to speak, to review the decisions of the other branches and agencies of the government to determine whether or not
they have acted within bounds of the Constitution (See Article VIII, Section 1, Constitution). Yet, in the exercise thereof, the Court is to merely check
whether or not the government branch or agency has gone beyond the Constitutional limits of its jurisdiction, not that it erred or has a different
view.
5. ID.; CONSTITUTIONAL PROVISIONS; HOW CONSTRUED; SPIRIT AND INTENDMENT MUST PREVAIL. — In construing the law, the Courts are not
always to be hedged in by the literal meaning of its language. The spirit and intendment thereof, must prevail over the letter, especially where
adherence to the latter would result in absurdity and injustice (Casela vs. Court of Appeals, 35 SCRA 279 [1970]). A Constitutional provision should be
construed so as to give it effective operation and suppress the mischief at which it is aimed, hence, it is the spirit of the provision which should
prevail over the letter thereof (Jarrolt vs. Mabberly, 103 U.S. 580). In the words of the Court in the case of J.M. Tuazon vs. LTA (31 SCRA 413 [1970]);
"To that primordial intent, all else is subordinated. Our Constitution,any constitution is not to be construed narrowly or pedantically, for the
prescriptions therein contained, to paraphrase Justice Holmes, are not mathematical formulas having their essence in their form but are organic
living institutions, the significance of which is vital not formal . . . ."
6. ID.; CITIZENSHIP; SECTION 1, PARAGRAPH 3 OF ARTICLE IV OF 1987 CONSTITUTION; CONSTRUED. — Article IV of the Constitution provides:
"Section 1. The following are citizens of the Philippines: . . . (3) Those born before January 17, 1973, of Filipino mothers, who elect Philippine
citizenship upon reaching the age of majority; and . . . Section 2. Natural-born Citizens are those who are citizens of the Philippines from birth
without having to perform any act to acquire or perfect their citizenship. Those who elect Philippine citizenship in accordance with paragraph 3
hereof shall be deemed natural-born citizens." The Court interprets Section 1, Paragraph 3 above as applying not only to those who elect Philippine
citizenship after February 2, 1987 but also to those who, having been born of Filipino mothers, elected citizenship before that date. The provision in
Paragraph 3 was intended to correct an unfair position which discriminates against Filipino women. To make the provision prospective from February
3, 1987 is to give a narrow interpretation resulting in an inequitable situation. It must also be retroactive. The provision in question was enacted to
correct the anomalous situation where one born of a Filipino father and an alien mother was automatically granted the status of a natural-born
citizen while one born of a Filipino mother and an alien father would still have to elect Philippine citizenship. If one so elected, he was not, under
earlier laws, conferred the status of a natural-born. Under the 1973 Constitution, those born of Filipino fathers and those born of Filipino mothers
with an alien father were placed in equal footing. They were both considered as natural-born citizens. Hence, the bestowment of the status of
"natural-born" cannot be made to depend on the fleeting accident of time or result in two kinds of citizens made up of essentially the same similarly
situated members. It is for this reason that the amendments were enacted, that is, in order to remedy this accidental anomaly, and, therefore, treat
equally all those born before the 1973 Constitution and who elected Philippine citizenship either before or after the effectivity of that Constitution.
7. ID.; ID.; SECTION 2 OF ARTICLE IV OF THE 1987 CONSTITUTION; ELECTION OF CITIZENSHIP; APPLIES ONLY TO THOSE BORN OF FILIPINO MOTHER
AND ALIEN FATHER BUT NOT TO ONE WHOSE FATHER HAS BEEN NATURALIZED WHEN MINOR WAS ONLY NINE (9) YEARS OF AGE. — There is no
dispute that respondent's mother was a natural born Filipina at the time of her marriage. Crucial to this case is the issue of whether or not the
respondent elected or chose to be a Filipino citizen. Election becomes material because Section 2 of Article IV of the Constitution accords natural
born status to children born of Filipino mothers before January 17, 1973, if they elect citizenship upon reaching the age of majority. To expect the
respondent to have formally or in writing elected citizenship when he came of age is to ask for the unnatural and unnecessary. The reason is obvious.
He was already a citizen. Not only was his mother a natural born citizen but his father had been naturalized when the respondent was only nine (9)
years old. He could not have divined when he came of age that in 1973 and 1987 the Constitution would be amended to require him to have filed a
sworn statement in 1969 electing citizenship in spite of his already having been a citizen since 1957. In 1969, election through a sworn statement
would have been an unusual and unnecessary procedure for one who had been a citizen since he was nine years old.
8. ID.; ID.; ID.; ID.; CASE OF IN RE: FLORENCIO MALLARE (59 SCRA 45 [1974]) APPLIES IN CASE AT BAR. — In the case of In Re: Florencio Mallare (59
SCRA 45 [1974]), the Court held that the exercise of the right of suffrage and the participation in election exercises constitute a positive act of
election of Philippine citizenship. In the exact pronouncement of the Court, we held: "Esteban's exercise of the right of suffrage when he came of age,
constitutes a positive act of election of Philippine citizenship." The private respondent did more than merely exercise his right of suffrage. He has
established his life here in the Philippines. For those in the peculiar situation of the respondent who cannot be expected to have elected citizenship
as they were already citizens, we apply the In Re Mallare rule. The filing of a sworn statement or formal declaration is a requirement for those who
still have to elect citizenship. For those already Filipinos when the time to elect came up, there are acts of deliberate choice which cannot be less
binding. Entering a profession open only to Filipinos, serving in public office where citizenship is a qualification, voting during election time, running
for public office, and other categorical acts of similar nature are themselves formal manifestations of choice for these persons.

9. ID.; ID.; AN ATTACK THERETO MAY ONLY BE DONE THROUGH A DIRECT ACTION. — The petitioners argue that the respondent's father was not,
validly, a naturalized citizen because of his premature taking of the oath of citizenship. The Court cannot go into the collateral procedure of stripping
Mr. Ong's father of his citizenship after his death and at this very late date just so we can go after the son. The petitioners question the citizenship of
the father through a collateral approach. This can not be done. In our jurisprudence, an attack on a person's citizenship may only be done through a
direct action for its nullity (See Queto vs. Catolico, 31 SCRA 52 [1970]).
10. ID.; ID.; TO DECLARE THE GRANT THEREOF AS NULL AND VOID VIOLATIVE OF THE DUE PROCESS CLAUSE WHERE PERSON INVOLVED HAS BEEN
LAID TO REST. — To ask the Court to declare that grant of Philippine citizenship to Jose Ong Chuan as null and void would run against the principle of
due process. Jose Ong Chuan has already been laid to rest. How can he be given a fair opportunity to defend himself. A dead man cannot speak. To
quote the words of the HRET: "Ong Chuan's lips have long been muted to perpetuity by his demise and obviously he could not rise beyond where his
mortal remains now lie to defend himself were this matter to be made a central issue in this case."
11. ID.; ID.; ARTICLE 17 OF THE CIVIL CODE OF SPAIN SUB-PARAGRAPH 4 THEREOF IN RELATION TO SECTION 4 OF THE PHILIPPINE BILL OF 1902,
APPLIED IN CASE AT BAR. —Article 17 of the Civil Code of Spain enumerates those who were considered Spanish Subjects, viz: "ARTICLE 17. The
following are Spaniards: . . . (4). Those without such papers, who may have acquired domicile in any town in the Monarchy." The domicile of a natural
person is the place of his habitual residence. This domicile, once established is considered to continue and will not be deemed lost until a new one is
established (Article 50, NCC; Article 40, Civil Code of Spain; Zuellig vs. Republic, 83 Phil. 768 [1949]). Ong Te became a permanent resident of Laoang,
Samar around 1895. Correspondingly, a certificate of residence was then issued to him by virtue of his being a resident of Laoang, Samar. The
domicile that Ong Te established in 1895 continued until April 11, 1899; it even went beyond the turn of the 19th century. It is also in this place
where Ong Te set up his business and acquired his real property. Ong Te falls within the meaning of sub-paragraph 4 of Article 17 of the Civil Code of
Spain. Although Ong Te made brief visits to China, he, nevertheless, always returned to the Philippines. The fact that he died in China, during one of
his visits in said country, was of no moment. This will not change the fact that he already had his domicile fixed in the Philippines and pursuant to the
Civil Code of Spain, he had become a Spanish subject. If Ong Te became a Spanish subject by virtue of having established his domicile in a town under
the Monarchy of Spain, necessarily, Ong Te was also an inhabitant of the Philippines for an inhabitant has been defined as one who has actual fixed
residence in a place; one who has a domicile in a place (Bouvier's Law Dictionary, Vol. II). A priori, there can be no other logical conclusion but to
educe that Ong Te qualified as a Filipino citizen under the provisions of Section 4 of the Philippine Bill of 1902.
12. ID.; ID.; "RESIDENCE"; MEANING THEREOF UNDER THE CONSTITUTION. — Under the Constitution, the term "residence" has been understood as
synonymous with domicile not only under the previous Constitutions but also under the 1987 Constitution. The term "domicile" denotes a fixed
permanent residence to which when absent for business or pleasure, one intends to return (Ong Huan Tin vs. Republic, 19 SCRA 966 [1967]). The
absence of a person from said permanent residence, no matter how long, notwithstanding, it continues to be the domicile of that person. In other
words, domicile is characterized by animus revertendi (Ujano vs. Republic, 17 SCRA 147 [1966]).
13. ID.; ID.; ID.; ESTABLISHMENT THEREOF; OWNERSHIP OF A HOUSE NOT NECESSARY. — The petitioners' allegation that since the private
respondent owns no property in Laoang, Samar, he cannot, therefore, be a resident of said place is misplaced. The properties owned by the Ong
family are in the name of the private respondent's parents. Upon the demise of his parents, necessarily, the private respondent, pursuant to the laws
of succession, became the co-owner thereof (as a co-heir), notwithstanding the fact that these were still in the names of his parents. Even assuming
that the private respondent does not own any property in Samar, the Supreme Court in the case of De los Reyes vs. Solidum (61 Phil. 893 [1935]) held
that it is not required that a person should have a house in order to establish his residence and domicile. It is enough that he should live in the
municipality or in a rented house or in that of a friend or relative.
14. ID.; ID.; ID.; TEMPORARY ABSENCE DOES NOT NECESSARILY CONNOTE CHANGE THEREOF; "ANIMUS REVERTENDI" ESTABLISHED IN CASE AT BAR.
— It has also been settled that absence from residence to pursue studies or practice a profession or registration as a voter other than in the place
where one is elected, does not constitute loss of residence (Faypon vs. Quirino, 96 Phil. 294 [1954]). The private respondent stayed in Manila for the
purpose of finishing his studies and later to practice his profession. There was no intention to abandon the residence in Laoang, Samar. On the
contrary, the periodical journeys made to his home province reveal that he always had the animus revertendi.
15. ID.; ID.; PROSPECTIVE JUDICIAL RECOMMENDATION; MORE HUMANE AND LESS TECHNICAL APPROACH TO CITIZENSHIP PROBLEMS. — Our
citizens no doubt constitute the country's greatest wealth. Citizenship is a special privilege which one must forever cherish. However, in order to truly
revere this treasure of citizenship, we do not, on the basis of too harsh an interpretation, have to unreasonably deny it to those who qualify to share
in its richness. Under the overly strict jurisprudence surrounding our antiquated naturalization laws only the very affluent backed by influential
patrons, who were willing to suffer the indignities of a lengthy, sometimes humiliating, and often corrupt process of clearances by minor bureaucrats
and whose lawyers knew how to overcome so many technical traps of the judicial process were able to acquire citizenship. It is time for the
naturalization law to be revised to enable a more positive, affirmative, and meaningful examination of an applicant's suitability to be a Filipino. A
more humane, more indubitable and less technical approach to citizenship problems is essential.
16. ID.; HOUSE OF REPRESENTATIVE; CANDIDATES; PROPERTY OWNERSHIP; NOT A QUALIFICATION. — To require the private respondent to own
property in order to be eligible to run for Congress would be tantamount to a property qualification. The Constitution only requires that the
candidate meet the age, citizenship, voting and residence requirements. Nowhere is it required by the Constitution that the candidate should also
own property in order to be qualified to run (see Maquera vs. Borra, 122 Phil. 412 [1965]).
17. REMEDIAL LAW; BEST EVIDENCE RULE; EXCEPTION; ORIGINAL HAS BEEN LOST; REQUIREMENTS THEREOF TO BE ADMISSIBLE; PROPERLY LAID IN
CASE AT BAR. — The petitioners' sole ground in disputing that respondent was a natural-born Filipino is that the documents presented to prove it
were not in compliance with the best evidence rule. The petitioners allege that the private respondent failed to present the original of the
documentary evidence, testimonial evidence and of the transcript of the proceedings of the body upon which the resolution of the 1971
Constitutional Convention was predicated. On the contrary, the documents presented by the private respondent fall under the exceptions to the best
evidence rule. It was established in the proceedings before the HRET that the originals of the Committee Report No. 12, the minutes of the plenary
session of the 1971 Constitutional Convention held on November 28, 1972 cannot be found. This was affirmed by Atty. Ricafrente, Assistant
Secretary of the 1971 Constitutional Convention; by Atty. Nolledo, Delegate to the 1971 Constitutional Convention; and by Atty. Antonio Santos,
Chief Librarian of the U.P. Law Center, in their respective testimonies given before the HRET to the effect that there is no governmental agency which
is the official custodian of the records of the 1971 Constitutional Convention. The execution of the originals was established by Atty. Ricafrente, who
as the Assistant Secretary of the 1971 Constitutional Convention was the proper party to testify to such execution. The inability to produce the
originals before the HRET was also testified to as aforestated by Atty. Ricafrente, Atty. Nolledo, and Atty. Santos. In proving the inability to produce,
the law does not require the degree of proof to be of sufficient certainty; it is enough that it be shown that after a bona fide diligent search, the same
cannot be found (see Government of P.I. vs. Martinez, 44 Phil. 817 [1918]). Since the execution of the document and the inability to produce were
adequately established, the contents of the questioned documents can be proven by a copy thereof or by the recollection of witnesses.
PADILLA, J., dissenting:
1. CONSTITUTIONAL LAW; SUPREME COURT; JURISDICTION THEREOF; EXPANDED UNDER THE 1987 CONSTITUTION; DECISION OF HOUSE ELECTORAL
TRIBUNAL SUBJECT TO JUDICIAL REVIEW. — I believe that, contrary to the respondents' contentions, the Court has the jurisdiction and competence
to review the questioned decision of the House Electoral Tribunal and to decide the present controversy. Article VIII, Section 1 of the 1987
Constitution provides that: "Judicial power includes the duty of the courts of justice to settle actual controversies involving rights which are legally
demandable and enforceable, and to determine whether or not there has been a grave abuse of discretion amounting to lack or excess of jurisdiction
on the part of any branch or instrumentality of the government." The Constitution, it is true, constitutes the tribunal as the sole judge of all contests
relating to the election, returns, and qualifications of Members of the House of Representatives. But as early as 1938, it was held in Morrero vs.
Bocar (66 Phil. 429), construing Section 4, Article VI of the 1935 Constitution which provided that " . . . The Electoral Commission shall be the sole
judge of all contests relating to the election, returns and qualifications of the Members of the National Assembly." that: "The judgment rendered by
the (electoral) commission in the exercise of such and acknowledged power is beyond judicial interference, except, in any event, 'upon a clear
showing of such arbitrary and improvident use of the power as will constitute a denial of due process of law' (Barry vs. US ex rel. Cunningham, 279 US
597; 73 Law. ed., 867; Angara vs. Electoral Commission, 35 Off. Gaz., 23)." And then under the afore-quoted provisions of Article VIII, Section 1 of
the 1987 Constitution, this Court is duty-bound to determine whether or not, in an actual controversy, there has been a grave abuse of discretion
amounting to lack or excess of jurisdiction on the part of any branch or instrumentality of the government.

2. ID.; ID.; ID.; ID.; APPLIED IN CASE AT BAR. — The present controversy, involves more than perceived irregularities in the conduct of a congressional
election or a disputed appreciation of ballots, in which cases, it may be contended with great legal force and persuasion that the decision of the
electoral tribunal should be final and conclusive, for it is, by constitutional directive, made the sole judge of contests relating to such matters. The
present controversy, however, involves no less than a determination of whether the qualifications for membership in the House of
Representatives, as prescribed by the Constitution, have been met. Indeed, this Court would be unforgivably remiss in the performance of its duties,
as mandated by the Constitution, were it to allow a person, not a natural-born Filipino citizen, to continue to sit as a Member of the House of
Representatives, solely because the House Electoral Tribunal has declared him to be so. In such a case, the tribunal would have acted with grave
abuse of discretion amounting to lack or excess of jurisdiction as to require the exercise by this Court of its power of judicial review. Besides, the
citizenship and residence qualifications of private respondent for the office of Member of the House of Representatives, are here controverted by
petitioners who, at the same time, claim that they are entitled to the office illegally held by private respondent. From this additional direction, where
one asserts and earnestly perceived right that in turn is vigorously resisted by another, there is clearly a justiciable controversy proper for this Court
to consider and decide.
3. ID.; ID.; ID.; EXERCISE OF JUDICIAL REVIEW NOT VIOLATIVE OF THE PRINCIPLE OF SEPARATION OF POWERS. — The Court, in reviewing the decision
of the tribunal, does not assert supremacy over it in contravention of the time-honored principle of constitutional separation of powers. The Court in
this instance simply performs a function entrusted and assigned to it by the Constitution of interpreting, in a justiciable controversy, the pertinent
provisions of the Constitution with finality. "It is the role of the Judiciary to refine and, when necessary, correct constitutional (and/or statutory)
interpretation, in the context of the interactions of the three branches of the government, almost always in situations where some agency of the
State has engaged in action that stems ultimately from some legitimate area of governmental power (the Supreme Court in Modern Role, C.B.
Sevisher, 1958, p. 36)." Moreover, it is decidedly a matter of great public interest and concern to determine whether or not private respondent is
qualified to hold so important and high a public office which is specifically reserved by the Constitution only to natural-born Filipino citizens.
4. ID.; CITIZENSHIP; NATURAL-BORN; REQUISITE; NOT COMPLIED WITH IN CASE AT BAR. — The records show that private respondent was born on 19
June 1948 to the spouses Jose Ong Chuan, a Chinese citizen, and Agrifina E. Lao, a natural-born Filipino citizen, in Laoang, Northern Samar. In other
words, at birth, private respondent was a Chinese citizen (not a natural-born Filipino citizen) because his father was then a Chinese citizen (not a
naturalized Filipino citizen). Under the 1935 Constitution which was enforced at the time of private respondent's birth on 19 June 1948, only those
whose fathers were citizens of the Philippines were considered Filipino citizens. Those whose mothers were citizens of the Philippines had to elect
Philippine citizenship upon reaching the age of majority, in order to be considered Filipino citizens. Following the basic definition in the 1987
Constitution of a natural-born citizen, in relation to the 1935 Constitution, private respondent is not a natural-born Filipino citizen, having been born
a Chinese citizen by virtue of the Chinese citizenship of his father at the time of his birth, although from birth, private respondent had the right to
elect Philippine citizenship, the citizenship of his mother, but only upon his reaching the age of majority.
5. ID.; ID.; ID.; SECTION 15 OF THE REVISED NATURALIZATION LAW (C.A. 473); DID NOT CONFER STATUS OF NATURAL-BORN IN CASE AT BAR. —
While under Section 15 of theRevised Naturalization Law (C.A. 473) minor children of a naturalized citizen (father), who were born in the Philippines
prior to the naturalization of the parent automatically become Filipino citizens, this does not alter the fact that private respondent was not born to a
Filipino father, and the operation of Section 15 of CA 473 did not confer upon him the status of a natural-born citizen merely because he did not have
to perform any act to acquire or perfect his status as a Filipino citizen.
6. ID.; ID.; NATURALIZATION; NATURE THEREOF; PRIVILEGE NOT A RIGHT. — "Naturalization is not a right, but a privilege of the most discriminating
as well as delicate and exacting nature, affecting public interest of the highest order, and which may be enjoyed only under the precise conditions
prescribed by law therefor."
7. ID.; ID.; ID.; PETITION; GRANT THEREOF; APPEALABLE; OATH TAKEN BEFORE EXPIRATION OF THE PERIOD OF APPEAL; IMPROPER. — It is settled
that an order granting a petition to take the requisite oath of allegiance of one who has previously obtained a decision favorable to his application for
naturalization, is appealable. It is, therefore, improper and illegal to authorize the taking of said oath upon the issuance of said order and before the
expiration of the reglementary period to perfect any appeal from said order. In Cua Sun Ke vs. Republic (159 SCRA 477), this Court held that:
"Administration of the oath of allegiance on the same day as issuance of order granting citizenship is irregular and makes the proceedings so taken
null and void (Republic vs. Guy, 115 SCRA 244 [1982]; citing the case of Ong So vs. Republic of the Philippines, 121 Phil. 1381)."
8. ID.; ID.; NATURAL-BORN; DEFINED AND INTERPRETED UNDER THE 1987 CONSTITUTION. — Article IV, Section 2 of the 1987 Constitution defines
natural-born (Filipino) citizens as: "Natural-born citizens are those who are citizens of the Philippines from birth without having to perform any act to
acquire or perfect their Philippine citizenship. Those who elect Philippine citizenship in accordance with paragraph (3), Section 1 hereof shall be
deemed natural-born citizens." Article IV, Section 1, paragraph (3) of the 1987 Constitution provides that: "Section 1. The following are citizens of the
Philippines: . . . (3) Those born before January 17, 1973, of Filipino mothers, who elect Philippine citizenship upon reaching the age of majority." It
would appear then that the intent of the framers of the 1987 Constitution in defining a natural-born Filipino citizen was to equalize the position of
Filipino fathers and Filipino mothers as to their children becoming natural-born Filipino citizens. In other words, after 17 January 1973, effectivity
date of the 1973 Constitution, all those born of Filipino fathers (with alien spouse) or Filipino mothers (with alien spouse) are natural-born Filipino
citizens. But those born to Filipino mothers prior to 17 January 1973 must still elect Philippine citizenship upon reaching the age of majority, in order
to be deemed natural-born Filipino citizens. The election, which is related to the attainment of the age of majority, may be made before or after 17
January 1973. This interpretation appears to be in consonance with the fundamental purpose of the Constitution which is to protect and enhance the
people's individual interests, and to foster equality among them.
9. ID.; ID.; ELECTION THEREOF; MUST BE MADE EXPRESSLY AS PROVIDED FOR UNDER COMMONWEALTH ACT NO. 625. — It is settled doctrine in this
jurisdiction that election of Philippine citizenship must be made in accordance with Commonwealth Act 625, Sections 1 and 2 of the Act mandate
that the option to elect Philippine citizenship must be effected expressly, not impliedly.
10. ID.; ID.; ID.; CASE OF IN RE: FLORENCIO MALLARE (ADMINISTRATIVE CASE NO. 533, SEPTEMBER 12, 1974, [59 SCRA 45]) NOT APPLICABLE IN CASE
AT BAR. — The respondent tribunal cites In re: Florencio Mallare which held that Esteban Mallare's exercise of the right of suffrage when he came of
age, constituted a positive act of election of Philippine citizenship. Mallare, cited by respondent tribunal as authority for the doctrine of implied
election of Philippine citizenship, is not applicable to the case at bar. The respondent tribunal failed to consider that Esteban Mallare reached the age
of majority in 1924, or seventeen (17) years before CA 625 was approved and, more importantly, eleven (11) years before the 1935
Constitution (which granted the right of election) took effect.
11. ID.; ID.; ID.; REQUISITE PROVIDED FOR UNDER COMMONWEALTH ACT NO. 625 NOT COMPLIED WITH IN CASE AT BAR. — The respondent
tribunal erred in ruling that by operation of CA 473, the Revised Naturalization Law, providing for private respondent's acquisition of Filipino
citizenship by reason of the naturalization of his father, the law itself had already elected Philippine citizenship for him. For, assuming arguendo that
the naturalization of private respondent's father was valid, and that there was no further need for private respondent to elect Philippine citizenship
(as he had automatically become a Filipino citizen) yet, this did not mean that the operation of the Revised Naturalization Law amounted to an
election by him of Philippine citizenship as contemplated by the Constitution. Besides, election of Philippine citizenship derived from one's Filipino
mother, is made upon reaching the age of majority, not during one's minority. There is no doubt in my mind, therefore, that private respondent did
not elect Philippine citizenship upon reaching the age of majority in 1969 or within a reasonable time thereafter as required by CA 625.
Consequently, he cannot be deemed a natural-born Filipino citizen under Sections 2 and 1 (3), Article IV of the 1987 Constitution.

12. ID.; ELECTION PROTEST; QUESTIONING ELIGIBILITY OF A CANDIDATE-ELECT; IN EFFECT A QUO WARRANTO PROCEEDING; INELIGIBILITY OF
CANDIDATE-ELECT RESULTS IN NO-CHOICE. — Neither of the petitioners may take the place of private respondent in the House of Representatives
representing the second district of Northern Samar. The ruling of this Court in Ramon L. Labo, Jr. vs. The Commission on Elections (COMELEC) EN
BANC and Luis L. Lardizabal (176 SCRA 1), is controlling. There we held that Luis L. Lardizabal, who filed the quo warranto petition, could not replace
Ramon L. Labo, Jr. as mayor of Baguio City for the simple reason that as he obtained only the second highest number of votes in the election, he was
obviously not the choice of the people of Baguio City for mayor of that City. A petition alleging that the candidate-elect is not qualified for the office
is, in effect, a quo warranto proceeding even if it is labelled an election protest. It is a proceeding to unseat the ineligible person from office but not
necessarily to install the protestant in his place. The general rule is that the fact that a plurality or a majority of the votes are cast for an ineligible
candidate in an election does not entitle the candidate receiving the next highest number of votes to be declared elected. In such a case, the electors
have failed to make a choice and the election is a nullity.
13. ID.; ID.; PHILIPPINE BILL OF 1902; REQUIREMENTS PROVIDED THEREIN; NOT COMPLIED WITH IN CASE AT BAR. — The "test," following the
premises of the 1971 Constitutional Convention, is whether or not Ong Te, private respondent's and Emil L. Ong's grandfather was "an inhabitant of
the Philippines who continued to reside therein and was a Spanish subject on April 11, 1899." If he met these requirements of the Philippine Bill of
1902, then, Ong Te was a Filipino citizen; otherwise, he was not a Filipino citizen. Petitioners (protestants) submitted and offered in evidence before
the House Electoral Tribunal exhibits W, X, Y, Z, AA, BB, CC, DD and EE which are copies of entries in the "Registro de Chinos" from years 1896 to
1897 which show that Ong Te was not listed as an inhabitant of Samar where he is claimed to have been a resident. Petitioners (protestants) also
submitted and offered in evidence before the House Electoral Tribunal Exhibit V, a certification of the Chief of the Archives Division, Records and
Management and Archives Office, stating that the name of Ong Te does not appear in the "Registro de Chinos" for the province of Samar for 1895.
These exhibits prove or at least, as petitioners validly argue, tend to prove that Ong Te was NOT a resident of Samar close to 11 April 1899 and,
therefore, could not continue residing in Samar, Philippines after 11 April 1899, contrary to private respondent's pretense. In the face of these proofs
or evidence, private respondent FAILED TO PRESENT ANY REBUTTAL OR COUNTERVAILING EVIDENCE.
14. ID.; ID.; RES JUDICATA; NOT APPLICABLE. — The decision of the 1971 Constitutional Convention in the case of Emil L. Ong was a decision of
a political body, not a court of law. And, even if we have to take such a decision as a decision of a quasi-judicial body (i.e., a political body exercising
quasi-judicial functions), said decision in the Emil L. Ong case can not have the category or character of res judicata in the present judicial
controversy, because between the two (2) cases, there is no identity of parties (one involves Emil L. Ong, while the other involves private
respondent) and, more importantly, there is no, identity of causes of action because the first involves the 1935 Constitution while the second
involves the 1987 Constitution. As held in Lee vs. Commissioners on Immigration (G.R. No. L-23446, 20 December 1971, 42 SCRA 561): " . . . Everytime
the citizenship of a person is material or indispensable in a judicial or administrative case, whatever the corresponding court or administrative
authority decides therein as to such citizenship is generally not considered as res judicata, hence it has to be threshed out again and again as the
occasion may demand."
15. ID; SUPREMACY OF THE CONSTITUTION; MUST BE ENFORCED. — It is regrettable that one (as private respondent) who unquestionably obtained
the highest number of votes for the elective position of Representative (Congressman) to the House of Representatives for the second district of
Northern Samar, would have to cease in office by virtue of this Court's decision, if the full membership of the Court had participated in this case, with
the result that the legislative district would cease to have, in the interim, a representative in the House of Representatives. But the fundamental
consideration in case of this nature is the Constitution and only the Constitution. It has to be assumed, therefore, that when the electorate in the
second legislative district of Northern Samar cast the majority of their votes for private respondent, they seemed and believed that he was fully
eligible and qualified for the office because he is a natural-born Filipino citizen. That erroneous assumption and belief can not prevail over, but must
yield to the majesty of the Constitution.
SARMIENTO, J., concurring:
1. CONSTITUTIONAL LAW; ELECTORAL TRIBUNAL OF THE HOUSE OF REPRESENTATIVES; AS SOLE JUDGE OF ALL CONTEST RELATING TO MEMBERS
THEREOF; ISSUE OF CITIZENSHIP INCLUDED; BEYOND JUDICIAL INTERVENTION. — The question of citizenship is a question of fact, and as a rule, the
Supreme Court leaves facts to the tribunal that determined them. I am quite agreed that the Electoral Tribunal of the House of Representatives, as
the "sole judge" of all contests relating to the membership in the House, as follows: "Sec. 17. The Senate and the House of Representatives shall each
have an Electoral Tribunal which shall be the sole judge of all contests relating to the election, returns, and qualifications of their respective
Members. Each Electoral Tribunal shall be composed of nine Members, three of whom shall be Justices of the Supreme Court to be designated by the
Chief Justice, and the remaining six shall be Members of the Senate or the House of Representatives, as the case may be, who shall be chosen on the
basis of proportional representation from the political parties and the parties or organizations registered under the party-list system represented
therein. The senior Justice in the Electoral Tribunal shall be its Chairman." is the best judge of facts and this Court can not substitute its judgment
because it thinks it knows better.
2. ID.; SUPREME COURT; EXPANDED JURISDICTION THEREOF; REVIEW OF FACTS NOT INCLUDED. — In the case of Aratuc vs. Commission on Elections
(88 SCRA 251), it was held that this Court can not review the errors of the Commission on Elections (then the "sole judge" of all election contests) —
in the sense of reviewing facts and unearthing mistakes — and that this Court's jurisdiction is to see simply whether or not it is guilty of a grave abuse
of discretion. It is true that the new Constitution has conferred expanded powers on the Court, but as the Charter states, our authority is "to
determine whether or not there has been a grave abuse of discretion amounting to lack or excess of jurisdiction on the part of any branch or
instrumentality of the government." It is not to review facts.
3. ID.; ID.; ID.; "GRAVE ABUSE OF DISCRETION" DEFINED. — "Grave abuse of discretion" has been defined as whimsical exercise of power amounting
to excess of jurisdiction, or otherwise, to denial of due process of law.
DECISION
GUTIERREZ, JR., J p:
The petitioners come to this Court asking for the setting aside and reversal of a decision of the House of Representatives Electoral Tribunal (HRET).
The HRET declared that respondent Jose Ong, Jr. is a natural born Filipino citizen and a resident of Laoang, Northern Samar for voting purposes. The
sole issue before us is whether or not, in making that determination, the HRET acted with grave abuse of discretion.
On May 11, 1987, the congressional election for the second district of Northern Samar was held.
Among the candidates who vied for the position of representative in the second legislative district of Northern Samar are the petitioners, Sixto
Balinquit and Antonio Co and the private respondent, Jose Ong, Jr.
Respondent Ong was proclaimed the duly elected representative of the second district of Northern Samar.
The petitioners filed election protests against the private respondent premised on the following grounds:
1) Jose Ong, Jr. is not a natural born citizen of the Philippines; and
2) Jose Ong, Jr. is not a resident of the second district of Northern Samar.
The HRET, in its decision dated November 6, 1989, found for the private respondent.
A motion for reconsideration was filed by the petitioners on November 12, 1989. This was, however, denied by the HRET, in its resolution dated
February 22, 1989.
Hence, these petitions for certiorari.
We treat the comments as answers and decide the issues raised in the petitions.
ON THE ISSUE OF JURISDICTION
The first question which arises refers to our jurisdiction.
The Constitution explicitly provides that the House of Representatives Electoral Tribunal (HRET) and the Senate Electoral Tribunal (SET) shall be
the sole judges of all contests relating to the election, returns, and qualifications of their respective members. (See Article VI, Section
17, Constitution). prLL
The authority conferred upon the Electoral Tribunal is full, clear and complete. The use of the word sole emphasizes the exclusivity of the jurisdiction
of these Tribunals.
The Supreme Court in the case of Lazatin v. HRET (168 SCRA 391 [1988]) stated that under the 1987 Constitution, the jurisdiction of the Electoral
Tribunal is original and exclusive, viz:
"The use of the word 'sole' emphasizes the exclusive character of the jurisdiction conferred (Angara v. Electoral
Commission, supra at p. 162). The exercise of power by the Electoral Commission under the 1935 Constitution has been
described as 'intended to be as complete and unimpaired as if it had originally remained in the legislature.' (id., at p. 175)
Earlier this grant of power to the legislature was characterized by Justice Malcolm as 'full, clear and complete'. (Veloso v. Board
of Canvassers of Leyte and Samar, 39 Phil. 886 [1919]) Under the amended 1935 Constitution, the power was unqualifiedly
reposed upon the Electoral Tribunal and it remained as full, clear and complete as that previously granted the Legislature and
the Electoral Commission, (Lachica v. Yap, 25 SCRA 140 [1968] The same may be said with regard to the jurisdiction of the
Electoral Tribunal under the 1987 Constitution." (p. 401).

The Court continued further, ". . . so long as the Constitution grants the HRET the power to be the sole judge of all contests relating to election,
returns and qualifications of members of the House of Representatives, any final action taken by the HRET on a matter within its jurisdiction shall, as
a rule, not be reviewed by this Court . . . the power granted to the Electoral Tribunal is full, clear and complete and excludes the exercise of any
authority on the part of this Court that would in any wise restrict it or curtail it or even affect the same." (pp. 403-404)
When may the Court inquire into acts of the Electoral Tribunals under our constitutional grants of power?
In the later case of Robles v. HRET (181 SCRA 780 [1990]) the Supreme Court stated that the judgments of the Tribunal are beyond judicial
interference save only "in the exercise of this Court's so-called extraordinary jurisdiction, . . . upon a determination that the Tribunal's decision or
resolution was rendered without or in excess of its jurisdiction, or with grave abuse of discretion or paraphrasing Morrero, upon a clear showing of
such arbitrary and improvident use by the Tribunal of its power as constitutes a denial of due process of law, or upon a demonstration of a very clear
unmitigated ERROR, manifestly constituting such GRAVE ABUSE OF DISCRETION that there has to be a remedy for such abuse." (at pp. 785-786)
In the leading case of Morrero v. Bocar (66 Phil. 429 [1938]) the Court ruled that the power of the Electoral Commission "is beyond judicial
interference except, in any event, upon a clear showing of such arbitrary and improvident use of power as will constitute a denial of due process."
The Court does not venture into the perilous area of trying to correct perceived errors of independent branches of the Government. It comes in only
when it has to vindicate a denial of due process or correct an abuse of discretion so grave or glaring that no less than the Constitution calls for
remedial action. LLjur
The Supreme Court under the 1987 Constitution, has been given an expanded jurisdiction, so to speak, to review the decisions of the other branches
and agencies of the government to determine whether or not they have acted within the bounds of the Constitution. (See Article VIII, Section 1,
Constitution)
Yet, in the exercise thereof, the Court is to merely check whether or not the governmental branch or agency has gone beyond the Constitutional
limits of its jurisdiction, not that it erred or has a different view. In the absence of a showing that the HRET has committed grave abuse of discretion
amounting to lack of jurisdiction, there is no occasion for the Court to exercise its corrective power; it will not decide a matter which by its nature is
for the HRET alone to decide. (See Marcos v. Manglapus, 177 SCRA 668 [1989]) It has no power to look into what it thinks is apparent error.
As constitutional creations invested with necessary power, the Electoral Tribunals, although not powers in the tripartite scheme of the government,
are, in the exercise of their functions independent organs — independent of Congress and the Supreme Court. The power granted to HRET by the
Constitution is intended to be as complete and unimpaired as if it had remained originally in the legislature. (Angara v. Electoral Commission, 63 Phil.
139 [1936])
In passing upon petitions, the Court with its traditional and careful regard for the balance of powers, must permit this exclusive privilege of the
Tribunals to remain where the Sovereign authority has place it. (See Veloso v. Boards of Canvassers of Leyte and Samar, 39 Phil. 886 [1919])
It has been argued that under Article VI, Section 17 of the present Constitution, the situation may exist as it exists today where there is an unhealthy
one-sided political composition of the two Electoral Tribunals. There is nothing in the Constitution, however, that makes the HRET because of its
composition any less independent from the Court or its constitutional functions any less exclusive. The degree of judicial intervention should not be
made to depend on how many legislative members of the HRET belong to this party or that party. The test remains the same — manifest grave abuse
of discretion.
In the case at bar, the Court finds no improvident use of power, no denial of due process on the part of the HRET which will necessitate the exercise
of the power of judicial review by the Supreme Court.
ON THE ISSUE OF CITIZENSHIP
The records show that in the year 1895, the private respondent's grandfather, Ong Te, arrived in the Philippines from China. Ong Te established his
residence in the municipality of Laoang, Samar on land which he bought from the fruits of hard work.
As a resident of Laoang, Ong Te was able to obtain a certificate of residence from the then Spanish colonial administration.
The father of the private respondent, Jose Ong Chuan was born in China in 1905. He was brought by Ong Te to Samar in the year 1915.
Jose Ong Chuan spent his childhood in the province of Samar. In Laoang, he was able to establish an enduring relationship with his neighbors,
resulting in his easy assimilation into the community.
As Jose Ong Chuan grew older in the rural and seaside community of Laoang, he absorbed Filipino cultural values and practices. He was baptized into
Christianity. As the years passed, Jose Ong Chuan met a natural born-Filipina, Agripina Lao. The two fell in love and, thereafter, got married in 1932
according to Catholic faith and practice.
The couple bore eight children, one of whom is the private respondent who was born in 1948.
The private respondent's father never emigrated from this country. He decided to put up a hardware store and shared and survived the vicissitudes
of life in Samar.
The business prospered. Expansion became inevitable. As a result, a branch was set-up in Binondo, Manila. In the meantime, the father of the private
respondent, unsure of his legal status and in an unequivocal affirmation of where he cast his life and family, filed with the Court of First Instance of
Samar of application for naturalization on February 15, 1954. LibLex
On April 28, 1955, the CFI of Samar, after trial, declared Jose Ong Chuan a Filipino citizen.
On May 15, 1957, the Court of First Instance of Samar issued an order declaring the decision of April 28, 1955 as final and executory and that Jose
Ong Chuan may already take his Oath of Allegiance.
Pursuant to said order, Jose Ong Chuan took his Oath of Allegiance; correspondingly, a certificate of naturalization was issued to him.
At the time Jose Ong Chuan took his oath, the private respondent then a minor of nine years was finishing his elementary education in the province
of Samar. There is nothing in the records to differentiate him from other Filipinos insofar as the customs and practices of the local populace were
concerned.
Fortunes changed. The house of the family of the private respondent in Laoang, Samar was burned to the ground.
Undaunted by the catastrophe, the private respondent's family constructed another one in place of their ruined house. Again, there is no showing
other than that Laoang was their abode and home.
After completing his elementary education, the private respondent, in search for better education, went to Manila in order to acquire his secondary
and college education.
In the meantime, another misfortune was suffered by the family in 1975 when a fire gutted their second house in Laoang, Samar. The respondent's
family constructed still another house, this time a 16-door apartment building, two doors of which were reserved for the family.
The private respondent graduated from college, and thereafter took and passed the CPA Board Examinations.
Since employment opportunities were better in Manila, the respondent looked for work here. He found a job in the Central Bank of the Philippines as
an examiner. Later, however, he worked in the hardware business of his family in Manila. In 1971, his elder brother, Emil, was elected as a delegate
to the 1971 Constitutional Convention. His status as a natural born citizen was challenged. Parenthetically, the Convention which in drafting the
Constitution removed the unequal treatment given to derived citizenship on the basis of the mother's citizenship formally and solemnly declared
Emil Ong, respondent's full brother, as a natural born Filipino. The Constitutional Convention had to be aware of the meaning of natural born
citizenship since it was precisely amending the article on this subject. cdll
The private respondent frequently went home to Laoang, Samar, where he grew up and spent his childhood days.
In 1984, the private respondent married a Filipina named Desiree Lim.
For the elections of 1984 and 1986, Jose Ong, Jr. registered himself as a voter of Laoang, Samar, and correspondingly, voted there during those
elections.
The private respondent after being engaged for several years in the management of their family business decided to be of greater service to his
province and ran for public office. Hence, when the opportunity came in 1987, he ran in the elections for representative in the second district of
Northern Samar.
Mr. Ong was overwhelmingly voted by the people of Northern Samar as their representative in Congress. Even if the total votes of the two
petitioners are combined, Ong would still lead the two by more than 7,000 votes.
The pertinent portions of the Constitution found in Article IV read:.
"SECTION 1, the following are citizens of the Philippines:
1. Those who are citizens of the Philippines at the time of the adoption of the Constitution;
2. Those whose fathers or mothers are citizens of the Philippines;
3. Those born before January 17, 1973, of Filipino mothers, who elect Philippine citizenship upon reaching the age of majority;
and
4. Those who are naturalized in accordance with law.
SECTION 2, Natural-born Citizens are those who are citizens of the Philippines from birth without having to perform any act to
acquire or perfect their citizenship. Those who elect Philippine citizenship in accordance with paragraph 3 hereof shall be
deemed natural born citizens."
The Court interprets Section 1, Paragraph 3 above as applying not only to those who elect Philippine citizenship after February 2, 1987 but also to
those who, having been born of Filipino mothers, elected citizenship before that date.
The provision in Paragraph 3 was intended to correct an unfair position which discriminates against Filipino women. There is no ambiguity in the
deliberations of the Constitutional Commission, viz:
"Mr. Azcuna:
With respect to the provision of section 4, would this refer only to those who elect Philippine citizenship after the effectivity of
the 1973 Constitution or would it also cover those who elected it under the 1973 Constitution?
Fr. Bernas:
It would apply to anybody who elected Philippine citizenship by virtue of the provision of the 1935 Constitution whether the
election was done before or after January l7, 1973." (Records of the Constitutional Commission, Vol. 1, p. 228;
Emphasis supplied).
xxx xxx xxx
"Mr. Trenas:
The Committee on Citizenship, Bill of Rights, Political Rights and Obligations and Human Rights has more or less decided to
extend the interpretation of who is a natural-born Citizen as provided in section 4 of the 1973 Constitution by adding
that persons who have elected Philippine Citizenship under the 1935 Constitution shall be natural-born? Am I right
Mr. Presiding Officer?
Fr. Bernas:
yes."
xxx xxx xxx
"Mr. Nolledo:
And I remember very well that in the Reverend Father Bernas' well written book, he said that the decision was designed
merely to accommodate former delegate Ernesto Ang and that the definition on natural-born has no retroactive
effect. Now it seems that the Reverend Father Bernas is going against this intention by supporting the amendment?
Fr. Bernas:
As the Commissioner can see, there has been an evolution in my thinking. (Records of the Constitutional Commission, Vol. 1,
p. 189)
xxx xxx xxx
"Mr. Rodrigo:
But this provision becomes very important because his election of Philippine citizenship makes him not only a Filipino citizen
but a natural-born Filipino citizen entitling him to run for Congress . . .
Fr. Bernas:
Correct. We are quite aware of that and for that reason we will leave it to the body to approve that provision of section 4.
Mr. Rodrigo:
I think there is a good basis for the provision because it strikes me as unfair that the Filipino citizen who was born a day before
January 17, 1973 cannot be a Filipino citizen or a natural born citizen." (Records of the Constitutional Commission,
Vol. 1, p. 231)
xxx xxx xxx
"Mr. Rodrigo:
The purpose of that provision is to remedy an inequitable situation. Between 1935 and 1973 when we were under the 1935
Constitution, those born of Filipino fathers but alien mothers were natural-born Filipinos. However, those born of
Filipino mothers but alien fathers would have to elect Philippine citizenship upon reaching the age of majority; and if
they do elect, they become Filipino citizens but not natural-born Filipino citizens." (Records of the Constitutional
Commission, Vol. 1, p. 356)
The foregoing significantly reveals the intent of the framers. To make the provision prospective from February 3, 1987 is to give a narrow
interpretation resulting in an inequitable situation. It must also be retroactive.
It should be noted that in construing the law, the Courts are not always to be hedged in by the literal meaning of its language. The spirit and
intendment thereof, must prevail over the letter, especially where adherence to the latter would result in absurdity and injustice. (Casela v. Court of
Appeals, 35 SCRA 279 [1970])
A Constitutional provision should be construed so as to give it effective operation and suppress the mischief at which it is aimed, hence, it is the spirit
of the provision which should prevail over the letter thereof. (Jarrolt v. Mabberly, 103 U.S. 580)
In the words of the Court in the case of J.M. Tuason v. LTA (31 SCRA 413 [1970]:
"To that primordial intent, all else is subordinated. Our Constitution,any constitution is not to be construed narrowly or
pedantically, for the prescriptions therein contained, to paraphrase Justice Holmes, are not mathematical formulas having their
essence in their form but are organic living institutions, the significance of which is vital not formal . . ." (p. 427)
The provision in question was enacted to correct the anomalous situation where one born of a Filipino father and an alien mother was automatically
granted the status of a natural-born citizen while one born of a Filipino mother and an alien father would still have to elect Philippine citizenship. If
one so elected, he was not, under earlier laws, conferred the status of a natural-born.
Under the 1973 Constitution, those born of Filipino fathers and those born of Filipino mothers with an alien father were placed on equal footing.
They were both considered as natural-born citizens.
Hence, the bestowment of the status of "natural-born" cannot be made to depend on the fleeting accident of time or result in two kinds of citizens
made up of essentially the same similarly situated members.
It is for this reason that the amendments were enacted, that is, in order to remedy this accidental anomaly, and, therefore, treat equally all those
born before the 1973 Constitution and who elected Philippine citizenship either before or after the effectivity of that Constitution. Cdpr
The Constitutional provision in question is, therefore curative in nature. The enactment was meant to correct the inequitable and absurd situation
which then prevailed, and thus, render those acts valid which would have been nil at the time had it not been for the curative provisions. (See
Development Bank of the Philippines v. Court of Appeals, 96 SCRA 342 [1980])
There is no dispute that the respondent's mother was a natural born Filipina at the time of her marriage. Crucial to this case is the issue of whether
or not the respondent elected or chose to be a Filipino citizen.
Election becomes material because Section 2 of Article IV of the Constitution accords natural born status to children born of Filipino mothers before
January 17, 1973, if they electcitizenship upon reaching the age of majority.
To expect the respondent to have formally or in writing elected citizenship when he came of age is to ask for the unnatural and unnecessary. The
reason is obvious. He was already a citizen. Not only was his mother a natural born citizen but his father had been naturalized when the respondent
was only nine (9) years old. He could not have divined when he came of age that in 1973 and 1987 the Constitution would be amended to require
him to have filed a sworn statement in 1969 electing citizenship inspite of his already having been a citizen since 1957. In 1969, election through a
sworn statement would have been an unusual and unnecessary procedure for one who had been a citizen since he was nine years old.
We have jurisprudence that defines "election" as both a formal and an informal process.
In the case of In Re: Florencio Mallare (59 SCRA 45 [1974]), the Court held that the exercise of the right of suffrage and the participation in election
exercises constitute a positive act of election of Philippine citizenship. In the exact pronouncement of the Court, we held:
"Esteban's exercise of the right of suffrage when he came of age, constitutes a positive act of election of Philippine citizenship".
(p. 52; emphasis supplied)
The private respondent did more than merely exercise his right of suffrage. He has established his life here in the Philippines.
For those in the peculiar situation of the respondent who cannot be expected to have elected citizenship as they were already citizens, we apply
the In Re Mallare rule.
The respondent was born in an outlying rural town of Samar where there are no alien enclaves and no racial distinctions. The respondent has lived
the life of a Filipino since birth. His father applied for naturalization when the child was still a small boy. He is a Roman Catholic. He has worked for a
sensitive government agency. His profession requires citizenship for taking the examinations and getting a license. He has participated in political
exercises as a Filipino and has always considered himself a Filipino citizen. There is nothing in the records to show that he does not embrace
Philippine customs and values, nothing to indicate any tinge of alien-ness, no acts to show that this country is not his natural homeland. The mass of
voters of Northern Samar are fully aware of Mr. Ong's parentage. They should know him better than any member of this Court will ever know him.
They voted by overwhelming numbers to have him represent them in Congress. Because of his acts since childhood, they have considered him as a
Filipino.
The filing of sworn statement or formal declaration is a requirement for those who still have to elect citizenship. For those already Filipinos when the
time to elect came up, there are acts of deliberate choice which cannot be less binding. Entering a profession open only to Filipinos, serving in public
office where citizenship is a qualification, voting during election time, running for public office, and other categorical acts of similar nature are
themselves formal manifestations of choice for these persons. LLjur
An election of Philippine citizenship presupposes that the person electing is an alien. Or his status is doubtful because he is a national of two
countries. There is no doubt in this case about Mr. Ong's being a Filipino when he turned twenty-one (21).
We repeat that any election of Philippine citizenship on the part of the private respondent would not only have been superfluous but it would also
have resulted in an absurdity. How can a Filipino citizen elect Philippine citizenship?
The respondent HRET has an interesting view as to how Mr. Ong elected citizenship. It observed that "when protestee was only nine years of age, his
father, Jose Ong Chuan became a naturalized Filipino. Section 15 of the Revised Naturalization Act squarely applies its benefit to him for he was then
a minor residing in this country. Concededly, it was the law itself that had already elected Philippine citizenship for protestee by declaring him as
such." (Emphasis supplied)

The petitioners argue that the respondent's father was not, validly, a naturalized citizen because of his premature taking of the oath of citizenship.
The Court cannot go into the collateral procedure of stripping Mr. Ong's father of his citizenship after his death and at this very late date just so we
can go after the son.
The petitioners question the citizenship of the father through a collateral approach. This can not be done. In our jurisdiction, an attack on a person's
citizenship may only be done through a direct action for its nullity. (See Queto v. Catolico, 31 SCRA 52 [1970]).
To ask the Court to declare the grant of Philippine citizenship to Jose Ong Chuan as null and void would run against the principle of due process. Jose
Ong Chuan has already been laid to rest. How can he be given a fair opportunity to defend himself. A dead man cannot speak. To quote the words of
the HRET: "Ong Chuan's lips have long been muted to perpetuity by his demise and obviously he could not rise beyond where his mortal remains now
lie to defend himself were this matter to be made a central issue in this case."
The issue before us is not the nullification of the grant of citizenship to Jose Ong Chuan. Our function is to determine whether or not the HRET
committed abuse of authority in the exercise of its powers. Moreover, the respondent traces his natural born citizenship through his mother, not
through the citizenship of his father. The citizenship of the father is relevant only to determine whether or not the respondent "chose" to be a
Filipino when he came of age. At that time and up to the present, both mother and father were Filipinos. Respondent Ong could not have elected any
other citizenship unless he first formally renounced Philippine citizenship in favor of a foreign nationality. Unlike other persons faced with a problem
of election, there was no foreign nationality of his father which he could possibly have chosen.
There is another reason why we cannot declare the HRET as having committed manifest grave abuse of discretion. The same issue of natural-born
citizenship has already been decided by the Constitutional Convention of 1971 and by the Batasang Pambansa convened by authority of the
Constitution drafted by that Convention. Emil Ong, full blood brother of the respondent, was declared and accepted as a natural born citizen by both
bodies.
Assuming that our opinion is different from that of the Constitutional Convention, the Batasang Pambansa, and the respondent HRET, such a
difference could only be characterized as error. There would be no basis to call the HRET decision so arbitrary and whimsical as to amount to grave
abuse of discretion.
What was the basis for the Constitutional Convention's declaring Emil Ong a natural born citizen?
Under the Philippine Bill of 1902, inhabitants of the Philippines who were Spanish subjects on the 11th day of April 1899 and then residing in said
islands and their children born subsequent thereto were conferred the status of a Filipino citizen.
Was the grandfather of the private respondent a Spanish subject?
Article 17 of the Civil Code of Spain enumerates those who were considered Spanish Subjects, viz:
"ARTICLE 17. The following are Spaniards:
1. Persons born in Spanish territory.
2. Children born of a Spanish father or mother, even though they were born out of Spain.
3. Foreigners who may have obtained naturalization papers.
4. Those without such papers, who may have acquired domicile in any town in the Monarchy." (Emphasis supplied)
The domicile of a natural person is the place of his habitual residence. This domicile, once established is considered to continue and will not be
deemed lost until a new one is established. (Article 50, NCC; Article 40, Civil Code of Spain; Zuellig v. Republic, 83 Phil. 768 [1949])
As earlier stated, Ong Te became a permanent resident of Laoang, Samar around 1895. Correspondingly, a certificate of residence was then issued to
him by virtue of his being a resident of Laoang, Samar. (Report of the Committee on Election Protests and Credentials of the 1971 Constitutional
Convention, September 7,1972, p. 3)
The domicile that Ong Te established m 1895 continued until April 11, 1899; it even went beyond the turn of the 19th century. It is also in this place
were Ong Te set-up his business and acquired his real property.
As concluded by the Constitutional Convention Ong Te falls within the meaning of sub-paragraph 4 of Article 17 of the Civil Code of Spain.
Although Ong Te made brief visits to China, he, nevertheless, always returned to the Philippines. The fact that he died in China, during one of his
visits in said country, was of no moment. This will not change the fact that he already had his domicile fixed in the Philippines and pursuant to
the Civil Code of Spain, he had become a Spanish subject. LibLex
If Ong Te became a Spanish subject by virtue of having established his domicile in a town under the Monarchy of Spain, necessarily, Ong Te was also
an inhabitant of the Philippines for an inhabitant has been defined as one who has actual fixed residence in a place; one who has a domicile in a
place. (Bouvier's Law Dictionary, Vol. II) A priori, there can be no other logical conclusion but to educe that Ong Te qualified as a Filipino citizen under
the provisions of section 4 of the Philippine Bill of 1902.
The HRET itself found this fact of absolute verity in concluding that the private respondent was a natural-born Filipino.
The petitioners' sole ground in disputing this fact is that the documents presented to prove it were not in compliance with the best evidence rule.
The petitioners allege that the private respondent failed to present the original of the documentary evidence, testimonial evidence and of the
transcript of the proceedings of the body which the aforesaid resolution of the 1971 Constitutional Convention was predicated.
On the contrary, the documents presented by the private respondent fall under the exceptions to the best evidence rule.
It was established in the proceedings before the HRET that the originals of the Committee Report No. 12, the minutes of the plenary session of 1971
Constitutional Convention held on November 28, 1972 cannot be found.
This was affirmed by Atty. Ricafrente, Assistant Secretary of the 1971 Constitutional Convention; by Atty. Nolledo, Delegate to the 1971
Constitutional Convention; and by Atty. Antonio Santos, Chief Librarian of the U.P. Law Center, in their respective testimonies given before the HRET
to the effect that there is no governmental agency which is the official custodian of the records of the 1971 Constitutional Convention. (TSN,
December 12, 1988, pp. 30-31; TSN, January 17, 1989, pp. 34-35; TSN, February 1, 1989, p. 44; TSN, February 6, 1989, pp. 28-29)
The execution of the originals was established by Atty. Ricafrente, who as the Assistant Secretary of the 1971 Constitutional Convention was the
proper party to testify to such execution. (TSN, December 12, 1989, pp. 11-24)
The inability to produce the originals before the HRET was also testified to as aforestated by Atty. Ricafrente, Atty. Nolledo, and Atty. Santos. In
proving the inability to produce, the law does not require the degree of proof to be of sufficient certainty; it is enough that it be shown that after a
bona fide diligent search, the same cannot be found. (see Government of P.I. v. Martinez, 44 Phil. 817 [1918])
Since the execution of the document and the inability to produce were adequately established, the contents of the questioned documents can be
proven by a copy thereof or by the recollection of witnesses.
Moreover, to erase all doubts as to the authenticity of the documentary evidence cited in the Committee Report, the former member of the 1971
Constitutional Convention, Atty. Nolledo, when he was presented as a witness in the hearing of the protest against the private respondent,
categorically stated that he saw the disputed documents presented during the hearing of the election protest against the brother of the private
respondent. (TSN, February 1, 1989, pp. 8-9)
In his concurring opinion, Mr. Justice Sarmiento, a vice-president of the Constitutional Convention, states that he was presiding officer of the plenary
session which deliberated on the report on the election protest against Delegate Emil Ong. He cites a long list of names of delegates present. Among
them are Mr. Chief Justice Fernan, and Mr. Justice Davide, Jr. The petitioners could have presented any one of the long list of delegates to refute Mr.
Ong's having been declared a natural-born citizen. They did not do so. Nor did they demur to the contents of the documents presented by the private
respondent. They merely relied on the procedural objections respecting the admissibility of the evidence presented.
The Constitutional Convention was the sole judge of the qualifications of Emil Ong to be a member of that body. The HRET, by explicit mandate of the
Constitution, is the sole judge of the qualifications of Jose Ong, Jr. to be a member of Congress. Both bodies deliberated at length on the
controversies over which they were sole judges. Decisions were arrived at only after a full presentation of all relevant factors which the parties
wished to present. Even assuming that we disagree with their conclusions, we cannot declare their acts as committed with grave abuse of discretion.
We have to keep clear the line between error and grave abuse.
ON THE ISSUE OF RESIDENCE
The petitioners question the residence qualification of respondent Ong.
The petitioners lose sight of the meaning of "residence" under the Constitution. The term "residence" has been understood as synonymous
with domicile not only under the previous Constitutions but also under the 1987 Constitution.
The deliberations of the Constitutional Commission reveal that the meaning of residence vis-a-vis the qualifications of a candidate for Congress
continues to remain the same as that of domicile, to wit:

"Mr. Nolledo:
With respect to Section 5, I remember that in the 1971 Constitutional Convention, there was an attempt to require residence
in the place not less than one year immediately preceding the day of the elections. So my question is: What is the
Committee's concept of residence of a candidate for the legislature? Is it actual residence or is it the concept of
domicile or constructive residence?
Mr. Davide:
Madame President, insofar as the regular members of the National Assembly are concerned, the proposed section merely
provides, among others, 'and a resident thereof, that is, in the district, for a period of not less than one year
preceding the day of the election'. This was in effect lifted from the 1973 Constitution, the interpretation given to it
was domicile." (Records of the 1987 Constitutional Convention, Vol. II, July 22, 1986, p. 87)
xxx xxx xxx
"Mrs. Rosario Braid:
The next question is on Section 7, page 2. I think Commissioner Nolledo has raised the same point that 'resident' has been
interpreted at times as a matter of intention rather than actual residence.
Mr. De los Reyes:
Domicile.
Ms. Rosario Braid:
Yes, So, would the gentlemen consider at the proper time to go back to actual residence rather than mere intention to reside?
Mr. De los Reyes:
But we might encounter some difficulty especially considering that a provision in the Constitution in the Article on Suffrage
says that Filipinos living abroad may vote as enacted by law. So, we have to stick to the original concept that it should
be by domicile and not physical and actual residence." (Records of the 1987 Constitutional Commission, Vol. II, July
22, 1986, p. 110)
The framers of the Constitution adhered to the earlier definition given to the word "residence" which regarded it as having the same meaning as
domicile.
The term "domicile" denotes a fixed permanent residence to which when absent for business or pleasure, one intends to return. (Ong Huan Tin v.
Republic, 19 SCRA 966 [1967]) The absence of a person from said permanent residence, no matter how long, notwithstanding, it continues to be the
domicile of that person. In other words, domicile is characterized by animus revertendi. (Ujano v. Republic, 17 SCRA 147 [1966]) cdphil
The domicile of origin of the private respondent, which was the domicile of his parents, is fixed at Laoang, Samar. Contrary to the petitioners'
imputation, Jose Ong, Jr. never abandoned said domicile; it remained fixed therein even up to the present.
The private respondent, in the proceedings before the HRET, sufficiently established that after the fire that gutted their house in 1961, another one
was constructed.
Likewise, after the second fire which again destroyed their house in 1975, a sixteen-door apartment was built by their family, two doors of which
were reserved as their family residence. (TSN, Jose Ong, Jr., November 18, 1988, p. 8)
The petitioners' allegation that since the private respondent owns no property in Laoang, Samar, he cannot, therefore, be a resident of said place is
misplaced.
The properties owned by the Ong Family are in the name of the private respondent's parents. Upon the demise of his parents, necessarily, the
private respondent, pursuant to the laws of succession, became the co-owner thereof (as a co-heir), notwithstanding the fact that these were still in
the names of his parents.
Even assuming that the private respondent does not own any property in Samar, the Supreme Court in the case of De los Reyes D. Solidum (61 Phil.
893 [1935]) held that it is not required that a person should have a house in order to establish his residence and domicile. It is enough that he should
live in the municipality or in a rented house or in that of a friend or relative. (Emphasis supplied)
To require the private respondent to own property in order to be eligible to run for Congress would be tantamount to a property qualification. The
Constitution only requires that the candidate meet the age, citizenship, voting and residence requirements. Nowhere is it required by the
Constitution that the candidate should also own property in order to be qualified to run. (see Maquera v. Borra, 122 Phil. 412 [1965])
It has also been settled that absence from residence to pursue studies or practice a profession or registration as a voter other than in the place
where one is elected, does not constitute loss of residence. (Faypon v. Quirino, 96 Phil. 294 [1954])
As previously stated, the private respondent stayed in Manila for the purpose of finishing his studies and later to practice his profession. There was
no intention to abandon the residence in Laoang, Samar. On the contrary, the periodical journeys made to his home province reveal that he always
had the animus revertendi.
The Philippines is made up not only of a single race; it has, rather, undergone an interracial evolution. Throughout our history, there has been a
continuing influx of Malays, Chinese, Americans, Japanese, Spaniards and other nationalities. This racial diversity gives strength to our country.
Many great Filipinos have not been whole-blooded nationals, if there is such a person, for there is none. To mention a few, the great Jose Rizal was
part Chinese, the late Chief Justice Claudio Teehankee was part Chinese, and of course our own President, Corazon Aquino is also part Chinese.
Verily, some Filipinos of whom we are proud were ethnically more Chinese than the private respondent.
Our citizens no doubt constitute the country's greatest wealth. Citizenship is a special privilege which one must forever cherish.
However, in order to truly revere this treasure of citizenship, we do not, on the basis of too harsh an interpretation, have to unreasonably deny it to
those who qualify to share in its richness.
Under the overly strict jurisprudence surrounding our antiquated naturalization laws only the very affluent backed by influential patrons, who were
willing to suffer the indignities of a lengthy, sometimes humiliating, and often corrupt process of clearances by minor bureaucrats and whose lawyers
knew how to overcome so many technical traps of the judicial process were able to acquire citizenship. It is time for the naturalization law to be
revised to enable a more positive, affirmative, and meaningful examination of an applicant's suitability to be a Filipino. A more humane, more
indubitable and less technical approach to citizenship problems is essential.
WHEREFORE, the petitions are hereby DISMISSED. The questioned decision of the house of Representatives Electoral Tribunal is AFFIRMED.
Respondent Jose Ong, Jr. is declared a natural-born citizen of the Philippines and a resident of Laoang, Northern Samar.
SO ORDERED.
Bidin, Griño-Aquino, Medialdea and Davide, Jr., JJ ., concur.
Fernan, C .J ., Melencio-Herrera, Cruz, Feliciano and Gancayco, JJ ., took no part.
||| (Co v. House of Representatives Electoral Tribunal, G.R. Nos. 92191-92, 92202-03, [July 30, 1991], 276 PHIL 758-830)
EN BANC
[G.R. No. L-21289. October 4, 1971.]
MOY YA LIM YAO alias EDILBERTO AGUINALDO LIM and LAU YUEN YEUNG, petitioners-appellants, vs. THE COMMISSIONER
OF IMMIGRATION, respondent-appellee.
Aruego, Mamaril & Associates for petitioners-appellants.
Solicitor General Arturo A. Alafriz, Asst. Sol. Gen. Frine C . Zaballero and Solicitor Sumilang V . Bernardo for respondent-appellee.
SYLLABUS
1. POLITICAL LAW; CITIZENSHIP; IMMIGRATION ACT; SECTION 9 (G) THEREOF, NOT APPLICABLE TO ALIEN WHO LEGITIMATELY BECOMES FILIPINO. —
Section 9 (g) of the Immigration Act does not apply to aliens who after coming into the Philippines as temporary visitors, legitimately become Filipino
citizens or acquire Filipino citizenship. Such change of nationality naturally bestows upon them the right to stay in the Philippines permanently or not,
as they may choose, and if they elect to reside here, the immigration authorities may neither deport them nor confiscate their bonds.
2. ID.; ID.; NATURALIZATION; EFFECTS. — The naturalization of an alien visitor as a Philippine citizen logically produces the effect of conferring upon
him ipso facto all the rights of citizenship including that of being entitled to permanently stay in the Philippines outside the orbit of authority of the
Commissioner of Immigration vis-avis aliens, if only because by its very nature and express provisions, the Immigration Law is a law only for aliens
and is inapplicable to citizens of the Philippines.
3. STATUTORY CONSTRUCTION; WHERE LANGUAGE OF STATUTE IS SUSCEPTIBLE OF TWO CONSTRUCTIONS, THAT WHICH CARRIES OUT OBJECT
PREVAILS. — A statute is to be construed with reference to its manifest object, and if the language is susceptible of two constructions, one which will
carry out and the other defeat such manifest object, it should receive the former construction. A construction will cause objectionable results should
be avoided and the court will, if possible, place on the statute a construction which will not result in injustice, and in accordance with the decisions
construing statutes, a construction will not result in oppression, hardship, or inconveniences will also be avoided, as will a construction which will
prejudice public interest, or construction resulting in unreasonableness, as well as a construction which will result in absurd consequences.
4. ID.; CONSTRUCTION AVOIDED IF INCONSISTENT WITH LEGISLATIVE INTENT. — So a construction should, if possible, be avoided if the result would
be an apparent inconsistency in legislative intent, as has been determined by the judicial decisions, or which would result in futility, redundancy, or a
conclusion not contemplated by the legislature; and the court should adopt that construction which will be the least likely to produce mischief.
Unless plainly shown to have been the intention of the legislature an interpretation which would render the requirements of the statute uncertain
and vague is to be avoided, and the court will not ascribe to the legislature an intent to confer an illusory right.
5. POLITICAL LAW; CITIZENSHIP; NATURALIZATION; POLICY OF SELECTIVE ADMISSION, EXPLAINED. — The avowed policy of "selective admission"
more particularly refers to a case where a citizenship is sought to be acquired in a judicial proceeding for naturalization. In such a case, the courts
should no doubt apply the national policy of selecting only those who are worthy to be come citizens. There is here a choice between accepting or
rejecting the application for citizenship. But this policy finds no application is cases where citizenship is conferred by operation of law. In such cases,
the courts have no choice to accept or reject. If the individual claiming citizenship by operations of law proves in legal proceedings that he satisfies
the statutory requirements, the cannot do otherwise than to declare that he is a citizens of the Philippines.
6. ID.; ID.; ID.; ALIEN WOMAN MARRYING FILIPINO IPSO FACTO BECOME CITIZEN PROVIDED NOT DISQUALIFIED BY LAW. — We now hold, all previous
decisions of this Court indicating otherwise notwithstanding, that under Section 15 of Commonwealth Act 473, an alien woman marrying a Filipino,
native-born or naturalized, becomes ipso facto a Filipina provided she is not disqualified to be a citizen of the Philippines under Section 4 of the same
law. Likewise, an alien woman married an alien who is subsequently naturalized here follows the Philippine citizenship of her husband the moment
he takes his oath as Filipino citizens, provided that she does not suffer from any of the disqualifications under said Section 4.
7. ID.; ID.; ID.; ID.; NATURALIZATION PROCEEDING, NOT REQUIRED. — Section 16 is a parallel provision to Section 15. If the widow of an applicant for
naturalization a Filipino, who dies during the proceedings, is not required to go through a naturalization proceeding, in order to be considered as a
Filipino citizen hereof, it should not follow that the wife of a living Filipino cannot be denied that same privilege. This is plain common sense and
there is absolutely no evidence that the Legislature intended to treat them differently.
8. ID.; ID.; ID.; MODES OF. — The Constitution itself recognizes as Philippine citizens "Those who are naturalized in accordance with law" (Section 1
[5], Article IV, Philippine Constitution). Citizens by naturalization, under this provision, include not only those who are naturalized in accordance with
legal proceedings for the acquisition of citizenship, but also those who acquire citizenship by "derivative naturalization" or by operation of law, as, for
example, the "naturalization" of an alien wife through the naturalization of her husband, or by marriage of an alien woman to a citizen.
9. ID.; ID.; ID.; SECTION 15 OF REVISED NATURALIZATION LAW; PURPOSE. — The leading idea or purpose of Section 15 was to confer Philippine
citizenship by operation of law upon certain classes of aliens as a legal consequence of their relationship, by blood or by affinity, to persons who are
already citizens of the Philippines. Whenever the fact of relationship of the persons enumerated in the provisions concurs with the fact of
citizenship of the person to who they are related, the effect is for said person to become ipso facto citizens of the Philippines. "Ipso facto" as here
used does not mean that all alien wives and all minor children of the Philippine citizens, from the mere fact of relationship, necessarily become such
citizens also. Those who do not meet the statutory requirements do not ipso facto become citizens; they must apply for naturalization in order to
acquire such status. What it does mean, however, is that in respect of those persons enumerated in Section 15, the relationship to a citizen of the
Philippines is the operative fact which establishes the acquisition of Philippine citizenship by them. Necessarily, it also determines the point of time at
which such citizenship commences.
10. ID.; ID.; ID.; ID.; ALIEN WIFE DEEMED A CITIZEN IF SHE MIGHT HERSELF BE NATURALIZED. — The legislature could not have intended that an alien
wife should not be deemed a Philippine citizen unless and until she proves that she might herself be lawfully naturalized. Far from it, the law states in
plain terms that she shall be deemed a citizen of the Philippines if she is one "who might herself be lawfully naturalized." The proviso that she must
be one "who might herself be lawfully naturalized" is not a condition precedent to the vesting or acquisition of citizenship; it is only a condition or a
state of fact necessary to establish her citizenship as a factum probandum, i.e., as a fact established and proved in evidence. The word "might," as
used in that phrase, precisely implies that at the time of her marriage to Philippine citizen, the alien woman "had (the) power" to become such a
citizen herself under the laws then in force.
11. ID.; ID.; RES JUDICATA NOT APPLICABLE TO RULINGS THEREON. — Everytime the citizenship of a person is material or indispensable in a judicial
or administrative case, whatever the corresponding court or administrative authority decides therein as to such citizenship is generally not
considered as res adjudicata, hence it has to be threshed out again and again as the occasion may demand.
12. ID.; ID.; NATURALIZATION; PROCEDURES FOR ALIEN WIFE TO ACQUIRE PHILIPPINE CITIZENSHIP. — Regarding the steps that should be taken by an
alien woman married to a Filipino citizen in order to acquire Philippine citizenship, the procedure followed in the Bureau of Immigration is as follows:
The alien woman must file a petition for the cancellation of her alien certificate of registration alleging, among other things, that she is married to a
Filipino citizen and that she is not disqualified from acquiring her husband's citizenship pursuant to Section 4 of Commonwealth Act No. 473, as
amended. Upon the filing of the said petition, which should be accompanied or supported by the joint affidavit of the petitioner and her Filipino
husband to the effect that the petitioner does not belong to any of the groups disqualified by the cited Section from becoming naturalized Filipino
citizen, the Bureau of Immigration conducts an investigation and thereafter promulgates its order or decision granting or denying the petition.
REYES, J.B.L., J., dissenting:
POLITICAL LAW; CITIZENSHIP; NATURALIZATION; ALIEN WOMAN MARRIED TO FILIPINO MUST PROVE QUALIFICATIONS UNDER SECTION 3. — Our
naturalization law separates qualifications from disqualifications; the positive qualifications under Section 3 thereof express a policy of restriction as
to candidates for naturalization as much as the disqualifications under Section 4. And it has been shown in our decision in the second Ly Giok Ha case
(Ly Giok Ha vs. Galang, L-21332 March 18, 1966, 16 SCRA 416) that those not disqualified under Section 4 would not necessarily qualify under Section
3, even if the residence qualification were disregarded. In other words, by giving to Section 15 of our Naturalization Law the effect of excluding only
those women suffering from disqualification under Section 3 could result in admitting to citizenship woman that Section 2 intends to exclude. In
these circumstances, I do not see why American interpretation of the words who might herself be lawfully naturalized should be considered hinding
in this jurisdiction.

DECISION
BARREDO, J p:
Appeal from the following decision of the Court of First Instance of Manila in its Civil Case No. 49705 entitled Moy Ya Lim Yao, etc., et al. vs. The
Commissioner of Immigration which, brief as it is, sufficiently depicts the factual setting of and the fundamental issues involved in this case thus:
"In the instant case, petitioners seek the issuance of a writ of injunction against the Commissioner of Immigration, 'restraining
the latter and/or his authorized representative from ordering plaintiff Lau Yuen Yeung to leave the Philippines and causing her
arrest and deportation and the confiscation of her bond, upon her failure to do so.'
"The prayer for preliminary injunction embodied in the complaint, having been denied, the case was heard on the merits and
the parties submitted their respective evidence.
"The facts of the case, as substantially and correctly stated by the Solicitor General are these:
'On February 8, 1961, Lau Yuen Yeung applied for a passport visa to enter the Philippines as a non-immigrant. In the
interrogation made in connection with her application for a temporary visitor's visa to enter the Philippines, she
stated that she was a Chinese residing at Kowloon, Hongkong, and that she desired to take a pleasure trip to the
Philippines to visit her great (grand) uncle Lau Ching Ping for a period of one month (Exhibits '1,' '1-a,' and '2'). She
was permitted to come into the Philippines on March 13, 1961, and was permitted to stay for a period of one month
which would expire on April 13, 1961. On the date of her arrival, Asher Y, Cheng filed a bond in the amount of
P1,000.00 to undertake, among others, that said Lau Yuen Yeung would actually depart from the Philippines on or
before the expiration of her authorized period of stay in this country or within the period as in his discretion the
Commissioner of Immigration or his authorized representative might properly allow. After repeated extensions,
petitioner Lau Yuen Yeung was allowed to stay in the Philippines up to February 13, 1962 (Exhibit '4'). On January 25,
1962, she contracted marriage with Moy Ya Lim Yao alias Edilberto Aguinaldo Lim an alleged Filipino citizen. Because
of the contemplated action of respondent to confiscate her bond and order her arrest and immediate deportation,
after the expiration of her authorized stay, she brought this action for injunction with preliminary injunction. At the
hearing which took place one and a half years after her arrival, it was admitted that petitioner Lau Yuen Yeung could
not write either English or Tagalog. Except for a few words, she could not speak either English or Tagalog. She could
not name any Filipino neighbor, with a Filipino name except one, Rosa. She did not know the names of her brothers-
in-law, or sisters-in-law.'
"Under the facts unfolded above, the Court is of the considered opinion, and so holds, that the instant petition for injunction
cannot be sustained for the same reasons set forth in the Order of this Court, dated March 19, 1962, the pertinent portions of
which read:
'First, Section 15 of the Revised Naturalization Law provides:
"'Effect of the naturalization on wife and children. — Any woman who is now or may hereafter be married to a citizen
of the Philippines, and who might herself be lawfully naturalized shall be deemed a citizen of the Philippines."
The above-quoted provision is clear and its import unequivocal and hence it should be held to mean what it plainly and
explicitly expresses in unmistakable terms. The clause 'who might herself be lawfully naturalized' incontestably implies that an
alien woman may be deemed a citizen of the Philippines by virtue of her marriage to a Filipino citizenonly if she possesses all
the qualifications and none of the disqualifications specified in the law, because these are the explicit requisites provided by
law for an alien to be naturalized. (Lee Suan Ay, Alberto Tan and Lee Chiao vs. Emilio Galang, etc., G. R. No. L-11855). However,
from the allegation of paragraph 3 of the complaint, to wit:
"'3. That plaintiff Lau Yuen Yeung, Chinese by birth, who might herself be lawfully naturalized as a Filipino citizen (not
being disqualified to become such by naturalization), is a Filipino citizen by virtue of her marriage on January 25,
1962 to plaintiff MOY YA LIM YAO alias EDILBERTO AGUINALDO LIM, under the Naturalization Laws of the
Philippines."
it can be deduced beyond debate that petitioner Lau Yuen Yeung while claiming not to be disqualified, does not and cannot
allege that she possesses all the qualifications to be naturalized, naturally because, having been admitted as a temporary
visitor only on March 13, 1961, it is obvious at once that she lacks at least, the requisite length of residence in the Philippines
(Revised Naturalization Law, Sec. 2, Case No. 2, Sec. 3, Case No. 3).
'Were if the intention of the law that the alien woman, to be deemed a citizen of the Philippines by virtue of
marriage to a Filipino citizen, need only be not disqualified under the Naturalization Law, it would have been worded
"and who herself is not disqualified to become a citizen of the Philippines."
'Second, Lau Yuen Yeung, a temporary Chinese woman visitor, whose authorized stay in the Philippines, after repeated
extensions thereof, was to expire last February 28, 1962, having married her co-plaintiff only on January 25, 1962, or just a
little over one month before the expiry date of her stay, it is evident that said marriage was effected merely for convenience to
defeat or avoid her then impending compulsory departure, not to say deportation. This cannot be permitted.
'Third, as the Solicitor General has well stated:
"'5. That petitioner Lau Yuen Yeung, having been admitted as a temporary alien visitor on the strength of a deliberate
and voluntary representation that she will enter and stay only for a period of one month and thereby secured a visa,
cannot go back on her representation to stay permanently without first departing from the Philippines as she had
promised." (Chung Tiao Bing, et al. vs. Commissioner of Immigration, G. R. No. L-9966, September 29, 1956; Ong Se
Lun vs. Board of Commissioners, G. R. No. L-6017, September 16, 1954; Sec. 9, last par., Phil. Immigration Law).
The aforequoted argument of the Solicitor General is well buttressed, not only by the decided cases of the Supreme Court on
the point mentioned above, but also on the very provisions of Section 9, sub-paragraph (g) of the Philippine Immigration Act of
1940 which reads:
" 'An alien who is admitted as a non-immigrant cannot remain in the Philippines permanently. To obtain permanent
admission, a non-immigrant alien must depart voluntarily to some foreign country and procure from the appropriate
Philippine Consul the proper visa and thereafter undergo examination by the Officers of the Bureau of Immigration at
a Philippine port of entry for determination of his admissibility in accordance with the requirements of this Act. (This
paragraph is added by Republic Act 503).'" (Sec. 9, subparagraph (g) of the Philippine Immigration Act of 1940).
'And fourth, respondent Commissioner of Immigration is charged with the administration of all laws relating to immigration
(Sec. 3, Com. Act No. 613) and in the performance of his duties in relation to alien immigrants, the law gives the Commissioner
of Immigration a wide discretion, a quasi-judicial function in determining cases presented to him (Pedro Uy So vs.
Commissioner of Immigration CA-G. R. No. 23336-R, Dec 15, 1960), so that his decision thereon may not be disturbed unless he
acted with abuse of discretion or in excess of his jurisdiction.'
"It may also be not amiss to state that wife Lau Yuen Yeung, while she barely and insufficiently talk in broken Tagalog and
English, she admitted that she cannot write either language."
The only matter of fact not clearly passed upon by His Honor which could have some bearing in the resolution of this appeal is the allegation in
the brief of petitioners-appellants, not denied in the government's brief, that "in the hearing . . . , it was shown thru the testimony of the
plaintiff Lau Yuen Yeung that she does not possess any of the disqualifications for naturalization." Of course, as an additional somehow relevant
factual matter, it is also emphasized by said appellants that during the hearing in the lower court, held almost ten months after the alleged
marriage of petitioners, "Lau Yuen Yeung was already carrying in her womb for seven months a child by her husband."
Appellants have assigned six errors allegedly committed by the court a quo, thus:
I
THE LOWER COURT ERRED IN HOLDING THAT THE CLAUSE 'WHO MIGHT HERSELF BE LAWFULLY NATURALIZED' (OF SECTION
15, REVISED NATURALIZATION LAW) INCONTESTABLY IMPLIES THAT AN ALIEN WOMAN MAY BE DEEMED A CITIZEN OF THE
PHILIPPINES BY VIRTUE OF HER MARRIAGE TO A FILIPINO CITIZEN, ONLY IF SHE POSSESSES ALL THE QUALIFICATIONS AND
NONE OF THE DISQUALIFICATIONS SPECIFIED IN THE LAW.
II
THE LOWER COURT ERRED IN HOLDING THAT A WOMAN FOREIGNER WHO DOES NOT POSSESS ANY OF THE
DISQUALIFICATIONS FOR CITIZENSHIP AND WHO MARRIED A FILIPINO CITIZEN IS STILL CONSIDERED AN ALIEN EVEN AFTER
SUCH MARRIAGE AS TO FALL WITHIN THE REQUIREMENT OF SECTION 9, SUB-PARAGRAPH (9) OF THE PHILIPPINE
IMMIGRATION ACT OF 1940.
III
THE COURT ERRED IN CONCLUDING THAT LAU YUEN YEUNG'S MARRIAGE TO A FILIPINO CITIZEN WAS ONLY FOR
CONVENIENCE, MERELY BECAUSE THE SAME WAS CELEBRATED JUST OVER A MONTH BEFORE THE EXPIRY DATE OF HER
AUTHORIZED STAY.
IV
THE LOWER COURT ERRED IN FAILING TO FIND THAT THE COMMISSIONER OF IMMIGRATION ACTED WITH ABUSE OF
DISCRETION OR IN EXCESS OF HIS JURISDICTION WHEN SAID OFFICER THREATENED TO SEND OUT OF THE COUNTRY PLAINTIFF
LAU YUEN YEUNG WITH WARNING THAT HER FAILURE TO DO SO WOULD MEAN CONFISCATION OF HER BOND, ARREST AND
IMMEDIATE DEPORTATION, IN SPITE OF THE FACT THAT LAU YUEN YEUNG IS NOW A FILIPINO CITIZEN.
V
THE LOWER COURT ERRED IN DISMISSING PLAINTIFFS-APPELLANTS' COMPLAINT AND IN REFUSING TO PERMANENTLY ENJOIN
THE COMMISSIONER FROM ORDERING PLAINTIFF LAU YUEN YEUNG TO LEAVE THE PHILIPPINES AS A TEMPORARY VISITOR
WHICH SHE IS NOT.

VI
THE LOWER COURT ERRED IN REFUSING TO GRANT PLAINTIFFS-APPELLANTS' MOTION FOR PRELIMINARY INJUNCTION
EMBODIED IN THEIR COMPLAINT, IN AN ORDER DATED MARCH 19, 1962. (PAGES 36-41, RECORD ON APPEAL).
We need not discuss these assigned errors separately. In effect, the above decision upheld the two main grounds of objection of the Solicitor General
to the petition in the court below, viz:
"That petitioner Lau Yuen Yeung, having been admitted as a temporary alien visitor on the strength of a deliberate and
voluntary representation that she will enter and stay only for a period of one month and thereby secured a visa, cannot go
back on her representation to stay permanently without first departing from the Philippines as she had promised (Chung Tiao
Bing, et al. vs. Commissioner of Immigration, G.R. No. L-9966, September 29, 1956; Ong Se Lun vs. Board of Commissioners,
G.R. No. L-6017, Sept. 16, 1954, Sec. 9, last par. Phil. Immigration Law);
"That the mere marriage of a Filipino citizen to an alien does not automatically confer on the latter Philippine citizenship. The
alien wife must possess all the qualifications required by law to become a Filipino citizen by naturalization and none of the
disqualifications. (Lee Suan Ay, Alberto Tan and Lee Chiao vs. Galang, etc., G. R. No. L-11855, Dec. 25, 1959)"
It is obvious from the nature of these objections that their proper resolution would necessarily cover all the points raised in appellants'
assignments of error, hence, We will base our discussions, more or less, on said objections.
I.
The first objection of the Solicitor General which covers the matters dealt with in appellants' second and fourth assignments of error does not require
any lengthy discussion. As a matter of fact, it seems evident that the Solicitor General's pose that an alien who has been admitted into the Philippines
as a non-immigrant cannot remain here permanently unless he voluntarily leaves the country first and goes to a foreign country to secure thereat
from the appropriate Philippine consul the proper visa and thereafter undergo examination by officers of the Bureau of Immigration at a Philippine
port of entry for determination of his admissibility in accordance with the requirements of the Philippine Immigration Act of 1940, as amended by
Republic Act 503, is premised on the assumption that petitioner Lau Yuen Yeung is not a Filipino citizen. We note the same line of reasoning in the
appealed decision of the court a quo. Accordingly, it is but safe to assume that were the Solicitor General and His Honor of the view that said
petitioner had become ipso facto a Filipina by virtue of her marriage to her Filipino husband, they would have held her as entitled to assume the
status of a permanent resident without having to depart as required of aliens by Section 9(g) of the law.
In any event, to set this point at rest, We hereby hold that portion of Section 9(g) of the Immigration Act providing:
"An alien who is admitted as a non-immigrant cannot remain in the Philippines permanently. To obtain permanent admission,
a non-immigrant alien must depart voluntarily to some foreign country and procure from the appropriate Philippine consul the
proper visa and thereafter undergo examination by the officers of the Bureau of Immigration at a Philippine port of entry for
determination of his admissibility in accordance with the requirements of this Act."
does not apply to aliens who after coming into the Philippines as temporary visitors, legitimately become Filipino citizens or acquire Filipino
citizenship. Such change of nationality naturally bestows upon them the right to stay in the Philippines permanently or not, as they may choose,
and if they elect to reside here, the immigration authorities may neither deport them nor confiscate their bonds. True it is that this Court has
vehement]y expressed disapproval of convenient ruses employed by aliens to convert their status from temporary visitors to permanent
residents in circumvention of the procedure prescribed by the legal provision already mentioned, such as in Chiong Tiao Bing vs. Commissioner
of Immigration, 99 Phil. 1020, wherein, thru Mr. Justice J.B.L. Reyes, the Court, reiterating the ruling in Ong Se Lun vs. Board of Immigration
Commissioners, 95 Phil. 785, said:
". . . It is clear that if an alien gains admission to the Islands on the strength of a deliberate and voluntary representation that
he will enter only for a limited time, and thereby secures the benefit of a temporary visa, the law will not allow him
subsequently to go back on his representation and stay permanently, without first departing from the Philippines as he had
promised. No officer can relieve him of the departure requirements of section 9 of the Immigration Act, under the guise of
'change' or 'correction', for the law makes no distinctions, and no officer is above the law. Any other ruling would, as stated in
our previous decision, encourage aliens to enter the Islands on false pretences; every alien so permitted to enter for a limited
time, might then claim a right to permanent admission, however flimsy such claim should be, and thereby compel our
government to spend time, money and effort to examining and verifying whether or not every such alien really has a right to
take up permanent residence here. In the meanwhile, the alien would be able to prolong his stay and evade his return to the
port whence he came, contrary to what he promised to do when he entered. The damages inherent in such ruling are self-
evident."
On the other hand, however, We cannot see any reason why an alien who has been here as a temporary visitor but who has in the meanwhile
become a Filipino should be required to still leave the Philippines for a foreign country, only to apply thereat for a re-entry here and undergo the
process of showing that he is entitled to come back, when after all, such right has become incontestible as a necessary concomitant of his
assumption of our nationality by whatever legal means this hag been conferred upon him. Consider, for example, precisely the case of the minor
children of an alien who is naturalized. It is indubitable that they become ipso facto citizens of the Philippines. Could it be the law that before they
can be allowed permanent residence, they still have to be taken abroad so that they may be processed to determine whether or not they have a right
to have permanent residence here? The difficulties and hardships which such a requirement entails and its seeming unreasonableness argue against
such a rather absurd construction. Indeed, as early as 1957, in Ly Giok Ha vs. Galang, 101 Phil. 459, Mr. Justice Concepcion, our present Chief Justice,
already ruled thus:
". . . (P)etitioners allege that, upon her marriage to a Filipino, Ly Giok Ha became also a citizen of the Philippines. Indeed, if this
conclusion were correct, it would follow that, in consequence of her marriage, she had been naturalized as such citizen, and,
hence the decision appealed from would have to be affirmed, for section 40(c) of Commonwealth Act 613 provides that 'in the
event of the naturalization as a Philippine citizen . . . of the alien on whose behalf the bond deposit is given, the bond shall be
cancelled or the be deposited shall be returned to the depositor or his legal representative.'" (At. pp. 462-463) In other words,
the applicable statute itself more than implies that the naturalization of an alien visitor as a Philippine citizen logically produces
the effect of conferring upon him ipso facto all the rights of citizenship including that of being entitled to permanently stay in
the Philippines outside the orbit of authority of the Commissioner of Immigration vis-a-vis aliens, if only because by its very
nature and express provisions, theImmigration Law is a law only for aliens and is inapplicable to citizens of the Philippines. In
the sense thus discussed, therefore, appellants' second and fourth assignments of error are well taken.
II.
Precisely, the second objection of the Solicitor General sustained by the trial judge is that appellant Lau Yuen Yeung's marriage to appellant Moya Lim
Yao alias Edilberto Aguinaldo whose Filipino citizenship is not denied did not have the effect of making her a Filipino, since it has not been shown
that she "might herself be lawfully naturalized," it appearing clearly in the record that she does not possess all the qualifications required of
applicants for naturalization by the Revised Naturalization Law, Commonwealth Act 473, even if she has proven that she does not suffer from any of
the disqualifications thereunder. In other words, the Solicitor General implicitly concedes that had it been established in the proceedings below that
appellant Lau Yuen Yeung possesses all the qualifications required by the law of applicants for naturalization, she would have been recognized by the
respondent as a Filipino citizen in the instant case, without requiring her to submit to the usual proceedings for naturalization.
To be sure, this position of the Solicitor General is in accord with what used to be the view of this Court since Lee Suan Ay, et al. v. Emilio Galang, etc.,
et al., G.R. No. L-11855, promulgated December 23, 1959, 106 Phil., 706, 713, 1 for it was only in Zita Ngo Burca vs. Republic, G.R. No. L-24252 which
was promulgated on January 30, 1967 (19 SCRA 186), that over the pen of Mr. Justice Conrado Sanchez, this Court held that for an alien woman who
marries a Filipino to be deemed a Filipina, she has to apply for naturalization in accordance with the procedure prescribed by the Revised
Naturalization Law and prove in said naturalization proceeding not only that she has all the qualifications and none of the disqualifications provided
in the law but also that she has complied with all the formalities required thereby like any other applicant for naturalization, 2 albeit said decision is
not yet part of our jurisprudence inasmuch as the motion for its reconsideration is still pending resolution. Appellants are in effect urging Us,
however, in their first and second assignments of error, not only to reconsider Burca but to even reexamine Lee Suan Ay which, as a matter of fact, is
the prevailing rule, having been reiterated in all subsequent decisions up to Go Im Ty. 3

Actually, the first case in which Section 15 of the Naturalization Law, Commonwealth Act 473, underwent judicial construction was in the first Ly Giok
Ha case, 4 one almost identical to the one at bar. Ly Giok Ha, a woman of Chinese nationality, was a temporary visitor here whose authority to stay
was to expire on March 14, 1956. She filed a bond to guaranty her timely departure. On March 8, 1956, eight days before the expiration of her
authority to stay, she married a Filipino by the name of Restituto Lacasta. On March 9, 1956, her husband notified the Commissioner of Immigration
of said marriage and, contending that his wife had become a Filipina by reason of said marriage, demanded for the cancellation of her bond, but
instead of acceding to such request, the Commissioner required her to leave, and upon her failure to do so, on March 16, 1956, the Commissioner
confiscated her bond; a suit was filed for the recovery of the bond; the lower court sustained her contention that she had no obligation to leave
because she had become Filipina by marriage, hence her bond should be returned. The Commissioner appealed to this Court. In the said appeal, Mr.
Justice Roberto Concepcion, our present Chief Justice, spoke for the Court, thus:
"The next and most important question for determination is whether her marriage to a Filipino justified or, at least, excused
the aforesaid failure of Ly Giok Ha to depart from the Philippines on or before March 14, 1956. In maintaining the affirmative
view, petitioners alleged that, upon her marriage to a Filipino, Ly Giok Ha became, also, a citizen of the Philippines. Indeed, if
this conclusion were correct, it would follow that, in consequence of her marriage, she had been naturalized as such citizen,
and, hence, the decision appealed from would have to be affirmed, for section 40(c) of Commonwealth Act No. 613 provides
that 'in the event of the naturalization as a Philippine citizen . . . of the alien on whose behalf the bond deposit is given, the
bond shall be cancelled or the sum deposited shall be returned to the depositor or his legal representative." Thus the issue boils
down to whether an alien female who marries a male citizen of the Philippines follows ipso facto his political status.
"The pertinent part of section 15 of Commonwealth Act No. 473, upon which petitioners rely, reads:
'Any woman who is now or may hereafter be married to a citizen of the Philippines, and who might herself be
lawfully naturalized shall be deemed a citizen of the Philippines.'
"Pursuant thereto, marriage to a male Filipino does not vest Philippine citizenship to his foreign wife, unless she 'herself may
be lawfully naturalized.' As correctly held in an opinion of the Secretary of Justice (O.p. No. 52, series of 1950), * this limitation
of section 15 excludes, from the benefits of naturalization by marriage, those disqualified from being naturalized as citizens of
the Philippines under section 4 of said Commonwealth Act No. 473, namely:
'(a) Persons opposed to organized government or affiliated with any association or group of persons who uphold and
teach doctrines opposing all organized governments;
'(b) Persons defending or teaching the necessity or propriety of violence, personal assault, or assassination for the
success and predominance of their ideas;
'(c) Polygamists or believers in the practice of polygamy;
'(d) Persons convicted of crimes involving moral turpitude;
'(e) Persons suffering from mental alienation or incurable contagious diseases;
'(f) Persons who, during the period of their residence in the Philippines, have not mingled socially with the Filipinos,
or who have not evinced a sincere desire to learn and embrace the customs, traditions, and ideals of the Filipinos;
'(g) Citizens or subjects of nations with whom the . . . Philippines are at war, during the period of such war;
'(h) Citizens or subjects of a foreign country other than the United States, whose laws does not grant Filipinos the
right to become naturalized citizens or subjects thereof.'
"In the case at bar, there is neither proof nor allegation in the pleadings that Ly Giok Ha does not fall under any of the classes
disqualified by law. Moreover, as the parties who claim that, despite her failure to depart from the Philippines within the
period specified in the bond in question, there has been no breach thereof, petitioners have the burden of proving her alleged
change of political status, from alien to citizen. Strictly speaking, petitioners have not made out, therefore a case against the
respondents-appellants.
"Considering, however, that neither in the administrative proceedings, nor in the lower court, had the parties seemingly felt
that there was an issue on whether Ly Giok Ha may 'be lawfully naturalized,' and this being a case of first impression in our
courts, we are of the opinion that, in the interest of equity and justice, the parties herein should be given an opportunity to
introduce evidence, if they have any, on said issue." (At pp. 462-464.).
As may be seen, although not specifically in so many words, no doubt was left in the above decision as regards the following propositions:
1. That under Section 15 of Commonwealth Act 473, the Revised Naturalization Law, the marriage of an alien woman to a Filipino makes her a
Filipina, if she "herself might be lawfully naturalized";
2. That this Court declared as correct the opinion of the Secretary of Justice that the limitation of Section 15 of the Naturalization Law excludes from
the benefits of naturalization by marriage, only those disqualified from being naturalized under Section 4 of the law quoted in the decision;
3. That evidence to the effect that she is not disqualified may be presented in the action to recover her bond confiscated by the Commissioner of
Immigration;
4. That upon proof of such fact, she may be recognized as Filipina; and
5. That in referring to the disqualifications enumerated in the law, the Court somehow left the impression that no inquiry need be made as to
qualifications, 5 specially considering that the decision cited and footnoted several opinions of the Secretary of Justice, the immediate superior of the
Commissioner of Immigration, the most important of which are the following:
"Paragraph (a), section 13 of Act No. 2927, as amended, (now section 15, Commonwealth Act No. 473), provided that 'any
woman who is now or may hereafter be married to a citizen of the Philippines, and who might herself be lawfully naturalized
shall be deemed a citizen of the Philippines.' A similar provision in the naturalization law of the United States has been
construed as not requiring the woman to have the qualifications of residence, good character, etc., as in the case of
naturalization by judicial proceedings, but merely that she is of the race of persons who may be naturalized. (Kelly v. Owen
[Dist. Col. 1868] 7 Wall 496, 5F, 11, 12; ex parte Tryason [D. C. Wash. 1914] 215 F. 449, 27 Op. Atty. Gen. 507). (Op. No. 168, s.
1940 of Justice Sec. Jose Abad Santos.)
"In a previous opinion rendered for your Office, I stated that the clause 'who might herself be lawfully naturalized', should be
construed as not requiring the woman to have the qualifications of residence, good character, etc., as in cases of naturalization
by judicial proceedings, but merely that she is of the race of persons who may be naturalized. (Op. No. 79, s. 1940)
"Inasmuch as the race qualification has been removed by the Revised Naturalization Law, it results that any woman who
married a citizen of the Philippines prior to or after June 17, 1939, and the marriage not having been dissolved, and on the
assumption that she possesses none of the disqualifications mentioned in Section 4 of Commonwealth Act No. 473, follows the
citizenship of her husband." (Op. No. 176, v. 1940 of Justice Sec. Jose Abad Santos.)
"From the foregoing narration of facts, it would seem that the only material point of inquiry is as to the citizenship of Arce
Machura. If he shall be found to be a citizen of the Philippines, his wife, Mrs. Lily James Machura, shall likewise be deemed a
citizen of the Philippines pursuant to the provision of Section 15, Commonwealth Act No. 473, which reads in part as follows:
'Any woman who is now or may hereafter be married to a citizen of the Philippines, and who might herself be
lawfully naturalized shall be deemed a citizen of the Philippines.'
"The phrase 'who might herself be lawfully naturalized', as contained in the above provision, means that the woman who is
married to a Filipino citizen must not belong to any of the disqualified classes enumerated in Section 4 of the Naturalization
Law (Ops., Sec. of Jus., No. 28, s. 1950; No. 43, s. 1948, No. 95, s. 1941: Nos. 79 and 168, s. 1940). Under the facts stated in the
within papers, Mrs. Machura does not appear to be among the disqualified classes mentioned in the law.
"It having been shown that Arce Machura or Arsenio Guevara was born as an illegitimate of a Filipino mother, he should be
considered as a citizen of the Philippines in consonance with the well-settled rule that an illegitimate child follows the
citizenship of his only legally recognized parent, the mother (Op., Sec. of Jus., Nos. 58, 98 & 281, s. 1948; No. 96, s. 1949). Her
husband being a Filipino, Mrs. Machura must necessarily be deemed as a citizen of the Philippines by marriage (Sec. 15,
Com. Act No. 473.) (Op. No. 52, s. 1950 of Justice Sec. Ricardo Nepomuceno.)
The logic and authority of these opinions, compelling as they are, must have so appealed to this Court that five days later, on May 22, 1957, in
Ricardo Cua v. The Board of Commissioners, 101 Phil. 521, Mr. Justice J.B.L. Reyes, reiterated the same ruling on the basis of the following facts:
Tjioe Wu Suan, an Indonesian, arrived in Manila on November 1, 1952, but it turned out that her passport was forged. On December 10, 1953, a
warrant was issued for her arrest for purposes of deportation. Later, on December 20, 1953, she married Ricardo Cua, a Filipino, and because of said
marriage, the Board of Special Inquiry considered her a Filipina. Upon a review of the case, however, the Board of Immigration Commissioners
insisted on continuing with the deportation proceedings and so, the husband filed prohibition and mandamus proceedings. The lower court denied
the petition. Although this Court affirmed said decision, it held, on the other hand, that:
"Granting the validity of marriage, this Court has ruled in the recent case of Ly Giok Ha v. Galang, supra, p. 459, that the bare
fact of a valid marriage to a citizen does not suffice to confer his citizenship upon the wife. Section 15 of the Naturalization Law
requires that the alien woman who marries a Filipino must show, in addition, that she 'might herself be lawfully naturalized' as
a Filipino citizen. As construed in the decision cited, this last condition requires proof that the woman who married a Filipino is
herself not disqualified under section 4 of the Naturalization Law.
"No such evidence appearing on record, the claim of assumption of Filipino citizenship by Tjioe Wu Suan, upon her marriage to
petitioner, is untenable. The lower court, therefore, committed no error in refusing to interfere with the deportation
proceedings, where she can anyway establish the requisites indispensable for her acquisition of Filipino citizenship, as well as
the alleged validity of her Indonesian passport." (Ricardo Cua v. The Board of Immigration Commissioners, G. R. No. L-9997,
May 22, 1957, 101 Phil. 521, 523.) [Emphasis supplied]
For emphasis, it is reiterated that in the above two cases, this Court expressly gave the parties concerned opportunity to prove the fact that they
were not suffering from any of the disqualifications of the law without the need of undergoing any judicial naturalization proceeding. It may be
stated, therefore, that according to the above decisions, the law in this country, on the matter of the effect of marriage of an alien woman to a
Filipino is that she thereby becomes a Filipina, if it can be proven that at the time of such marriage, she does not possess any of the disqualifications
enumerated in Section 4 of the Naturalization Law, without the need of submitting to any naturalization proceedings under said law.
It is to be admitted that both of the above decisions made no reference to qualifications, that is, as to whether or not they need also to be proved,
but, in any event, it is a fact that the Secretary of Justice understood them to mean that such qualifications need not be possessed nor proven. Then
Secretary of Justice Jesus Barrera, who later became a distinguished member of this Court, 6 so ruled in opinions rendered by him subsequent to Ly
Giok Ha, the most illustrative of which held:
"At the outset it is important to note that an alien woman married to a Filipino citizen needs only to show that she 'might
herself be lawfully naturalized' in order to acquire Philippine citizenship. Compliance with other conditions of the statute, such
as those relating to the qualifications of an applicant for naturalization through judicial proceedings, is not necessary (See:
Leonard v. Grant, 5 Fed. 11; 27 Ops. Atty. Gen [U.S.] 507; Ops Sec. of Justice, No. 776, s. 1940, and No. 111, s. 1953.
"This view finds support in the case of Ly Giok Ha et al. v. Galang et al., G.R. No. L-10760, promulgated May 17, 1957, where
the Supreme Court, construing the abovequoted section of the Naturalization Law, held that 'marriage to a male Filipino does
not vest Philippine citizenship to his foreign wife, unless she 'herself may he lawfully naturalized,' and that 'this limitation of
Section 15 excludes, from the benefits of naturalization by marriage, those disqualified from being naturalized as citizens of the
Philippines under Section 4 of said Commonwealth Act No. 473.' In other words, disqualification for any of the causes
enumerated in Section 4 of the Act is the decisive factor that defeats the right of the foreign wife of a Philippine citizen to
acquire Philippine citizenship.
xxx xxx xxx
"Does petitioner, Lim King Bian, belong to any of these groups ? The Commissioner of Immigration does not say so but merely
predicates his negative action on the ground that a warrant of deportation for 'overstaying' is pending against the petitioner.
"We do not believe the position is well taken. Since the grounds for disqualification for naturalization are expressly
enumerated in the law, a warrant of deportation not based on a finding of unfitness to become naturalized for any of those
specified causes may not be invoked to negate acquisition of Philippine citizenship by a foreign wife of a Philippine citizen
under Section 15 of the Naturalization Law. (Inclusio unius est exclusio alterius)" (Op. No. 12, s. 1958 of Justice Undersec. Jesus
G. Barrera.)
"Regarding the steps that should be taken by an alien woman married to a Filipino citizen in order to acquire Philippine
citizenship, the procedure followed in the Bureau of Immigration is as follows: The alien woman must file a petition for the
cancellation of her alien certificate of registration alleging, among other things, that she is married to a Filipino citizen and that
she is not disqualified from acquiring her husband's citizenship pursuant to section 4 of Commonwealth Act No. 473, as
amended. Upon the filing of said petition, which should be accompanied or supported by the joint affidavit of the petitioner
and her Filipino husband to the effect that the petitioner does not belong to any of the groups disqualified by the cited section
from becoming naturalized Filipino citizen (please see attached CEB Form 1), the Bureau of Immigration conducts an
investigation and thereafter promulgates its order or decision granting or denying the petition." (Op. No. 38, B. 1958 of Justice
Sec. Jesus G. Barrera.)
"This view finds support in the case of Ly Giok Ha et al., v. Galang et al. (G.R. No. L-10760, promulgated May 17, 1957), where
the Supreme Court, construing the above-quoted section in the Revised Naturalization Law, held that 'marriage to a male
Filipino does not vest Philippine citizenship to his foreign wife, unless she 'herself may be lawfully naturalized,' and that 'this
limitation of Section 15 excludes from the benefits of naturalization by marriage those disqualified from being naturalized as
citizens of the Philippines under Section 4 of said Commonwealth Act No. 473.' In other words, disqualification for any of the
causes enumerated in section 4 of the Act is the decisive factor that defeats the right of an alien woman married to a Filipino
citizen to acquire Philippine citizenship." (Op. 57, s. 1958 of Justice Sec. Jesus G. Barrera.)
"The contention is untenable. The doctrine enunciated in the Ly Giok Ha case is not a new one. In that case, the Supreme Court
held that under paragraph 1 of Section 15 ofCommonwealth Act No. 473, 'marriage to a male Filipino does not vest Philippine
citizenship to his foreign wife unless she "herself may be lawfully naturalized"', and, quoting several earlier opinions of the
Secretary of Justice, namely: No. 52, s. 1950; No. 168, s. 1940; No. 95, s. 1941; No. 63, s. 1948; No. 28, s. 1950, 'this limitation
of section 15 excludes from the benefits of naturalization by marriage, those disqualified from being naturalized as citizens of
the Philippines under section 4 of said Commonwealth Act No. 473." (Op. 134, B. 1962 of Justice Undersec. Magno S.
Gatmaitan.)
It was not until more than two years later that, in one respect, the above construction of the law was importantly modified by this Court in Lee Suan
Ay, supra, in which the facts were as follows:
"Upon expiration of the appellant Lee Suan Ay's authorized period of temporary stay in the Philippines (25 March 1955), on 26
March 1955 the Commissioner of Immigration asked the bondsman to present her to the Bureau of Immigration within 24
hours from receipt of notice, otherwise the bond will be confiscated (Annex 1). For failure of the bondsman to comply with the
foregoing order, on 1 April 1955 the Commissioner of Immigration ordered the cash bond confiscated (Annex E). Therefore,
there was an order issued by the Commissioner of Immigration confiscating or forfeiting the cash bond. Unlike in forfeiture of
bail bonds in criminal proceedings, where the Court must enter an order forfeiting the bail bond and the bondsman must be
given an opportunity to present his principal or give a satisfactory reason for his inability to do so, before final judgment may
be entered against the bondsman, (section 15, Rule 110; U.S. v. Bonoan, 22 Phil. 1.) in forfeiture of bonds posted for the
temporary stay of an alien in the Philippines, no court proceeding is necessary. Once a breach of the terms and conditions of
the undertaking in the bond is committed, the Commissioner of Immigration may, under the terms and conditions thereof,
declare it forfeited in favor of the Government." (In the meanwhile, on April 1, 1955, Lee Suan Ay and Alberto Tan, a Filipino,
were joined in marriage by the Justice of the Peace of Las Piñas, Rizal.)
Mr. Justice Sabino Padilla speaking for a unanimous court which included Justices Concepcion and Reyes who had penned Ly Giok Ha and
Ricardo Cua, ruled thus:
"The fact that Lee Suan Ay (a Chinese) was married to a Filipino citizen does not relieve the bondsman from his liability on the
bond. The marriage took place on 1 April 1955, and the violation of the terms and conditions of; the undertaking in the bond —
failure to depart from the Philippines upon expiration of her authorized period of temporary stay in the Philippines (25 March
1955) and failure to report to the Commissioner of Immigration within 24 hours from receipt of notice — were committed
before the marriage. Moreover, the marriage of a Filipino citizen to an alien does not automatically confer Philippine
citizenship upon the latter. She must possesses the qualifications required by law to become a Filipino citizen by
naturalization. ** There is no showing that the appellant Lee Suan Ay possesses all the qualifications and none of the
disqualifications provided for by law to become a Filipino citizen by naturalization."
Pertinently to be noted at once in this ruling, which, to be sure, is the one relied upon in the appealed decision now before Us, is the fact that the
footnote of the statement therein that the alien wife "must possess the qualifications required by law to become a Filipino citizen by naturalization"
makes reference to Section 15, Commonwealth Act 473and precisely, also to Ly Giok Ha v. Galang, supra. As will be recalled, on the other hand, in
the opinions of the Secretary of Justice explicitly adopted by the Court in Ly Giok Ha, among them, Opinion No. 176, Series of 1940, above-quoted, it
was clearly held that "(I)n a previous opinion rendered for your Office, I stated that the clause 'who might herself be lawfully naturalized', should be
construed as not requiring the woman to have the qualifications of residence, good character, etc., as in cases of naturalization by judicial
proceedings, but merely that she is of the race by persons who may be naturalized." (Op. Na. 79, s. 1940)

Since Justice Padilla gave no reason at all for the obviously significant modification of the construction of the law, it could be said that there was need
for clarification of the seemingly new posture of the Court. The occasion for such clarification should have been in Kua Suy, etc., et al. vs. The
Commissioner of Immigration, G.R. No. L-13790, October 31, 1963, penned by Mr. Justice J.B.L. Reyes, who had rendered the opinion in Ricardo
Cua, supra, which followed that in Ly Giok Ha, supra, but apparently seeing no immediate relevancy in the case on hand then of the particular point
in issue now, since it was not squarely raised therein similarly as in Lee Suan Ay, hence, anything said on the said matter would at best be no more
than obiter dictum, Justice Reyes limited himself to holding that "Under Section 15 of the Naturalization Act, the wife is deemed a citizen of the
Philippines only if she 'might herself be lawfully naturalized,' so that the fact of marriage to a citizen, by itself alone, does not suffice to confer
citizenship, as this Court has previously ruled in Ly Giok Ha v. Galang, 54 O.G. 356, and in Cua v. Board of Immigration Commissioners, 53 O.G. 8567;
and there is here no evidence of record as to the qualifications or absence of disqualifications of appellee Kua Suy", without explaining the apparent
departure already pointed out from Ly Giok Ha and Ricardo Cua. Even Justice Makalintal, who wrote a separate concurring and dissenting opinion
merely lumped together Ly Giok Ha, Ricardo Cua and Lee Suan Ay and opined that both qualifications and non-disqualifications have to be shown
without elucidating on what seemed to be departure from the said first two decisions.
It was only on November 30, 1963 that to Mr. Justice Roberto Regala fell the task of rationalizing the Court's position. In La San Tuang v. Galang, G.R.
No. L-18775, November 30, 1963, 9 SCRA 638, the facts were simply these: 10 San Tuang, a Chinese woman, arrived in the Philippines on July 1, 1960
as a temporary visitor with authority to stay up to June 30, 1961. She married a Filipino on January 7, 1961, almost six months before the expiry date
at her permit, and when she was refused to leave after her authority to stay had expired, she refused to do so, claiming she had become a Filipina by
marriage, and to bolster her position, she submitted an affidavit stating explicitly that she does not possess any of the disqualifications enumerated
in the Naturalization Law, Commonwealth Act 473. When the case reached the court, the trial judge held for the government that in addition to not
having any of the disqualifications referred to, there was need that Lo San Tuang should have also possessed all the qualifications of residence, moral
character, knowledge of a native principal dialect, etc., provided by the law. Recognizing that the issue squarely to be passed upon was whether or
not the possession of all the qualifications were indeed needed to be shown apart from non-disqualification, Justice Regala held affirmatively for the
Court, reasoning out thus:
"It is to be noted that the petitioner has anchored her claim for citizenship on the basis of the decision laid down in the case of
Leonard v. Grant, 5 Swy. 603, 5 F 11, where the Circuit Court of Oregon held that it was only necessary that the woman 'should
be a person of the class or race permitted to be naturalized by existing laws, and that in respect of the qualifications arising out
of her conduct or opinions, being the wife of a citizen, she is to be regarded as qualified for citizenship, and therefore
considered a citizen.' (In explanation of its conclusion, the Court said: 'If, whenever during the life of the woman or afterwards,
the question of her citizenship arises in a legal proceeding, the party asserting her citizenship by reason of her marriage with a
citizen must not only prove such marriage, but also that the woman then possessed all the further qualifications necessary to
her becoming naturalized under existing laws, the statute will be practically nugatory, if not a delusion and a snare. The proof
of the facts may have existed at the time of the marriage, but years after, when a controversy arises upon the subject, it may
be lost or difficult to find.')
"In other words, all that she was required to prove was that she was a free white woman or a woman of African descent or
nativity, in order to be deemed an American citizen, because, with respect to the rest of the qualifications on residence, moral
character, etc., she was presumed to be qualified.
"Like the law in the United States, our former Naturalization Law (Act No. 2927, as amended by Act No. 3448) specified the
classes of persons who alone might become citizens of the Philippines, even as it provided who were disqualified. Thus, the
pertinent provisions of that law provided:
'Section 1. Who may become Philippine citizens. — Philippine citizenship may be acquired by (a) natives of the
Philippines who are not citizens thereof under the Jones Law; (b) natives of the Insular possessions of the United
States; (c) citizens of the United States, or foreigners who under the laws of the United States may become citizens of
said country if residing therein.
'Section 2. Who are disqualified. — The following cannot be naturalized as Philippine citizens: (a) Persons opposed to
organized government or affiliated with any association or group of persons who uphold and teach doctrines
opposing all organized government; (b) persons defending or teaching the necessity or propriety of violence,
personal assault or assassination for the success and predominance of their ideas; (c) polygamists or believers in the
practice of polygamy; (d) persons convicted of crimes involving moral turpitude; (e) persons suffering from mental
alienation or incurable contagious diseases; (f) citizens or subjects of nations with whom the United States and the
Philippines are at war, during the period of such war.
'Section 3. Qualifications. — The persons comprised in subsection (a) of section one of this Act, in order to be able to
acquire Philippine citizenship, must be not less than twenty-one years of age on the day of the hearing of their
petition.
'The persons comprised in subsections (b) and (c) of said section one shall, in addition to being not less than twenty-
one years of age on the day of the hearing of the petition, have all and each of the following qualifications:
'First. Residence in the Philippine Islands for a continuous period of not less than five years, except as provided in the
next following section;
'Second. To have conducted themselves in a proper and irreproachable manner during the entire Period of their
residence in the Philippine Islands, in their relation with the constituted government as well as with the community
in which they are living;
'Third. To hold in the Philippine Islands real estate worth not less than one thousand pesos, Philippine currency, or
have some known trade or profession; and
'Fourth. To speak and write English, Spanish, or some native tongue.
'In case the petitioner is a foreign subject, he shall, besides, declare in writing and under oath his intention of
renouncing absolutely and perpetually all faith and allegiance to the foreign authority, state or sovereignty of which
he was a native, citizen or subject.'
"Applying the interpretation given by Leonard v. Grant, supra, to our law as it then stood, alien women married to citizens of
the Philippines must, in order to be deemed citizens of the Philippines, be either (1) natives of the Philippines who were not
citizens thereof under the Jones Law, or (2) natives of other Insular possessions of the United States, or (3) citizens of the
United States or foreigners who under the laws of the United States might become citizens of that country if residing therein.
With respect to the qualifications set forth in Section 3 of the former law, they were deemed to have the same for all intents
and purposes.
"But, with the approval of the Revised Naturalization Law (Commonwealth Act No. 473) on June 17, 1939, Congress has since
discarded class or racial consideration from the qualifications of applicants for naturalization (according to its proponent, the
purpose in eliminating this consideration was, first, to remove the features of the existing naturalization act which
discriminated in favor of the Caucasian} and against Asiatics who are our neighbors, and are related to us by racial affinity and,
second, to foster amity with all nations [Sinco, Phil. Political Law 502 — 11 ed.]), even as it retained in Section 15 the phrase in
question. The result is that the phrase 'who might herself be lawfully naturalized' must be understood in the context in which it
is now found, in a setting so different from that in which it was found by the Court in Leonard v. Grant.
"The only logical deduction from the elimination of class or racial consideration is that, as the Solicitor General points out, the
phrase 'who might herself be lawfully naturalized' must now be understood as referring to those who under Section 2 of the
law are qualified to become citizens of the Philippines.
"There is simply no support for the view that the phrase 'who might herself be lawfully naturalized' must now be understood
as requiring merely that the alien woman must not belong to the class of disqualified persons under Section 4 of the Revised
Naturalization Law. Such a proposition misreads the ruling laid down in Leonard v. Grant. A person who is not disqualified is
not necessarily qualified to become a citizen of the Philippines, because the law treats 'qualifications' and 'disqualifications' in
separate sections. And then it must not be lost sight of that even under the interpretation given to the former law, it was to be
understood that the alien woman was not disqualified under Section 2 of that law. Leonard v. Grant did not rule that it was
enough if the alien woman does not belong to the class of disqualified persons in order that she may be deemed to follow the
citizenship of her husband: What that case held was that the phrase 'who might herself be lawfully naturalized, merely means
that she belongs to the class or race of persons qualified to become citizens by naturalization — the assumption being always
that she is not otherwise disqualified.
"We therefore hold that under the first paragraph of Section 15 of the Naturalization Law, an alien woman, who is married to a
citizen of the Philippines, acquires the citizenship of her husband only if she has all the qualifications and none of the
disqualifications provided by law. Since there is no proof in this case that petitioner has all the qualifications and is not in any
way disqualified, her marriage to a Filipino citizen does not automatically make her a Filipino citizen. Her affidavit to the effect
that she is not in any way disqualified to become a citizen of this country was correctly disregarded by the trial court, the same
being self-serving."
Naturally, almost a month later in Sun Peck Yong V. Commissioner of Immigration, G.R. No L-20784, December 27, 1963, 9 SCRA 875, wherein the
Secretary of Foreign Affairs reversed a previous resolution of the preceding administration to allow Sun Peck Yong and her minor son to await the
taking of the oath of Filipino citizenship of her husband two years after the decision granting him nationalization and required her to leave and this
order was contested in court, Justice Barrera held:
"In the case of Lo San Tuang v. Commissioner of Immigration (G.R. No. L-18775, promulgated November 30, 1963; Kua Suy vs.
Commissioner of Immigration, L-13790, promulgated October 31, 1963), we held that the fact that the husband became a
naturalized citizen does not automatically make the wife a citizen of the Philippines. It must also be shown that she herself
possesses all the qualifications, and none of the disqualifications, to become a citizen. In this case, there is no allegation, much
less showing, that petitioner-wife is qualified to become a Filipino citizen herself. Furthermore, the fact that a decision was
favorably made on the naturalization petition of her husband is no assurance that he (the husband) would become a citizen, as
to make a basis for the extension of her temporary stay."
On the same day, in Tong Siok Sy v. Vivo, G.R. No. L-21136, December 27, 1963, 9 SCRA 876, Justice Barrera reiterated the same ruling and citing
particularly Lo San Tuang and Kua Suy, held that the marriage of Tong Siok Sy to a Filipino on November 12, 1960 at Taichung, Taiwan and her
taking oath of Filipino citizenship before the Philippine Vice Consul at Taipeh, Taiwan on January 6, 1961 did not make her a Filipino citizen,
since she came here only in 1961 and obviously, she had not had the necessary ten-year residence in the Philippines required by the law.
Such then was the status of the jurisprudential law on the matter under discussion when Justice Makalintal sought a reexamination thereof in Choy
King Tee v. Galang, G.R. No. L-18351, March 26, 1965, 13 SCRA 402. Choy King Tee's husband was granted Philippine citizenship on January 13, 1959
and took the oath on January 31 of the same year, Choy King Tee first came to the Philippines in 1955 and kept commuting between Manila and
Hongkong since then, her last visa before the case being due to expire on February 14, 1961. On January 27, 1961, her husband asked the
Commissioner of Immigration to cancel her alien certificate of registration, as well as their child's, for the reason that they were Filipinos, and when
the request was denied as to the wife, a mandamus was sought, which the trial court granted. Discussing anew the issue of the need for
qualifications, Justice Makalintal not on]y reiterated the arguments of Justice Regala in Lo San Tuang but added further that the ruling is believed to
be in line with the national policy of selective admission to Philippine citizenship. 7
No wonder, upon this authority, in Austria v. Conchu, G.R. No. L-20716, June 22, 1965, 14 SCRA 336, Justice J.P. Bengzon readily reversed the decision
of the lower court granting the writs of mandamus and prohibition against the Commissioner of Immigration, considering that Austria's wife, while
admitting she did not possess all the qualifications for naturalization, had submitted only an affidavit that she had none of the disqualifications
therefor. So also did Justice Dizon similarly hold eight days later in Brito v. Commissioner, G.R. No. L-16829, June 30, 1965, 14 SCRA 539.
Then came the second Ly Giok Ha case 8 wherein Justice J. B. L. Reyes took occasion to expand on the reasoning of Choy King Tee by illustrating with
examples "the danger of relying exclusively on the absence of disqualifications, without taking into account the other affirmative requirements of the
law." 9
Lastly, in Go Im Ty v. Republic, G.R. No. L-17919, decided on July 30, 1966, 10 Justice Zaldivar held for the Court that an alien woman who is widowed
during the pendency of the naturalization proceedings of her husband, in order that she may be allowed to take the oath as Filipino, must, aside from
proving compliance with the requirements of Republic Act 530, show that she possesses all the qualifications and does not suffer from any of the
disqualifications under the Naturalization Law, citing in the process the decision to such effect discussed above, 1 1 even as he impliedly reversed pro
tanto the ruling in Tan Lin v. Republic, G.R. No. L-13786, May 31, 1961, 2 SCRA 383.
Accordingly, in Burca, Justice Sanchez premised his opinion on the assumption that the point now under discussion is settled law.
In the case now at bar, the Court is again called upon to rule on the same issue. Under Section 15 of the Naturalization Law, Commonwealth Act 473,
providing that:
"SEC. 15. Effect of the naturalization on wife and children. — Any woman, who is now or may hereafter be married to a citizen
of the Philippines, and who might herself be lawfully naturalized shall be deemed a citizen of the Philippines.
"Minor children of persons naturalized under this law who have been born in the Philippines shall be considered citizens
thereof.
"A foreign-born minor child, if dwelling in the Philippines at the time of the naturalization of the parent, shall automatically
become a Philippine citizen, and a foreign-born child, who is not in the Philippines at the time the parent is naturalized, shall be
deemed a Philippine citizen only during his minority, unless he begins to reside permanently in the Philippines when still a
minor, in which case, he will continue to be a Philippine citizen even after becoming of age.
"A child born outside of the Philippines after the naturalization of his parent, shall be considered a Philippine citizen, unless
within one year after reaching the age of majority he fails to register himself as a Philippine citizen at the American Consulate
of the country where he resides, and to take the necessary oath of allegiance.
is it necessary, in order that an alien woman who marries a Filipino or who is married to a man who subsequently becomes a Filipino, may
become a Filipino citizen herself, that, aside from not suffering from any of the disqualifications enumerated in the law, she must also possess
all the qualifications required by said law? If nothing but the unbroken line from Lee Suan Ay to Go Im Ty, as recounted above, were to be
considered, it is obvious that an affirmative answer to the question would be inevitable, specially, if it is noted that the present case was actually
submitted for decision on January 21, 1964 yet, shortly after Lo San Tuang, Tong Siok Sy and Sun Peck Yong, all supra, and even before Choy King
Tee, supra, were decided. There are other circumstances, however, which make it desirable, if not necessary, that the Court take up the matter
anew. There has been a substantial change in the membership of the Court since Go Im Ty, and of those who were in the Court already when
Burca was decided, two members, Justice Makalintal and Castro concurred only in the result, precisely, according to them, because they wanted
to leave the point now under discussion open in so far as they are concerned. 12 Truth to tell, the views and arguments discussed at length with
copious relevant authorities, in the motion for reconsideration as well as in the memorandum of the amici curiae 13 in the Burca case cannot
just be taken lightly and summarily ignored, since they project in the most forceful manner, not only the legal and logical angles of the issue, but
also the imperative practical aspects thereof in the light of the actual situation of the thousands of alien wives of Filipinos who have so long,
even decades, considered themselves as Filipinas and have always lived and acted as such, officially or otherwise, relying on the long standing
continuous recognition of their status as such by the administrative authorities in charge of the matter, as well as by the courts. Under these
circumstances, and if only to afford the Court an opportunity to consider the views of the five justices who took no part in Ga Im Ty (including
the writer of this opinion), the Court decided to further reexamine the matter. After all, the ruling first laid inLee Suan Ay, and later in Lo San
Tuang, Choy King Tee and the second (1966) Ly Giok Ha, did not categorically repudiate the opinions of the Secretary of Justice relied upon by
the first (1959) Ly Giok Ha. Besides, some points brought to light during the deliberations in this case would seem to indicate that the premises
of the later cases can still bear further consideration.
Whether We like it or not, it is undeniably factual that the legal provision We are construing, Section 15, aforequoted, of the Naturalization Law has
been taken directly, copied and adopted from its American counterpart. To be more accurate, said provision is nothing less than a reenactment of
the American provision. A brief review of its history proves this beyond per adventure of doubt.
The first Naturalization Law of the Philippines approved by the Philippine Legislature under American sovereignty was that of March 26, 1920, Act
No. 2927. Before then, as a consequence of the Treaty of Paris, our citizenship laws were found only in the Organic Laws, the Philippine Bill of 1902,
the Act of the United States Congress of March 23, 1912 and later the Jones Law of 1916. In fact, Act No. 2927 was enacted pursuant to express
authority granted by the Jones Law. For obvious reasons, the Philippines gained autonomy on the subjects of citizenship and immigration only after
the effectivity of the Philippine Independence Act. This made it practically impossible for our laws on said subject to have any perspective or
orientation of our own; everything was American.

The Philippine Bill of 1902 provided pertinently:


"SECTION 4. That all inhabitants of the Philippine Islands continuing to reside therein who were Spanish subjects on the
eleventh day of April, eighteen-hundred and ninety-nine, and then resided in said Islands, and their children born subsequent
thereto, shall be deemed and held to be citizens of the Philippine Islands and as such entitled to the protection of the United
States, except such as shall have elected to preserve their allegiance to the Crown of Spain in accordance with the provisions of
the treaty of peace between the United States and Spain signed at Paris December tenth, eighteen hundred and ninety-eight."
This Section 4 of the Philippine Bill of 1902 was amended by Act of Congress of March 23, 1912, by adding a provision as follows:
"Provided, That the Philippine Legislature is hereby authorized to provide by law for the acquisition of Philippine citizenship by
those natives of the Philippine Islands who do not come within the foregoing provisions, the natives of other insular
possessions of the United States, and such other persons residing in the Philippine Islands who would become citizens of the
United States, under the laws of the United States, if residing therein."
The Jones Law reenacted these provisions substantially:
"SECTION 2. That all inhabitants of the Philippine Islands who were Spanish subjects on the eleventh day of April, eighteen
hundred and ninety-nine, and then resided in said islands, and their children born subsequent thereto, shall be deemed and
held to be citizens of the Philippine Islands, except such as shall have elected to preserve their allegiance to the Crown of Spain
in accordance with the provisions of the treaty of peace between the United States and Spain, signed at Paris December tenth,
eighteen hundred and ninety-eight and except such others as have since become citizens of some other country: Provided,
That the Philippine Legislature, herein provided for, is hereby authorized to provide by law for the acquisition of Philippine
citizenship by those natives of the Philippine Islands who do not come within the foregoing provisions, the natives of the
insular possessions of the United States, and such other persons residing in the Philippine Islands who are citizens of the
United States under the laws of the United States if residing therein."
For aught that appears, there was nothing in any of the said organic laws regarding the effect of marriage to a Filipino upon the nationality of an alien
woman, albeit under the Spanish Civil Code provisions on citizenship, Articles 17 to 27, which were, however, abrogated upon the change of
sovereignty, it was unquestionable that the citizenship of the wife always followed that of the husband. Not even Act 2927 contained any provision
regarding the effect of naturalization of an alien upon the citizenship of his alien wife, nor of the marriage of such alien woman with a native born
Filipino or one who had become a Filipino before the marriage, although Section 13 thereof provided thus:
"SEC. 13. Right of widow and children of petitioners who have died. — In case a petitioner should die before the final decision
has been rendered, his widow and minor children may continue the proceedings. The decision rendered in the case shall, so far
as the widow and minor children are concerned, produce the same legal effect as if it had been rendered during the life of the
petitioner."
It was not until November 30, 1928, upon the approval of Act 3448, amending Act 2977, that the following provisions were added to the above
Section 13:
"SECTION 1. The following new sections are hereby inserted between sections thirteen and fourteen of Act Numbered Twenty-
nine hundred and Twenty-seven:
'SEC. 13 (a). Any woman who is now or may hereafter be married to a citizen of the Philippine Islands and who might
herself be lawfully naturalized, shall be deemed a citizen of the Philippine Islands.
'SEC. 13 (b). Children of persons who have been duly naturalized under this law, being under the age of twenty-one
years at the time of the naturalization of their parents, shall, if dwelling in the Philippine Islands, be considered
citizens thereof.
'SEC. 13 (c). Children of persons naturalized under this law who have been born in the Philippine Islands after the
naturalization of their parents shall be considered citizens thereof.' "
When Commonwealth Act 473, the current naturalization law, was enacted on June 17, 1939, the above Section 13 became its Section 15 which has
already been quoted earlier in this decision. As can be seen, Section 13(a) abovequoted was re-enacted practically word for word in the first
paragraph of this Section 15 except for the change of Philippine Islands to Philippines. And it could not have been on any other basis than this
legislative history of our naturalization law that each and everyone of the decisions of this Court from the first Ly Giok Ha to Go Im Ty, discussed
above, were rendered.
As stated earlier, in the opinion of Chief Justice Concepcion in the first Ly Giok Ha, it was quite clear that for an alien woman who marries a Filipino to
become herself a Filipino citizen, there is no need for any naturalization proceeding because she becomes a Filipina ipso facto from the time of such
marriage, provided she does not suffer any of the disqualifications enumerated in Section 4 of Commonwealth Act 473, with no mention being made
of whether or not the qualifications enumerated in Section 2 thereof need be shown. It was only in Lee Suan Ay in 1959 that the possession of
qualifications were specifically required, but it was not until 1963, in Lo San Tuang, that Justice Regala reasoned out why the possession of the
qualifications provided by the law should also be shown to be possessed by the alien wife of a Filipino, for her to become a Filipina by marriage.
As may be recalled, the basic argument advanced by Justice Regala was briefly as follows: That "like the law in the United States, our Naturalization
Law specified the classes of persons who alone might become citizens, even as it provided who were disqualified," and inasmuch as Commonwealth
Act 473, our Naturalization Law since 1939 did not reenact the section providing who might become citizens, allegedly in order to remove racial
discrimination in favor of Caucasians and against Asiatics, "the only logical deduction . . . is that the phrase 'who might herself be lawfully naturalized'
must now be understood as referring to those who under Section 2 of the law are qualified to become citizens of the Philippines" and "there is
simply no support for the view that the phrase 'who might herself be lawfully naturalized' must now be understood as requiring merely that the alien
woman must not belong to the class of disqualified persons under Section 4 of the Revised Naturalization Law." 14
A similar line of reasoning was followed in Choy King Tee, which for ready reference may be quoted:
"The question has been settled by the uniform ruling of this Court in a number of cases. The alien wife of a Filipino citizen must
first prove that she has all the qualifications required by Section 2 and none of the disqualifications enumerated in Section 4 of
the Naturalization Law before she may he deemed a Philippine citizen (Lao Chay v. Galang, L-19977, Oct. 30, 1964, citing Lo San
Tuang v. Galang, L-18775, Nov. 30, 1963; Sun Peck Yong v. Commissioner of Immigration, L-20784, December 27, 1963; Tong
Siok Sy v. Vivo, L-21136, December 27, 1963). The writer of this opinion has submitted the question anew to the court for a
possible reexamination of the said ruling in the light of the interpretation of a similar law in the United States after which
Section 15 of our Naturalization Law was patterned. That law was section 2 of the Act of February 10, 1855 (Section 1994 of
the Revised Statutes of the U.S.). The local law, Act No. 3448, was passed on November 30, 1928 as an amendment to the
former Philippine Naturalization Law, Act No. 2927, which was approved on March 26, 1920. Under this Naturalization Law,
acquisition of Philippine citizenship was limited to three classes of persons, (a) Natives of the Philippines who were not citizens
thereof; (b) natives of the other insular possessions of the United States; and (c) citizens of the United States, or foreigners
who, under the laws of the United States, may become citizens of the latter country if residing therein. The reference in
subdivision (c) to foreigners who may become American Citizens is restrictive in character, for only persons of certain specified
races were qualified thereunder. In other words, in so far as racial restrictions were concerned there was at the time a
similarity between the naturalization laws of the two countries, and hence there was reason to accord here persuasive force to
the interpretation given in the United States to the statutory provision concerning the citizenship of alien women marrying
American citizens.
"This Court, however, believes that such reason has ceased to exist since the enactment of the Revised Naturalization
Law (Commonwealth Act No. 473) on June 17, 1939. The racial restrictions have been eliminated in this Act, but the provision
found in Act No. 3448 has been maintained. It is logical to presume that when Congress chose to retain the said provision —
that to be deemed a Philippine citizen upon marriage the alien wife must be one 'who might herself be lawfully naturalized,'
the reference is no longer to the class or race to which the woman belongs, for class or race has become immaterial, but to the
qualifications and disqualifications for naturalization as enumerated in Sections 2 and 4 of the statute. Otherwise the
requirement that the woman 'might herself be lawfully naturalized' would be meaningless surplusage, contrary to settled
norms of statutory construction.
"The rule laid down by this Court in this and in other cases heretofore decided is believed to be in line with the national policy
of selective admission to Philippine citizenship, which after all is a privilege granted only to those who are found worthy
thereof, and not indiscriminately to anybody at all on the basis alone of marriage to a man who is a citizen of the Philippines,
irrespective of moral character, ideological beliefs, and identification with Filipino ideals, customs and traditions.

"Appellee here having failed to prove that she has all the qualifications for naturalization, even, indeed, that she has none of
the disqualifications, she is not entitled to recognition as a Philippine citizen."
In the second Ly Giok Ha, the Court further fortified the arguments in favor of the same conclusion thus:
"On cross-examination, she (Ly Giok Ha) failed to establish that: (1) she has been residing in the Philippines for a continuous
period of at least (10) years (p. 27, t.s.n., id.); (2) she has a lucrative trade, profession, or lawful occupation (p. 13. t.s.n., id.);
and (3) she can speak and write English, or any of the principal Philippine languages (pp. 12, 13, t.s.n., id.)
"While the appellant Immigration Commissioner contends that the words emphasized indicate that the present Naturalization
Law requires that an alien woman who marries a Filipino husband must possess the qualifications prescribed by section 2 in
addition to not being disqualified under any of the eight ('a' to 'h') subheadings of section 4 ofCommonwealth Act No. 473, in
order to claim our citizenship by marriage, both the appellee and the court below (in its second decision) sustain the view that
all that the law demands is that the woman be not disqualified under section 4.
"At the time the present case was remanded to the court of origin (1960) the question at issue could be regarded as not
conclusively settled, there being only the concise pronouncement in Lee Suan Ay, et al. v. Galang, G. R. No. L-11855, Dec. 23,
1959, to the effect that:
'The marriage of a Filipino citizen to an alien does not automatically confer Philippine citizenship upon the latter. She
must possess the qualifications required by law to become a Filipino citizen by naturalization.'
"Since that time, however, a long line of decisions of this Court has firmly established the rule that the requirement of section
15 of Commonwealth Act 473 (the Naturalization Act), that an alien woman married to a citizen should be one who 'might
herself be lawfully naturalized," means not only woman free from the disqualifications enumerated in section 4 of the Act but
also one who possesses the qualifications prescribed by section 2 of Commonwealth Act 473 (San Tuan v. Galang, L-18775,
Nov. 30, 1963; Sun Peck Yong v. Com. of Immigration, L-20784, Dec. 27, 1963; Tong Siok Sy v. Vivo, L-21136, Dec. 27, 1963;
Austria v. Conchu, L-20716, June 22, 1965; Choy King Tee v. Galang, L-18351, March 26, 1965; Brito v. Com. of Immigration, L-
16829, June 30, 1965).
"Reflection will reveal why this must be so. The qualifications prescribed under section 2 of the Naturalization Act, and the
disqualifications enumerated in its section 4 are not mutually exclusive; and if all that were to be required is that the wife of a
Filipino be not disqualified under section 4, the result might well be that citizenship would be conferred upon persons in
violation of the policy of the statute. For example, section 4 disqualifies only —
'(c) Polygamists or believers in the practice of polygamy; and
'(d) Persons convicted of crimes involving moral turpitude,'
so that a blackmailer, or a maintainer of gambling or bawdy houses, not previously convicted by a competent court would not
be thereby disqualified; still, it is certain that the law did not intend such person to be admitted as a citizen in view of the
requirement of section 2 that an applicant for citizenship 'must be of good moral character.'
"Similarly, the citizen's wife might be a convinced believer in racial supremacy, in government by certain selected classes, in
the right to vote exclusively by certain 'herrenvolk', and thus disbelieve in the principles underlying the Philippine Constitution;
yet she would not be disqualified under section 4, as long as she is not 'opposed to organized government,' nor affiliated to
groups 'upholding or teaching doctrines opposing all organized governments', nor 'defending or teaching the necessity or
propriety of violence, personal assault or assassination for the success or predominance of their ideas.' Et sic de caeteris.
"The foregoing instances should suffice to illustrate the danger of relying exclusively on the absence of disqualifications,
without taking into account the other affirmative requirements of the law, which, in the case at bar, the appellee Ly Giok Ha
admittedly does not possess.
"As to the argument that the phrase 'might herself be lawfully naturalized' was derived from the U.S. Revised Statutes (section
1994) and should be given the same territorial and racial significance given to it by American courts, this Court has rejected the
same in Lon San Tuang v. Galang, L-18775, November 30, 1963; and in Choy King Tee v. Galang, L-18351, March 26, 1965."
It is difficult to minimize the persuasive force of the foregoing rationalizations, but a closer study thereof cannot but reveal certain relevant
considerations which adversely affect the premises on which they are predicated, thus rendering the conclusions arrived thereby not entirely
unassailable.
1. The main proposition, for instance, that in eliminating Section 1 of Act 2927 providing who are eligible for Philippine citizenship, the purpose
of Commonwealth Act 473, the Revised Naturalization Law, was to remove the racial requirements for naturalization, thereby opening the door of
Filipino nationality to Asiatics instead of allowing the admission thereto of Caucasians only, suffers from lack of exact accuracy. It is important to
note, to start with, that Commonwealth Act 473 did away with the whole Section 1 of Act 2927 which reads thus:
"SECTION 1. Who may become Philippines citizens. — Philippine citizenship may be acquired by: (a) natives of the Philippines
who are not citizens thereof under the Jones Law; (b) natives of the other Insular possessions of the United States; (c) citizens
of the United States, or foreigners who under the laws of the United States may become citizens of said country if residing
therein."
and not only subdivision (c) thereof. Nowhere in this whole provision was there any mention of race or color of the persons who were then
eligible for Philippine citizenship. What is more evident from said provision is that it reflected the inevitable subordination of our legislation
during the pre-Commonwealth American regime to the understandable limitations flowing from our status as a territory of the United States by
virtue of the Treaty of Paris. In fact, Section 1 of Act 2927 was precisely approved pursuant to express authority, without which it could not have
been done, granted by an amendment to Section 4 of the Philippine Bill of 1902 introduced by the Act of the United States Congress of March
23, 1912 and which was reenacted as part of the Jones Law of 1916, the pertinent provisions of which have already been quoted earlier. In
truth, therefore, it was because of the establishment of the Philippine Commonwealth and in the exercise of our legislative autonomy on
citizenship matters under the Philippine Independence Act that Section 1 of Act 2927 was eliminated, 15 and not purposely to eliminate any
racial discrimination contained in our Naturalization Law. The Philippine Legislature naturally wished to free our Naturalization Law from the
impositions of American legislation. In other words, the fact that such discrimination was removed was one of the effects rather than the
intended purpose of the amendment.
2. Again, the statement in Choy King Tee to the effect that "the reference in subdivision (c) (of Section 1 of Act 2927) to foreigners who may become
American citizens is restrictive in character, for only persons of certain specified races were qualified thereunder" fails to consider the exact import of
the said subdivision. Explicitly, the thrust of the said subdivision was to confine the grant under it of Philippine citizenship only to the three classes of
persons therein mentioned, the third of which were citizens of the United States and, corollarily, persons who could be American citizens under her
laws. The words used in the provision do not convey any idea of favoring aliens of any particular race or color and of excluding others, but more
accurately, they refer to all the disqualifications of foreigners for American citizenship under the laws of the United States. The fact is that even as of
1906, or long before 1920, when our Act 2927 became a law, the naturalization laws of the United States already provided for the following
disqualifications in the Act of the Congress of June 29, 1906:
"SEC. 7. That no person who disbelieves in or who is opposed to organized government, or who is a member of or affiliated
with any organization entertaining and teaching such disbelief in or opposition to organized government, or who advocates or
teaches the duty, necessity, or propriety of the unlawful assaulting or killing of any officer or officers, either of specific
individuals or of officers generally, of the Government of the United States, or of any other organized government, because of
his or their official character, or who is a polygamist, shall be naturalized or be made a citizen of the United States."
and all these disqualified persons were, therefore, ineligible for Philippine citizenship under Section 1 of Act 2927 even if they happened to be
Caucasians. More importantly, as a matter of fact, said American law, which was the first "Act to Establish a Bureau of Immigration and
Naturalization and to Provide for a Uniform Rule for Naturalization of Aliens throughout the United States" contained no racial disqualification
requirement, except as to Chinese, the Act of May 6, 1882 not being among those expressly repealed by this law, hence it is clear that when Act
2927 was enacted, subdivision (c) of its Section 1 could not have had any connotation of racial exclusion necessarily, even if it were traced back
to its origin in the Act of the United States Congress of 1912 already mentioned above. 16 Thus, it would seem that the nationalization in the
quoted decisions predicated on the theory that the elimination of Section 1 of Act 2927 by Commonwealth Act 473 was purposely for no other
end than the abolition of racial discrimination in our naturalization law has no clear factual basis. 17

3. In view of these considerations, there appears to be no cogent reason, why the construction adopted in the opinions of the Secretary of Justice
referred to in the first Ly Giok Ha decision of the Chief Justice should not prevail. It is beyond dispute that the first paragraph of Section 15
of Commonwealth Act 473 is a reenactment of Section 13(a) of Act 2927, as amended by Act 3448, and that the latter is nothing but an exact copy,
deliberately made, of Section 1994 of the Revised Statutes of the United States as it stood before it repeal in 1922. 18 Before such repeal, the phrase
"who might herself be lawfully naturalized" found in said Section 15 had a definite unmistakable construction uniformly followed in all courts of the
United States that had occasion to apply the same and which, therefore, must be considered as if it were written in the statute itself. It is almost trite
to say that when our legislators enacted said section, they knew of its unvarying construction in the United States and that, therefore, in adopting
verbatim the American statute, they have in effect incorporated into the provision, as thus enacted, the construction given to it by the American
courts as well as the Attorney General of the United States and all administrative authorities charged with the implementation of the naturalization
and immigration laws of that country. (Lo Cham v. Ocampo, 77 Phil., 635 [1946]; Laxamana v. Baltazar, 92 Phil., 32 [1952]; Hartley v. Commissioner,
295 U.S. 216, 79 L. ed. 1399, 55 S Ct. 756 [1935]; Helvering v. Windmill, 305 U.S. 79, 83 L ed. 52, 59 S Ct. 45 [1938]; Helvering v. R. J. Reynolds
Tobacco Co., 306 U.S. 110, 83 L ed. 536, 59 S Ct. 423 [1939]. [p. 32, Memo of Amicus Curiae]).
A fairly comprehensive summary of the said construction by the American courts and administrative authorities is contained in United Stats of
America ex rel. Dora Sejnensky v. Robert E. Tod, Commissioner of Immigration, Appt., 285 Fed. 523, decided November 14, 1922, 26 A. L. R. 1316 as
follows:
"Section 1994 of the Revised Statutes (Comp. Stat. § 3948, 2 Fed. Sta. Anno. 2d ed. p. 117) provides as follows: 'Any woman
who is now or may hereafter be married to a citizen of the United States, and who might herself be lawfully naturalized, shall
be deemed a citizen.'
"Section 1944 of the Revised Stat. is said to originate in the Act of Congress of February 10, 1855 (10 Stat. at L. 604, chap. 71),
which in its second section provided 'that any woman, who might lawfully be naturalized under the existing laws, married, or
who shall be married to a citizen of the United States, shall be deemed and taken to be a citizen.'
"And the American Statute of 1855 is substantially a copy of the earlier British Statute 7 & 8 Vict. chap. 66, . . . 16, 1844, which
provided that 'any woman married, or who shall be married, to a natural-born subject or person naturalized, shall be deemed
and taken to be herself naturalized, and have all the rights and privileges of a natural born subject.'
"The Act of Congress of September 22, 1922 (42 Stat. at L. 1021, chap. 411, Comp. Stat. § 4358b, Fed. Stat. Anno. Supp. 1922,
p. 255), being 'An Act Relative to the Naturalization and Citizenship of Married Women,' in § 2, provides 'that any woman who
marries a citizen of the United States after the passage of this Act, . . . shall not become a citizen of the United States by reason
of such marriage . . .'
"Section 6 of the act also provides 'that . . . 1994 of the Revised Statutes . . . are repealed.'
"Section 6 also provides that 'such repeal shall not terminate citizenship acquired or retained under either of such sections, . . .'
meaning § § 2 and 6. So that this Act of September 22, 1922, has no application to the facts of the present case, as the
marriage of the relator took place prior to its passage. This case, therefore, depends upon the meaning to be attached to §
1994 of the Revised Statutes.
"In 1868 the Supreme Court, in Kelly v. Owen, 7 Wall. 496, 498, 19 L. ed. 283, 284, construed this provision as found in the Act
of 1855 as follows: 'The term, "who might lawfully be naturalized under the existing laws," only limits the application of the law
to free white women. The previous Naturalization Act, existing at the time, only required that the person applying for its
benefits should be "a free white person," and not an alien enemy.'
"This construction limited the effect of the statute to those aliens who belonged to the class or race which might be lawfully
naturalized, and did not refer to any of the other provisions of the naturalization laws as to residence or moral character, or to
any of the provisions of the immigration laws relating to the exclusion or deportation of aliens.
"In 1880, in Leonard v. Grant (C. C.) 5 Fed. 11, District Judge Deady also construed the Act of 1855, declaring that 'any woman
who is now or may hereafter be married to a citizen of the United States, and might herself be lawfully naturalized, shall be
deemed a citizen.' He held that 'upon the authorities, and the reason, if not the necessity, of the case,' the statute must be
construed as in effect declaring that an alien woman, who is of the class or race that may be lawfully naturalized under the
existing laws, and who marries a citizen of the United States, is such a citizen also, and it was not necessary that it should
appear affirmatively that she possessed the other qualifications at the time of her marriage to entitle her to naturalization.
"In 1882, the Act of 1855 came before Mr. Justice Harlan, sitting in the circuit court, in United States v. Kellar, 13 Fed. 82. An
alien woman, a subject of Prussia came to the United States and married here a naturalized citizen. Mr. Justice Harlan, with the
concurrence of Judge Treat, held that upon her marriage she became ipso facto a citizen of the United States as fully as if she
had complied with all of the provisions of the statutes upon the subject of naturalization. He added: 'There can be no doubt of
this, in view of the decision of the Supreme Court of the United States in Kelly v. Owen, 7 Wall. 496, 19 L. ed. 283.' The alien
'belonged to the class of persons' who might be lawfully naturalized.
"In 1904, in Hopkins v. Fachant, 65 C. C. A. 1, 130 Fed. 839, an alien woman came to the United States from France and entered
the country contrary to the immigration laws. The immigration authorities took her into custody at the port of New York, with
the view of deporting her. She applied for her release under a writ of habeas corpus, and pending the disposition of the matter
she married a naturalized American citizen. The circuit court of appeals for the ninth circuit held, affirming the court below,
that she was entitled to be discharged from custody. The court declared: 'The rule is well settled that her marriage to a
naturalized citizen of the United States entitled her to be discharged. The status of the wife follows that of her husband, . . .
and by virtue of her marriage her husband's domicil became her domicil.'
"In 1908, the circuit court for the district of Rhode Island in Re Rustigian, 165 Fed. 980, had before it the application of a
husband for his final decree of naturalization. It appeared that at that time his wife was held by the immigration authorities at
New York on the ground that she was afflicted with a dangerous and contagious disease. Counsel on both sides agreed that the
effect of the husband's naturalization would be to confer citizenship upon the wife. In view of that contingency District Judge
Brown declined to pass upon the husband's application for naturalization, and thought it best to wait until it was determined
whether the wife's disease was curable. He placed his failure to act on the express ground that the effect of naturalizing the
husband might naturalize her. At the same time he expressed his opinion that the husband's naturalization would not effect
her naturalization, as she was not one who could become lawfully naturalized. 'Her own capacity (to become naturalized),' the
court stated, 'is a prerequisite to her attaining citizenship. If herself lacking in that capacity, the married status cannot confer it
upon her.' Nothing, however, was actually decided in that case, and the views expressed therein are really nothing more than
mere dicta. But, if they can be regarded as something more than that, we find ourselves, with all due respect for the learned
judge, unable to accept them.
"In 1909, in United States ex rel. Nicola v. Williams, 173 Fed, 626, District Judge Learned Hand held that an alien woman, a
subject of the Turkish Empire, who married an American citizen while visiting Turkey, and then came to the United States,
could not be excluded, although she had, at the time of her entry, a disease which under the immigration laws would have
been sufficient ground for her exclusion, if she had not had the status of a citizen. The case was brought into this court on
appeal, and in 1911 was affirmed, in 106 C. C. A. 464, 184 Fed. 322. In that case, however at the time the relators married, they
might have been lawfully naturalized, and we said: 'Even if we assume the contention of the district attorney to be correct that
marriage will not make a citizen of a woman who would be excluded under our immigration laws, it does not affect these
relators.'
"We held that, being citizens, they could not be excluded as aliens; and it was also said to be inconsistent with the policy of our
law that the husband should be a citizen and the wife an alien. The distinction between that case and the one now before the
court is that, in the former case, the marriage took place before any order of exclusion had been made, while in this the
marriage was celebrated after such an order was made. But such an order is a mere administrative provision, and has not the
force of a judgment of a court, and works no estoppel. The administrative order is based on the circumstances that existed at
the time the order of exclusion was made. If the circumstances change prior to the order being carried into effect, it cannot be
executed. For example, if an order of exclusion should be based on the ground that the alien was at the time afflicted with a
contagious disease, and it should be made satisfactorily to appear, prior to actual deportation, that the alien had entirely
recovered from the disease, we think it plain that the order could not be carried into effect. So, in this case, if, after the making
of the order of exclusion and while she is permitted temporarily to remain, she in good faith marries an American citizen, we
cannot doubt the validity of her marriage, and that she thereby acquired, under international law and under § 1994 of the
Revised Statutes, American citizenship, and ceased to be an alien. There upon, the immigration authorities lost their
jurisdiction over her, as that jurisdiction applies only to aliens, and not to citizens.

"In 1910, District Judge Dodge, in Ex parte Kaprielian, 188 Fed. 694, sustained the right of the officials to deport a woman
under the following circumstances: She entered this country in July, 1910, being an alien and having been born in Turkey. She
was taken into custody by the immigration authorities in the following September, and in October a warrant for her
deportation was issued. Pending hearings as to the validity of that order, she was paroled in the custody of her counsel. The
ground alleged for her deportation was that she was afflicted with a dangerous and contagious disease at the time of her
entry. One of the reasons assigned to defeat deportation was that the woman had married a citizen of the United States
pending the proceedings for her deportation. Judge Dodge declared himself unable to believe that a marriage under such
circumstances 'is capable of having the effect claimed, in view of the facts shown.' He held that it was no part of the intended
policy of § 1994 to annul or override the immigration laws, so as to authorize the admission into the country of the wife of a
naturalized alien not otherwise entitled to enter, and that an alien woman, who is of a class of persons excluded by law from
admission to the United States does not come within the provisions of that section. The court relied wholly upon the dicta
contained in the Rustigian Case. No other authorities were cited.
"In 1914, District Judge Neterer, in Ex parte Grayson, 215 Fed. 449, construed § 1994 and held that where, pending
proceedings to deport an alien native of France as an alien prostitute, she was married to a citizen of the United States, she
thereby became a citizen, and was not subject to deportation until her citizenship was revoked by due process of law. It was
his opinion that if, as was contended, her marriage was conceived in fraud, and was entered into for the purpose of evading
the immigration laws and preventing her deportation, such fact should be established in a court of competent jurisdiction in an
action commenced for the purpose. The case was appealed and the appeal was dismissed. 134 C. C. A. 666, 219 Fed. 1022.
"It is interesting also to observe the construction placed upon the language of the statute by the Department of Justice. In
1874, Attorney General Williams, 14 Ops. Atty. Gen. 402, passing upon the Act of February 10, 1855, held that residence within
the United States for the period required by the naturalization laws was not necessary in order to constitute an alien woman a
citizen, she having married a citizen of the United States abroad, although she never resided in the United States, she and her
husband having continued to reside abroad after the marriage.
"In 1909, a similar construction was given to the Immigration Act of May 5, 1907, in an opinion rendered by Attorney General
Wickersham. It appeared an unmarried woman, twenty-eight years of age and a native of Belgium, arrived in New York and
went at once to a town in Nebraska, where she continued to reside. About fifteen months after her arrival she was taken
before a United States commissioner by way of instituting proceedings under the Immigration Act (34 Stat. at L. 898, chap.
1134, Comp. Stat. § 4242, 3 Fed. Stat. Anno. 2d ed. p. 637) for her deportation, on the ground that she had entered this
country for the purpose of prostitution, and had been found an inmate of a house of prostitution and practicing the same
within three years after landing. It appeared, however, that after she was taken before the United States commissioner, but
prior to her arrest under a warrant by the Department of Justice, she was lawfully married to a native-born citizen of the
United States. The woman professed at the time of her marriage an intention to abandon her previous mode of life and to
remove with her husband to his home in Pennsylvania. He knew what her mode of life had been, but professed to believe in
her good intentions. The question was raised as to the right to deport her, the claim being advance that by her marriage she
had become an American citizen and therefore could not be deported. The Attorney General ruled against the right to deport
her as she had become an American citizen. He held that the words, 'who might herself be lawfully naturalized,' refer to a class
or race who might be lawfully naturalized, and that compliance with the other conditions of the naturalization laws was not
required. 27 Ops. Atty. Gen. 507.
"Before concluding this opinion, we may add that it has not escaped our observation that Congress, in enacting the
Immigration Act of 1917, co as to provide, in § 19, 'that the marriage to an American citizen of a female of the sexually immoral
classes . . . shall not invest such female with United States citizenship if the marriage of such alien female shall be solemnized
after her arrest or after the commission of acts which make her liable to deportation under this act.'
"Two conclusions seem irresistibly to follow from the above change in the law:
"(1) Congress deemed legislation essential to prevent women of the immoral class avoiding deportation through the device of
marrying an American citizen.
"(2) If Congress intended that the marriage of an American citizen with an alien woman of any other of the excluded classes,
either before or after her detention should not confer upon her American citizenship, thereby entitling her to enter the
country, its intention would have been expressed, and § 19 would not have been confined solely to women of the immoral
class."
Indeed, We have examined all the leading American decisions on the, subject and We have found no warrant for the proposition that the phrase
"who might herself be lawfully naturalized" in Section 1994 of the Revised Status was meant solely as a racial bar, even if loose statements in some
decisions and other treaties and other writings on the subject would seem to give such impression. The case of Kelly v. Owen, supra, which appears
to be the most cited among the first of these decisions 19 simply held:
"As we construe this Act, it confers the privileges of citizenship upon women married to citizens of the United States, if they
are of the class of persons for whose naturalization the previous Acts of Congress provide. The terms 'married' or 'who shall be
married,' do not refer, in our judgment, to the time when the ceremony of marriage is celebrated, but to a state of marriage.
They mean that, whenever a woman, who under previous Acts might be naturalized, is in a state of marriage to a citizen,
whether his citizenship existed at the passage of the Act or subsequently, or before or after the marriage, she becomes, by that
fact, a citizen also. His citizenship, whenever it exists, confers, under the Act, citizenship upon her. The construction which
would restrict the Act to women whose husbands, at the time of marriage, are citizens, would exclude far the greater number,
for whose benefit, as we think, the Act was intended. Its object, in our opinion, was to allow her citizenship to follow that of
her husband, without the necessity of any application for naturalization on her part; and, if this was the object, there is no
reason for the restriction suggested.
"The terms, 'who might lawfully be naturalized under the existing laws,' only limit the application of the law to free white
women. The previous Naturalization Act, existing at the time only required that the person applying for its benefits should be 'a
free white person,' and not an alien enemy. Act of April 14th, 1802, 2 Stat. at L. 153.
"A similar construction was given to the Act by the Court of Appeals of New York, in Burton v. Burton, 40 N. Y. 373; and is the
one which gives the widest extension to its provisions"
Note that while the court did say that "the terms, 'who might lawfully be naturalized under existing laws' only limit the application to free white
women" 20 it hastened to add that "the previous Naturalization Act, existing at the time, . . . required that the person applying for its benefits should
be (not only) a 'free white person' (but also) . . . not an alien enemy." This is simply because under the Naturalization Law of the United States at the
time the case was decided, the disqualification of enemy aliens had already been removed by the Act of July 30, 1813, as may be seen in the
corresponding footnote hereof anon. In other words, if in the case of Kelly v. Owen only the race requirement was mentioned, the reason was that
there was no other non-racial requirement or no more alien-enemy disqualification at the time; and this is demonstrated by the fact that the court
took care to make it clear that under the previous naturalization law, there was also such requirement in addition to race. This is important, since as
stated in re Rustigian, 165 Fed. Rep. 980, "The expression used by Mr. Justice Field, (in Kelly v. Owen) the terms 'who might lawfully be naturalized
under existing laws' only limit the application of the law to free white women, must be interpreted in the application to the special facts and to the
incapacities under the then existing laws," (at p. 982) meaning that whether or not an alien wife marrying a citizen would be a citizen was dependent,
not only on her race and nothing more necessarily, but on whether or not there were other disqualifications under the law in force at the time of her
marriage or the naturalization of her husband.
4. As already stated, in Lo San Tuang, Choy King Tee and the second Ly Giok Ha, the Court drew the inference that because Section 1 of Act 2927 was
eliminated byCommonwealth Act 473, it. follows that in place of the said eliminated section, particularly its subdivision (c), being the criterion of
whether or not an alien wife "may be lawfully naturalized," what should be required is not only that she must not be disqualified under Section 4 but
that she must also possess the qualifications enumerated in Section 2, such as those of age, residence, good moral character, adherence to the
underlying principles of the Philippine Constitution, irreproachable conduct, lucrative employment or ownership of real estate, capacity to speak and
write English or Spanish and one of the principal local languages, education of children in certain schools, etc., thereby implying that, in effect, said
Section 2 has been purposely intended to take the place of Section 1 of Act 2927. Upon further consideration of the proper premises, We have come
to the conclusion that such inference is not sufficiently justified.

To begin with, nothing extant in the legislative history, which We have already examined above of the mentioned provisions has been shown or can
be shown to indicate that such was the clear intent of the legislature. Rather, what is definite is that Section 15 an exact copy of Section 1994 of the
Revised Statutes of the United States, which, at the time of the approval of Commonwealth Act 473 had already a settled construction by American
courts and administrative authorities.
Secondly, as may be gleaned from the summary of pertinent American decisions quoted above, there can be no doubt that in the construction of the
identically worded provision in the Revised Statutes of the United States, (Section 1994, which was taken from the Act of February 10, 1855) all
authorities in the United States are unanimously agreed that the qualifications of residence, good moral character, adherence to the Constitution,
etc. are not supposed to be considered, and that the only eligibility to be taken into account is that of the race or class to which the subject belongs,
the conceptual scope of which, We have just discussed. 21 In the very case of Leonard v. Grant, supra,discussed by Justice Regala in Lo San Tuang,
the explanation for such posture of the American authorities was made thus:
"The phrase, 'shall be deemed a citizen,' in section 1994 Rev. St., or as it was in the Act of 1855, supra, 'shall be deemed and
taken to be a citizen,' while it may imply that the person to whom it relates has not actually become a citizen by ordinary
means or in the usual way, as by the judgment of a competent court, upon a proper application and proof, yet it does not
follow that such person is on that account practically any the less a citizen. The word 'deemed' is the equivalent of 'considered'
or 'judged'; and, therefore, whatever an act of Congress requires to be 'deemed' or 'taken' as true of any person or thing, must,
in law, he considered as having been duly adjudged or established concerning such person or thing, and have force and effect
accordingly. When, therefore, Congress declares that an alien woman shall, under certain circumstances, be 'deemed' an
American citizen, the effect when the contingency occurs, is equivalent to her being naturalized directly by an act of Congress,
or in the usual mode thereby prescribed."
Unless We disregard now the long settled familiar rule of statutory construction that in a situation like this wherein our legislature has copied an
American statute word for word, it is understood that the construction already given to such statute before its being copied constitute part of
our own law, there seems to be no reason how We can give a different connotation or meaning to the provision in question. At least, We have
already seen that the views sustaining the contrary conclusion appear to be based on inaccurate factual premises related to the real legislative
background of the framing of our naturalization law in its present form.
Thirdly, the idea of equating the qualifications enumerated in Section 2 of Commonwealth Act 473 with the eligibility requirements of Section 1 of
Act 2927 cannot bear close scrutiny from any point of view. There is no question that Section 2 of Commonwealth Act 473 is more or less
substantially the same as Section 3 of Act 2927. In other words, Section 1 of Act 2927 co-existed already with practically the same provision as
Section 2 of Commonwealth Act 473. If it were true that the phrase "who may be lawfully naturalized" in Section 13(a) of Act 2927, as amended by
Act 3448, referred to the so called racial requirement in Section 1 of the same Act, without regard to the provisions of Section 3 thereof, how could
the elimination of Section 1 have the effect of shifting the reference to Section 3, when precisely, according to the American jurisprudence, which
was prevailing at the time Commonwealth Act 473 was approved, such qualifications as were embodied in said Section 3, which had their
counterpart in the corresponding American statutes, are not supposed to be taken into account and that what should be considered only are the
requirements similar to those provided for in said Section 1 together with the disqualifications enumerated in Section 4?
Fourthly, it is difficult to conceive that the phrase "who might be lawfully naturalized" in Section 15 could have been intended to convey a meaning
different than that given to it by the American courts and administrative authorities. As already stated, Act 3448 which contained said phrase and
from which it was taken by Commonwealth Act 473, was enacted in 1928. By that time, Section 1994 of the Revised Statutes of the United States was
no longer in force because it had been repealed expressly the Act of September 22, 1922 which did away with the automatic naturalization of alien
wives of American citizens and required, instead, that they submit to regular naturalization proceedings, albeit under more liberal terms than those
of other applicants. In other words, when our legislature adopted the phrase in question, which, as already demonstrated, had a definite
construction in American law, the Americans had already abandoned said phraseology in favor of a categorical compulsion for alien wives to be
naturalized judicially. Simple logic would seem to dictate that, since our lawmakers, at the time of the approval of Act 3448, had two choices, one to
adopt the phraseology of Section 1994 with its settled construction and the other to follow the new posture of the Americans of requiring judicial
naturalization, and it appears that they have opted for the first, We have no alternative but to conclude that our law still follows the old or previous
American law on the subject. Indeed, when Commonwealth Act 473 was approved in 1939, the Philippine Legislature, already autonomous then from
the American Congress, had a clearer chance to disregard the old American law and make one of our own, or, at least, follow the trend of the Act of
the U.S. Congress of 1922, but still, our legislators chose to maintain the language of the old law. What then is significantly important is not that the
legislature maintained said phraseology after Section 1 of Act 2927 was eliminated, but that it continued insisting on using it even after the
Americans had amended their law in order to provide for what is now contended to be the construction that should be given to the phrase in
question. Stated differently, had our legislature adopted a phrase from an American statute before the American courts had given it a construction
which was acquiesced to by those given upon to apply the same, it would be possible for Us to adopt a construction here different from that of the
Americans, but as things stand, the fact is that our legislature borrowed the phrase when there was already a settled construction thereof, and what
is more, it appears that our legislators even ignored the modification of the American law and persisted in maintaining the old phraseology. Under
these circumstances, it would be in defiance of reason and the principles of Statutory construction to say that Section 15 has a nationalistic and
selective orientation and that it should be construed independently of the previous American posture because of the difference of circumstances
here and in the United States. It is always safe to say that in the construction of a statute, We cannot fall on possible judicial fiat or perspective when
the demonstrated legislative point of view seems to indicate otherwise.
5. Viewing the matter from another angle, there is need to emphasize that in reality and in effect, the so-called racial requirements, whether under
the American laws or the Philippine laws, have hardly been considered as qualifications in the same sense as those enumerated in Section 3 of Act
2927 and later in Section 2 of Commonwealth Act 473. More accurately, they have always been considered as disqualifications, in the sense that
those who did not possess them were the ones who could not "be lawfully naturalized," just as if they were suffering from any of the
disqualifications under Section 2 of Act 2927 and later those under Section 4 of Commonwealth Act 473, which, incidentally, are practically identical
to those in the former law, except those in paragraphs (f) and (h) of the latter. 22 Indeed, such is the clear impression anyone will surely get after
going over all the American decisions and opinions quoted and/or cited in the latest USCA (1970), Title 8, section 1430, pp. 598-602, and the first
decisions of this Court on the matter, Ly Giok Ha (1959) and Ricardo Cua, citing with approval the opinions of the Secretary of Justice. 23 Such being
the case, that is, that the so-called racial requirements were always treated as disqualifications in the same light as the other disqualifications under
the law, why should their elimination not be viewed or understood as a subtraction from or a lessening of the disqualifications? Why should such
elimination have instead the meaning that what were previously considered as irrelevant qualifications have become disqualifications, as seems to
be the import of the holding in Choy King Tee to the effect that the retention in Section 15 of Commonwealth Act 473 of the same language of what
used to be Section 13 (a) of Act 2927 (as amended by Act 3448), notwithstanding the elimination of Section 1 of the latter, necessarily indicates that
the legislature had in mind making the phrase in question "who may be lawfully naturalized" refer no longer to any racial disqualification but to the
qualification under Section 2 of Commonwealth Act 473? Otherwise stated, under Act 2927, there were two groups of persons that could not be
naturalized, namely, those falling under Section 1 and those falling under Section 2, and surely, the elimination of one group, i.e. those belonging to
Section 1, could not have had, by any process of reasoning, the effect of increasing, rather than decreasing the disqualifications that used to be
before such elimination. We cannot see by what alchemy of logic such elimination could have converted qualifications into disqualifications, specially
in the light of the fact that, after all, these are disqualifications clearly set out as such in the law distinctly and separately from qualifications and, as
already demonstrated, in American jurisprudence, qualifications had never been considered to be of any relevance in determining "who might be
lawfully naturalized," as such phrase is used in the statute governing the status of alien wives of American citizens, and our law on the matter was
merely copied verbatim from the American statutes.

6. In addition to these arguments based on the applicable legal provisions and judicial opinions, whether here or in the United States, there are
practical considerations that militate towards the same conclusions. As aptly stated in the motion for reconsideration of counsel for petitioner-
appellee dated February 23, 1967, filed in the case of Zita Ngo Burca v. Republic, supra:
"Unreasonableness of requiring alien wife to prove 'qualifications —
"There is one practical consideration that strongly militates against a construction that Section 15 of the law requires that an
alien wife of a Filipino must affirmatively prove that she possesses the qualifications prescribed under Section 2, before she
may be deemed a citizen. Such condition, if imposed upon an alien wife, becomes unreasonably onerous and compliance
therewith manifestly difficult. The unreasonableness of such requirement is shown by the following:
"1. One of the qualifications required of an applicant for naturalization under Section 2 of the law is that the
applicant 'must have resided in the Philippines for a continuous period of not less than ten years.' If this requirement
is applied to an alien wife married to a Filipino citizen, this means that for a period of ten years at least, she cannot
hope to acquire the citizenship of her husband. If the wife happens to be a citizen of a country whose law declares
that upon her marriage to a foreigner she automatically loses her citizenship and acquires the citizenship of her
husband, this could mean that for a period of ten years at least, she would be stateless. And even after having
acquired continuous residence in the Philippines for ten years, there is no guarantee that her petition for
naturalization will be granted, in which case she would remain stateless for an indefinite period of time.
"2. Section 2 of the law likewise requires of the applicant for naturalization that he 'must own real estate in the
Philippines worth not less than five thousand pesos, Philippine currency, or must have some known lucrative trade,
profession, or lawful occupation.' Considering the constitutional prohibition against acquisition by an alien of real
estate except in cases of hereditary succession (Art. XIII, Sec. 5, Constitution), an alien wife desiring to acquire the
citizenship of her husband must have to prove that she has a lucrative income derived from a lawful trade, profession
or occupation. The income requirement has been interpreted to mean that the petitioner herself must be the one to
possess the said income. (Uy v. Republic, L-19578, Oct. 27, 1964; Tanpa Ong vs. Republic, L-20605, June 30, 1965; Li
Tong Pek v. Republic, L-20912, November 29, 1965). In other words, the wife must prove that she has a lucrative
income derived from sources other than her husband's trade, profession or calling. It is of common knowledge, and
judicial notice may be taken of the fact that most wives in the Philippines do not have gainful occupations of their
own. Indeed, Philippine law, recognizing the dependence of the wife upon the husband, imposes upon the latter the
duty of supporting the former. (Art. 291, Civil Code). It should be borne in mind that universally, it is an accepted
concept that when a woman marries, her primary duty is to be a wife, mother and housekeeper. If an alien wife is
not to be remiss in this duty, how can she hope to acquire a lucrative income of her own to qualify her for
citizenship?
"3. Under Section 2 of the law, the applicant for naturalization 'must have enrolled his minor children of school age,
in any of the public schools or private schools recognized by the Office of the Private Education of the Philippines,
where Philippine history, government and civics are taught or prescribed as part of the school curriculum during the
entire period of residence in the Philippines required of him prior to the hearing of his petition for naturalization as
Philippine citizen.' If an alien woman has minor children by a previous marriage to another alien before she marries a
Filipino, and such minor children had not been enrolled in Philippine schools during her period of residence in the
country, she cannot qualify for naturalization under the interpretation of this Court. The reason behind the
requirement that children should be enrolled in recognized educational institutions is that they follow the citizenship
of their father. (Chan Ho Lay v. Republic, L-5666, March 30, 1954; Tan Hi v. Republic, 88 Phil. 117 [1951]; Hao Lian
Chu v. Republic, 87 Phil. 668 [950]; Yap Chin v. Republic, L-4177, May 29, 1953; Lim Lian Hong v. Republic, L-3575,
Dec. 26, 1950). Considering that said minor children by her first husband generally follow the citizenship of their alien
father, the basis for such requirement as applied to her does not exist. Cessante ratione legis cessat ipsa lex.
"4. Under Section 3 of the law, the 10-year continuous residence prescribed by Section 2 'shall be understood as
reduced to five years for any petitioner (who is) married to a Filipino woman.' It is absurd that an alien male married
to a Filipino wife should be required to reside only for five years in the Philippines to qualify for citizenship, whereas
an alien woman married to a Filipino husband must reside for ten years.
"Thus under the interpretation given by this Court, it is more difficult for an alien wife related by marriage to a Filipino citizen
to become such citizen, than for a foreigner who is not so related. And yet, it seems more than clear that the general purpose
of the first paragraph of Section 15 was obviously to accord to an alien woman, by reason of her marriage to a Filipino, a
privilege not similarly granted to other aliens. It will be recalled that prior to the enactment of Act No. 3448 in 1928,
amending Act No. 2927 (the old Naturalization Law), there was no law granting any special privilege to alien wives of Filipinos.
They were treated as any other foreigner. It was precisely to remedy this situation that the Philippine legislature enacted Act
No. 3448. On this point, the observation made by the Secretary of Justice in 1941 is enlightening:
'It is true that under Article 22 of the (Spanish) Civil Code, the wife follows the nationality of the husband; but the
Department of State of the United States on October 31, 1921, ruled that the alien wife of a Filipino citizen is not a
Filipino citizen, pointing out that our Supreme Court in the leading case of Roa v. Collector of Customs (23 Phil. 315)
held that Articles 17 to 27 of the Civil Code being political have been abrogated upon the cession of the Philippine
Islands to the United States. Accordingly, the stand taken by the Attorney-General prior to the enactment of Act No.
3448, was that marriage of alien women to Philippine citizens did not make the former citizens of this country.' (Op.
Atty. Gen., March 16, 1928).
'To remedy this anomalous condition, Act No. 5448 was enacted in 1928 adding section 13(a) to Act No. 2997 which
provides that "any woman who is now or may hereafter be married to a citizen of the Philippine Islands, and who
might herself be lawfully naturalized, shall be deemed a citizen of the Philippine Islands.' (Op. No. 22, s. 1941;
emphasis ours)
"If Section 15 of the Revised Naturalization Law were to be interpreted, as this Court did, in such a way as to require that the
alien wife must prove the qualifications prescribed in Section 2, the privilege granted to alien wives would become illusory. It is
submitted that such a construction, being contrary to the manifested object of the statute, must be rejected.
'A statute is to be construed with reference to its manifest object, and if the language is susceptible of two
constructions, one which will carry out and the other defeat such manifest object, it should receive the former
construction.' (In re National Guard, 71 Vt. 493, 45 A. 1051; Singer v. United States, 323 U.S. 333, 89 L. ed. 285. See
also, U.S. v. Navarro, 19 Phil. 134 [1911]; U. S. v. Toribio, 15 Phil. 85 [1910]).
'. . . A construction which will cause objectionable results should be avoided and the court will, if possible, place on
the statute a construction which will not result in injustice, and in accordance with the decisions construing statutes,
a construction which will result in oppression, hardship, or inconveniences will also be avoided, as will a construction
which will prejudice public interest, or construction resulting in unreasonableness, as well as a construction which
will result in absurd consequences.'
'So a construction should, if possible, be avoided if the result would be an apparent inconsistency in legislative intent,
as has been determined by the judicial decisions, or which would result in futility, redundancy, or a conclusion not
contemplated by the legislature; and the court should adopt that construction which will be the least likely to
produce mischief. Unless plainly shown to have been the intention of the legislature, an interpretation which would
render the requirements of the statute uncertain and vague is to be avoided, and the court will not ascribe to the
legislature an intent to confer an illusory right . . .' (82 C.J.S., Statutes, sec. 326, pp. 623-632)."
7. In Choy King Tee and the second Ly Giok Ha, emphasis was laid on the need for aligning the construction of Section 15 with "the national policy of
selective admission to Philippine citizenship." But the question may be asked, is it reasonable to suppose that in the pursuit of such policy, the
legislature contemplated to make it more difficult, if not practically impossible in some instances, for an alien woman marrying a Filipino to become a
Filipina than any ordinary applicant for naturalization, as has just been demonstrated above? It seems but natural and logical to assume that Section
15 was intended to extend special treatment to alien women who by marrying a Filipino irrevocably deliver themselves, their possessions, their fate
and fortunes and all that marriage implies to a citizen of this country, "for better or for worse." Perhaps there can and will be cases wherein the
personal conveniences and benefits arising from Philippine citizenship may motivate such marriage, but must the minority, as such cases are bound
to be, serve as the criterion for the construction of law? Moreover, it is not farfetched to believe that in joining a Filipino family, the alien woman is
somehow disposed to assimilate the customs, beliefs and ideals of Filipinos among whom, after all, she has to live and associate, but surely, no one
should expect her to do so even before marriage. Besides, it may be considered that in reality the extension of citizenship to her is made by the law
not so much for her sake as for the husband. Indeed, We find the following observations anent the national policy rationalization in Choy King Tee
and Ly Giok Ha (the second) to be quite persuasive:

"We respectfully suggest that this articulation of the national policy begs the question. The avowed policy of 'selective
admission' more particularly refers to a case where citizenship is sought to be acquired in a judicial proceeding for
naturalization. In such a case, the courts should no doubt apply the national policy of selecting only those who are worthy to
become citizens. There is here a choice between accepting or rejecting the application for citizenship. But this policy finds no
application in cases where citizenship is conferred by operation of law. In such cases, the courts have no choice to accept or
reject. If the individual claiming citizenship by operation of law proves in legal proceedings that he satisfies the statutory
requirements, the courts cannot do otherwise than to declare that he is a citizen of the Philippines. Thus, an individual who is
able to prove that his father is a Philippine citizen, is a citizen of the Philippines, 'irrespective of his moral character, ideological
beliefs, and identification with Filipino ideals, customs, and traditions.' A minor child of a person naturalized under the law,
who is able to prove the fact of his birth in the Philippines, is likewise a citizen, regardless of whether he has lucrative income,
or he adheres to the principles of the Constitution. So it is with an alien wife of a Philippine citizen. She is required to prove
only that she may herself be lawfully naturalized, i.e., that she is not one of the disqualified persons enumerated in Section 4 of
the law, in order to establish her citizenship status as a fact.
"A paramount policy consideration of graver import should not be overlooked in this regard, for it explains and justifies the
obviously deliberate choice of words. It is universally accepted that a State, in extending the privilege of citizenship to an alien
wife of one of its citizens could have had no other objective than to maintain a unity of allegiance among the members of the
family. (Nelson v. Nelson, 113 Neb. 453, 203 N. W. 640 [1925]; see also 'Convention on the Nationality of Married Women:
Historical Background and Commentary.' UNITED NATIONS, Department of Economic and Social Affairs E/CN, 6/399, pp. 8 et
seq.). Such objective can only be satisfactorily achieved by allowing the wife to acquire citizenship derivatively through the
husband. This is particularly true in the Philippines where tradition and law has placed the husband as head of the family,
whose personal status and decisions govern the life of the family group. Corollary to this, our laws look with favor on the unity
and solidarity of the family (Art. 220, Civil Code), in whose preservation of State as a vital and enduring interest. (See Art. 216,
Civil Code). Thus, it has been said that by tradition in our country, there is a theoretic identity of person and interest between
husband and wife, and from the nature of the relation, the home of one is that of the other. (See De la Viña v. Villareal, 41 Phil.
13). It should likewise be said that because of the theoretic identity of husband and wife, and the primacy of the husband, the
nationality of husband should be the nationality of the wife, and the laws upon one should be the law upon the other. For as
the court, in Hopkins v. Fachant (9th Cir., 1904) 65 C.C.A., 1, 130 Fed. 839, held: 'The status of the wife follows that of the
husband, . . . and by virtue of her marriage her husband's domicile became her domicile.' And the presumption under
Philippine law being that the property relations of husband and wife are under the regime of conjugal partnership (Art. 119,
Civil Code), the income of one is also that of the other.
"It is, therefore, not congruent with our cherished traditions of family unity and identity that a husband should be a citizen and
the wife an alien, and that the national treatment of one should be different from that of the other. Thus, it cannot be that the
husband's interests in property and business activities reserved by law to citizens should not form part of the conjugal
partnership and be denied to the wife, nor that she herself cannot, through her own efforts but for the benefit of the
partnership, acquire such interests. Only in rare instances should the identity of husband and wife be refused recognition, and
we submit that in respect of our citizenship laws, it should only be in the instances where the wife suffers from the
disqualifications stated in Section 4 of the Revised Naturalization Law." (Motion for Reconsideration, Burca vs. Republic, supra.)
With all these considerations in mind, We are persuaded that it is in the best interest of all concerned that Section 15 of the Naturalization Law be
given effect in the same way as it was understood and construed when the phrase "who may be lawfully naturalized," found in the American statute
from which it was borrowed and copied verbatim, was applied by the American courts and administrative authorities. There is merit, of course, in the
view that Philippine statutes should be construed in the light of Philippine circumstances, and with particular reference to our naturalization laws.
We should realize the disparity in the circumstances between the United States, as the so-called "melting pot" of peoples from all over the world,
and the Philippines as a developing country whose Constitution is nationalistic almost in the extreme. Certainly, the writer of this opinion cannot be
the last in rather passionately insisting that our jurisprudence should speak our own concepts and resort to American authorities, to be sure, entitled
to admiration and respect, should not be regarded as source of pride and indisputable authority. Still, We cannot close our eyes to the undeniable
fact that the provision of law now under scrutiny has no local origin and orientation; it is purely American, factually taken bodily from American law
when the Philippines was under the dominating influence of statutes of the United States Congress. It is indeed a sad commentary on the work of our
own legislature of the late 1920's and 1930's that given the opportunity to break away from the old American pattern, it took no step in that
direction. Indeed, even after America made it patently clear in the Act of Congress of September 22, 1922 that alien women marrying Americans
cannot be citizens of the United States without undergoing naturalization proceedings, our legislators still chose to adopt the previous American law
of August 10, 1855 as embodied later in Section 1994 of the Revised Statutes of 1874, which, it is worth reiterating, was consistently and uniformly
understood as conferring American citizenship to alien women marrying Americans ipso facto, without having to submit to any naturalization
proceeding and without having to prove that they possess the special qualifications of residence, moral character, adherence to American ideals and
American constitution, provided they show they did not suffer from any of the disqualifications enumerated in the American Naturalization Law.
Accordingly, We now hold, all previous decisions of this Court indicating otherwise notwithstanding, that under Section 15 ofCommonwealth Act 473,
an alien woman marrying a Filipino, native born or naturalized, becomes ipso facto a Filipina provided she is not disqualified to be a citizen of the
Philippines under Section 4 of the same law. Likewise, an alien woman married to an alien who is subsequently naturalized here follows the
Philippine citizenship of her husband the moment he takes his oath as Filipino citizen, provided that she does not suffer from any of the
disqualifications under said Section 4.
As under any other law rich in benefits for those coming under it, doubtless there will be instances where unscrupulous persons will attempt to take
advantage of this provision of law by entering into fake and fictitious marriages or mala fide matrimonies. We cannot as a matter of law hold that just
because of these possibilities, the construction of the provision should be otherwise than as dictated inexorably by more ponderous relevant
considerations, legal, juridical and practical. There can always be means of discovering such undesirable practices and every case can be dealt with
accordingly as it arises.
III.
The third aspect of this case requires necessarily a re-examination of the ruling of this Court in Burca, supra, regarding the need of judicial
naturalization proceedings before the alien wife of a Filipino may herself be considered or deemed a Filipino. If this case which, as already noted, was
submitted for decision in 1964 yet, had only been decided earlier, before Go Im Ty, the foregoing discussions would have been sufficient to dispose
of it. The Court could have held that despite her apparent lack of qualifications, her marriage to her co-petitioner made her a Filipina, without her
undergoing any naturalization proceedings, provided she could sustain her claim that she is not disqualified under Section 4 of the law. But as things
stand now, with the Burca ruling, the question We have still to decide is, may she be deemed a Filipina without submitting to a naturalization
proceeding?
Naturally, if Burca is to be followed, it is clear that the answer to this question must necessarily be in the affirmative. As already stated, however, the
decision in Burca has not yet become final because there is still pending with Us a motion for its reconsideration which vigorously submits grounds
worthy of serious consideration by this Court. On this account, and for the reasons expounded earlier in this opinion, this case is as good an occasion
as any other to re-examine the issue.
In the said decision, Justice Sanchez held for the Court:
"We accordingly rule that: (1) An alien woman married to a Filipino who desires to be a citizen of this country must apply
therefore by filing a petition for citizenship reciting that she possesses all the qualifications set forth in Section 2 and none of
the disqualifications under Section 4, both of the Revised Naturalization Law; (2) Said petition must be filed in the Court of First
Instance where petitioner has resided at least one year immediately preceding the filing of the petition; and (3) Any action by
any other office, agency, board or official, administrative or otherwise — other than the judgment of a competent court of
justice — certifying or declaring that an alien wife of the Filipino citizen is also a Filipino citizen, is hereby declared null and
void.

"3. We treat the present petition as one for naturalization. Or, in the words of law, a 'petition for citizenship'. This is as it
should be. Because a reading of the petition will reveal at once that efforts were made to act forth therein. and to prove
afterwards, compliance with Sections 2 and 4 of the Revised Naturalization law. The trial court itself apparently considered the
petition as one for naturalization, and, in fact, declared petitioner 'a citizen of the Philippines.'"
In other words, under this holding, in order for an alien woman marrying a Filipino to be vested with Filipino citizenship, it is not enough that she
possesses the qualifications prescribed by Section 2 of the law and none of the disqualifications enumerated in its Section 4. Over and above all
these, she has to pass thru the whole process of judicial naturalization, apparently from declaration of intention to oath-taking, before she can
become a Filipina. In plain words, her marriage to a Filipino is absolutely of no consequence to her nationality vis-a-vis that of her Filipino husband;
she remains to be the national of the country to which she owed allegiance before her marriage, and if she desires to be of one nationality with her
husband, she has to wait for the same time that any other applicant for naturalization needs to complete, the required period of ten year residence,
gain the knowledge of English or Spanish and one of the principal local languages, make her children study in Filipino schools, acquire real property
or engage in some lawful occupation of her own independently of her husband, file her declaration of intention and after one year her application for
naturalization, with the affidavits of two credible witnesses of her good moral character and other qualifications, etc., etc., until a decision is
rendered in her favor, after which, she has to undergo the two years of probation, and only then, but not before she takes her oath as citizen, will
she begin to be considered and deemed to be a citizen of the Philippines. Briefly; she can become a Filipino citizen only by judicial declaration.
Such being the import of, the Court's ruling, and it being quite obvious, on the other hand, upon a cursory reading of the provision, in question, that
the law intends by it to spell out what is the "effect of naturalization on (the) wife and children" of an alien, as plainly indicated by its title, and
inasmuch as the language of the provision itself clearly conveys the thought that some effect beneficial to the wife is intended by it, rather than that
she is not in any manner to be benefited thereby, it behooves Us to take a second hard look at the ruling, if only to see whether or not the Court
might have overlooked any relevant consideration warranting a conclusion different from that contained therein. It is undeniable that the issue
before Us is of grave importance, considering its consequences upon tens of thousands of persons affected by the ruling therein made by the Court,
and surely, it is for Us to avoid, whenever possible, that Our decision in any case should produce any adverse effect upon them not contemplated
either by the law or by the national policy it seeks to enforce.
AMICI CURIAE in the Burca case, respectable and impressive by their number and standing in the Bar and well known for their reputation for
intellectual integrity, legal acumen and incisive and comprehensive resourcefulness in research, truly evident in the quality of the memorandum they
have submitted in said case, invite Our attention to the impact of the decision therein thus:
"The doctrine announced by this Honorable Court for the first time in the present case — that an alien woman who marries a
Philippine citizen not only does not ipso factoherself become a citizen but can acquire such citizenship only through ordinary
naturalization proceedings under the Revised Naturalization Law, and that all administrative actions 'certifying or declaring'
such woman to be a Philippine citizen are 'null and void' — has consequences that reach far beyond the confines of the present
case. Considerably more people are affected, and affected deeply, than simply Mrs. Zita N. Burca. The newspapers report that
as many as 15 thousand women married to Philippine citizens are affected by this decision of the Court. These are women of
many and diverse nationalities, including Chinese, Spanish, British, American, Columbian, Finnish, Japanese, Chilean, and so on.
These members of the community, some of whom have been married to citizens for two or three decades, have all exercised
rights and privileges reserved by law to Philippine citizens. They will have acquired, separately or in conjugal partnership with
their citizen husbands, real property, and they will have sold and transferred such property. Many of these women may be in
professions membership in which is limited to citizens. Others are doubtless stockholders or officers or employees in
companies engaged in business activities for which a certain percentage of Filipino equity content is prescribed by law. All
these married women are now faced with possible divestment of personal status and of rights acquired and privileges
exercised in reliance, in complete good faith, upon a reading of the law that has been accepted as correct for more than two
decades by the very agencies of government charged with the administration of that law. We must respectfully suggest that
judicial doctrines which would visit such comprehensive and far-reaching injury upon the wives and mothers of Philippine
citizens deserve intensive scrutiny and re-examination."
To be sure, this appeal can be no less than what this Court attended to in Gan Tsitung vs. Republic, G.R. No L-20819, Feb. 21, 1967, 19 SCRA
401—when Chief Justice Concepcion observed:
"The Court realizes, however, that the rulings in the Barretto and Delgado cases — although referring to situations the equities
of which are not identical to those obtaining in the case at bar — may have contributed materially to the irregularities
committed therein and in other analogous cases, and induced the parties concerned to believe, although erroneously, that the
procedure followed was valid under the law.
"Accordingly, and in view of the implications of the issue under consideration, the Solicitor General was required, not only, to
comment thereon, but, also, to state 'how many cases there are, like the one at bar, in which certificates of naturalization have
been issued after notice of the filing of the petition for naturalization had been published in the Official Gazette only once,
within the periods (a) from January 28, 1950' (when the decision in Delgado v. Republic was promulgated) 'to May 29, 1957'
(when the Ong Son Cui was decided) 'and (b) from May 29, 1957 to November 29, 1965' (when the decision in the present case
was rendered).
"After mature deliberation, and in the light of the reasons adduced in appellant's motion for reconsideration and in the reply
thereto of the Government, as well as of the data contained in the latter, the Court holds that the doctrine laid down in the
Ong Son Cui case shall apply and affect the validity of certificates of naturalization issued after, not on or before May 29,
1957."
Here We are met again by the same problem. In Gan Tsitung, the Court had to expressly enjoin the prospective application of its construction of the
law made in a previous decision 24 which had already become final, to serve the ends of justice and equity. In the case at bar, We do not have to go
that far. As already observed, the decision in Burca is still under reconsideration, while the ruling in Lee Suan Ay, Lo San Tuang, Choy King Tee and
other that followed them have at the most become the law of the case only for the parties thereto. If there are good grounds therefor, all We have
to do now is to re-examine the said rulings and clarify them.
For ready reference, We requote Section 15:
"Sec. 15. Effect of the naturalization on wife and children .— Any woman who is now or may hereafter be married to a citizen
of the Philippines, and who might herself be lawfully naturalized shall be deemed a citizen of the Philippines.
"Minor children of persons naturalized under this law who have been born in the Philippines shall be consider citizens thereof.
"A foreign-born minor child, if dwelling in the Philippines at the time of naturalization of the parents, shall automatically
become a Philippine citizen, and a foreign-born minor child, who is not in the Philippines at the time the parent is naturalized,
shall be deemed a Philippines citizen only during his minority, unless he begins to reside permanently in the Philippines when
still a minor, in which case, he will continue to be a Philippine citizen even after becoming of age.
"A child born outside of the Philippines after the naturalization of his parent, shall be considered a Philippine citizen, unless
within one year after reaching the age of minority, he fails to register himself as a Philippine citizen at the American Consulate
of the country where he resides, and to take the necessary oath of allegiance."
It is obvious that the main subject-matter and purpose of the statute, the Revised Naturalization Law or Commonwealth Act 473, as a whole is to
establish a complete procedure for the judicial conferment of the of the status of citizenship upon qualified aliens. After having out such a procedure,
remarkable for its elaborate and careful inclusion of all safeguards against the possibility of any undesirable persons becoming a part of our citizenry,
it carefully but categorically states the consequence of the naturalization of an alien undergoing such procedure it prescribes upon members of his
immediate family, his wife and children, 25 and, to that end, in no uncertain terms it ordains that: (a) all his minor children who have been born in
the Philippines shall be "considered citizens" also; (b) all such minor children, if born outside the Philippines but dwelling here at the time of such
naturalization "shall automatically become" Filipinos also, but those not born in the Philippines and not in the Philippines at the time of such
naturalization, are also "deemed citizens" of this country provided that they shall lose said status if they transfer their permanent residence to a
foreign country before becoming of age; (c) all such minor children, if born outside of the Philippines after such naturalization, shall also be
"considered" Filipino citizens, unless they expatriate themselves by failing to register as Filipinos at the Philippine (American) Consulate of the
country where they reside and take the necessary oath of allegiance; and (d) as to the wife, she "shall be deemed a citizen of the Philippines" if she is
one "who might herself be lawfully naturalized". 26

No doubt whatever is entertained, so Burca holds very correctly, as to the point that the minor children, failing within the conditions of place and
time of birth and residence prescribed in the provision, are vested with Philippines citizenship directly by legislative fiat or by force of the law itself
and without the need for any judicial proceeding or declaration. (At p. 192 SCRA). Indeed, the language of the provision is not susceptible of any
other interpretation. But it is claimed that the same expression "shall be deemed a citizen of the Philippines" in reference to the wife, does not
necessarily connote the vesting of citizenship status upon her by legislative fiat because the antecedent phrase requiring that she must be one "who
might herself be lawfully naturalized" implies that such status is intended to attach only after she has undergone the whole process of judicial
naturalization required of any person desiring to become a Filipino. Stated otherwise, the ruling in Burca is that while Section 15 envisages and
intends legislative naturalization as to the minor children, the same section deliberately treats the wife differently and leaves her out for ordinary
judicial naturalization.
Of course, it goes without saying that it is perfectly within the constitutional authority of the Congress of the Philippines to confer or vest citizenship
status by legislative fiat. (U.S. v. Wong Kim Ark, 169 U.S. 649, 42 L ed. 890 [1898]; See, 1 Tañada & Carreon, Political Law of the Philippines 152 [1961
ed.] ) In fact, it has done so for particular individuals, like two foreign religious prelates, 27 hence there is no reason it cannot do it for classes or
groups of persons under general conditions applicable to all of the members of such class or group, like women who marry Filipinos, whether native-
born or naturalized. The issue before Us in this case is whether or not the legislature has done so in the disputed provisions of Section 15 of the
Naturalization Law. And Dr. Vicente G. Sinco, one of the most respected authorities on political law in the Philippines 28 observes in this connection
thus: " A special form of naturalization is often observed by some states with respect to women. Thus in the Philippines a foreign woman married to
a Filipino citizen becomesipso facto naturalized, if she belongs to any of the classes who may apply for naturalization under the Philippine Laws"
(Sinco, Phil. Political Law 498-499 [10th ed. 1954]; emphasis ours; this comment is substantially reiterated in the 1962 edition, citing Ly Giok Ha and
Ricardo Cua , supra.)
More importantly, it may be stated at this juncture, that in construing the provision of the United States statutes from which our law has been
copies, 28a the American citizenship by choice but by operation of law. "In the Revised Statutes the words 'and taken' are omitted. The effect of this
statute is that every alien woman who marries a citizen of the United States becomes perforce a citizen herself, without the formality of
naturalization, and regardless of her wish in that respect." (USCA 8, p. 601 [1970 ed.], citing Mackenzie v. Hare, 1913, 134 P. 713, 165 Cal. 766,
affirmed 36 S. Ct. 106, 239 U.S. 299, 60 L ed. 297.)
We need not recount here again how this provision in question was first enacted as paragraph (a) of Section 13, by way of an insertion into Act 2927
by Act 3448 of November 30, 1928, and that , in turn, and paragraph was copied verbatim from Section 1994 of the Revised Statutes of the United
States, which by that time already had a long accepted construction among the courts and administrative authorities in that country holding that
under such provision an alien woman who married a citizen became, upon such marriage, likewise a citizen by force of law and as a consequence of
the marriage itself without having to undergo any naturalization proceedings, provided that it could be shown that at the time of such marriage, she
was not disqualified to be naturalized under the laws then in force. To repeat the discussion We already made of these undeniable facts would
unnecessarily make this decision doubly extensive. The only point which might be reiterated for emphasis at this juncture is that whereas in the
United States, the American Congress, recognizing the uniform construction of Section 1994 of the Revised Statutes to be as stated above, and
finding it desirable to avoid the effects of such construction, approved the Act of September 22, 1922 explicitly requiring all such alien wives to
submit to judicial naturalization, albeit under more liberal terms than those for other applicants for citizenship, on the other hand, the Philippines
Legislature, instead of following suit and adopting a requirement, enacted Act 3448 on November 30, 1928 which copied verbatim the
aforementioned Section 1994 of the Revised Statutes, thereby indicating its preferences to adopts the latter law and its settled constitution rather
than the reform introduced by the Act of 1992.
Obviously, these considerations leave Us no choice. Much as this Court may feel that as the United States herself has evidently found it to be an
improvement of her national policy vis-a-vis the alien wives of her citizens to discontinue their automatic incorporation into the body of her citizenry
without passing through the judicial scrutiny of a naturalization proceeding, as it used to be before 1922, it seems but proper, without evidencing any
bit of colonial mentality, that as a developing country, the Philippines adopt a similar policy, unfortunately, the manner in which our own legislature
has enacted our laws on the subject, as recounted above, provides no basis for Us to construe said law along the line of the 1922 modification of the
American Law. For Us to do so would be to indulge in judicial legislation which it is not constitutionally permissible for this Court to do. Worse, this
Court would be going precisely against the grain of the implicit Legislative intent.
There is at least one decision of this Court before Burca wherein it seems it is quite clearly implied that this Court is of the view that under Section 16
of the Naturalization Law, the widow and children of an applicant for naturalization who dies during the proceeding do not have to submit
themselves to another naturalization proceeding in order to avail of the benefits of the proceedings involving the husband. Section 16 provides:
"SEC. 16. Right of widow and children of petitioners who have died. — In case a petitioner should die before the final decision
has been rendered, his widow and minor children may continue the proceedings. The decision rendered in the case shall, so far
same legal effect as if it had been rendered during the life of the petitioner."
In Tan Lin v. Republic, G.R. No. L-13706, May 31, 1961, 2 SCRA 383 this Court held:
"Invoking the above provisions in their favor, petitioners-appellants argue (1) that under said Sec. 16, the widow and minor
children are allowed to continue the same proceedings and are not substituted for the original petitioner; (2) that the
qualifications of the original petitioner remain to be in issue and not those of the widow and minor children, and (3) that said
Section 16 applies whether the petitioner dies before or after final decision is rendered, but before the judgment becomes
executory.
"There is force in the first and second arguments. Even the second sentence of said Section 16 contemplates the fact that the
qualifications of the original petitioner remains the subject of inquiry, for the simple reason that it states that "The decision
rendered in the case shall, so far as the widow and minor children are concerned, produce the same legal effect as if it had
been rendered during the life of the petitioner.' This phraseology emphasizes the intent of the law to continue the proceedings
with the deceased as the theoretical petitioner, for if it were otherwise, it would have been unnecessary to consider the
decision rendered, as far as it effected the widow and the minor children.
xxx xxx xxx
"The Chua Chian case (supra), cited by the appellee, declared that a dead person can not be bound to do things stipulated in
the oath of allegiance, because an oath is a personal matter. Therein, the widow prayed that she be allowed to take the oath of
allegiance for the deceased. IN the case at bar, petitioner Tan Lin merely asked that she be allowed to take the oath of
allegiance and the proper certificate of naturalization, once the naturalization proceedings of her deceased husband, shall have
been completed, not on behalf of the deceased, but on her own behalf and of her children, as recipients of the benefits of his
naturalization. In other words, the herein petitioner proposed to take the oath of allegiance, as a citizen of the Philippines, by
virtue of the legal provision that 'any woman who is now or may hereafter be married to a citizen of the Philippines and who
might be lawfully naturalized shall be deemed a citizen of the Philippines. Minor children of persons naturalized under this law
who have been born in the Philippines shall be considered citizens thereof.' (Section 15, Commonwealth Act No. 473). The
decision granting citizenship to Lee Pa and the record of the case at bar, do not show that the petitioning widow could not
have been lawfully naturalized, at the time Lee Pa filed his petition, apart from the fact that his 9 minor children were all born
in the Philippines. (Decision, In the Matter of the P)etition of Lee Pa to be admitted a citizen of the Philippines, Civil Case No.
16287, CFI, Manila, Annex A; Record on Appeal, pp. 8-11). The reference for Chua case is, therefore, premature."
Section 16, as may be seen, is a parallel provision to Section 15. If the widow of an applicant for naturalization as Filipino, who dies during the
proceedings, is not required to go through a naturalization proceedings, in order to be considered as a Filipino citizen hereof, it should follow that the
wife of a living Filipino cannot be denied the same privilege. This is plain common sense and there is absolutely no evidence that the Legislature
intended to treat them differently.

Additionally, We have carefully considered the arguments advanced in the motion for reconsideration in Burca, and We see no reason to disagree
with the following views of counsel:
"It is obvious that the provision itself is a legislative declaration of who may be considered citizens of the Philippines. It is a
proposition too plain to be disputed that Congress has the power not only to prescribe the mode or manner under which
foreigners may acquire citizenship, but also the very power of conferring citizenship by legislative fiat. (U.S. v. Wong Kim Ark,
169 U.S. 649, 42 L. Ed. 890 [1898]; see 1 Tañada and Carreon, Political Law of the Philippine citizens ed.]). The constitutional
itself recognizes as Philippines citizens 'Those who are naturalized in accordance with law' (Section 1[5], Article IV, Philippine
Constitution). Citizens by naturalization, under this provision, include not only those who are naturalized in accordance with
legal proceedings for the acquisition of citizenship, but also those who acquire citizenship by 'derivative naturalization' or by
operation of law, as. for example, the 'naturalization' of an alien wife through the naturalization of her husband, or by marriage
of an alien woman to a citizen. (See Tañada & Carreon, op. cit supra, at 152 172; Velayo, Philippine Citizenship and
Naturalization 2 [1965 ed.]: 1 Paras, Civil code 186 [1967 ed.]; see also 3 Hackworth, Digest of International Law 3).
"The phrase 'shall be deemed a citizen of the Philippines found in Section 14 of the Revised Naturalization Law clearly
manifests an intent to confer citizenship. Construing a similar phrase found in the old U.S. naturalization law (Revised Statutes,
1994) , American courts have uniformly taken it to mean that upon her marriage, the alien woman becomes by operation of
law a citizen of the United States as fully as if she had complied with all the provisions of the statutes upon the subject of
naturalization. (U.S. v. Keller, 13 F. 82; U.S. Opinions of the US Attorney General dated June 4, 1874 [14 Op. 402], July 20, 1909
[27 Op. 507], December 1, 1910 [28 Op. 508], Jan. 15, 1920 [32 Op. 209] and Jan. 12, 1923 [23 398] ).
'The phrase "shall be deemed a citizen, " in Section 1994 Revised Statute (U.S. Comp. Stat. 1091 1268) or as it was in
the Act of 1855 910 Stat. at L. 604, Chapt. 71, Sec. 2), "shall be deemed and taken to be a citizen", while it may imply
that the person to whom it relates has not actually become a citizen by the ordinary means or in the usual way, as by
the judgment of a competent court, upon a proper application and proof, yet it does not follow that such person is
on that account practically any the less a citizen. The word "deemed" is the equivalent of "considered" or "judged,"
and therefore, whatever an Act of Congress requires to be "deemed" or "taken" as true of any person or thing must,
in law, be considered as having been duly adjudged or established concerning such person or thing, and have force
and effect accordingly. When, therefore, Congress declares that an alien woman shall, under certain circumstances,
be "deemed" an American to her being naturalized directly by an Act of Congress or in the usual mode thereby
prescribed.' (Van Dyne, Citizenship of the United States 239, cited in Velayo, Philippine Citizenship and Naturalization
146-147 [1965 ed.] ; italics ours).
"That this was likewise the intent of the Philippine legislature when it enacted the first paragraph of Section 15 of the Revised
Naturalization provision. In its entirely, Section 15 reads:
(See supra)
The phrases 'shall be deemed,' shall be considered,' and 'shall automatically become,' as used in the above provision , are
undoubtedly synonymous. The leading idea or purpose of the provision was to confer Philippine citizenship by operation of law
upon certain classes of aliens as a legal consequence of their relationship, by blood affinity, to persons who are already citizens
of the Philippines. Whenever the fact of relationship of the persons enumerated in the provision concurs related, the effect is
for said persons to becomeipso facto citizens of the Philippines. 'Ipso facto' as here used does not mean that all alien wives and
all minor children of Philippine citizens, from the mere fact of relationship, necessarily become such citizens also. Those who
do not meet the statutory requirements do not ipso facto become citizens; they must apply for naturalization in order to
acquire such status. What it does mean, however, is that in respect of those persons numerated in Section 15, the relationship
to a citizen of the Philippines is the operative fact which establishes the acquisition of Philippine citizenship by them.
Necessarily, it also determines the points of time at which such citizenship commences. Thus, under the second paragraph of
Section 15, a minor child of a Filipino naturalized under the law, who was born in the Philippines, becomes ipso facto a citizen
of the Philippines from the time the fact of relationship concurs with the fact of a citizenship of his parent, and the time when
child became a citizen does not depend upon the time that he is able to prove that he was born in the Philippines. The child
may prove some 25 years after the naturalization of his father that he was born in the Philippines and should, therefore, be
'considered' a citizen thereof. It does not mean that he became a Philippine citizen only at that later time. Similarly, an alien
woman who married a Philippine citizen may be able to prove only some 25 years after her marriage (perhaps, because it was
only 25 years after the marriage that her citizenship status became in question), that she is one who 'might herself be lawfully
naturalized.' It is not reasonable to conclude that she acquired Philippine citizenship only after she had proven that she 'might
herself be lawfully naturalized.'
"The point that bears emphasis in this regard is that in adopting the very phraseology of the law , the legislature could not have
intended that an alien wife should not be deemed a Philippine citizen unless and until she proves that she might herself be
lawfully naturalized' is not a condition precedent to the vesting or acquisition of citizenship; it is only a condition or a state of
fact necessary to establish her citizenship as a factum probandum i.e., as a fact established and proved in evidence. The word
'might,' as used in that phrase, precisely implies that at the time of her marriage to a Philippine citizen, the alien woman 'had
(the) power' to become such a citizen herself under the laws then in force. (Owen v. Kelly, 6 DC 191 [1867], aff'd Kelly v. Owen,
power long after her marriage does not alter the fact that at her marriage, she became a citizen.
"(This Court has held) that 'an alien wife of a Filipino citizen may not acquire the status of a citizen of the Philippines unless
there is proof that she herself may be lawfully naturalized' (Decision, pp. 3-4). Under this view, the acquisition' of citizenship by
the alien wife depends on her having proven her qualifications for citizenship, that is, she is not a lawfully naturalized. It is clear
from the words of the law that the proviso does not mean that she must first prove that deemed (by Congress, not by the
courts) a citizen. Even the 'uniform' decisions cited by this Court (at fn. 2) to support its holding did not rule that the alien wife
becomes a citizen only after she has proven her qualifications for citizenship. What those decisions ruled was that the alien
wives in those cases failed to prove their qualifications and therefore they failed to establish their claim to citizenship. Thus in
Ly Giok Ha v. Galang, 101 Phil. 459 [1957], the case was remanded to the lower court for determination of whether petitioner,
whose claim to citizenship by marriage to a Filipino was disputed by the Government, 'might herself be lawfully naturalized,'
for the purpose of 'proving her alleged change of political status from alien to citizen' (at 464). In Cua v. Board, 101 Phil. 521
[1957], the alien wife who was being deported, claimed she was a Philippine citizen by marriage to a Filipino. This Court finding
that there was no proof that she was not disqualified under Section 4 of the Revised Naturalization Law, ruled that: 'No such
evidence appearing on record, the claim of assumption of Philippine citizenship by Tijoe Wu Suan, upon her marriage to
petitioner, is untenable.' (at 523) It will be observed that in these decisions cited by this Court, the lack of proof that the alien
wives 'might (themselves) be lawfully naturalized' did not necessarily imply that they did not become, in truth and in fact,
citizens upon their marriage to Filipinos. What the decisions merely held was that these wives failed to establish their claim to
that status as a proven fact.
"In all instances where citizenship is conferred by operation of law, the time when citizenship is conferred should not be
confused with the time when citizenship status is established as a proven fact. Thus, even a natural-born citizen of the
Philippines, whose citizenship status is put in issue in any proceeding would be required to prove, for instance, that his father is
a citizen of the Philippines in order to factually establish his claim to citizenship. *** His citizenship status commences from the
time of birth, although his claim thereto is established as a fact only at a subsequent time. Likewise, an alien woman who might
herself be a lawfully naturalized becomes a Philippine citizen at the time of her marriage to a Filipino husband, not at the time
she is able to establish that status as a proven fact by showing that she might herself be lawfully naturalized. Indeed, there is
no difference between a statutory declaration that a person is deemed a citizen of the Philippines provided his father is such
citizen from a declaration that an alien woman married to a Filipino citizen of the Philippines provided his father is such citizen
from a declaration that an alien woman married to a Filipino citizen of the Philippines provided she might herself be lawfully
naturalized. Both become citizens by operation of law; the former becomes a citizen ipso facto upon birth; the later ipso facto
upon marriage.

"It is true that unless and until the alien wife proves that she might herself be lawfully naturalized, it cannot be said that she
has established her status as a proven fact. But neither can it be said that on account, she did not become a citizen of the
Philippines. If her citizenship status is not questioned in any legal proceeding, she obviously has no obligation to establish her
status as a fact. In such a case, the presumption of law should be what she claims to be. (U.S. v. Roxas, 5 Phil. 375 [1905] :
Hilado v. Assad, 51 O.G. 4527 [1955] ). There is a presumption that a representation shown to have been made is true. (Aetna
Indemnity Co. v. George A. Fuller, Co., 73 A. 738 A. 369, 111 ME. 321)."
The question that keeps bouncing back as a consequence of the foregoing views is, what substitute is there for naturalization proceeding to enable
the alien wife of a Philippine citizen to have the matter of her own citizenship settled and established so that she may not have to be called upon to
prove it everytime she has to perform an act or enter into a transaction or business or exercise a right reserved only to Filipinos? The ready answer to
such question is that as the laws of our country, both substantive and procedural, stand today, there is no such procedure, but such is no proof that
the citizenship under discussion is not vested as of the date of marriage or the husband's acquisition of citizenship, as the case may be, for the truth
is that the situation obtains even as to native-born Filipinos. Everytime the citizenship of a person is material or indispensible in a judicial or
administrative case, Whatever the corresponding court or administrative authority decides therein as to such citizenship is generally not considered
as res adjudicata, hence it has to be threshed out again and again as the occasion may demand. This, as We view it, is the sense in which Justice
Dizon referred to "appropriate proceeding" in Brito v. Commissioner, supra. Indeed, only the good sense and judgment of those subsequently
inquiring into the matter may make the effort easier or simpler for the persons concerned by relying somehow on the antecedent official findings,
even if these are not really binding.
It may not be amiss to suggest, however, that in order to have a good starting point and so that the most immediate relevant public records may be
kept in order the following observations in Opinion No. 38, series of 1958, of then Acting Secretary of Justice Jesus G. Barrera, may be considered as
the most appropriate initial step by the interested parties.
"Regarding the steps that should be taken by an alien woman married to a Filipino citizen in order to acquire Philippine
citizenship, the procedure followed in the Bureau of Immigration is as follows: The alien woman must file a petition for the
cancellation of her alien certificate of registration alleging, among other things, that she is married to a Filipino citizen and that
she is not disqualified from acquiring her husband's citizenship pursuant to section 4 of Commonwealth Act No. 473, as
amended. Upon the filing of said petition, which should be accompanied or supported by the joint affidavit of the petitioner
and her Filipino husband to the effect that the petitioner does not belong to any of the groups disqualified by the cited section
from becoming naturalized Filipino citizen (please see attached CEB Form 1), the Bureau of Immigration conducts an
investigation and thereafter promulgates its order or decision granting or denying the petition."
Once the Commissioner of Immigration cancels the subject's registration as an alien, there will probably be less difficulty in establishing her
Filipino citizenship in any other proceeding, depending naturally on the substance and vigor of the opposition.
Before closing, it is perhaps best to clarify that this third issue We have passed upon was not touched by the trial court, but as the point is decisive in
this case, the Court prefers that the matter be settled once and for all now.
IN VIEW OF ALL THE FOREGOING, the judgment of the Court a quo dismissing appellants' petition for injunction is hereby reversed and the
Commissioner of Immigration and/or his authorized representative is permanently enjoined from causing the arrest and deportation and the
confiscation of the bond of appellant Lau Yuen Yeung, who is hereby declared to have become a Filipino citizen from and by virtue of her marriage to
her co-appellant Moy Ya Lim Yao al as Edilberto Aguinaldo Lim, a Filipino citizen of January 25, 1962. No costs.
Dizon, Castro, Teehankee and Villamor, JJ ., concur.
Makalintal J ., reserves his separate concurring opinion.
Fernando, J ., concurs except as the interpretation accorded some American decisions as to which he is not fully persuaded.
APPENDIX
The following review of all naturalization statutes of the United States from 1790 to 1970 ravel: (1) that aside from race, various other
disqualifications have also been provided for in the said statutes from time to time, although it was only in 1906 that the familiar and usual grounds
of disqualification, like not being anarchists, polygamists, etc. were incorporated therein, and (2) that qualifications of applicants for naturalization
also varied from time to time.
A — DISQUALIFICATIONS
1. In the first naturalization statute of March 26, 1790, only a "free white person" could be naturalized, provided he was not "proscribed" by any
state, unless it be with the consent of such state. (Chap. V. 1 Stat. 103)
2. In the Act of January 29, 1795, to the same provisions was added the disqualification of those "legally convicted of having joined the army of Great
Britain, during the late war." (Chap. XX, 1 Stat. 414).
3. In the Act of June 18, 1798, Section 1 thereto provided:
"SECTION 1. Be it enacted by the Senate and House of Representatives of the United States of America in Congress
assembled, That no alien shall be admitted to become a citizen of the United States, or of any state, unless in the
manner prescribed by the act, entitled 'An Act to establish an uniform rule of naturalization; and to repeal the act heretofore
passed on that subject, 'he shall have declared his intention to become a citizen of the United States, five years, at least, before
his admission, and shall, at the time of his application to be admitted, declare and prove, to the satisfaction of the court having
jurisdiction in the case, that he has resided within the United States fourteen years, at least, and within the state or territory
where, or for which such court is at the time held five years, at least, besides conforming to the other declarations,
renunciations and proofs, by the said act required, any thing therein to the contrary hereof notwithstanding: Provided, that
any alien, who was residing within the limelights, and under the jurisdiction of the United States, before the twenty-ninth day
of January, one thousand seven hundred and ninety-five, may, within one year after the passing of this act—and any alien who
shall have made the declaration of his intention to become a citizen of the United States, in conformity to the provisions of the
act, entitled 'An act to establish an uniform rule of naturalization, and to repeal the act heretofore passed on that subject,'
may, within four years after having made the declaration aforesaid, be admitted to become a citizen, in the manner prescribed
by the said act, upon his making proof that he has resided five years, at least, within the limits, and under the jurisdiction of the
United States: And provided also, that no alien, who shall be a native, citizen, denizen or subject of any nation or state with
whom the United States shall be at war, at the time of his application, shall be then admitted to become a citizen of the United
States."
There is here no mention of "white persons." (Chap. LIV, 1 Stat. 566).
4. In the Act of April 14, 1802, mentioned in Kelly v. Owen, supra, reference was made again to "free white persons," and the same enemy alien and
"state-proscribed" disqualifications in the former statutes were carried over. (Chap. XXVIII, 2 Stat. 153.)
5. The Act of March 26, 1804 provided in its Section 1 thus:
"Be it enacted by the Senate and House of Representatives of the United States of America in Congress assembled, That any
alien, being a free white person, who was residing within the limits and under the jurisdiction of the United States, at any time
between the eighteenth day of June, one thousand seven hundred and ninety-eight, and the fourteenth day of April one
thousand eight hundred and two, and who has continued to reside within the same, may be admitted to become a citizen of
the United States, without a compliance with the first condition specified in the first section of the act, entitled 'An act to
establish an uniform rule of naturalization, and to repeal the acts heretofore passed on that subject.' "
In its Section 2, this Act already provided that:
"SEC. 2. And be it further enacted, That when any alien who shall have complied with the first condition specified in ,the first
section of the said original act, and who shall have pursued the directions prescribed in the second section of the said act, may
die, before he is actually naturalized, the widow and the children of such alien shall be considered as citizens of the United
States, and shall be entitled to rights and privileges as such, upon taking the oaths prescribed by law." (CHAP. XLVII, 2 Stat. 292)
6. In the Act of July 30, 1813, the disqualification of enemy aliens was removed as follows:
"CHAP. XXXVI. — An Act supplementary to the acts heretofore passed on the subject of an uniform rule of
naturalization. (a)
"Be it enacted by the Senate and House of Representatives of the United States of America in Congress assembled, That persons
resident within the United States, or the territories thereof, on the eighteenth day of June, in the year one thousand eight
hundred and twelve, who had before that day made declaration according to law, of their intention to become citizens of the
United States, or who by the existing laws of the United States, were on that day entitled to become citizens, without making
such declaration, may be admitted to become citizens thereof, notwithstanding they shall be alien enemies at the times and in
the manner prescribed by the laws heretofore passed on that subject: Provided, That nothing herein contained shall be taken
or construed to interfere with or prevent the apprehension and removal, agreeably to law, of any alien enemy at any time
previous to the actual naturalization of such alien." (Chap. XXXVI, 3 Stat. 53)

7. Neither the Act of March 22, 1816 nor those of May 26, 1824 and May 24, 1828 made any change in the above requirements. (Chap. XXXII, 3 Stat.
258; Chap. CLXX-XVI, 4 Stat. 69; and Chap. CXVI, 4 Stat. 310).
8. Then the Act of February 10, 1855, important because it gave alien wives of citizens ,the status of citizens, was enacted providing:
"CHAP. LXXI. — An Act to secure the Right of Citizenship to Children of Citizens of the United States born out of the Limits
thereof.
"Be it enacted by the Senate and House of Representatives of the United States of America in Congress assembled, That persons
heretofore born, or hereafter to be born, out of the limits and jurisdiction of the United States, whose fathers were or shall be
at the time of their birth citizens of the United States, shall be deemed and considered and are hereby declared to be citizens
of the United States: Provided, however, That the rights of citizenship shall not descend to persons whose fathers never resided
in the United States.
"SEC. 2. And be it further enacted, That any woman who might lawfully be naturalized under the existing laws, married, or who
shall be married to a citizen of the United States, shall loyal be deemed and taken to be a citizen." (Chap. LXXI, 10 Stat. 604.)
9. The Act of July 14, 1870 mainly provided only for penalties for certain acts related to naturalization, as punished thereby, but added in its Section 7
"that the naturalization laws are hereby extended to aliens of African nativity and to African descent." (Chap. CCLIV, 16 Stat. 254.)
10. The Act of February 1, 1876 contained no relevant amendment. (Chap. 5, 19 Stat. 2.)
11. When the statutes of the United States were revised on June 22, 1874, the naturalization law of the country was embodied in Sections 2165-2174
of saddle Revised Statutes. This contained no racial disqualification. In fact, it reenacted ;Section 2 of the Act of February 10, 1855 as its Section 1994
thereof, thus:
"SEC. 1994. Any person who is now or may hereafter be married to a citizen of the United States, and who might herself be
lawfully naturalized, shall be deemed a citizen." (18 Stat. 351.)
12. The Act of May 6, 1882 provided expressly that no State court or court of the United State shall admit Chinese to citizenship. (Chap. 126, Sec. 14,
22 Stat. , 61.)
13. The Act of August 9, 1888 extended the benefits of American citizenship to Indian woman married to Americans thus:
"CHAP. 818. — An Act in relation to marriage between white men and Indian women.
"Be it enacted, That no white man, not otherwise a member of any tribe of Indians, who may hereafter marry, an Indian
woman, member of any Indian tribe in the United States, or any of its Territories except the five civilized tribes in the Indian
Territory, shall by such marriage hereafter acquire any right to any tribal property, privilege, or interest whatever to which any
member of such tribe is entitled.
"SEC. 2. That every Indian woman, member of any such tribe of Indians, who may hereafter be married to any citizen of the
United States, is hereby declared to become by such marriage a citizen of the United States, with all the right, privileges, and
immunities of any such citizen, being a married woman:
"Provided, That nothing in this act contained shall impair or in any way affect the right or title of such married woman to any
tribal property or any interest therein.
"SEC. 2. That whenever the marriage of any white man with any Indian woman, a member of any such tribe of Indians, is
required or offered to be proved in any judicial ,proceeding, evidence of the admission of such fact by the party against whom
the proceeding is had, or evidence of general repute, or of cohabitation as married persons, or any other circumstantial or
presumptive evidence from which the fact may be inferred, shall be competent. (Aug. 9, 1888) " [25 Stat. 392, Suppl. 1.]
14. The Act of April 19, 1900 extended American citizenship to all citizens of the Republic of Hawaii on August 12, 1898 as well as the laws of the
United States to said Republic, including, of course, those on naturalization. (Chap. 339, Sec. 4, 31 Stat. 141.)
15. On June 29, 1906. "An Act to establish a Bureau of Immigration and Naturalization, and to provide a uniform rule for the naturalization of aliens
throughout the United States" was approved. No reference was made therein to "free white persons''; it merely provided in its Section 7 that:
"SEC. 7. That no person who disbelieve in or who is opposed to organized government, or who is a member of or affiliated with
any organization entertaining and teaching such disbelief in or opposition to organized government, or who advocates or
teaches the duty, necessity, or propriety of the unlawful assaulting or killing of any officer or officers, either of specific
individuals or of officers generally of the Government of the United States, or of any other organized government, because of
his or their official character, or who is a polygamist, shall be naturalized or be made a citizen of the United States." (36 Stat.
598)
Incidentally, the 6th paragraph of its Section 4 provided:
"Sixth. When any alien who has declared his intention to become a citizen of the United States dies before he is actually
naturalized the widow and minor children of such alien may, by complying with the other provisions of this Act, be naturalized
without making any declaration of intention." (36 Stat. 598)
16. By the Act of March 2, 1907, alien women who acquired American citizenship by marriage retained said citizenship, if she continued to reside in
the United States and did not renounce it, or, if she resided outside of the United States by registering with the U.S. Consul of her place of residence.
(CHAP. 2534, Sec. 4, 34 Stat. 1229.)
17. Since United States legislation treats naturalization and citizenship per se separately, Section 1994 of the Revised Statutes remained untouched.
In the Act of February 24, 1911 it was provided:
"Be it enacted by the Senate and House of Representatives of the United States of America in Congress assembled, That when
any alien, who has declared his intention to become a citizen of the United States, becomes insane before he is actually
naturalized, and his wife shall thereafter make a homestead entry under the land laws of the United States, she and their
minor children may, by complying with the other provisions of the naturalization laws be naturalized without making any
declaration of intention." (36 Stat. 929.)
18. The Act of August 11, 1916 merely validated entries filed in certain countries. (CHAP. 316, 39 Stat. 926.)
19. In the Act of May 9, 1918, the U.S. Congress amended the naturalization laws to make possible the admission of Filipino navy servicemen, and
understandably, because of the war then, it provided:
"Seventh. Any native-born Filipino of the age of twenty-one years and upward who has declared his intention to become a
citizen of the United States and who has enlisted or may hereafter enlist in the United States Navy or Marine Corps or the
Naval Auxiliary Service, and who, after service of not less than three years, may be honorably discharged therefrom, or who
may receive an ordinary discharge with recommendation for reenlistment; or any alien, or any Porto Rican not a citizen of the
United States, . . . ." (40 Stat. 542.)
20. On September 22, 1922, "An Act Relative to the Naturalization and citizenship of married women" was appareled repeating Section 1994 of the
Revised Statutes and otherwise adopting a different attitude as regards the citizenship and naturalization of married women thus:
"Be it enacted by the Senate cleaned House of Representatives of the United States of America in Congress assembled, That the
right of any woman to become a naturalized citizen of the United States shall not be denied or abridged because of her sex or
because she is a married woman.
"Sec. 2. That any woman who marries a citizen of the United States after the passage of this Act, or any woman whose husband
is naturalized after the passage of this Act, shall not become a citizen of the United States by reason of such marriage or
naturalization; but, if eligible to citizenship, she may be naturalized upon full and complete compliance with all the
requirements of the naturalization laws, with the following exceptions:
(a) No declaration of intention shall be required;
(b) In lieu of the five-year period of residence within the United States and the one-year period of residence within the State or
Territory where the naturalization court is held, she shall have resided continuously in the United States Hawaii, Alaska, or
Porto Rico for at least one year immediately preceding the filing of the petition.
"Sec. 3. That a woman citizen of the United States shall not cease to be a citizen of the United States by reason of her marriage
after the passage of this Act, unless she makes a formal renunciation of her citizenship before a court having jurisdiction over
naturalization of aliens; Provided, That any woman citizen who marries an alien ineligible to citizenship shall cease to be a
citizen of the United States. If at the termination of the marital status she is a citizen of the United States she shall retain her
citizenship regardless of her residence. If during the continuance of the marital status she resides continuously for two years in
a foreign State of which her husband is a citizen or subject, or for five years continuously outside the United States, she shall
thereafter be subject to the same presumption as is a naturalized citizen of the United States under the second paragraph of
section 2 of the Act entitled "An Act in reference to the expatriation of citizens and their protection abroad," approved March 2
1907. Nothing herein shall be construed to repeal or amend the provisions of Revised Statutes 1999 or of section 2 of the
Expatriation Act of 1907 with reference to expatriation.
"Sec. 4. That a woman who, before the passage of this Act, has lost her United States citizenship by reason of her marriage to
an alien eligible for citizenship, may be naturalized as provided by section 2 of this Act: Provided, That no certificate of arrival
shall be required to be filed with her petition if during the continuance of the marital status she; shall have resided within the
United States. After her naturalization she shall have the same citizenship status as if her marriage had taken place after the
passage of this Act.

"Sec. 5. That no woman whose husband is not eligible to citizenship shall be naturalized during the continuance of the marital
status.
"Sec. 6. That section 1994 of the Revised Statutes and section 4 of the Expatriation Act of 1907 are repealed. Such repeal shall
not terminate citizenship acquired or retained under either of such sections nor restore citizenship lost under section 4 of the
Expatriation Act of 1907.
"Sec. 7. That section 3 of the Expatriation Act of 1901 is repealed. Such repeal shall not restore citizenship lost under such
section nor terminate citizenship resumed under such section. A woman who has resumed under such section citizenship lost
by marriage shall, upon the passage of this Act, have for all purposes the same citizenship status as immediately preceding her
marriage." (Chap. 411, 42 Stat. 10211022.)
21. When "The Code of the Laws of to United States of America of a General and Permanent Character in Force on December 7, 1925" was approved,
the provisions, corresponding to the disqualifications for naturalization and the citizenship and naturalization of women embodied therein were:
"367. Naturalization of woman; sex or marriage not a bar. — The right of any woman to become a naturalized citizen of the
United States shall not be denied or abridged because of her sex or because she is a married woman. ( Sept. 22, 1922, c.411, 1,
42 Stat. 1021.)
"368. Same; women marrying citizens or persons becoming naturalized; procedure. — Any woman who marries a citizen of the
United States after September 22, 1922, or any woman whose husband is naturalized after that date, shall not become a
citizen of the United States by reason of such marriage or naturalization; but, if eligible to citizenship, she may be naturalized
upon full and complete compliance with the following exceptions:
(a) No declaration of intention shall be required;
(b) In lieu of the five-year period of residence within the United States and the one-year period of residence within the State or
Territory where the naturalization court is held, she shall have resided continuously in the United States, Hawaii, Alaska, or
Porto Rico for at least one year immediately preceding the filing of the petition. (Sept. 22, 1922, c. 411, § 2, 42 Stat. 1022.)
"369. Same; women who have lost citizenship by Marrying aliens eligible to citizenship; procedure. — A woman, who, before
September 22, 1922, has lost her United States citizenship by reason for her marriage to an alien eligible for citizenship, may
be naturalized as provided in the preceding section. No certificate of arrival shall be required to be filed with her petition if
during the continuance of the marital status she shall have resided within the United Stators. After her naturalization she shall
have the same citizenship status as if her marriage had taken place after September 22, 1922. (Swept. 22, 1922, c. 411, § 4, 42
Stat. 1022.)
"370. Same; Women married to persons ineligible to citizenship. — No woman whose husband is not eligible to citizenship shall
be naturalized during the continuance of the marital status. (Swept. 22, 1922, c. 411, 5, 42 Stat. 1022.)
"371. Same, wife of alien declaring becoming insane before naturalization; minor children. — When any alien, who has
declared his intention to become a citizen of the United States, becomes insane before he is actually naturalized, and his wife
shall thereafter make a homestead entry under the land laws of the United States, she and their minor children may, by
complying with the other provisions of the naturalization laws be naturalized without making any declaration of intention.
(Feb. 24, 1911, c. 151, 36 Stat. 929.)" (Chap. 9, 44 Stat. 156, 158.)
which, of course, must be read together with the provisions on inadmissibility of Chinese, anarchists, polygamists, non-English speaking persons,
etc. in Sections 363-365 of the same Code.
22. The Act of May 26, 1926 extended naturalization privileges to alien veterans of World War I, thus:
"Be it enacted by the Senate anal House of Representatives of the United States of America in Congress assembled, That (a) as
used in this Act, the term "alien veteran" means an individual, a member of the military or naval forces of the United States at
any time after April 5, 1917, and before November 12, 1918, who is now an alien not ineligible to citizenship; but does not
include (1) any individual at anytime during such period or thereafter separated from such forces under other than honorable
conditions, (2) any conscientious objector who performed no military duty whatever or refused to wear the uniform, or (3) any
alien at any time during such period or thereafter discharged from the military or naval forces on account of his alienage.
(b) Terms defined in the Immigration Act of 1924 shall, when used in this Act, have the meaning assigned to such terms in that
Act.
"Sec. 2. An alien veteran shall for the purposes of the Immigration Act of 1924 be considered as a non-quota immigrant, but
shall be subject to all the other provisions of that Act and of the immigration laws, except that —
(a) He shall not be subject to the head tax imposed by section 2 of the Immigration Act of 1917;
(b) He shall not be required to pay any fee under section 2 or section 7 of the Immigration Act of 1924;
(c) If otherwise admissible, he shall not be excluded under section 3 of the Immigration Act of 1917, unless excluded under the
provisions of that section relating to —
(1) Persons afflicted with a loathsome or dangerous contagious disease, except tuberculosis in any form;
(2) Polygamy;
(3) Prostitutes, procurers, or other like immoral persons;
(4) Contract laborers;
(5) Persons previously deported;
(6) Persons convicted of crime.
"Sec. 3. The unmarried child under eighteen years of age, the wife, or the husband, of an alien veteran shall, for the purposes
of the Immigration Act of 1924, be considered as a non quota immigrant when accompanying or following within six months to
join him, but shall be subject to all other provisions of that Act and of the immigration laws.
'Sec. 4. The foregoing provisions of this Act shall not apply to any alien unless the immigration visa is issued to him before the
expiration of one year after the enactment of this Act." (Chap. 398, 44 Stat. 654-655.)
23. The Act of June 21, 1930 authorized repatriation of certain veterans of World War I. (Chap. 559, 46 Stat. 791.)
24. On March 3, 1931, the Act of September 22, 1922 as amended as follows:
"Sec. 4.(a) Section 3 of the Act entitled "An Act relative to the naturalization and citizenship of married women," approved
September 22, 1922, as amended, is amended to read as follows:
'Sec. 3.(a) A woman citizen of the United States shall not cease to be a citizen of the United States by reason of her marriage
after this section, as amended, takes effect, unless she makes a formal renunciation of her citizenship before a court having
jurisdiction over naturalization of aliens.
'(b) Any woman who before this section, as amended takes effect, has lost her United States citizenship by residence abroad
after marriage to an alien or by marriage to an alien ineligible to citizenship may, if she has not acquired any other nationality
by affirmative act, be naturalized in the manner prescribed in section 4 of this Act, as amended. Any woman who was a citizen
of the United States at birth shall not be denied naturalization under section 4 on account of her race.
'(c) No woman shall be entitled to naturalization under section 4 of this Act, as amended, if her United States citizenship
originated solely by a reason of her marriage to a citizen of the United States or by reason of the acquisition of United States
citizenship by her husband.'
"(b) Section 5 of such Act of September 22, 1922, is repealed." (Chap. 442, 46 Stat. 1511-1512.)
25. The Act of May 25, 1932 contained the following somewhat pertinent provisions:
"Be it enacted by the Senate and House of Representatives of the United States of America in Congress assembled, That
(a) an alien veteran, as defined in section 1 of the Act of May 26, 1926 (ch. 398, 44 Stat. 654; title 8, sec. 241, U.S. C. Supp. 1), if
residing in the United States, be entitled at any time within two years after the enactment of this Act to naturalization upon
the same terms, conditions, and exemptions which would have been accorded to such alien if he had petitioned before the
armistice of the World War, except that (1) such alien shall be required to prove that immediately preceding the date of this
petition he has resided continuously within the United States for at least two years, in pursuance of a legal admission for
permanent residence, and that during all such period he has behaved as a person of good moral character; (2) if such
admission was subsequent to March 3, 1924, such alien shall file with his petition a certificate of arrival issued by the
Commissioner of Naturalization; (3) final action shall not be had upon the petition until at least ninety days have elapsed after
filing of such petition; and (4) such alien shall be required to appear and file his petition in person, and to take the prescribed
oath of allegiance in open court. Such residence and good moral character shall be proved either by the affidavits of two
credible witnesses who are citizens of the United States, or by depositions by two such witnesses made before a naturalization
examiner, for each place of residence.
"(b) All petitions for citizenship made outside the United States in accordance with the seventh subdivision of section 4 of the
Naturalization Act of June 29, 1906, as amended, upon which naturalization has not been heretofore granted, are hereby
declared to be invalid for all purposes.
"Sec. 2. (a) The seventh subdivision of section 4 of the Naturalization Act of June 29, 1906, as amended, is amended by striking
out 'the National Guard or Naval Militia of any State, Territory, or the District of Columbia, or the State Militia in Federal
Service.'

"(b) This section shall not be applied in the case of any individual whose petition for naturalization has been filed before the
enactment of this Act.
"Sec. 3. The last proviso in the first paragraph of the seventh subdivision of section 4 of such Act of June 29, 1906, as amended,
is amended by striking out the period at the end thereof and inserting in lieu thereof a semicolon and the following: 'except
that this proviso shall not apply in the case of service on American-owned vessels by an alien who has been lawfully admitted
to the United States for permanent residence.'
"Sec. 4. Section 32 of such Act of June 29, 1906, as amended, is amended by adding at the end thereof the following new
subdivisions:
'(c) If the name of any naturalized citizen has, subsequent to naturalization, been changed by order of a court of competent
jurisdiction, or by marriage, the citizen may, upon the payment to the commissioner of a fee of $10, make application
(accompanied by two photographs of the applicant) for a new certificate of citizenship in the new name of such citizen. If the
commissioner finds the name of the applicant to have been changed as claimed he shall issue to the applicant a new certificate
with one of such photographs of the applicant affixed thereto.
'(d) The Commissioner of Naturalization is authorized to make and issue, without fee, certifications of any part of the
naturalization records of any court, or of any certificate of citizenship, for use in complying with any statute, State or Federal,
or in any judicial proceeding. Any such certification shall be admitted in evidence equally with the original from which such
certification was made in any case in which the original thereof might be admissible as evidence. No such certification shall be
made by any clerk of court except upon order of the court.'
"Sec. 5. So much of subdivision (a) of section 33 of such Act of June 29, 1906, as amended, as read 'Upon obtaining a certificate
from the Secretary of Labor showing the date, place, and manner of arrival in the United States,' is hereby repealed.
"Sec. 6. Section 4 of the Act entitled 'An Act to supplement the naturalization laws, and for other purposes,' approved March 2,
1929, is amended by striking out the period at the end thereof and inserting in lieu thereof a semicolon and the following:
'except that no such certificate shall be required if the entry was on or before June 29, 1906.'
"Sec. 7. Despite the provisions of subdivision (a) of section 1 of the Act entitled 'An Act making it a felony with penalty for
certain aliens to enter the United States of America under certain conditions in violation of law,' approved March 4, 1929, as
amended, an alien, if otherwise admissible, shall not be excluded from admission to the United States under the provisions of
such subdivision after the expiration of one year after the date of deportation if, prior to his reembarkation at a place outside
of the United States, or prior to his application in foreign contiguous territory for admission to the United States, the Secretary
of Labor, in his discretion, shall have granted such alien permission to reapply for admission.
"Sec. 8. The compilation of the statistics to show races nationalities, and other information, authorized and directed to be
prepared by the Commissioner of Naturalization, shall be completed and published at the same time, as near as practicable, as
the Publication of the statistics of the 1930 census except that reports covering the census of 1910 shall be completed and
submitted not later than January 31, 1933, and reports covering the census of 1920 not later than December 31, 1938. Such
statistics shall show the records of registry made under the provisions of the Act entitled 'An Act to supplement the
naturalization laws, and for other purposes,' approved March 2, 1929. Payment for the equipment used in preparing such
compilation shall be made from appropriations for miscellaneous expenses of the Bureau of Naturalization.
"Sec. 9. The Secretary of the Treasury, upon the recommendation of the Secretary of Labor, is authorized to provide quarters
without payment of rent, in the building occupied by the Naturalization Service in New York City, for a photographic studio
operated by welfare organizations without profit and solely for the benefit of aliens seeking naturalization. Such studio shall be
under the supervision of the Commissioner of Naturalization.
"Sec. 10. The tenth subdivision of section 4 of the Act of June 29, 1906 (ch. 3592, 34 Stat. 598), as amended by the Act of May
9, 1918 (ch. 69, 40, 40 Stat. 545; U.S.C., title 8 sec. 377), is hereby amended to read as follows:
'Tenth. That any person not an alien enemy, who resided uninterruptedly within the United States during the period of five
years next preceding July 1, 1920, and was on that date otherwise qualified to become a citizen of the United States, except
that he had not made a declaration of intention required by law and who during or prior to that time, because of
misinformation regarding his citizenship status erroneously exercised the rights and performed the duties of a citizen of the
United States in good faith, may file the petition for naturalization prescribed by law without making the preliminary
declaration of intention required of other aliens, and upon satisfactory proof to the court that he has so acted may be
admitted as a citizen of the United States upon complying in all respects with the other requirements of the naturalization law.'
(Chap. 203, 47 Stat. 165-167.)
26. By June 27, 1952, the right of a person to be naturalized could no longer be denied by reason of race or sex or because such person was married,
although various disqualifications were still maintained, such as lack understanding, capacity to read and write English, or of the principles of the
constitution and form of government of the United States, being opposed to organized government of law, favoring totalitarian forms of
government, deserters from the armed forces, etc. (Secs. 1422 to 1426, USCA 8-9, 1953; See also Secs. 1421 et seq., USCA 8, 1970.)
B — QUALIFICATIONS
Apart from the above disqualifications, the statutes referred tea contained express requirements as to qualifications as follows:
(1) The Act of 1790 required residence, good moral character and adherence to the principles of the United States Constitution.
(2) That of 1795 required a declaration of intention. residence, adherence to the U.S. Constitution, good moral character and no title of nobility.
(3) That of 1798 referred only declaration of intention and residence.
(4) That of 1802 required residence, renunciation of allegiance to former government, adherence to U.S. Constitution, good moral character and
declaration of intention.
(5) That of 1804 was practically I the same as that of 1802.
(6) So also were those of 1813, 1816 and 1824.
(7) That of 1828 mentioned only residence and declaration of intention.
(8) Those of 1855, 1870 and 1888 amended the law in other respects.
(9) That of 1906 contained the following provisions:
"SEC. 4. That an alien may be admitted to become a citizen of the United States in the following manner and not otherwise:
"First. He A hall declare on oath before the clerk of any court authorized by this Act to naturalize aliens, or his authorized
deputy, in the district in which such alien resides, two years at least prior to his admission, and after he has reached the age of
eighteen years, that it is bona fide his intention to become a citizen of the united States, and to renounce forever all allegiance
and fidelity to any foreign prince, potentate, state, or sovereignty, and particularly, by name, to the prince, potentate, state, or
sovereignty of which the alien may be at the time a citizen or subject. And such declaration shall set forth, the name, age,
occupation, personal description, place of birth, last foreign residence and allegiance, the date of arrival, the, name of the
vessel, if any, in which he came to the United states, and the present place of residence in the United States of said
alien: Provided, however, That no alien who, in conformity with the law in force at the date of his declaration, has declared his
intention to become a citizen of the United States shall be required to renew such declaration.
"Second. Not less than two years nor more than seven years after he has made such declaration of intention he shall make and
file, in duplicate, a petition in writing, signed by the applicant in his own handwriting and duly verified, in which petition such
applicant shall state his full name, his place of residence (by street and number, if possible), his occupation, and, if possible, the
date and place of his birth; the place from which he emigrated, and the date and place of his arrival in the United States, and, if
he entered through a port, the name of the vessel on which he arrived; the time when and the place and name of the court
where he declared his intention to become a citizen of the United States; if he is married he shall state the name of his wife
and, if possible, the country of her nativity and her place of residence at the time of filing his petition; and if he has children,
the name, date, and place of birth and place of residence of each child living at the time of his petition: Provided, That if he has
filed his declaration before the passage of this Act he shall not be required to sign the petition in his own handwriting.
"The petition shall set forth that he is not a disbeliever in or opposed to organized government, or a member of or affiliated
with any organization or body of persons teaching disbelief in or opposed to organized government, a polygamist or believer in
the practice of polygamy, and that it is his intention to become a citizen of the United States and to renounce absolutely and
forever all allegiance and fidelity to any foreign prince, potentate, state, or sovereignty, and particularly by name to the prince,
potentate, state, or sovereignty of which he at the time of filing of his petition may be a citizen or subject, and that it is his
intention to reside permanently within the United States, and whether or not he has been denied admission a, a citizen of the
United States, and, if denied, the ground or grounds of such denial, the court or courts in which such decision was rendered,
and that the cause for such denial has since been cured or removed, and every fact material to his naturalization ailed required
to be proved upon the final hearing of his application.

"The petition shall also be verified by the affidavits of at least two credible witnesses, who are citizens of the United States, and
who shall state in their affidavits that they have personally known the applicant to be a resident of the United States for a
period of at least five years continuously, and of the State, Territory, or district in which the application is made for a period of
at least one year immediately preceding the date of the filing of his petition, and that they each have personal knowledge that
the petitioner is a person of good moral character, and that he is in every way qualified, in their opinion, to be admitted as a
citizen of the United States.
"At the time of filing of his petition there shall be filed with the clerk of the court a certificate from the Department of
Commerce and Labor, if the petitioner arrives in the United States after the passage of this Act, stating the date, place and
manner of his arrival in the United States, and the declaration of intention of such petitioner, which certificate and declaration
shall be attached to and made a part of said petition.
"Third. He shall, before he is admitted to citizenship, declare on oath in open court that he will support the Constitution of the
United States, and he absolutely and entirely renounces and abjures all allegiance and fidelity to any foreign prince potentate,
state, or sovereignty, and particularly by name to the prince, potentate, state, or sovereignty of which he was before a citizen
or subject; that he will support and defend the Constitution and laws of the United States against all enemies, foreign and
domestic, and bear true faith and allegiance to the same.
"Fourth. It shall be made to appear to the satisfaction of the court admitting any alien to citizenship that immediately
preceding the date of his application he has resided continuously within the United States five years at least, and within the
State or Territory where such court is at the time held one year at least, and that during that time he has behaved as a man of
good moral character, attached to the principles of the Constitution of the United States, and well disposed to the good order
and happiness of the same. In addition to the oath of the applicant, the testimony of at least two witnesses, citizens of the
United States, as to the facts of residence, moral character, and attachment to the principles of the Constitution shall be
required, and the name, ,place of residence, and occupation of each witness shall be set forth in the record.
"Fifth. In case the alien applying to be admitted to citizenship has borne any hereditary title, or has been of any of the orders of
nobility in the kingdom or state from which he came, he shall, in addition to the above requisites make an express renunciation
of his title or order of nobility in the court to which his application is made, and his renunciation shall be recorded in the court.
"Sixth. When any alien who has declared his intention to become a citizen of the United States dies before he is actually
naturalized the widow and minor children of such alien may, by complying with the other provisions of this Act, be naturalized
without making any declaration of intention." (34 stat. 596-98.)
10. Those of 1911 and 1916 contained amendments as to other matters.
11. That of 1918 provided for different qualifications for Filipinos, Porto Ricans, etc. for naturalization in addition to service in the U.S. Navy or
Philippine Constabulary.
12. Those of years after 1922 when Section 1994 was repealed would have no material bearing in this case.
Amen.
||| (Moy Ya Lim Yao v. Commissioner of Immigration, G.R. No. L-21289, [October 4, 1971], 148-B PHIL 773-877)
EN BANC
[G.R. No. 104654. June 6, 1994.]
REPUBLIC OF THE PHILIPPINES, petitioner, vs. HON. ROSALIO G. DE LA ROSA, PRESIDING JUDGE OF THE REGIONAL TRIAL
COURT, BRANCH 28, MANILA and JUAN G. FRIVALDO, respondents.
[G.R. No. 105715. June 6, 1994.]
RAUL R. LEE, petitioner, vs. COMMISSION ON ELECTIONS and JUAN G. FRIVALDO, respondents.
[G.R. No. 105735. June 6, 1994.]
RAUL R. LEE, petitioner, vs. COMMISSION ON ELECTIONS and JUAN G. FRIVALDO, respondents.
The Solicitor General for petitioner in G.R. No. 104654.
Yolando F. Lim counsel for private respondent.
SYLLABUS
1. CONSTITUTIONAL LAW; REVISED NATURALIZATION LAW; PROCEDURAL REQUIREMENTS, JURISDICTIONAL; CASE AT BAR. — Private respondent,
having opted to reacquire Philippine citizenship thru naturalization under the Revised Naturalization Law, is duty bound to follow the procedure
prescribed by the said law. It is not for an applicant to decide for himself and to select the requirement which he believes, even sincerely, are
applicable to his case and discard those which be believes are inconvenient or merely of nuisance value. The law does not distinguish between an
applicant who was formerly a Filipino citizen and one who was never such a citizen. It does not provide a special procedure for the reacquisition of
Philippine citizenship by former Filipino citizens akin to the repatriation of a woman who had lost her Philippine citizenship by reason of her marriage
to an alien. The trial court never acquired jurisdiction to hear the petition for naturalization of private respondent. The proceedings conducted, the
decision rendered and the oath of allegiance taken therein, are null and void or failure to comply with the publication and posting requirements
under the Revised Naturalization Law. Under Section 9 of the said law, both the petition for naturalization and the order setting it for hearing must
be published once a week for three consecutive weeks in the Official Gazette and a newspaper of general circulation. Compliance therewith is
jurisdictional (Po Yi Bo v. Republic, 205 SCRA 400 [1992]). Moreover, the publication and posting of the petition and the order must be in its full text
for the court to acquire jurisdiction (Sy v. Republic, 55 SCRA 724 [1974]). The petition for naturalization lacks several allegations required by Sections
2 and 6 of the Revised Naturalization Law, particularly: (1) that the petitioner is of good moral character; (2) that he resided continuously in the
Philippines for at least ten years; (3) that he is able to speak and write English and any one of the principal dialects; (4) that he will reside
continuously in the Philippines from the date of the filing of the petition until his admission to Philippine citizenship; and (5) that he has filed a
declaration of intention or if he is excused from said filing, the justification therefor. The absence of such allegations is fatal to the petition (Po Yi Bi v.
Republic, 205 SCRA 400 [1992]). Likewise the petition is not supported by the affidavit of at least two credible persons who vouched for the good
moral character of private respondent as required by Section 7 of the Revised Naturalization Law. Private respondent also failed to attach a copy of
his certificate of arrival to the petition as required by Section 7 of the said law. The proceedings of the trial court was marred by the following
irregularities: (1) the hearing of the petition was set ahead of the scheduled date of hearing, without a publication of the order advancing the date of
hearing, and the petition itself; (2) the petition was heard within six months from the last publication of the petition; (3) petitioner was allowed to
take his oath of allegiance before the finality of the judgment; and (4) petitioner took his oath of allegiance without observing the two-year waiting
period. Private respondent is declared NOT a citizen of the Philippines and therefore DISQUALIFIED from continuing to serve as GOVERNOR of the
Province of Sorsogon. He is ordered to VACATE his office and to SURRENDER the same to the Vice-Governor of the Province of Sorsogon once this
decision becomes final and executory.
2. ID.; ID.; DECISION THEREON BECOMES FINAL ONLY AFTER THIRTY (30) DAYS FROM PROMULGATION. — A decision in a petition for naturalization
becomes final only after 30 days from its promulgation and, insofar as the Solicitor General is concerned, that period is counted from the date of his
receipt of the copy of the decision (Republic v. Court of First Instance of Albay, 60 SCRA 195 [1974]).
3. ID.; ID.; DECISION GRANTING NATURALIZATION SHALL BE EXECUTORY AFTER TWO (2) FROM PROMULGATION; REASON. — Section 1 of R.A. No.
530 provides that no decision granting citizenship in naturalization proceedings shall be executory until after two years from its promulgation in
order to be able to observe if: (1) the applicant has left the country; (2) the applicant has dedicated himself continously to a lawful calling or
profession; (3) the applicant has not been convicted of any offense or violation of government promulgated rules; and (4) the applicant has
committed any act prejudicial to the interest of the country or contrary to government announced policies.
4. REMEDIAL LAW; SPECIAL CIVIL ACTION; QUO WARRANTO; PETITION NOT COVERED BY THE TEN (10) DAY PERIOD FOR APPEAL PRESCRIBED IN
SECTION 253 OF THE OMNIBUS ELECTION CODE. — In Frivaldo v. Commission on Elections, 174 SCRA 245 (1989), we held that a petition for quo
warranto, questioning the respondent's title and seeking to prevent him from holding office as Governor for alienage, is not covered by the ten-day
period for appeal prescribed in Section 253 of the Omnibus Election Code.
5. POLITICAL LAW; PUBLIC OFFICE; QUALIFICATIONS THEREON ARE CONTINUING REQUIREMENTS. — We explained that "qualifications for public
office are continuing requirements and must be possessed not only at the time of appointment or election or assumption of office but during the
officer's entire tenure; once any of the required qualification is lost, his title may be seasonably challenged."
6. ID.; ID.; ONLY FILIPINO CITIZENS CAN RUN AND BE ELECTED THERETO. — Petitioner's argument, that to unseat him will frustrate the will of the
electorate, is untenable. Both the Local Government Code and the Constitution require that only Filipino citizens can run and be elected to public
office. We can only surmise that the electorate, at the time they voted for private respondent, was of the mistaken belief that he had legally
reacquired Filipino citizenship.
7. ID.; ELECTIONS; WHERE THE CANDIDATE WHO OBTAINED THE HIGHEST NUMBER OF VOTES IS DISQUALIFIED, THE CANDIDATE WHO GARNERED
THE SECOND HIGHEST NUMBER OF VOTES IS NOT ENTITLED TO BE DECLARED WINNER. — Petitioner in G.R. No. 105715, prays that the votes cast in
favor of private respondent be considered stray and that he, being the candidate obtaining the second highest number of votes, be declared winner.
In Labo, Jr. v. COMELEC , 176 SCRA 1 (1989), we ruled that where the candidate who obtained the highest number of votes is later declared to be
disqualified to hold the office to which he was elected, the candidate who garnered the second highest number of votes is not entitled to be declared
winner (See also Geronimo v. Ramos, 136 SCRA 435 [1985]; Topacio v. Paredes, 23 Phil. 238 [1912]).
DECISION
QUIASON, J p:
In Frivaldo v. Commission on Elections, 174 SCRA 245 (1989), this Court declared private respondent, Juan G. Frivaldo, an alien and therefore
disqualified from serving as Governor of the Province of the Sorsogon.
Once more, the citizenship of private respondent is put in issue in these petitions docketed as G.R. No. 104654, G.R. No. 105715 and G.R. No.
105735. The petitions were consolidated since they principally involved the same issues and parties. LibLex
I
G.R. No. 104654
This is a petition for certiorari under Rule 45 of the Revised Rules of Court in relation to R.A. No. 5440 and Section 25 of the Interim Rules, filed by the
Republic of the Philippines: (1) to annul the Decision dated February 27, 1992 of the Regional Trial Court, Branch 28, Manila, in SP Proc. No. 91-
58645, which re-admitted private respondent as a Filipino citizen under the Revised Naturalization Law (C.A. No. 63 as amended by C.A. No. 473);
and (2) to nullify the oath of allegiance taken by private respondent on February 27, 1992.
On September 20, 1991, petitioner filed a petition for naturalization captioned: "In the Matter of Petition of Juan G. Frivaldo to be Re-admitted as a
Citizen of the Philippines under Commonwealth Act No. 63" (Rollo, pp. 17-23).
In an Order dated October 7, 1991 respondent Judge set the petition for hearing on March 16, 1992, and directed the publication of the said order
and petition in the Official Gazette and a newspaper of general circulation, for three consecutive weeks, the last publication of which should be at
least six months before the said date of hearing. The order further required the posting of a copy thereof and the petition in a conspicuous place in
the Office of the Clerk of Court of the Regional Trial Court, Manila (Rollo, pp. 24- 26).
On January 14, 1992, private respondent filed a "Motion to Set Hearing Ahead of Schedule," where he manifested his intention to run for public
office in the May 1992 elections. He alleged that the deadline for filing the certificate of candidacy was March 15, one day before the scheduled
hearing. He asked that the hearing set on March 16 be cancelled and be moved to January 24 (Rollo, pp. 27-28).
The Motion was granted in an Order dated January 24, 1992, wherein the hearing of the petition was moved to February 21, 1992. The said order
was not published nor a copy thereof posted. cdrep
On February 21, the hearing proceeded with private respondent as the sole witness. He submitted the following documentary evidence: (1) Affidavit
of Publication of the Order dated October 7, 1991 issued by the publisher of The Philippine Star (Exh. "A"); (2) Certificate of Publication of the order
issued by the National Printing Office (Exh. "B"); (3) Notice of Hearing of Petition (Exh. "B-1"); (4) Photocopy of a Citation issued by the National Press
Club with private respondent's picture (Exhs. "C" and "C-2"); (5) Certificate of Appreciation issued by the Rotary Club of Davao (Exh. "D"); (6)
Photocopy of a Plaque of Appreciation issued by the Republican College, Quezon City (Exh. "E"); (7) Photocopy of a Plaque of Appreciation issued by
the Davao-Bicol Association (Exh. "F"); (8) Certification issued by the Records Management and Archives Office that the record of birth of private
respondent was not on file (Exh. "G"); and (9) Certificate of Naturalization issued by the United States District Court (Exh. "H").

Six days later, on February 27, respondent Judge rendered the assailed Decision, disposing as follows:
"WHEREFORE, the petition is GRANTED, Petitioner JUAN G. FRIVALDO, is re-admitted as a citizen of the Republic of the
Philippines by naturalization, thereby vesting upon him, all the rights and privileges of a natural born Filipino citizen" (Rollo, p.
33).
On the same day, private respondent was allowed to take his oath of allegiance before respondent Judge (Rollo, p. 34).
On March 16, a "Motion for Leave of Court to Intervene and to Admit Motion for Reconsideration" was filed by Quiterio H. Hermo. He alleged that
the proceedings were tainted with jurisdictional defects, and prayed for a new trial to conform with the requirements of the Naturalization Law.
After receiving a copy of the Decision on March 18, 1992, the Solicitor General interposed a timely appeal directly with the Supreme Court.
G.R. No. 105715
This is a petition for certiorari, mandamus with injunction under Rule 65 of the Revised Rules of Court in relation to Section 5(2) of Article VIII of the
Constitution with prayer for temporary restraining order filed by Raul R. Lee against the Commission on Elections (COMELEC) and private respondent,
to annul the en banc Resolution of the COMELEC, which dismissed his petition docketed as SPC Case No. 92-273. The said petition sought to annul
the proclamation of private respondent as Governor-elect of the Province of Sorsogon.
Petitioner was the official candidate of the Laban ng Demokratikong Pilipino (LDP) for the position of governor of the Province of Sorsogon in the
May 1992 elections. Private respondent was the official candidate of the Lakas-National Union of Christian Democrats (Lakas-NUCD) for the same
position.
Private respondent was proclaimed winner on May 22, 1992.
On June 1, petitioner filed a petition with the COMELEC to annul the proclamation of private respondent as Governor-elect of the Province of
Sorsogon on the grounds: (1) that the proceedings and composition of the Provincial Board of Canvassers were not in accordance with law; (2) that
private respondent is an alien, whose grant of Philippine citizenship is being questioned by the State in G.R. No. 104654; and (3) that private
respondent is not a duly registered voter. Petitioner further prayed that the votes cast in favor of private respondent be considered as stray votes,
and that he, on the basis of the remaining valid votes cast, be proclaimed winner. llcd
On June 10, the COMELEC issued the questioned en banc resolution which dismissed the petition for having been filed out of time, citing Section 19
of R.A. No. 7166. Said section provides that the period to appeal a ruling of the board of canvassers on questions affecting its composition or
proceedings was three days.
In this petition, petitioner argues that the COMELEC acted with grave abuse of discretion when it ignored the fundamental issue of private
respondent's disqualification in the guise of technicality.
Petitioner claims that the inclusion of private respondent's name in the list of registered voters in Sta. Magdalena, Sorsogon was invalid because at
the time he registered as a voter in 1987, he was an American citizen.
Petitioner further claims that the grant of Filipino citizenship to private respondent is not yet conclusive because the case is still on appeal before us.
Petitioner prays for: (1) the annulment of private respondent's proclamation as Governor of the Province of Sorsogon; (2) the deletion of private
respondent's name from the list of candidates for the position of governor; (3) the proclamation of the governor-elect based on the remaining votes,
after the exclusion of the votes for private respondent; (4) the issuance of a temporary restraining order to enjoin private respondent from taking his
oath and assuming office; and (5) the issuance of a writ of mandamus to compel the COMELEC to resolve the pending disqualification case docketed
as SPA Case No. 92-016, against private respondent. LLphil
G.R. No. 105735
This is a petition for mandamus under Rule 65 of the Revised Rules of Court in relation to Section 5(2) of Article VIII of the Constitution, with prayer
for temporary restraining order. The parties herein are identical with the parties in G.R. No. 105715.
In substance, petitioner prays for the COMELEC's immediate resolution of SPA Case No. 92-016, which is a petition for the cancellation of private
respondent's certificate of candidacy filed on March 23, 1992 by Quiterio H. Hermo, the intervenor in G.R. No. 104654 (Rollo, p. 18).
The petition for cancellation alleged: (1) that private respondent is an American citizen, and therefore ineligible to run as candidate for the position
of governor of the Province of Sorsogon; (2) that the trial court's decision re-admitting private respondent as a Filipino citizen was fraught with legal
infirmities rendering it null and void; (3) that assuming the decision to be valid, private respondent's oath of allegiance, which was taken on the same
day the questioned decision was promulgated, violated Republic Act No. 530, which provides for a two-year waiting period before the oath of
allegiance can be taken by the applicant; and (4) that the hearing of the petition on February 27, 1992, was held less than four months from the date
of the last publication of the order and petition. The petition prayed for the cancellation of private respondent's certificate of candidacy and the
deletion of his name from the list of registered voters in Sta. Magdalena, Sorsogon.
In his answer to the petition for cancellation, private respondent denied the allegations therein and averred: (1) that Quiterio H. Hermo, not being a
candidate for the same office for which private respondent was aspiring, had no standing to file the petition; (2) that the decision re-admitting him to
Philippine citizenship was presumed to be valid; and (3) that no case had been filed to exclude his name as a registered voter. LLjur
Raul R. Lee intervened in the petition for cancellation of private respondent's certificate of candidacy (Rollo, p. 37).
On May 13, 1992, said intervenor urged the COMELEC to decide the petition for cancellation, citing Section 78 of the Omnibus Election Code, which
provides that all petitions on matters involving the cancellation of a certificate of candidacy must be decided "not later than fifteen days before
election," and the case of Alonto v. Commission on Elections, 22 SCRA 878 (1968), which ruled that all pre-proclamation controversies should be
summarily decided (Rollo, p. 50).
The COMELEC concedes that private respondent has not yet reacquired his Filipino citizenship because the decision granting him the same is not yet
final and executory (Rollo, p. 63). However, it submits that the issue of disqualification of a candidate is not among the grounds allowed in a pre-
proclamation controversy, like SPC Case No. 92-273. Moreover, the said petition was filed out of time.
The COMELEC contends that the preparation for the elections occupied much of its time, thus its failure to immediately resolve SPA Case No. 92-016.
It argues that under Section 5 of Rule 25 of the COMELEC Rules of Procedure, it is excused from deciding a disqualification case within the period
provided by law for reasons beyond its control. It also assumed that the same action was subsequently abandoned by petitioner when he filed before
it a petition for quo warranto docketed as EPC No. 92-35. The quo warrantoproceedings sought private respondent's disqualification because of his
American citizenship. LLjur
II
G.R. No. 104654
We shall first resolve the issue concerning private respondent's citizenship.
In his comment to the State's appeal of the decision granting him Philippine citizenship in G.R. No. 104654, private respondent alleges that the
precarious political atmosphere in the country during Martial Law compelled him to seek political asylum in the United States, and eventually to
renounce his Philippine citizenship.
He claims that his petition for naturalization was his only available remedy for his reacquisition of Philippine citizenship. He tried to reacquire his
Philippine citizenship through repatriation and direct act of Congress. However, he was later informed that repatriation proceedings were limited to
army deserters or Filipino women who had lost their citizenship by reason of their marriage to foreigners (Rollo, pp. 49-50). His request to Congress
for sponsorship of a bill allowing him to reacquire his Philippine citizenship failed to materialize, notwithstanding the endorsement of several
members of the House of Representatives in his favor (Rollo, p. 51). He attributed this to the maneuvers of his political rivals.
He also claims that the re-scheduling of the hearing of the petition to an earlier date, without publication, was made without objection from the
Office of the Solicitor General. He makes mention that on the date of the hearing, the court was jampacked. LLphil
It is private respondent's posture that there was substantial compliance with the law and that the public was well-informed of his petition for
naturalization due to the publicity given by the media.
Anent the issue of the mandatory two-year waiting period prior to the taking of the oath of allegiance, private respondent theorizes that
the rationale of the law imposing the waiting period is to grant the public an opportunity to investigate the background of the applicant and to
oppose the grant of Philippine citizenship if there is basis to do so. In his case, private respondent alleges that such requirement may be dispensed
with, claiming that his life, both private and public, was well-known. Private respondent cites his achievements as a freedom fighter and a former
Governor of the Province of Sorsogon for six terms.
The appeal of the Solicitor General in behalf of the Republic of the Philippines is meritorious. The naturalization proceedings in SP Proc. No. 91-58645
was full of procedural flaws, rendering the decision an anomaly. LLphil
Private respondent, having opted to reacquire Philippine citizenship thru naturalization under the Revised Naturalization Law, is duty bound to follow
the procedure prescribed by the said law. It is not for an applicant to decide for himself and to select the requirements which he believes, even
sincerely, are applicable to his case and discard those which be believes are inconvenient or merely of nuisance value. The law does not distinguish
between an applicant who was formerly a Filipino citizen and one who was never such a citizen. It does not provide a special procedure for the
reacquisition of Philippine citizenship by former Filipino citizens akin to the repatriation of a woman who had lost her Philippine citizenship by reason
of her marriage to an alien.
The trial court never acquired jurisdiction to hear the petition for naturalization of private respondent. The proceedings conducted, the decision
rendered and the oath of allegiance taken therein, are null and void for failure to comply with the publication and posting requirements under
the Revised Naturalization Law.
Under Section 9 of the said law, both the petition for naturalization and the order setting it for hearing must be published once a week for three
consecutive weeks in the Official Gazette and a newspaper of general circulation. Compliance therewith is jurisdictional (Po Yi Bo v. Republic, 205
SCRA 400 [1992]). Moreover, the publication and posting of the petition and the order must be in its full text for the court to acquire jurisdiction (Sy
v. Republic, 55 SCRA 724 [1974]).
The petition for naturalization lacks several allegations required by Sections 2 and 6 of the Revised Naturalization Law, particularly: (1) that the
petitioner is of good moral character; (2) that he resided continuously in the Philippines for at least ten years; (3) that he is able to speak and write
English and any one of the principal dialects; (4) that he will reside continuously in the Philippines from the date of the filing of the petition until his
admission to Philippine citizenship; and (5) that he has filed a declaration of intention or if he is excused from said filing, the justification therefor.
The absence of such allegations is fatal to the petition (Po Yi Bi v. Republic, 205 SCRA 400 [1992]).
Likewise the petition is not supported by the affidavit of at least two credible persons who vouched for the good moral character of private
respondent as required by Section 7 of the Revised Naturalization Law. Private respondent also failed to attach a copy of his certificate of arrival to
the petition as required by Section 7 of the said law. LLphil
The proceedings of the trial court was marred by the following irregularities: (1) the hearing of the petition was set ahead of the scheduled date of
hearing, without a publication of the order advancing the date of hearing, and the petition itself; (2) the petition was heard within six months from
the last publication of the petition; (3) petitioner was allowed to take his oath of allegiance before the finality of the judgment; and (4) petitioner
took his oath of allegiance without observing the two-year waiting period.
A decision in a petition for naturalization becomes final only after 30 days from its promulgation and, insofar as the Solicitor General is concerned,
that period is counted from the date of his receipt of the copy of the decision (Republic v. Court of First Instance of Albay, 60 SCRA 195 [1974]).
Section 1 of R.A. No. 530 provides that no decision granting citizenship in naturalization proceedings shall be executory until after two years from its
promulgation in order to be able to observe if: (1) the applicant has left the country; (2) the applicant has dedicated himself continously to a lawful
calling or profession; (3) the applicant has not been convicted of any offense or violation of government promulgated rules; and (4) the applicant has
committed any act prejudicial to the interest of the country or contrary to government announced policies. prcd
Even discounting the provisions of R.A. No. 530, the courts cannot implement any decision granting the petition for naturalization before its finality.
G.R. No. 105715
In view of the finding in G.R. No. 104654 that private respondent is not yet a Filipino citizen, we have to grant the petition in G.R. No. 105715 after
treating it as a petition for certiorari instead of a petition for mandamus. Said petition assails the en banc resolution of the COMELEC, dismissing SPC
Case No. 92-273, which in turn is a petition to annul private respondent's proclamation on three grounds: 1) that the proceedings and composition of
the Provincial Board of Canvassers were not in accordance with law; 2) that private respondent is an alien, whose grant of Filipino citizenship is being
questioned by the State in G.R. No. 104654; and 3) that private respondent is not a duly registered voter. The COMELEC dismissed the petition on the
grounds that it was filed outside the three-day period for questioning the proceedings and composition of the Provincial Board of Canvassers under
Section 19 of R.A. No. 7166. prcd
The COMELEC failed to resolve the more serious issue — the disqualification of private respondent to be proclaimed Governor on grounds of lack of
Filipino citizenship. In this aspect, the petition is one for quo warranto. In Frivaldo v. Commission on Elections, 174 SCRA 245 (1989), we held that a
petition for quo warranto, questioning the respondent's title and seeking to prevent him from holding office as Governor for alienage, is not covered
by the ten-day period for appeal prescribed in Section 253 of the Omnibus Election Code. Furthermore, we explained that "qualifications for public
office are continuing requirements and must be possessed not only at the time of appointment or election or assumption of office but during the
officer's entire tenure; once any of the required qualification is lost, his title may be seasonably challenged."
Petitioner's argument, that to unseat him will frustrate the will of the electorate, is untenable. Both the Local Government Code and the Constitution
require that only Filipino citizens can run and be elected to public office. We can only surmise that the electorate, at the time they voted for private
respondent, was of the mistaken belief that he had legally reacquired Filipino citizenship.
Petitioner in G.R. No. 105715, prays that the votes cast in favor of private respondent be considered stray and that he, being the candidate obtaining
the second highest number of votes, be declared winner. In Labo, Jr. v. COMELEC , 176 SCRA 1 (1989), we ruled that where the candidate who
obtained the highest number of votes is later declared to be disqualified to hold the office to which he was elected, the candidate who garnered the
second highest number of votes is not entitled to be declared winner (See also Geronimo v. Ramos, 136 SCRA 435 [1985]; Topacio v. Paredes, 23 Phil.
238 [1912]). prLL
G.R. No. 105735
In view of the discussions of G.R. No. 104654 and G.R. No. 105715, we find the petition in G.R. No. 105735 moot and academic.
WHEREFORE, the petitions in G.R. No. 104654 and G.R. No. 105715 are both GRANTED while the petition in G.R. No. 105735 is DISMISSED. Private
respondent is declared NOT a citizen of the Philippines and therefore DISQUALIFIED from continuing to serve as GOVERNOR of the Province of
Sorsogon. He is ordered to VACATE his office and to SURRENDER the same to the Vice-Governor of the Province of Sorsogon once this decision
becomes final and executory. No pronouncement as to costs. cdll
SO ORDERED.
Feliciano, Padilla, Bidin, Regalado, Davide, Jr., Romero, Bellosillo, Melo, Puno, Vitug and Kapunan, JJ., concur.
Narvasa, C .J ., took no part. Related to a party.
Cruz, J., took no part. Related to one of the counsel in the proceedings before the COMELEC.
||| (Republic v. De La Rosa, G.R. No. 104654, 105715, 105735, [June 6, 1994])
EN BANC
[G.R. Nos. 178831-32. April 1, 2009.]
JOCELYN SY LIMKAICHONG, petitioner, vs. COMMISSION ON ELECTIONS, NAPOLEON N. CAMERO and RENALD F.
VILLANDO, respondents.
[G.R. No. 179120. April 1, 2009.]
LOUIS C. BIRAOGO, petitioner, vs. HON. PROSPERO NOGRALES, Speaker of the House of Representatives of the Congress of
the Philippines, and JOCELYN SY LIMKAICHONG, respondents.
[G.R. Nos. 179132-33. April 1, 2009.]
OLIVIA P. PARAS, petitioner, vs. HON. PROSPERO NOGRALES, in his capacity as Speaker of the House of Representatives;
HON. ROBERTO NAZARENO, in his capacity as Secretary General of the House of Representatives; HON. RHODORA SEVILLA,
in her capacity as Deputy Secretary General for Finance of the House of Representatives; THE COMMISSION ON ELECTIONS
and JOCELYN SY LIMKAICHONG, respondents.
[G.R. Nos. 179240-41. April 1, 2009.]
RENALD F. VILLANDO, petitioner, vs. COMMISSION ON ELECTIONS and JOCELYN SY LIMKAICHONG, respondents.
DECISION
PERALTA, J p:
Once a winning candidate has been proclaimed, taken his oath, and assumed office as a Member of the House of Representatives, the jurisdiction
of the House of Representatives Electoral Tribunal begins over election contests relating to his election, returns, and qualifications, and mere
allegation as to the invalidity of her proclamation does not divest the Electoral Tribunal of its jurisdiction. DACTSH
At the core of these contentious consolidated petitions are: ( 1 ) the Joint Resolution 1 of the Commission on Elections (COMELEC) Second Division
dated May 17, 2007, disqualifying Jocelyn D. Sy Limkaichong (Limkaichong) from running as a congressional candidate for the First District of Negros
Oriental; (2) the COMELEC En Banc Resolution 2dated June 29, 2007, affirming her disqualification; and (3) the COMELEC En Banc Resolution 3 dated
August 16, 2007, resolving that all pending incidents relating to her qualifications should now be determined by the House of Representatives
Electoral Tribunal (HRET).
The facts are uncontroverted. On March 26, 2007, Limkaichong filed with the COMELEC her Certificate of Candidacy 4 (COC) for the position of
Representative of the First District of Negros Oriental.
In the following weeks, two (2) petitions for her disqualification were instituted before the COMELEC by concerned citizens coming from her locality.
On April 4, 2007, Napoleon Camero, a registered voter of La Libertad, Negros Oriental, filed the petition for her disqualification on the ground that
she lacked the citizenship requirement of a Member of the House of Representatives. The petition, which was docketed as SPA No. (PES) A07-
006, 5 alleged that she is not a natural-born Filipino because her parents were Chinese citizens at the time of her birth. On April 11, 2007, Renald F.
Villando, also a registered voter of the same locality, filed the second petition on the same ground of citizenship, docketed as SPA (PES) No. A07-
007. 6 He claimed that when Limkaichong was born, her parents were still Chinese citizens as the proceedings for the naturalization of Julio Ong Sy,
her father, never attained finality due to procedural and substantial defects. Both petitions prayed for the cancellation of Limkaichong's COC and for
the COMELEC to strike out her name from the list of qualified candidates for the Representative of the First District of Negros Oriental. DTcASE
In her separate Answers 7 to the petitions, Limkaichong claimed that she is a natural-born Filipino since she was born to a naturalized Filipino
father and a natural-born Filipino mother, who had reacquired her status as such due to her husband's naturalization. Thus, at the time of her birth
on November 9, 1959, nineteen (19) days had already passed after her father took his Oath of Allegiance on October 21, 1959 and after he was
issued a Certificate of Naturalization on the same day. She contended that the COMELEC should dismiss the petitions outright for lack of cause of
action. Citing Salcedo II v. Commission on Elections, 8 she averred that a petition filed before an election, questioning the qualification of a candidate,
should be based on Section 78, 9 in relation to Section 74 10 of the Omnibus Election Code (OEC), 11 and not under Sections 68 12 and 74 thereof in
relation to Section 1, 13 Rule 25 of the COMELEC Rules of Procedure 14 and Section 5, 15 paragraph C (3.a) of COMELEC Resolution No. 7800. 16 She
also contended that the petitions were dismissible on the ground that they were in the nature of a collateral attack on her and her father's
citizenships, in contravention of the well-established rule that attack on one's citizenship may only be made through a direct action for its nullity.
The COMELEC consolidated the two (2) petitions and re-docketed them as SPA Nos. 07-247 17 and 07-248, 18 entitled IN THE MATTER OF THE
PETITION TO DISQUALIFY JOCELYN SY LIMKAICHONG FROM HER CANDIDACY AS FIRST DISTRICT REPRESENTATIVE OF NEGROS ORIENTAL (herein
referred to as the disqualification cases), which remained pending on May 14, 2007, when the National and Local Elections were conducted.
After the casting, counting and canvassing of votes in the said elections, Limkaichong emerged as the winner with 65,708 votes 19 or by a margin of
7,746 votes over another congressional candidate, Olivia Paras 20 (Paras), who obtained 57,962.
On May 15, 2007, Paras filed with the COMELEC a Very Urgent Motion for Leave to Intervene and to Suspend the Proclamation of Jocelyn Sy
Limkaichong as Winning Candidate of the First District of Negros Oriental. 21 IaTSED
In a Joint Resolution 22 dated May 17, 2007, the COMELEC Second Division granted the petitions in the disqualification cases, disqualified
Limkaichong as a candidate for Representative of the First District of Negros Oriental, directed the Provincial Supervisor of the COMELEC to strike out
her name from the list of eligible candidates, and for the Provincial Board of Canvassers (PBOC) to suspend her proclamation. In disposing the cases,
the COMELEC Second Division made the following ratiocination:
On the substantial issue of whether respondent Jocelyn Sy-Limkaichong is disqualified to run for the congressional seat of the
First District of Negros Oriental on the ground that she is not a natural-born Filipino, we hold that she is so disqualified.
Petitioners have successfully discharged their burden of proof and has convincingly shown with pieces of documentary
evidence that Julio Ong Sy, father of herein respondent Jocelyn Sy-Limkaichong, failed to acquire Filipino citizenship in the
naturalization proceedings which he underwent for the said purpose.
An examination of the records of Special Case No. 1043 would reveal that the Office of the Solicitor General was deprived of
its participation in all the stages of the proceedings therein, as required under Commonwealth Act No. 473 or the Revised
Naturalization Law and Republic Act No. 530, An Act Making Additional Provisions for Naturalization.
xxx xxx xxx
The documents presented by petitioners showed that the OSG was not furnished copies of two material orders of the trial
court in the said proceedings. One was the July 9, 1957 Order granting his petition for naturalization and the other was
the September 21, 1959 Order declaring Julio Ong Sy as a Filipino citizen.
Moreover, from a perusal of the same page 171 of the OSG logbook, we have determined that the OSG did not receive a
notice for the hearing conducted by the trial court on July 9, 1959, prior to its issuance of the September 12, 1959 Order
declaring Julio Ong Sy as a Filipino citizen. cCSHET
As correctly pointed out by petitioners, this was fatal to the naturalization proceedings of Julio Ong Sy, and prevented the
same from gaining finality. The leading case in the matter is Republic v. Hon. Gabriel V. Valero, 136 SCRA 617 (May 31, 1985),
wherein the Supreme Court declared:
And as though that was not enough, the hearing prior to the oathtaking of respondent Tan was conducted without
the required notice to the Solicitor General. It is true, as it appeared later, that Fiscal Veluz, Jr. was authorized by the
Solicitor General to represent the Government in the hearing of the application for naturalization. That authority,
however, does not extend to Fiscal [Veluz's] right to appear for the State in the hearing preparatory to the
oathtaking. Private respondent Tan was therefore under legal obligation to serve copy of his motion to be allowed to
take his oath of allegiance as a Filipino citizen upon the Solicitor General which was not done.
Respondent argues that upon his taking of the Oath of Allegiance, Julio Ong Sy became a Filipino citizen for all intents and
purposes, with all the rights appurtenant thereto.
This argument does not hold water, as was held by the Supreme Court in the same case of Republic v. Valero, supra:
That private respondent Tan had already taken his oath of allegiance does not in any way legalize the proceedings
relative thereto which is pregnant with legal infirmities. Compounding these irregularities is the fact that Tan was
allowed to take his oath even before the expiration of the thirty (30)-day period within which an appeal may be made
thus making the said oath not only highly improper but also illegal.
In the same case, the Supreme Court added:
To sustain the same would be to sanction a monstrosity known as citizenship by estoppel. The grant of naturalization
under such circumstances is illegal and cancellation thereof may be had at any time. Neither estoppel nor res
judicata may be set up as a bar from instituting the necessary proceedings to nullify the certificate of naturalization
so issued. cACEaI
Another glaring defect in the said proceedings was the fact that Julio Ong Sy took his Oath of Allegiance on October 21, 1959,
which was exactly thirty (30) days after his declaration as a naturalized Filipino.

Even granting that the OSG was notified of the September 21, 1959 Order, this was still one day short of the reglementary
period required under Sections 11 and 12 of C.A. No. 473, above-cited.
The thirty-day reglementary period is so required under the law so that the OSG could make known his objections and to
appeal from the order of the trial court declaring the petitioner a naturalized Filipino citizen. This is also the reason why a copy
of the petitioner's motion to take his oath of allegiance has to be furnished to the OSG.
The respondent insists that naturalization proceedings are in rem and are binding on the whole world.
She would have been correct had all the necessary parties to the case been informed of the same. The OSG, being the counsel
for the government, has to participate in all the proceedings so that it could be bound by what has transpired therein. Lacking
the participation of this indispensable party to the same, the proceedings are null and void and, hence, no rights could arise
therefrom.
From all the foregoing, therefore, it could be seen that Julio Ong Sy did not acquire Filipino citizenship through the
naturalization proceedings in Special Case No. 1043. Thus, he was only able to transmit to his offspring, Chinese citizenship.
Respondent Jocelyn Sy-Limkaichong being the daughter of Julio Ong Sy, and having been born on November 9, 1959, under
the 1935 Philippine Constitution, is a Chinese national, and is disqualified to run as First District Representative of Negros
Oriental.
WHEREFORE, the Petitions are GRANTED and Jocelyn D. Sy-Limkaichong is declared as DISQUALIFIED from her candidacy for
Representative of the First District of Negros Oriental. SCEDAI
The Provincial Supervisor of the Commission on Elections of Negros Oriental is hereby directed to strike out the name JOCELYN
SY-LIMKAICHONG from the list of eligible candidates for the said position, and the concerned Board of Canvassers is hereby
directed to hold and/or suspend the proclamation of JOCELYN SY-LIMKAICHONG as winning candidate, if any, until this decision
has become final.
SO ORDERED. 23
The PBOC received the Joint Resolution of the COMELEC Second Division on the evening of May 17, 2007, and accordingly suspended the
proclamation of Limkaichong. 24
The following day, or on May 18, 2007, the COMELEC En Banc issued Resolution No. 8062 25 adopting the policy-guidelines of not suspending the
proclamation of winning candidates with pending disqualification cases which shall be without prejudice to the continuation of the hearing and
resolution of the involved cases.
On May 20, 2007, Limkaichong filed with the COMELEC a Motion for Reconsideration of the Joint Resolution of May 17, 2007 and Urgent Motion to
Lift the Order Suspending Proclamation. 26
On May 22, 2007, Limkaichong filed another motion for the lifting of the directive suspending her proclamation, insisting that she should be
proclaimed as the winner in the congressional race pursuant to COMELEC Resolution No. 8062. 27 On same date, Villando, one of the petitioners in
the disqualification cases, filed an Urgent Manifestation Clarifying COMELEC Resolution No. 8062 with Motion, 28 praying that the COMELEC should
not lift the suspension of Limkaichong's proclamation.
On May 25, 2007, the PBOC, in compliance with COMELEC Resolution No. 8062, reconvened and proclaimed Limkaichong as the duly elected
Member of the House of Representatives for the First District of Negros Oriental. 29
Thereafter, or on May 30, 2007, Paras filed with the COMELEC a Petition to Nullify and/or Annul the Proclamation of Jocelyn Sy-Limkaichong as
First District Representative of Negros Oriental in relation to the May 17, 2007 Joint Resolution of the COMELEC Second Division, 30 stating,
among others, that Limkaichong's proclamation violated the earlier order of the COMELEC Second Division suspending her proclamation. The
petition, docketed as SPC No. 07-211, was dismissed by the COMELEC First Division, 31 ratiocinating that the disqualification cases were not yet final
when Limkaichong was proclaimed. Accordingly, her proclamation which was valid or legal, effectively divested the COMELEC of its jurisdiction over
the cases. The COMELEC First Division explained its ruling in this wise:
The Commission has made its intention in issuing Resolution No. 8062 very clear in that there shall be no suspension of
proclamation of winning candidates with pending disqualification cases involving, among others, issues of citizenship. As the
disqualification cases involving Limkaichong were still pending reconsideration by the en banc,the underlying policy which gave
rise to the issuance of the Resolution: to respect the will of the Filipino electorate, applies to the suspension of proclamation of
the winning congressional candidate for the First District of Negros Oriental.
WHEREFORE, the instant petition is dismissed.
SO ORDERED. (Emphasis ours)
Dissatisfied, Paras moved for the reconsideration of the above Resolution. 32
Meanwhile, in a Resolution 33 dated June 29, 2007, the COMELEC En Banc, in an equally divided vote of 3:3, denied Limkaichong's motion for
reconsideration of the Joint Resolution of the COMELEC Second Division in the disqualification cases. The pertinent portions of the Resolution
denying her motion reads: aESICD
Anent the issue of jurisdiction, We rule that the Commission has jurisdiction to rule on Respondent Limkaichong's Motion for
Reconsideration notwithstanding her proclamation as it is only this Commission, and not the House of Representatives
Electoral Tribunal (HRET), which has jurisdiction to review resolutions or decisions of the COMELEC, whether issued by a
division or en banc. As stated by the Supreme Court in the leading case of Codilla v. De Venecia, G.R. No. 150605, December 10,
2002, respondent herself seasonably challenged the validity of the resolution of the Second Division in her motion for
reconsideration. Hence, the issue of respondent's disqualification was still within the exclusive jurisdiction of the Comelec En
Banc to resolve, and HRET cannot assume jurisdiction on the matter, to wit:
To stress again, at the time of the proclamation of respondent Locsin, the validity of the Resolution of the COMELEC
Second Division was seasonably challenged by the petitioner in his Motion for Reconsideration. The issue was still
within the exclusive jurisdiction of the Comelec En Banc to resolve. Hence, the HRET cannot assume jurisdiction over
the matter.
In Puzon v. Cua, even the HRET ruled that the "doctrinal ruling that once a proclamation has been made and a
candidate-elect has assumed office, it is this Tribunal that has jurisdiction over an election contest involving members
of the House of Representatives, could not have been immediately applicable due to the issue regarding the validity
of the very COMELEC pronouncements themselves." This is because the HRET has no jurisdiction to review
resolutions or decisions of the COMELEC, whether issued by a division or en banc.
Finally, in disposing the Opposition to the Motion for Reconsideration with Partial Motion for Reconsideration filed by
intervenor Olivia P. Paras praying that she be proclaimed as the winning candidate for First District Representative, suffice it to
say that in the same case of Codilla v. De Venecia, supra, the Supreme Court held, thus:
More brazen is the proclamation of respondent Locsin which violates the settled doctrine that the candidate who
obtains the second highest number of votes may not be proclaimed winner in case the winning candidate is
disqualified. In every election, the people's choice is the paramount consideration and their expressed will must, at
all times, be given effect. When the majority speaks and elects into office a candidate by giving him the highest
number of votes cast in the election for the office, no one can be declared elected in his place. In Domino v.
COMELEC, this Court ruled, viz.:
It would be extremely repugnant to the basic concept of the constitutionally guaranteed right to suffrage if a
candidate who has not acquired the majority or plurality of votes is proclaimed winner and imposed as
representative of a constituency, the majority of which have positively declared through their ballots that they do
not choose him. To simplistically assume that the second placer would have received that (sic) other votes would be
to substitute our judgment for the mind of the voters. He could not be considered the first among the qualified
candidates because in a field which excludes the qualified candidate, the conditions would have substantially
changed. aIcDCT
xxx xxx xxx
The effect of a decision declaring a person ineligible to hold an office is only that the election fails entirely, that the
wreath of victory cannot be transferred from the disqualified winner to the repudiated loser because the law then as
now only authorizes a declaration in favor of the person who has obtained a plurality of votes, and does not entitle
the candidate receiving the next highest number of votes to be declared elected. In such case, the electors have
failed to make a choice and the election is a nullity. To allow the defeated and repudiated candidate to take over the
elective position despite his rejection by the electorate is to disenfranchise the electorate without any fault on their
part and to undermine the importance and meaning of democracy and the people's right to elect officials of their
choice.
All told, We find no cogent reason to disturb the findings of this Commission (Second Division) in its Joint Resolution
promulgated on May 17, 2007.
WHEREFORE, premises considered, the instant Motion for Reconsideration of Respondent Jocelyn Sy-Limkaichong is
hereby DENIED.
The Opposition to the Motion for Reconsideration with Partial Motion for Reconsideration filed by Intervenor Olivia P. Paras
praying that she be proclaimed as the winning candidate for the First District Representative of Negros Oriental is hereby
denied for lack of merit.

SO ORDERED. 34
On July 3, 2007, Limkaichong filed in the disqualification cases against her a Manifestation and Motion for Clarification and/or To Declare the
Petitions as Dismissed in Accordance with Section 6, Rule 18 of the COMELEC Rules of Procedure. 35 She contended that, with her proclamation,
her having taken her oath of office and her assumption of the position, the COMELEC was divested of jurisdiction to hear the disqualification cases.
She further contended that, following Section 6, 36 Rule 18 of the COMELEC Rules of Procedure, the disqualification cases would have to be reheard,
and if on rehearing, no decision would be reached, the action or proceedings should be dismissed, because the COMELEC En Banc was equally
divided in opinion when it resolved her motion for reconsideration. EScIAa
On an even date, Paras wrote the House of Representatives informing it of the COMELEC En Banc Resolution dated June 29, 2007 upholding the Joint
Resolution of the COMELEC Second Division dated May 17, 2007, which disqualified Limkaichong as a congressional candidate. 37
In the interim, then Speaker of the House of Representatives Jose de Venecia, Jr. (De Venecia) allowed Limkaichong to officially assume the office as a
Member of the House of Representatives on July 23, 2007, as shown in the Journal of the House of Representatives. 38
Despite Limkaichong's repeated pleas for the resolution of her manifestation and motion for clarification, 39 the COMELEC did not resolve the same.
Hence, on August 1, 2007, she filed with this Court a Petition for Certiorari 40 under Rule 65, in relation to Rule 64 of the 1997 Rules of Civil
Procedure docketed as G.R. Nos. 178831-32 praying for the annulment of the May 17, 2007 Joint Resolution of the COMELEC Second Division and
the June 29, 2007 Resolution of the COMELEC En Banc in the disqualification cases for having been issued with grave abuse of discretion amounting
to lack of jurisdiction. She averred that since she was already proclaimed on May 25, 2007 as Representative of the First District of Negros Oriental,
had assumed office on June 30, 2007, and had started to perform her duties and functions as such, the COMELEC had lost its jurisdiction and it is now
the HRET which has jurisdiction over any issue involving her qualifications for the said office.
On August 16, 2007, the COMELEC En Banc ruled on Limkaichong's manifestation and motion for clarification, 41 with the following disquisition:
In view of the proclamation of Limkaichong and her subsequent assumption of office on June 30, 2007, this Commission rules
that all pending incidents relating to the qualifications of Limkaichong should now be determined by the House of
Representatives Electoral Tribunal in accordance with the above-quoted provision of the Constitution.
WHEREFORE, premises considered, this Commission resolved, as it hereby resolves, that all pending incidents relating to the
qualifications of Jocelyn S. Limkaichong as Member of the House of Representatives should now be determined by the House
of Representatives Electoral Tribunal. aTEScI
SO ORDERED. (Emphasis ours)
On August 24, 2007, Louis Biraogo (Biraogo), as a citizen and a taxpayer, filed with the Court a Petition for Prohibition and Injunction with
Preliminary Injunction and/or Temporary Restraining Order 42 under Section 2, Rule 65 of the 1997 Rules of Civil Procedure, docketed as G.R. No.
179120, seeking to enjoin and permanently prohibit: (a) De Venecia from allowing Limkaichong to sit in the House of Representatives and participate
in all its official activities; and (b) Limkaichong from holding office as its Member. 43
Meanwhile, on August 28, 2007, Paras has instituted before the Court a Petition for Quo Warranto, Prohibition and Mandamus with Prayer for the
Issuance of a Temporary Restraining Order and/or Writ of Preliminary Injunction 44 under Rule 65 of the 1997 Rules of Civil Procedure, docketed
as G.R. Nos. 179132-33, seeking, among others, the ouster of Limkaichong from the House of Representatives on account of her disqualification and
for the holding of special elections to fill the vacancy created by such.45
On even date, the COMELEC Second Division promulgated a Resolution 46 denying Villando's motion to suspend the proclamation of Limkaichong,
which denial was affirmed by the COMELEC En Banc in a Resolution 47 dated February 1, 2008.
On September 5, 2008, Villando also filed with this Court a Petition for Certiorari and Injunction with Preliminary Injunction and Temporary
Restraining Order 48 under Rule 65 of the 1997 Rules of Civil Procedure, docketed as G.R. Nos. 179240-41, contending, among others, that the
COMELEC En Banc gravely abused its discretion in issuing the August 16, 2007 Resolution 49 because it still acted on Limchaikong's manifestation and
motion for clarification, notwithstanding that the same was not set for hearing and considering that its June 29, 2007 Resolution had already become
final and executory. cIaCTS
As the four (4) petitions are interrelated, the Court resolved to consolidate them in its Resolutions dated September 4 and 11, 2007.
The Court heard the parties in oral argument on August 26, 2008, during which the following issues were tackled:
1. Whether the proclamation of Limkaichong by the Provincial Board of Canvassers of Negros Oriental is valid;
2. Whether said proclamation divested the Commission on Elections of jurisdiction to resolve the issue of Limkaichong's
citizenship;
3. Whether the House of Representatives Electoral Tribunal shall assume jurisdiction, in lieu of the COMELEC, over the issue of
Limkaichong's citizenship;
4. Whether the COMELEC Second Division and the COMELEC En Banc correctly ruled that Limkaichong is disqualified from
running as a Member of the House of Representatives on the ground that she is not a natural-born citizen;
5. Whether the COMELEC disqualification of Limkaichong is final and executory; and,
6. Whether the Speaker of the House of Representatives may be compelled to prohibit Limkaichong from assuming her duties
as a Member of the House of Representatives.
On same day, the Court required the parties to simultaneously file within twenty (20) days their respective memoranda, after which the petitions
shall be deemed submitted for resolution, with or without the memoranda.
Section 6, Article VI of the 1987 Philippine Constitution provides for the qualification of a Member of the House of Representatives, thus: SETaHC
Section 6. No person shall be a Member of the House of Representatives unless he is a natural-born citizen of the
Philippines and, on the day of the election, is at least twenty-five years of age, able to read and write, and, except the party-list
representatives, a registered voter in the district in which he shall be elected, and a resident thereof for a period of not less
than one year immediately preceding the day of the election.
When Limkaichong filed her COC, she stated therein that she is a natural-born Filipino citizen. It was not true, according to the petitioners in the
disqualification cases, because her father remained a Chinese citizen at the time of her birth. The COMELEC Second Division has sided with Camero
and Villando, and disqualified Limkaichong to run as a congressional candidate in the First District of Negros Oriental for having failed to comply with
the citizenship requirement. Accordingly, her proclamation was ordered suspended notwithstanding that she obtained the highest number of votes
during the elections. Nonetheless, she was proclaimed by the PBOC pursuant to the policy guidelines of COMELEC En Banc Resolution No. 8062, and
she has since assumed her position and performed her functions as a Member of the House of Representatives.
I
Whether Limkaichong's proclamation was valid.
The proclamation of Limkaichong was valid. The COMELEC Second Division rendered its Joint Resolution dated May 17, 2007. On May 20, 2007,
Limkaichong timely filed with the COMELEC En Banc her motion for reconsideration as well as for the lifting of the incorporated directive suspending
her proclamation. The filing of the motion for reconsideration effectively suspended the execution of the May 17, 2007 Joint Resolution. 50 Since
the execution of the May 17, 2007 Joint Resolution was suspended, there was no impediment to the valid proclamation of Limkaichong as the
winner. Section 2, Rule 19 of the COMELEC Rules of Procedure provides:
Sec. 2. Period for Filing Motions for Reconsideration. — A motion to reconsider a decision, resolution, order or ruling of a
Division shall be filed within five (5) days from the promulgation thereof. Such motion, if not pro forma, suspends the
execution for implementation of the decision, resolution, order and ruling. aCSDIc
In G.R. Nos. 179132-33, Paras, however, maintained that Limkaichong was a Chinese citizen who was disqualified to run as a congressional candidate
by way of a final judgment of the COMELEC. With that, her proclamation was questionable and the same was done in open defiance of the Joint
Resolution dated May 17, 2007 of the COMELEC Second Division. She also stressed that Limkaichong's proclamation was procedurally defective, it
appearing that one of the PBOC members was not present on May 25, 2007, and that it took place in a restaurant and not at the provincial capitol.
Finally, she argued that Limkaichong's proclamation was void in accordance with the Court's pronouncement in the case of Codilla v. De Venecia. 51
The Office of the Solicitor General (OSG) filed its Comment on the petition of Paras, expressing its support for the position taken by the latter.
A perusal of the arguments advanced by Paras and the OSG does not sway the Court to rule against the validity of Limkaichong's proclamation. No
less than the COMELEC First Division has sustained the validity of her proclamation when it dismissed, by way of a Resolution dated June 29, 2007,
the petition filed by Paras to nullify the proclamation. Not only that. The COMELEC First Division has also adopted Limkaichong's argument that
following her valid proclamation, the COMELEC's jurisdiction over the disqualification cases has ceased and that the same should be threshed out in
the proper proceedings filed before the HRET. Notably, the dismissal of Paras' petition was affirmed by the COMELEC in its Omnibus Order dated
January 28, 2008.

In addition, the validity of Limkaichong's proclamation is in accordance with COMELEC En Banc Resolution No. 8062. The disqualification cases filed
against her remained pending as a result of her timely motion for reconsideration. Villando (in G.R. Nos. 179240-41), however, maintained that
Resolution No. 8062 is invalid; hence, it could not be used as basis to validate Limkaichong's proclamation. He argued that it must be published since
it is a "policy-guideline" in the exercise of the COMELEC's rule-making power. As such, it cannot supersede the Joint Resolution of the Second Division
which was rendered pursuant to the COMELEC's quasi-judicial power.
His argument is specious. Resolution No. 8062 is not only a policy-guideline. It is also an administrative interpretation of the two (2) provisions of
the 1987 Constitution, namely: (i) Section 17, 52 Article VI (ii); Section 2 (2), 53 Article IX-C; Section 6 54 of R.A. 6646; and Sections 241 55 and
243, 56 Article XX of the OEC. As such, it does not have to comply with the due process requirement. The term "administrative" connotes or pertains
to "administration, especially management, as by managing or conducting, directing or superintending, the execution, application, or conduct of
persons or things." It does not entail an opportunity to be heard, the production and weighing of evidence, and a decision or resolution
thereon. 57 This is to be distinguished from "quasi-judicial function", a term which applies, among others, to the action or discretion of public
administrative officers or bodies, who are required to investigate facts, or ascertain the existence of facts, hold hearings, and draw conclusions from
them, as a basis for their official action and to exercise discretion of a judicial nature. 58 HIAEcT
Resolution No. 8062 is a valid exercise of the COMELEC's constitutionally mandated power to promulgate its own rules of procedure relative to the
conduct of the elections. 59 In adopting such policy-guidelines for the May 14, 2007 National and Local Elections, the COMELEC had in mind the
objective of upholding the sovereign will of the people and in the interest of justice and fair play. Accordingly, those candidates whose
disqualification cases are still pending at the time of the elections, should they obtain the highest number of votes from the electorate, shall be
proclaimed but that their proclamation shall be without prejudice to the continuation of the hearing and resolution of the involved cases. Whereas,
in this case, the COMELEC Second Division having failed to act on the disqualification cases against Limkaichong until after the conduct of the
elections, with her obtaining the highest number of votes from the electorate, her proclamation was properly effected by the PBOC pursuant to
Resolution No. 8062.
The Court has held in the case of Planas v. COMELEC, 60 that at the time of the proclamation of Defensor, the respondent therein who garnered the
highest number of votes, the Division Resolution invalidating his certificate of candidacy was not yet final. As such, his proclamation was valid or
legal, as he had at that point in time remained qualified. Limkaichong's situation is no different from that of Defensor, the former having been
disqualified by a Division Resolution on the basis of her not being a natural-born Filipino citizen. When she was proclaimed by the PBOC, she was the
winner during the elections for obtaining the highest number of votes, and at that time, the Division Resolution disqualifying her has not yet became
final as a result of the motion for reconsideration.
II
Whether, upon Limkaichong's proclamation, the HRET, instead of the COMELEC, should assume jurisdiction over the disqualification cases.
In her petition (G.R. Nos. 178831-32), Limkaichong argued that her proclamation on May 25, 2007 by the PBOC divested the COMELEC of its
jurisdiction over all issues relating to her qualifications, and that jurisdiction now lies with the HRET.
Biraogo, on the other hand, believed otherwise. He argued (in G.R. No. 179120) that the issue concerning Limkaichong's disqualification is still within
the exclusive jurisdiction of the COMELEC En Banc to resolve because when Limkaichong was proclaimed on May 25, 2007, the matter was still
pending resolution before the COMELEC En Banc.
We do not agree. The Court has invariably held that once a winning candidate has been proclaimed, taken his oath, and assumed office as a
Member of the House of Representatives, the COMELEC's jurisdiction over election contests relating to his election, returns, and qualifications
ends, and the HRET's own jurisdiction begins. 61It follows then that the proclamation of a winning candidate divests the COMELEC of its jurisdiction
over matters pending before it at the time of the proclamation. The party questioning his qualification should now present his case in a proper
proceeding before the HRET, the constitutionally mandated tribunal to hear and decide a case involving a Member of the House of Representatives
with respect to the latter's election, returns and qualifications. The use of the word "sole" in Section 17, Article VI of the Constitution and in Section
250 62 of the OEC underscores the exclusivity of the Electoral Tribunals' jurisdiction over election contests relating to its members. 63 aCSDIc
Section 17, Article VI of the 1987 Constitution provides:
Sec. 17. The Senate and the House of Representatives shall each have an Electoral Tribunal which shall be the sole judge of all
contests relating to the election, returns, and qualifications of their respective Members. Each Electoral Tribunal shall be
composed of nine Members, three of whom shall be Justices of the Supreme Court to be designated by the Chief Justice, and
the remaining six shall be Members of the Senate or the House of Representatives, as the case may be, who shall be chosen on
the basis of proportional representation from the political parties and the parties or organizations registered under the party-
list system represented therein. The senior Justice in the Electoral Tribunal shall be its Chairman.
Corollary thereto is Rule 14 of the 1998 Rules of the HRET, as amended, which states:
RULE 14. Jurisdiction. — The Tribunal is the sole judge of all contests relating to the election, returns,
and qualifications of the Members of the House of Representatives.
The COMELEC En Banc, in its Resolution dated August 16, 2007, had given paramount consideration to the two (2) aforementioned provisions when
it stated that:
In view of the proclamation of Limkaichong and her subsequent assumption of office on June 30, 2007, this Commission rules
that all pending incidents relating to the qualifications of Limkaichong should now be determined by the House of
Representatives Electoral Tribunal in accordance with the above-quoted provision of the Constitution.
WHEREFORE, premises considered, this Commission resolved, as it hereby resolves, that all pending incidents relating to the
qualifications of Jocelyn S. Limkaichong as Member of the House of Representatives should now be determined by the House
of Representatives Electoral Tribunal. SaIACT
SO ORDERED. (Emphasis supplied)
Worth citing also is the ratiocination of the COMELEC First Division when it dismissed the petition of Paras seeking the nullity of Limkaichong's
proclamation, thus:
The present situation is similar not to the factual circumstances of Codilla, which Paras invokes, but rather to that
in Planas which adheres to the general rule giving jurisdiction to the House of Representatives Electoral Tribunal. As at the time
of Limkaichong's proclamation, her disqualification was not yet final, her proclamation was valid or legal. This Commission no
longer has jurisdiction over the case. This, notwithstanding the Second Division's directive suspending Limkaichong's
proclamation.
The Commission has made its intention in issuing Resolution No. 8062 very clear in that there shall be no suspension of
proclamation of winning candidates with pending disqualification cases, involving, among others, issues of citizenship. As the
disqualification cases involving Limkaichong were still pending reconsideration by the En Banc, the underlying policy which
gave rise to the issuance of the resolution: to respect the will of the Filipino electorate, applies to the suspension of
proclamation of the winning Congressional candidate for the First District of Negros Oriental.
WHEREFORE, the instant petition is DISMISSED.
SO ORDERED.
Petitioners (in G.R. Nos. 179120, 179132-33, and 179240-41) steadfastly maintained that Limkaichong's proclamation was tainted with irregularity,
which will effectively prevent the HRET from acquiring jurisdiction. TASCEc
The fact that the proclamation of the winning candidate, as in this case, was alleged to have been tainted with irregularity does not divest the HRET
of its jurisdiction. 64 The Court has shed light on this in the case of Vinzons-Chato, 65 to the effect that:
In the present case, it is not disputed that respondent Unico has already been proclaimed and taken his oath of office as a
Member of the House of Representatives (Thirteenth Congress); hence, the COMELEC correctly ruled that it had already lost
jurisdiction over petitioner Chato's petition. The issues raised by petitioner Chato essentially relate to the canvassing of returns
and alleged invalidity of respondent Unico's proclamation. These are matters that are best addressed to the sound judgment
and discretion of the HRET. Significantly, the allegation that respondent Unico's proclamation is null and void does not divest
the HRET of its jurisdiction:
. . . [I]n an electoral contest where the validity of the proclamation of a winning candidate who has taken his oath of
office and assumed his post as congressman is raised, that issue is best addressed to the HRET. The reason for this
ruling is self-evident, for it avoids duplicity of proceedings and a clash of jurisdiction between constitutional bodies,
with due regard to the people's mandate.

Further, for the Court to take cognizance of petitioner Chato's election protest against respondent Unico would be to usurp the
constitutionally mandated functions of the HRET.
In fine, any allegations as to the invalidity of the proclamation will not prevent the HRET from assuming jurisdiction over all matters essential to a
member's qualification to sit in the House of Representatives.
The 1998 HRET Rules, as amended, provide for the manner of filing either an election protest or a petition for quo warranto against a Member of the
House of Representatives, to wit:
Rule 16. Election protest. — A verified petition contesting the election of any Member of the House of Representatives shall be
filed by any candidate who has duly filed a certificate of candidacy and has been voted for the same office, within ten (10) days
after the proclamation of the winner. The party filing the protest shall be designated as the protestant while the adverse party
shall be known as the protestee.
xxx xxx xxx
Rule 17. Quo Warranto. — A verified petition for quo warranto contesting the election of a Member of the House of
Representatives on the ground of ineligibility or of disloyalty to the Republic of the Philippines shall be filed by any voter within
ten (10) days after the proclamation of the winner. The party filing the petition shall be designated as the petitioner while the
adverse party shall be known as the respondent.
xxx xxx xxx
Rule 19. Periods Non-Extendible. — The ten-day period mentioned in Rules 16 and 17 is jurisdictional and cannot be extended.
Accordingly, after the proclamation of the winning candidate in the congressional elections, the remedy of those who may assail one's
eligibility/ineligibility/qualification/disqualification is to file before the HRET a petition for an election protest, or a petition for quo warranto, within
the period provided by the HRET Rules. In Pangilinan v. Commission on Elections, 66 we ruled that where the candidate has already been proclaimed
winner in the congressional elections, the remedy of petitioner is to file an electoral protest with the Electoral Tribunal of the House of
Representatives. HTaIAC
The PBOC proclaimed Limkaichong as the winner on May 25, 2007. Thus, petitioners (in G.R. Nos. 179120, 179132-33, and 179240-41) should have
filed either an election protest or petition for quo warranto within ten days from May 25, 2007. But they did not. In fact, to date, no petition of
protest or petition for quo warranto has been filed with the HRET. Verily, the ten-day prescriptive period for initiating a contest against Limkaichong
has long expired.
However, the said ten-day prescriptive period under the 1998 HRET Rules does not apply to disqualification cases based on citizenship. Under
the 1987 Constitution, Members of the House of Representatives must be natural-born citizens not only at the time of their election but during
their entire tenure. Being a continuing requirement, one who assails a member's citizenship or lack of it may still question the same at any time, the
ten-day prescriptive period notwithstanding.
In Frivaldo v. Commission on Elections, 67 the Court held that:
The argument that the petition filed with the Commission on Elections should be dismissed for tardiness is not well-taken. The
herein private respondents are seeking to prevent Frivaldo from continuing to discharge his office as governor because he is
disqualified from doing so as a foreigner. Qualifications for public office are continuing requirements and must be possessed
not only at the time of appointment or election or assumption of office but during the officer's entire tenure. Once any of
the required qualifications is lost, his title may be seasonably challenged. If, say, a female legislator were to marry a
foreigner during her term and by her act or omission acquires his nationality, would she have the right to remain in office
simply because the challenge to her title may not longer be made within ten days from her proclamation? . . .
This Court will not permit the anomaly of a person sitting as provincial governor in this country while owing exclusive
allegiance to another country. The fact that he was elected by the people of Sorsogon does not excuse this patent violation of
the salutary rule limiting public office and employment only to the citizens of this country. The qualifications prescribed for
elective office cannot be erased by the electorate alone. The will of the people as expressed through the ballot cannot cure
the vice of ineligibility, especially if they mistakenly believed, as in this case, that the candidate was qualified. Obviously,
this rule requires strict application when the deficiency is lack of citizenship. If a person seeks to serve in the Republic of the
Philippines, he must owe his total loyalty to this country alone, abjuring and renouncing all fealty to any other state.
However, in assailing the citizenship of the father, the proper proceeding should be in accordance with Section 18 of Commonwealth Act No.
473 which provides that:
Sec. 18. Cancellation of Naturalization Certificate Issued. — Upon motion made in the proper proceedings by the Solicitor
General or his representative, or by the proper provincial fiscal, the competent judge may cancel the naturalization
certificate issued and its registration in the Civil Register:
1. If it is shown that said naturalization certificate was obtained fraudulently or illegally;
2. If the person naturalized shall, within five years next following the issuance of said naturalization certificate, return
to his native country or to some foreign country and establish his permanent residence there: Provided,
That the fact of the person naturalized remaining more than one year in his native country or the country
of his former nationality, or two years in any other foreign country, shall be considered as prima
facie evidence of his intention of taking up his permanent residence in the same:
3. If the petition was made on an invalid declaration of intention;
4. If it is shown that the minor children of the person naturalized failed to graduate from a public or private high
schools recognized by the Office of Private Education of the Philippines, where Philippine history,
government or civics are taught as part of the school curriculum, through the fault of their parents either
by neglecting to support them or by transferring them to another school or schools. A certified copy of the
decree canceling the naturalization certificate shall be forwarded by the clerk of Court of the Department
of Interior and the Bureau of Justice; aCSHDI
5. If it is shown that the naturalized citizen has allowed himself to be used as a dummy requiring Philippine
citizenship as a requisite for the exercise, use or enjoyment of a right, franchise or privilege. (Emphasis
supplied)
As early as the case of Queto v. Catolico, 68 where the Court of First Instance judge motu proprio and not in the proper denaturalization proceedings
called to court various grantees of certificates of naturalization (who had already taken their oaths of allegiance) and cancelled their certificates of
naturalization due to procedural infirmities, the Court held that:
. . . It may be true that, as alleged by said respondents, that the proceedings for naturalization were tainted with certain
infirmities, fatal or otherwise, but that is beside the point in this case. The jurisdiction of the court to inquire into and rule
upon such infirmities must be properly invoked in accordance with the procedure laid down by law. Such procedure is the
cancellation of the naturalization certificate. [Section 1(5), Commonwealth Act No. 63], in the manner fixed in Section 18
of Commonwealth Act No. 473, hereinbefore quoted, namely, "upon motion made in the proper proceedings by the Solicitor
General or his representatives, or by the proper provincial fiscal." In other words, the initiative must come from these
officers, presumably after previous investigation in each particular case. (Emphasis supplied)
Clearly, under law and jurisprudence, it is the State, through its representatives designated by statute, that may question the illegally or invalidly
procured certificate of naturalization in the appropriate denaturalization proceedings. It is plainly not a matter that may be raised by private persons
in an election case involving the naturalized citizen's descendant. DTIcSH
III
Whether the COMELEC Second Division and the COMELEC En Banc correctly disqualified Limkaichong on the ground that she is not a natural-
born Filipino citizen.
In resolving the disqualification cases, the COMELEC Second Division relied on the entries in the docket book of the OSG, 69 the only remaining
record of the naturalization proceedings, 70 and ruled on the basis thereof that the naturalization proceedings of Julio Ong Sy, Limkaichong's father,
in Special Case No. 1043, were null and void. The COMELEC Second Division adopted Villando and Camero's arguments that the OSG was deprived of
its participation in the said case for it was not furnished copies of the following: (a) the July 9, 1957 Order of the Court of First Instance (CFI) granting
the petition for naturalization; and (b) the September 21, 1959 Order of the CFI declaring Julio Ong Sy a Filipino citizen. Thus, when the latter took his
oath of allegiance on October 21, 1959, it was exactly 30 days after his declaration as a naturalized Filipino, or one day short of the reglementary
period required under Sections 11 and 12 of Commonwealth Act No. 473. Such defects were fatal to the naturalization proceedings of Julio Ong Sy
and prevented the same from gaining finality. The COMELEC Second Division concluded that since Julio Ong Sy did not acquire Philippine citizenship
through the said naturalization proceedings, it follows that Limkaichong remains a Chinese national and is disqualified to run as candidate and be
elected as a Member of the House of Representatives.

We cannot resolve the matter of Limkaichong's citizenship as the same should have been challenged in appropriate proceedings as earlier stated.
IV
Whether the COMELEC's disqualification of Limkaichong is final and executory.
In resolving this issue, pertinent is the provision of Section 13 (b), Rule 18 of the 1993 COMELEC Rules of Procedure:
Sec. 13. Finality of Decisions or Resolutions. — . . .
(b) In Special Actions and Special Cases, a decision or resolution of the Commission en banc shall become final and executory
after five (5) days from its promulgation unless restrained by the Supreme Court.
In his Memorandum dated June 27, 2008, Biraogo stated that the Resolution of the COMELEC En Banc in the disqualification cases became final and
executory after five (5) days from its promulgation and that the same was not restrained by this Court pursuant to Section 13 (b), Rule 18 of the 1993
COMELEC Rules of Procedure. He averred that since Limkaichong received a copy of the COMELEC En Banc Resolution dated June 29, 2007 on July 3,
2007, she had until July 8, 2007 within which to obtain a restraining order from the Court to prevent the same from becoming final and executory.
However, she did not do anything to that effect. Biraogo also averred that Limkaichong is guilty of forum shopping; hence, her petition must be
dismissed by the Court.
Instead of asking the Court for what Biraogo opined as a restraining order, Limkaichong filed with this Court, on August 1, 2007, her petition
for certiorari assailing the said COMELEC En Banc Resolution pursuant to Section 2, 71 Rule 64, in relation to Rule 65, 1997 Rules of Civil Procedure,
postulating that she had thirty (30) days from July 4, 2007 within which to file the petition, or until August 3, 2007. She cited Section 7, Article IX of
the 1987 Constitution, which prescribes the power of this Court to review decisions of the COMELEC, 72 thus: AICTcE
SEC. 7. Each Commission shall decide by a majority vote of all its Members any case or matter brought before it within sixty
days from the date of its submission for decision or resolution. A case or matter is deemed submitted for decision or resolution
upon the filing of the last pleading, brief, or memorandum required by the rules of the Commission or by the Commission
itself. Unless otherwise provided by this Constitution or by law, any decision, order, or ruling of each Commission may be
brought to the Supreme Court on certiorari by the aggrieved party within thirty days from receipt of a copy thereof.
In his Comment on the petition, Villando prayed for the outright dismissal of Limkaichong's petition as (a) it was filed beyond the reglementary
period; (b) Limkaichong engaged in prohibited forum shopping; and (c) Limkaichong admitted that the issues raised have become moot and
academic. He also sought to declare Limkaichong in contempt of court for forum shopping.
The COMELEC, through the OSG, also filed its Comment, praying for the denial of Limkaichong's petition and its dismissal for being moot, contending
that: (a) the COMELEC En Banc Resolution dated August 16, 2007 has rendered the instant petition moot and academic; and (b) Limkaichong
knowingly and intentionally engaged in forum shopping. The OSG argued that, without waiting for the resolution of her Motion for Clarification and
two (2) successive motions to resolve said motions which are pending before the COMELECEn Banc, Limkaichong filed the present petition to
question the Joint Resolution dated May 17, 2007 of the COMELEC Second Division, which issues were pending before the COMELEC En Banc. Her act
of seeking relief from this Court while there were several other incidents pending before the COMELEC, the final resolution in either one of which will
amount to res judicata in the other, clearly showed forum shopping on her part.
In her Reply to the above Comments, Limkaichong countered that she did not engage in forum shopping, for had she waited for the COMELEC to rule
on her manifestation and other motions, it would have resulted in the expiration of the reglementary period for filing a petition for certiorari before
the Court. cDCSTA
The May 17, 2007 Joint Resolution of the COMELEC Second Division disqualifying Limkaichong and suspending her proclamation cannot yet be
implemented considering that she timely filed a motion for reconsideration. Thus, pursuant to Section 13 (c), Rule 18 and Section 2 Rule 19 of the
COMELEC Rules of Procedure, the Joint Resolution has not yet attained finality for it to be implemented.
Notably, the seeming impropriety of the Resolution of the COMELEC En Banc dated June 29, 2007 has since been remedied by the promulgation of its
Resolution dated August 16, 2007, recognizing that it no longer has jurisdiction over the disqualification cases following the valid proclamation of
Limkaichong and her assumption of office as a Member of the House of Representatives.
V
Whether the Speaker of the House of Representatives may be compelled to prohibit Limkaichong from assuming her duties as a Member of the
House of Representatives.
Biraogo's contention was that De Venecia 73 should be stopped from entering Limkaichong's name in the Roll of Members of the House of
Representatives because he has no power to allow an alien to sit and continue to sit therein as it would amount to an unlawful exercise of his legal
authority. Moreover, Biraogo opposes Limkaichong's assumption of office in the House of Representatives since she is not qualified to sit therein,
being a Chinese citizen and, thus, disqualified by virtue of a final and executory judgment of the COMELEC En Banc. He relied on the COMELEC En
Banc Resolution dated June 29, 2007, which affirmed the COMELEC Second Division Joint Resolution dated May 17, 2007 disqualifying Limkaichong
from holding public office. He contended that the said Resolution dated June 29, 2007 is already final and executory; hence, it should be respected
pursuant to the principle of res judicata.
De Venecia, on the other hand, argued that he should not be faulted for honoring the proclamation of Limkaichong, because it had the hallmarks of
regularity, and he had no power to exclude any Member of the House of Representatives motu proprio. In their Comment on the petition,
respondents De Venecia, et al., contended that the enrollment of a Member in the Roll of Members of the House of Representatives and his/her
recognition as such becomes the ministerial duty of the Secretary General and the House of Representatives upon presentation by such Member of
a valid Certificate of Proclamation and Oath of Office. ECTIHa
Respondent Nograles, as De Venecia's substitute, filed a Memorandum dated July 16, 2008 stating that under the circumstances, the House of
Representatives, and its officials, are without recourse except to honor the validity of the proclamation of Limkaichong until the same is canceled,
revoked or nullified, and to continue to recognize her as the duly elected Representative of the First District of Negros Oriental until it is ordered by
this Court, as it was in Codilla, to recognize somebody else. He went on to state that after assumption by the Member-elect, or having acquired a
presumptively valid title to the office, the House of Representatives cannot, motu proprio, cancel, revoke, withdraw any recognition given to a sitting
Member or to "remove" his name from its roll, as such would amount to a removal of such Member from his office without due process of law.
Verily, it is only after a determination by the appropriate tribunal (as in this case, the HRET), pursuant to a final and executory order, that the
Member does not have a right to the office (i.e., not being a duly elected Member), that the House of Representatives is directed to exclude the said
Member.
Their contentions are meritorious. The unseating of a Member of the House of Representatives should be exercised with great caution and after the
proper proceedings for the ouster has been validly completed. For to arbitrarily unseat someone, who obtained the highest number of votes in the
elections, and during the pendency of the proceedings determining one's qualification or disqualification, would amount to disenfranchising the
electorate in whom sovereignty resides. 74
WHEREFORE, premises considered, the petition in G.R. Nos. 178831-32 is GRANTED and the Joint Resolution of the COMELEC Second Division dated
May 17, 2007 in SPA Nos. 07-247 and 07-248 is REVERSED and SET ASIDE. All the other petitions (G.R. Nos. 179120, 179132-33, 179240-41) are
hereby DISMISSED.
SO ORDERED.
Puno, C.J., Quisumbing, Ynares-Santiago, Carpio, Corona, Carpio-Morales, Tinga, Chico-Nazario, Nachura, Leonardo-de Castro and Brion, JJ., concur.
Austria-Martinez, J., is on leave.
Velasco, Jr., J., Pls. see dissenting opinion.
||| (Limkaichong v. Commission on Elections, G.R. Nos. 178831-32, 179120, 179132-33, 179240-41, [April 1, 2009], 601 PHIL 751-794)
EN BANC
[G.R. Nos. 178831-32. July 30, 2009.]
JOCELYN SY LIMKAICHONG, petitioner, vs. COMMISSION ON ELECTIONS, NAPOLEON N. CAMERO and RENALD F.
VILLANDO, respondents.
[G.R. No. 179120. July 30, 2009.]
LOUIS C. BIRAOGO, petitioner, vs. HON. PROSPERO NOGRALES, Speaker of the House of Representatives of the Congress of
the Philippines, and JOCELYN SY LIMKAICHONG, respondents.
[G.R. Nos. 179132-33. July 30, 2009.]
OLIVIA P. PARAS, petitioner, vs. HON. PROSPERO NOGRALES, in his capacity as Speaker of the House of Representatives;
HON. ROBERTO NAZARENO, in his capacity as Secretary General of the House of Representatives; HON. RHODORA SEVILLA,
in her capacity as Deputy Secretary General for Finance of the House of Representatives; THE COMMISSION ON ELECTIONS
and JOCELYN SY LIMKAICHONG, respondents.
[G.R. Nos. 179240-41. July 30, 2009.]
RENALD F. VILLANDO, petitioner, vs. COMMISSION ON ELECTIONS and JOCELYN SY LIMKAICHONG, respondents.
RESOLUTION
PERALTA, J p:
The instant motion with prayer for oral argument filed by Louis C. Biraogo, petitioner in G.R. No. 179120, seeks a reconsideration of the Court's April
1, 2009 Decision, which granted Jocelyn D. Sy Limkaichong's petition for certiorari in G.R. Nos. 178831-32. The Court dismissed all the other petitions,
including Biraogo's petition, and reversed the Joint Resolution of the Commission on Election's (COMELEC) Second Division dated May 17, 2007 in
SPA Nos. 07-247 and 07-248 disqualifying Limkaichong from running as a congressional candidate in the First District of Negros Oriental due to lack of
citizenship requirement. SHDAEC
Biraogo prefaced his motion by stating that justice and constitutionalism must remain entrenched in Philippine case law. To achieve this end, he
maintained that the Court should reconsider its April 1, 2009 Decision. He also prayed for an oral argument, which he posited, would help the Court
in the just and proper disposition of the pending incident.
After an assiduous review of the motion for reconsideration, we resolve that the same should be denied for lack of merit.
Most of the arguments advanced by Biraogo are a mere rehash of his previous arguments, which we have all considered and found without merit in
the Decision dated April 1, 2009. Nonetheless, in order to lay to rest once and for all Biraogo's misgivings, we shall discuss only the relevant issues
and revalidate our Decision by ruling on his motion as follows:
The core issue in the consolidated petitions is the qualification of Limkaichong to run for, be elected to, and assume and discharge, the position of
Representative for the First District of Negros Oriental. The contention of the parties who sought her disqualification is that she is not a natural-born
citizen, hence, she lacks the citizenship requirement in Section 6, 1 Article VI of the 1987 Constitution. In the election that ensued, she was voted for
by the constituents of Negros Oriental and garnered the highest votes. She was eventually proclaimed as the winner and has since performed her
duties and responsibilities as Member of the House of Representatives.
Indeed, the citizenship requirement was enshrined in our Constitution in order to ensure that our people and country do not end up being governed
by aliens. 2 With this principle in mind, we have said in Aquino v. COMELEC 3 that if one of the essential qualifications for running for membership in
the House of Representatives is lacking, then not even the will of a majority or plurality of the voters would substitute for a requirement mandated
by the fundamental law itself. Hence assuming, time constraints notwithstanding, and after proper proceedings before the proper tribunal be had,
that Limkaichong would prove to be an alien, the court of justice would tilt against her favor and would not sanction such an imperfection in her
qualification to hold office. But, first things first.
The proponents against Limkaichong's qualification stated that she is not a natural-born citizen because her parents were Chinese citizens at the time
of her birth. They went on to claim that the proceedings for the naturalization of Julio Ong Sy, her father, never attained finality due to procedural
and substantial defects.
In our Decision, We held that:
However, in assailing the citizenship of the father, the proper proceeding should be in accordance with Section 18
of Commonwealth Act No. 473 which provides that:
Sec. 18.Cancellation of Naturalization Certificate Issued. — Upon motion made in the proper proceedings by the
Solicitor General or his representative, or by the proper provincial fiscal, the competent judge may cancel the
naturalization certificate issued and its registration in the Civil Register:
1.If it is shown that said naturalization certificate was obtained fraudulently or illegally;
2.If the person naturalized shall, within five years next following the issuance of said naturalization
certificate, return to his native country or to some foreign country and establish his permanent
residence there: Provided, That the fact of the person naturalized remaining more than one year
in his native country or the country of his former nationality, or two years in any other foreign
country, shall be considered as prima facie evidence of his intention of taking up his permanent
residence in the same;
3.If the petition was made on an invalid declaration of intention;
4.If it is shown that the minor children of the person naturalized failed to graduate from a public or private
high school recognized by the Office of Private Education [now Bureau of Private Schools] of the
Philippines, where Philippine history, government or civics are taught as part of the school
curriculum, through the fault of their parents either by neglecting to support them or by
transferring them to another school or schools. A certified copy of the decree canceling the
naturalization certificate shall be forwarded by the Clerk of Court of the Department of Interior
[now Office of the President] and the Bureau of Justice [now Office of the Solicitor General];
5.If it is shown that the naturalized citizen has allowed himself to be used as a dummy in violation of the
constitutional or legal provisions requiring Philippine citizenship as a requisite for the exercise,
use or enjoyment of a right, franchise or privilege. (Emphasis supplied) EHITaS
As early as the case of Queto v. Catolico, where the Court of First Instance judge motu propio and not in the proper
denaturalization proceedings called to court various grantees of certificates of naturalization (who had already taken their
oaths of allegiance) and cancelled their certificates of naturalization due to procedural infirmities, the Court held that:
. . . It may be true that, as alleged by said respondents, that the proceedings for naturalization were tainted with
certain infirmities, fatal or otherwise, but that is beside the point in this case. The jurisdiction of the court to inquire
into and rule upon such infirmities must be properly invoked in accordance with the procedure laid down by law.
Such procedure is the cancellation of the naturalization certificate. [Section 1(5), Commonwealth Act No. 63], in the
manner fixed in Section 18 ofCommonwealth Act No. 473, hereinbefore quoted, namely, "upon motion made in the
proper proceedings by the Solicitor General or his representatives, or by the proper provincial fiscal". In other words,
the initiative must come from these officers, presumably after previous investigation in each particular case.
(Emphasis supplied)
Clearly, under law and jurisprudence, it is the State, through its representatives designated by statute, that may question the
illegally or invalidly procured certificate of naturalization in the appropriate denaturalization proceedings. It is plainly not a
matter that may be raised by private persons in an election case involving the naturalized citizen's descendant.
Accordingly, it is not enough that one's qualification, or lack of it, to hold an office requiring one to be a natural-born citizen, be attacked and
questioned before any tribunal or government institution. Proper proceedings must be strictly followed by the proper officers under the law. Hence,
in seeking Limkaichong's disqualification on account of her citizenship, the rudiments of fair play and due process must be observed, for in doing so,
she is not only deprived of the right to hold office as a Member of the House of Representative but her constituents would also be deprived of a
leader in whom they have put their trust on through their votes. The obvious rationale behind the foregoing ruling is that in voting for a candidate
who has not been disqualified by final judgment during the election day, the people voted for her bona fide, without any intention to misapply their
franchise, and in the honest belief that the candidate was then qualified to be the person to whom they would entrust the exercise of the powers of
government. 4
These precepts, notwithstanding, Biraogo remained firm in his belief that this Court erred in its Decision and that the COMELEC Joint Resolution
dated May 17, 2007 disqualifying Limkaichong should have been affirmed. He even went to a great extent of giving a dichotomy of the said Joint
Resolution by stating that it was composed of two parts, the first part of which is the substantive part, and the second, pertains to the injunctive part.
For this purpose, the dispositive portion of the said COMELEC Joint Resolution is reproduced below:
WHEREFORE, the Petitions are GRANTED and Jocelyn D. Sy-Limkaichong is declared as DISQUALIFIED from her candidacy for
Representative of the First District of Negros Oriental.
The Provincial Supervisor of the Commission on Elections of Negros Oriental is hereby directed to strike out the name JOCELYN
SY-LIMKAICHONG from the list of eligible candidates for the said position, and the concerned Board of Canvassers is hereby
directed to hold and/or suspend the proclamation of JOCELYN SY-LIMKAICHONG as winning candidate, if any, until this decision
has become final.
SO ORDERED. 5
Biraogo maintained that the Motion for Reconsideration filed by Limkaichong suspended only the execution of the substantive relief or the first part
of the above-quoted COMELEC Joint Resolution. However, it did not suspend the execution of the injunctive part and, accordingly, the Provincial
Supervisor of the COMELEC should not have proceeded with Limkaichong's proclamation as the winning candidate in the elections.
His argument has no leg to stand on. We cannot take a decision or resolution on a piece-meal basis and apply only that part which is seemingly
beneficial to one's cause and discard the prejudicial part which, obviously, would just be a hindrance in advancing one's stance or interests. Besides,
the COMELEC Joint Resolution which Biraogo dichotomized was effectively suspended when Limkaichong timely filed her Motion for Reconsideration
pursuant to Section 13 (c), 6 Rule 18 and Section 2, 7 Rule 19 of the COMELEC Rules of Procedure. Hence, it cannot as yet be implemented for not
having attained its finality.
Nevertheless, events have already transpired after the COMELEC has rendered its Joint Resolution. Limkaichong was proclaimed by the Provincial
Board of Canvassers, she had taken her oath of office, and she was allowed to officially assume the office on July 23, 2007. Accordingly, we ruled in
our April 1, 2009 Decision that the House of Representatives Electoral Tribunal (HRET), and no longer the COMELEC, should now assume jurisdiction
over the disqualification cases. Pertinently, we held:
. . . The Court has invariably held that once a winning candidate has been proclaimed, taken his oath, and assumed office as a
Member of the House of Representatives, the COMELEC's jurisdiction over election contests relating to his election, returns,
and qualifications ends, and the HRET's own jurisdiction begins. 8 It follows then that the proclamation of a winning
candidate divests the COMELEC of its jurisdiction over matters pending before it at the time of the proclamation. The party
questioning his qualification should now present his case in a proper proceeding before the HRET, the constitutionally
mandated tribunal to hear and decide a case involving a Member of the House of Representatives with respect to the latter's
election, returns and qualifications. The use of the word "sole" in Section 17, Article VI of the Constitution and in Section
2509 of the OEC underscores the exclusivity of the Electoral Tribunals' jurisdiction over election contests relating to its
members. 10
Section 17, Article VI of the 1987 Constitution provides:
Sec. 17.The Senate and the House of Representatives shall each have an Electoral Tribunal which shall be the sole
judge of all contests relating to the election, returns, and qualifications of their respective Members. Each Electoral
Tribunal shall be composed of nine Members, three of whom shall be Justices of the Supreme Court to be designated
by the Chief Justice, and the remaining six shall be Members of the Senate or the House of Representatives, as the
case may be, who shall be chosen on the basis of proportional representation from the political parties and the
parties or organizations registered under the party-list system represented therein. The senior Justice in the Electoral
Tribunal shall be its Chairman. acCTSE
xxx xxx xxx
Petitioners (in G.R. Nos. 179120, 179132-33, and 179240-41) steadfastly maintained that Limkaichong's proclamation was
tainted with irregularity, which will effectively prevent the HRET from acquiring jurisdiction.
The fact that the proclamation of the winning candidate, as in this case, was alleged to have been tainted with irregularity does
not divest the HRET of its jurisdiction. 11 The Court has shed light on this in the case of Vinzons-Chato, 12 to the effect that:
In the present case, it is not disputed that respondent Unico has already been proclaimed and taken his oath of office
as a Member of the House of Representatives (Thirteenth Congress); hence, the COMELEC correctly ruled that it had
already lost jurisdiction over petitioner Chato's petition. The issues raised by petitioner Chato essentially relate to the
canvassing of returns and alleged invalidity of respondent Unico's proclamation. These are matters that are best
addressed to the sound judgment and discretion of the HRET. Significantly, the allegation that respondent Unico's
proclamation is null and void does not divest the HRET of its jurisdiction:
. . . [I]n an electoral contest where the validity of the proclamation of a winning candidate who has taken his oath of
office and assumed his post as congressman is raised, that issue is best addressed to the HRET. The reason for this
ruling is self-evident, for it avoids duplicity of proceedings and a clash of jurisdiction between constitutional bodies,
with due regard to the people's mandate. SHCaDA
Further, for the Court to take cognizance of petitioner Chato's election protest against respondent Unico would be to
usurp the constitutionally mandated functions of the HRET.
In fine, any allegations as to the invalidity of the proclamation will not prevent the HRET from assuming jurisdiction over all
matters essential to a member's qualification to sit in the House of Representatives.
The 1998 HRET Rules, as amended, provide for the manner of filing either an election protest or a petition for quo warranto against a Member of the
House of Representatives. In our Decision, we ruled that the ten-day prescriptive period under the 1998 HRET Rules does not apply to disqualification
based on citizenship, because qualifications for public office are continuing requirements and must be possessed not only at the time of appointment
or election or assumption of office but during the officer's entire tenure. Once any of the required qualifications is lost, his title may be seasonably
challenged. 13 Accordingly, the 1987 Constitution requires that Members of the House of Representatives must be natural-born citizens not only at
the time of their election but during their entire tenure. Being a continuing requirement, one who assails a member's citizenship or lack of it may still
question the same at any time, the ten-day prescriptive period notwithstanding.
In fine, we hold that Biraogo had not successfully convinced us to reconsider our Decision and grant his motion for reconsideration.
In a last-ditched attempt to muddle the issues, Biraogo observed that the Decision dated April 1, 2009 is a complete turn-around from the ruling
embodied in the Decision written by Justice Ruben T. Reyes which, although unpromulgated, was nonetheless signed by fourteen (14) Associate
Justices and approved by the Court en banc on July 15, 2008. He decried the absence of an explanation in the Decision dated April 1, 2009 for the
said departure or turn-around.
Such a position deserves scant consideration.
The Court in Belac v. Commission on Elections, 14 held that a decision must not only be signed by the Justices who took part in the deliberation, but
must also be promulgated to be considered a Decision, to wit:
[A] true decision of the Court is the decision signed by the Justices and duly promulgated. Before that decision is so signed
and promulgated, there is no decision of the Court to speak of. The vote cast by a member of the Court after the deliberation
is always understood to be subject to confirmation at the time he has to sign the decision that is to be promulgated. The vote is
of no value if it is not thus confirmed by the Justice casting it. The purpose of this practice is apparent. Members of this Court,
even after they have cast their votes, wish to preserve their freedom of action till the last moment when they have to sign the
decision, so that they may take full advantage of what they may believe to be the best fruit of their most mature reflection and
deliberation. In consonance with this practice, before a decision is signed and promulgated, all opinions and conclusions
stated during and after the deliberation of the Court, remain in the breasts of the Justices, binding upon no one, not even
upon the Justices themselves. Of course, they may serve for determining what the opinion of the majority provisionally is and
for designating a member to prepare the decision of the Court, but in no way is that decision binding unless and until signed
and promulgated.
We add that at any time before promulgation, the ponencia may be changed by the ponente. Indeed, if any member of the
court who may have already signed it so desires, he may still withdraw his concurrence and register a qualification or dissent as
long as the decision has not yet been promulgated. A promulgation signifies that on the date it was made the judge or judges
who signed the decision continued to support it.
Thus, an unpromulgated decision is no decision at all. At the very least, they are part of the confidential internal deliberations of the Court which
must not be released to the public. A decision becomes binding only after it is validly promulgated. 15 Until such operative act occurs, there is really
no decision to speak of, even if some or all of the Justices have already affixed their signatures thereto. During the intervening period from the time
of signing until the promulgation of the decision, any one who took part in the deliberation and had signed the decision may, for a reason, validly
withdraw one's vote, thereby preserving one's freedom of action. ECaHSI
In sum, we hold that Biraogo's Motion for Reconsideration with Prayer for Oral Argument must be denied. This Court did not err in ruling that the
proper remedy of those who may assail Limkaichong's disqualification based on citizenship is to file before the HRET the proper petition at any time
during her incumbency.
WHEREFORE, the Motion for Reconsideration with Prayer for Oral Argument filed by petitioner Louis C. Biraogo in G.R. No. 179120 is DENIED with
FINALITY.
SO ORDERED.
Puno, C.J., Quisumbing, Ynares-Santiago, Carpio, Corona, Carpio Morales, Chico-Nazario, Nachura, Leonardo-de Castro and Bersamin, JJ., concur.
Velasco, Jr., J., dissents. I adopt my dissent to the April 1, 2009 Decision.
Brion, J., is on official leave.
||| (Limkaichong v. Commission on Elections, G.R. Nos. 178831-32, 179120, 179132-33, 179240-41, [July 30, 2009], 611 PHIL 817-831)
EN BANC
[G.R. No. 151914. July 31, 2002.]
TEODULO M. COQUILLA, petitioner, vs. THE HON. COMMISSION ON ELECTIONS and MR. NEIL M. ALVAREZ, respondents.
Franklin Delano M. Sacmar for petitioner.
The Solicitor General for public respondent.
Jonathan M. Agnes and Christoper L. Moscare for private respondent.
SYNOPSIS
Petitioner was born of Filipino parents in Oras, Eastern Samar. He joined the United States Navy and was subsequently naturalized as U.S. citizen.
Thereafter, he applied for repatriation and consequently took his oath as a citizen of the Philippines. Petitioner filed his certificate of candidacy
stating therein that he had been a resident of Oras, Eastern Samar for two years. Respondent sought the cancellation of petitioner's certificate of
candidacy on the ground that the latter had made a material misrepresentation therein by stating that he had been a resident of Oras for two years
when in truth he had resided therein for only six months since he took his oath as a citizen of the Philippines. The Second Division of the COMELEC
ordered the cancellation of petitioner's certificate of candidacy.
The Supreme Court ruled that petitioner lacked the requisite residency to qualify him for the mayorship of Oras, Eastern Samar. The term "residence"
is to be understood not in its common acceptation as referring to "dwelling" of "habitation," but rather to "domicile" or legal residence, that is "the
place where a party actually or constructively has his permanent home, where he, no matter where he may be found at any given time, eventually
intends to return and remain. A domicile of origin is acquired by every person at birth. It is usually the place where the child's parents reside and
continues until the same is abandoned by acquisition of a new domicile. In the case at bar, petitioner lost his domicile of origin in Oras by becoming a
U.S. citizen after enlisting in the U.S. Navy. From then on and until the time when he acquired Philippine citizenship, petitioner was an alien without
any right to reside in the Philippines save as the immigration laws may have allowed him to stay as a visitor or as a resident alien. HDATSI
SYLLABUS
1. POLITICAL LAW; ELECTION LAWS; COMELEC RULES OF PROCEDURE; MOTION FOR RECONSIDERATION; REGLEMENTARY PERIOD FOR FILING; CASE
AT BAR. — The five-day period for filing a motion for reconsideration under Rule 19, §2 should be counted from the receipt of the decision,
resolution, order, or ruling of the COMELEC Division. In this case, petitioner received a copy of the resolution of July 19, 2001 of the COMELEC's
Second Division on July 28, 2001. Five days later, on August 2, 2001, he filed his motion for reconsideration. On February 6, 2002, he received a copy
of the order, dated January 30, 2002, of the COMELEC en banc denying his motion for reconsideration. Five days later, on February 11, 2002, he filed
this petition for certiorari. There is no question, therefore, that petitioner's motion for reconsideration of the resolution of the COMELEC Second
Division, as well as his petition for certiorari to set aside of the order of the COMELEC en banc, was filed within the period provided for in Rule 19, §2
of the COMELEC Rules of Procedure and in Art. IX(A), §7 of the Constitution.
2. ID.; ID.; ID.; ID.; SUSPENDS THE RUNNING OF THE PERIOD TO ELEVATE THE MATTER TO THE SUPREME COURT, IF MOTION FOR RECONSIDERATION
IS NOT PRO FORMA; CASE AT BAR. — The motion for reconsideration was not pro forma and its filing did suspend the period for filing the petition
for certiorari in this case. The mere reiteration in a motion for reconsideration of the issues raised by the parties and passed upon by the court does
not make a motion pro forma; otherwise, the movant's remedy would not be a reconsideration of the decision but a new trial or some other remedy.
3. REMEDIAL LAW; ACTIONS; MOTION FOR RECONSIDERATION; WHEN CONSIDERED PRO FORMA. — [I]n the cases where a motion for
reconsideration was held to be pro forma, the motion was so held because (1) it was a second motion for reconsideration, or (2) it did not comply
with the rule that the motion must specify the findings and conclusions alleged to be contrary to law or not supported by the evidence, or (3) it failed
to substantiate the alleged errors, or (4) it merely alleged that the decision in question was contrary to law, or (5) the adverse party was not given
notice thereof. TcDIEH
4. POLITICAL LAW; ADMINISTRATIVE LAW; LOCAL GOVERNMENT CODE; ELECTIVE LOCAL OFFICIALS; QUALIFICATIONS; RESIDENCY REQUIREMENT;
RESIDENCE, DEFINED. — The term "residence" is to be understood not in its common acceptation as referring to "dwelling" or "habitation," but
rather to "domicile" or legal residence, that is, "the place where a party actually or constructively has his permanent home, where he, no matter
where he may be found at any given time, eventually intends to return and remain (animus manendi)." A domicile of origin is acquired by every
person at birth. It is usually the place where the child's parents reside and continues until the same is abandoned by acquisition of new domicile
(domicile of choice).
5. ID.; ID.; ID.; ID.; ID.; ID.; NOT ESTABLISHED IN CASE AT BAR. — In the case at bar, petitioner lost his domicile of origin in Oras by becoming a U.S.
citizen after enlisting in the U.S. Navy in 1965. From then on and until November 10, 2000, when he reacquired Philippine citizenship, petitioner was
an alien without any right to reside in the Philippines save as out immigration laws may have allowed him to stay as a visitor or as a resident alien.
6. ID.; ID.; ID.; ID.; ID.; CITIZENSHIP REQUIREMENT; MAY BE POSSESSED EVEN ON THE DAY THE CANDIDATE ASSUMES OFFICE. — [C]itizenship may be
possessed even on the day the candidate assumes office. But in the case of residency, as already noted, §39(a) of the Local Government Code
requires that the candidate must have been a resident of the municipality "for at least one (1) year immediately preceding the day of the election."
DECISION
MENDOZA, J p:
This is a petition for certiorari to set aside the resolution, 1 dated July 19, 2001, of the Second Division of the Commission on Elections (COMELEC),
ordering the cancellation of the certificate of candidacy of petitioner Teodulo M. Coquilla for the position of mayor of Oras, Eastern Samar in the May
14, 2001 elections and the order, dated January 30, 2002, of the COMELEC en banc denying petitioner's motion for reconsideration. acHDTE
The facts are as follows:
Petitioner Coquilla was born on February 17, 1938 of Filipino parents in Oras, Eastern Samar. He grew up and resided there until 1965, when he
joined the United States Navy. He was subsequently naturalized as a U.S. citizen. 2 From 1970 to 1973, petitioner thrice visited the Philippines while
on leave from the U.S. Navy. 3 Otherwise, even after his retirement from the U.S. Navy in 1985, he remained in the United States.
On October 15, 1998, petitioner came to the Philippines and took out a residence certificate, although he continued making several trips to the
United States, the last of which took place on July 6, 2000 and lasted until August 5, 2000. 4 Subsequently, petitioner applied for repatriation
under R.A. No. 8171 5 to the Special Committee on Naturalization. His application was approved on November 7, 2000, and, on November 10, 2000,
he took his oath as a citizen of the Philippines. Petitioner was issued Certificate of Repatriation No. 000737 on November 10, 2000 and Bureau of
Immigration Identification Certificate No. 115123 on November 13, 2000.
On November 21, 2000, petitioner applied for registration as a voter of Butnga, Oras, Eastern Samar. His application was approved by the Election
Registration Board on January 12, 2001. 6 On February 27, 2001, he filed his certificate of candidacy stating therein that he had been a resident of
Oras, Eastern Samar for "two (2) years." 7
On March 5, 2001, respondent Neil M. Alvarez, who was the incumbent mayor of Oras and who was running for reelection, sought the cancellation
of petitioner's certificate of candidacy on the ground that the latter had made a material misrepresentation in his certificate of candidacy by stating
that he had been a resident of Oras for two years when in truth he had resided therein for only about six months since November 10, 2000, when he
took his oath as a citizen of the Philippines.
The COMELEC was unable to render judgment on the case before the elections on May 14, 2001. Meanwhile, petitioner was voted for and received
the highest number of votes (6,131) against private respondent's 5,752 votes, or a margin of 379 votes. On May 17, 2001, petitioner was proclaimed
mayor of Oras by the Municipal Board of Canvassers. 8 He subsequently took his oath of office.
On July 19, 2001, the Second Division of the COMELEC granted private respondent's petition and ordered the cancellation of petitioner's certificate of
candidacy on the basis of the following findings:
Respondent's frequent or regular trips to the Philippines and stay in Oras, Eastern Samar after his retirement from the U.S.
Navy in 1985 cannot be considered as a waiver of his status as a permanent resident or immigrant . . . of the U.S.A. prior to
November 10, 2000 as would qualify him to acquire the status of residency for purposes of compliance with the one-year
residency requirement of Section 39(a) of the Local Government Code of 1991 in relation to Sections 65 and 68 of the Omnibus
Election Code. The one (1) year residency requirement contemplates of the actual residence of a Filipino citizen in the
constituency where he seeks to be elected.
All things considered, the number of years he claimed to have resided or stayed in Oras, Eastern Samar since 1985 as an
American citizen and permanent resident of the U.S.A. before November 10, 2000 when he reacquired his Philippine
citizenship by [repatriation] cannot be added to his actual residence thereat after November 10, 2000 until May 14, 2001 to
cure his deficiency in days, months, and year to allow or render him eligible to run for an elective office in the Philippines.
Under such circumstances, by whatever formula of computation used, respondent is short of the one-year residence
requirement before the May 14, 2001 elections. 9

Petitioner filed a motion for reconsideration, but his motion was denied by the COMELEC en banc on January 30, 2002. Hence this petition.
I.
Two questions must first be resolved before considering the merits of this case: (a) whether the 30-day period for appealing the resolution of the
COMELEC was suspended by the filing of a motion for reconsideration by petitioner and (b) whether the COMELEC retained jurisdiction to decide this
case notwithstanding the proclamation of petitioner.
A. With respect to the first question, private respondent contends that the petition in this case should be dismissed because it was filed late; that the
COMELEC en banc had denied petitioner's motion for reconsideration for being pro forma; and that, pursuant to Rule 19, §4 of the COMELEC Rules of
Procedure, the said motion did not suspend the running of the 30-day period for filing this petition. He points out that petitioner received a copy of
the resolution, dated July 19, 2001, of the COMELEC's Second Division on July 28, 2001, so that he had only until August 27, 2001 within which to file
this petition. Since the petition in this case was filed on February 11, 2002, the same should be considered as having been filed late and should be
dismissed.
Private respondent's contention has no merit.
Rule 19 of the COMELEC Rules of Procedure provides in pertinent parts:
Sec. 2. Period for Filing Motions for Reconsideration. — A motion to reconsider a decision, resolution, order, or ruling of a
Division shall be filed within five days from the promulgation thereof. Such motion, if not pro-forma, suspends the execution
for implementation of the decision, resolution, order, or ruling.
Sec. 4. Effect of Motion for Reconsideration on Period to Appeal. — A motion to reconsider a decision, resolution, order, or
ruling, when not pro-forma, suspends the running of the period to elevate the matter to the Supreme Court.
The five-day period for filing a motion for reconsideration under Rule 19, §2 should be counted from the receipt of the decision, resolution, order, or
ruling of the COMELEC Division. 10 In this case, petitioner received a copy of the resolution of July 19, 2001 of the COMELEC's Second Division on July
28, 2001. Five days later, on August 2, 2001, he filed his motion for reconsideration. On February 6, 2002, he received a copy of the order, dated
January 30, 2002, of the COMELEC en banc denying his motion for reconsideration. Five days later, on February 11, 2002, he filed this petition
for certiorari. There is no question, therefore, that petitioner's motion for reconsideration of the resolution of the COMELEC Second Division, as well
as his petition for certiorari to set aside of the order of the COMELEC en banc, was filed within the period provided for in Rule 19, §2 of the COMELEC
Rules of Procedure and in Art. IX (A), §7 of the Constitution. EAcTDH
It is contended, however, that petitioner's motion for reconsideration before the COMELEC en banc did not suspend the running of the period for
filing this petition because the motion was pro forma and, consequently, this petition should have been filed on or before August 27, 2001. It was
actually filed, however, only on February 11, 2002. Private respondent cites the finding of the COMELEC en banc that —
An incisive examination of the allegations in the Motion for Reconsideration shows that the same [are] a mere rehash of his
averments contained in his Verified Answer andMemorandum. Neither did respondent raise new matters that would
sufficiently warrant a reversal of the assailed resolution of the Second Division. This makes the said Motionpro forma. 11
We do not think this contention is correct. The motion for reconsideration was not pro forma and its filing did suspend the period for filing the
petition for certiorari in this case. The mere reiteration in a motion for reconsideration of the issues raised by the parties and passed upon by the
court does not make a motion pro forma; otherwise, the movant's remedy would not be a reconsideration of the decision but a new trial or some
other remedy. 12 But, as we have held in another case: 13
Among the ends to which a motion for reconsideration is addressed, one is precisely to convince the court that its ruling is
erroneous and improper, contrary to the law or the evidence; and in doing so, the movant has to dwell of necessity upon the
issues passed upon by the court. If a motion for reconsideration may not discuss these issues, the consequence would be that
after a decision is rendered, the losing party would be confined to filing only motions for reopening and new trial.
Indeed, in the cases where a motion for reconsideration was held to be pro forma, the motion was so held because (1) it was a second motion for
reconsideration, 14 or (2) it did not comply with the rule that the motion must specify the findings and conclusions alleged to be contrary to law or
not supported by the evidence, 15 or (3) it failed to substantiate the alleged errors, 16 or (4) it merely alleged that the decision in question was
contrary to law, 17 or (5) the adverse party was not given notice thereof. 18 The 16-page motion for reconsideration filed by petitioner in the
COMELEC en banc suffers from none of the foregoing defects, and it was error for the COMELEC en banc to rule that petitioner's motion for
reconsideration was pro forma because the allegations raised therein are a mere "rehash" of his earlier pleadings or did not raise "new matters."
Hence, the filing of the motion suspended the running of the 30-day period to file the petition in this case, which, as earlier shown, was done within
the reglementary period provided by law.
B. As stated before, the COMELEC failed to resolve private respondent's petition for cancellation of petitioner's certificate of candidacy before the
elections on May 14, 2001. In the meantime, the votes were canvassed and petitioner was proclaimed elected with a margin of 379 votes over
private respondent. Did the COMELEC thereby lose authority to act on the petition filed by private respondent?
R.A. No. 6646 provides:
SECTION 6. . Effect of Disqualification Case. — Any candidate who has been declared by final judgment to be disqualified shall
not be voted for, and the votes cast for him shall not be counted. If for any reason a candidate is not declared by final
judgment before an election to be disqualified and he is voted for and receives the winning number of votes in such election, the
Court or Commission shall continue with the trial and hearing of the action, inquiry, or protest and, upon motion of the
complainant or any intervenor, may during the pendency thereof order the suspension of the proclamation of such
candidate whenever the evidence of his guilt is strong. (Emphasis added)
SECTION 7 . Petition to Deny Due Course To or Cancel a Certificate of Candidacy. — The procedure hereinabove provided shall
apply to petitions to deny due course to or cancel a certificate of candidacy as provided in Section 78 of Batas Pambansa Blg.
881.
The rule then is that candidates who are disqualified by final judgment before the election shall not be voted for and the votes cast for them shall not
be counted. But those against whom no final judgment of disqualification had been rendered may be voted for and proclaimed, unless, on motion of
the complainant, the COMELEC suspends their proclamation because the grounds for their disqualification or cancellation of their certificates of
candidacy are strong. Meanwhile, the proceedings for disqualification of candidates or for the cancellation or denial of certificates of candidacy,
which have been begun before the elections, should continue even after such elections and proclamation of the winners. In Abella v.
COMELEC 19 and Salcedo II v. COMELEC, 20 the candidates whose certificates of candidacy were the subject of petitions for cancellation were voted
for and, having received the highest number of votes, were duly proclaimed winners. This Court, in the first case, affirmed and, in the second,
reversed the decisions of the COMELEC rendered after the proclamation of candidates, not on the ground that the latter had been divested of
jurisdiction upon the candidates' proclamation but on the merits.
II.
On the merits, the question is whether petitioner had been a resident of Oras, Eastern Samar at least one (1) year before the elections held on May
14, 2001 as he represented in his certificate of candidacy. We find that he had not.
First, §39(a) of the Local Government Code (R.A No. 7160) provides:
Qualifications. — (a) An elective local official must be a citizen of the Philippines; a registered voter in the barangay,
municipality, city, or province or, in the case of a member of the sangguniang panlalawigan, sangguniang
panlungsod, or sangguniang bayan, the district where he intends to be elected; a resident therein for at least one (1) year
immediately preceding the day of the election; and able to read and write Filipino or any other local language or dialect.
(Emphasis supplied)
The term "residence" is to be understood not in its common acceptation as referring to "dwelling" or "habitation," 21 but rather to "domicile" or
legal residence, 22 that is, "the place where a party actually or constructively has his permanent home, where he, no matter where he may be found
at any given time, eventually intends to return and remain (animus manendi)." 23 A domicile of origin is acquired by every person at birth. It is
usually the place where the child's parents reside and continues until the same is abandoned by acquisition of new domicile (domicile of choice). 24
In the case at bar, petitioner lost his domicile of origin in Oras by becoming a U.S. citizen after enlisting in the U.S. Navy in 1965. From then on and
until November 10, 2000, when he reacquired Philippine citizenship, petitioner was an alien without any right to reside in the Philippines save as our
immigration laws may have allowed him to stay as a visitor or as a resident alien.

Indeed, residence in the United States is a requirement for naturalization as a U.S. citizen. Title 8, §1427(a) of the United States Code provides:
Requirements of naturalization. — Residence
(a) No person, except as otherwise provided in this subchapter, shall be naturalized unless such applicant, (1)
immediately preceding the date of filing his application for naturalization has resided continuously, after being
lawfully admitted for permanent residence, within the United States for at least five years and during the five years
immediately preceding the date of filing his petition has been physically present therein for periods totaling at least
half of that time, and who has resided within the State or within the district of the Service in the United States in
which the applicant filed the application for at least three months, (2) has resided continuously within the United
States from the date of the application up to the time of admission to citizenship, and (3) during all the period
referred to in this subsection has been and still is a person of good moral character, attached to the principles of the
Constitution of the United States, and well disposed to the good order and happiness of the United States. (Emphasis
added)
In Caasi v. Court of Appeals, 25 this Court ruled that immigration to the United States by virtue of a "greencard," which entitles one to reside
permanently in that country, constitutes abandonment of domicile in the Philippines. With more reason then does naturalization in a foreign
country result in an abandonment of domicile in the Philippines.
Nor can petitioner contend that he was "compelled to adopt American citizenship" only by reason of his service in the U.S. armed forces. 26 It is
noteworthy that petitioner was repatriated not under R.A. No. 2630, which applies to the repatriation of those who lost their Philippine citizenship
by accepting commission in the Armed Forces of the United States, but under R.A. No. 8171, which, as earlier mentioned, provides for the
repatriation of, among others, natural-born Filipinos who lost their citizenship on account of political or economic necessity. In any event, the fact is
that, by having been naturalized abroad, he lost his Philippine citizenship and with it his residence in the Philippines. Until his reacquisition of
Philippine citizenship on November 10, 2000, petitioner did not reacquire his legal residence in this country. cSEaDA
Second, it is not true, as petitioner contends, that he reestablished residence in this country in 1998 when he came back to prepare for the mayoralty
elections of Oras by securing a Community Tax Certificate in that year and by "constantly declaring" to his townmates of his intention to seek
repatriation and run for mayor in the May 14, 2001 elections. 27 The status of being an alien and a non-resident can be waived either separately,
when one acquires the status of a resident alien before acquiring Philippine citizenship, or at the same time when one acquires Philippine
citizenship. As an alien, an individual may obtain an immigrant visa under §13 28 of the Philippine Immigration Act of 1948 and an Immigrant
Certificate of Residence (ICR) 29 and thus waive his status as a non-resident. On the other hand, he may acquire Philippine citizenship by
naturalization under C.A. No. 473, as amended, or, if he is a former Philippine national, he may reacquire Philippine citizenship by repatriation or by
an act of Congress, 30 in which case he waives not only his status as an alien but also his status as a non-resident alien.
In the case at bar, the only evidence of petitioner's status when he entered the country on October 15, 1998, December 20, 1998, October 16, 1999,
and June 23, 2000 is the statement "Philippine Immigration [-] Balikbayan" in his 1998-2008 U.S. passport. As for his entry on August 5, 2000, the
stamp bore the added inscription "good for one year stay." 31 Under §2 of R.A. No. 6768 (An Act Instituting a Balikbayan Program), the
term balikbayan includes a former Filipino citizen who had been naturalized in a foreign country and comes or returns to the Philippines and, if so, he
is entitled, among others, to a "visa-free entry to the Philippines for a period of one (1) year" (§3(c)). It would appear then that when petitioner
entered the country on the dates in question, he did so as a visa-free balikbayan visitor whose stay as such was valid for one year only. Hence,
petitioner can only be held to have waived his status as an alien and as a non-resident only on November 10, 2000 upon taking his oath as a citizen of
the Philippines under R.A. No. 8171. 32 He lacked the requisite residency to qualify him for the mayorship of Oras, Eastern Samar.
Petitioner invokes the ruling in Frivaldo v. Commission on Elections 33 in support of his contention that the residency requirement in §39(a) of the
Local Government Code includes the residency of one who is not a citizen of the Philippines. Residency, however, was not an issue in that case and
this Court did not make any ruling on the issue now at bar. The question in Frivaldo was whether petitioner, who took his oath of repatriation on the
same day that his term as governor of Sorsogon began on June 30, 1995, complied with the citizenship requirement under §39(a). It was held that he
had, because citizenship may be possessed even on the day the candidate assumes office. But in the case of residency, as already noted, §39(a) of
the Local Government Code requires that the candidate must have been a resident of the municipality "for at least one (1) year immediately
preceding the day of the election."
Nor can petitioner invoke this Court's ruling in Bengzon III v. House of Representatives Electoral Tribunal. 34 What the Court held in that case was
that, upon repatriation, a former natural-born Filipino is deemed to have recovered his original status as a natural-born citizen.
Third, petitioner nonetheless says that his registration as a voter of Butnga, Oras, Eastern Samar in January 2001 is conclusive of his residency as a
candidate because §117 of theOmnibus Election Code requires that a voter must have resided in the Philippines for at least one year and in the city
or municipality wherein he proposes to vote for at least six months immediately preceding the election. As held in Nuval v. Guray, 35 however,
registration as a voter does not bar the filing of a subsequent case questioning a candidate's lack of residency.
Petitioner's invocation of the liberal interpretation of election laws cannot avail him any. As held in Aquino v. Commission on Elections: 36
A democratic government is necessarily a government of laws. In a republican government those laws are themselves ordained
by the people. Through their representatives, they dictate the qualifications necessary for service in government positions. And
as petitioner clearly lacks one of the essential qualifications for running for membership in the House of Representatives, not
even the will of a majority or plurality of the voters of the Second District of Makati City would substitute for a requirement
mandated by the fundamental law itself.
Fourth, petitioner was not denied due process because the COMELEC failed to act on his motion to be allowed to present evidence. Under §5(d), in
relation to §7, of R.A. No. 6646 (Electoral Reforms Law of 1987), proceedings for denial or cancellation of a certificate of candidacy are summary in
nature. The holding of a formal hearing is thus not de rigeur. In any event, petitioner cannot claim denial of the right to be heard since he filed a
Verified Answer, a Memorandum and a Manifestation, all dated March 19, 2001, before the COMELEC in which he submitted documents relied by
him in this petition, which, contrary to petitioner's claim, are complete and intact in the records. IaHDcT
III.
The statement in petitioner's certificate of candidacy that he had been a resident of Oras, Eastern Samar for "two years" at the time he filed such
certificate is not true. The question is whether the COMELEC was justified in ordering the cancellation of his certificate of candidacy for this reason.
We hold that it was. Petitioner made a false representation of a material fact in his certificate of candidacy, thus rendering such certificate liable to
cancellation. The Omnibus Election Code provides:
SEC. 74. Contents of certificate of candidacy. — The certificate of candidacy shall state that the person filing it is announcing his
candidacy for the office stated therein and that he is eligible for said office; if for Member of the Batasang Pambansa, the
province, including its component cities, highly urbanized city or district or sector which he seeks to represent; the political
party to which he belongs; civil status; his date of birth; residence; his post office address for all election purposes; his
profession or occupation; that he will support and defend the Constitution of the Philippines and will maintain true faith and
allegiance thereto; that he will obey the laws, legal orders, and decrees promulgated by the duly constituted authorities; that
he is not a permanent resident or immigrant to a foreign country; that the obligation imposed by his oath is assumed
voluntarily, without mental reservation or purpose of evasion; and that the facts stated in the certificate of candidacy are true
to the best of his knowledge.
SEC. 78. Petition to deny due course to or cancel a certificate of candidacy. — A verified petition seeking to deny due course or
to cancel a certificate of candidacy may be filed by any person exclusively on the ground that any material representation
contained therein as required under Section 74 hereof is false. The petition may be filed at any time not later than twenty-five
days from the time of the filing of the certificate of candidacy and shall be decided, after due notice and hearing, not later than
fifteen days before the election.

Indeed, it has been held that a candidate's statement in her certificate of candidacy for the position of governor of Leyte that she was a resident of
Kananga, Leyte when this was not so 37 or that the candidate was a "natural-born" Filipino when in fact he had become an Australian
citizen 38 constitutes a ground for the cancellation of a certificate of candidacy. On the other hand, we held in Salcedo II v. COMELEC 39 that a
candidate who used her husband's family name even though their marriage was void was not guilty of misrepresentation concerning a material fact.
In the case at bar, what is involved is a false statement concerning a candidate's qualification for an office for which he filed the certificate of
candidacy. This is a misrepresentation of a material fact justifying the cancellation of petitioner's certificate of candidacy. The cancellation of
petitioner's certificate of candidacy in this case is thus fully justified.
WHEREFORE, the petition is DISMISSED and the resolution of the Second Division of the Commission on Elections, dated July 19, 2001, and the order,
dated January 30, 2002 of the Commission on Elections en banc are AFFIRMED.
SO ORDERED.
Davide, Jr., C.J., Bellosillo, Puno, Vitug, Kapunan, Panganiban, Quisumbing, Ynares-Santiago, Sandoval-Gutierrez, Carpio, Austria-
Martinez and Corona, JJ., concur.

||| (Coquilla v. Commission on Elections, G.R. No. 151914, [July 31, 2002], 434 PHIL 861-879)
EN BANC
[G.R. No. 120295. June 28, 1996.]
JUAN G. FRIVALDO, petitioner, vs. COMMISSION ON ELECTIONS, and RAUL R. LEE, respondents.
[G.R. No. 123755. June 28, 1996.]
RAUL R. LEE, petitioner, vs. COMMISSION ON ELECTIONS and JUAN G. FRIVALDO, respondents.
Sixto S. Brillantes, Jr., Juanito G. Arcilla and Teodoro M. Jumamil for Juan G. Frivaldo.
Felix Carao, Jr., Ferdinand Laguna, Gavinoo Barlin and Bernardo P. Fernandez for Raul Lee.
SYLLABUS
1. POLITICAL LAW; CITIZENSHIP; QUALIFICATION REQUIRED FOR ALL ELECTIVE LOCAL OFFICIALS. — The Local Government Code of 1991 [Republic Act
No. 7160] expressly requires Philippine citizenship as a qualification for elective local officials, including that of provincial governor. Philippine
citizenship is an indispensable requirement for holding an elective public office, and the purpose of the citizenship qualification is none other than to
ensure that no alien, i.e., no person owing allegiance to another nation, shall govern our people and our country or a unit of territory thereof. Now,
an official begins to govern or to discharge his functions only upon his proclamation and on the day the law mandates his term of office to begin. The
law intended CITIZENSHIP to be a qualification distinct from being a VOTER, even if being a voter presumes being a citizen first. It also stands to
reason that the voter requirement was included as another qualification (aside from "citizenship"), not to reiterate the need for nationality but to
require that the official be registered as a voter IN THE AREA OR TERRITORY he seeks to govern. i.e., the law states: "a registered voter in the
barangay, municipality, city, or province . . . where he intends to be elected." It should be emphasized that the Local Government Code requires an
elective official to be a registered voter. It does not require him to vote actually. Hence, registration — not the actual voting — is the core of this
"qualification". In other words, the law's purpose in this second requirement is to ensure that the prospective official is actually registered in the area
he seeks to govern — and not anywhere else.
2. ID.; ID.; MANNERS OF REACQUISITION UNDER PHILIPPINE LAWS. — Under Philippine law, citizenship may be reacquired by direct act of Congress,
by naturalization or by repatriation.
3. ID.; ID.; IMMIGRATION LAW; P.D. 725 CREATED A RIGHT AND REMEDY TO REACQUIRE PHILIPPINE CITIZENSHIP NOT ONLY FOR THE BENEFIT OF
FILIPINO WOMEN WHO MARRIED ALIENS BEFORE THE 1973 CONSTITUTION TOOK EFFECT BUT ALSO OF OTHER NATURAL BORN FILIPINO WHO LOST
THEIR PHILIPPINE CITIZENSHIP. — A reading of P.D. 725 immediately shows that it creates a new right, and also provides for a new remedy, thereby
filling certain voids in our laws. Thus, in its preamble, P.D. 725 expressly recognizes the plight of "many Filipino women (who) had lost their Philippine
citizenship by marriage to aliens" and who could not, under the existing law (C.A. No. 63, as amended) avail of repatriation until "after the death of
their husbands or the termination of their marital status" and who could neither be benefited by the 1973 Constitution's new provision allowing "a
Filipino woman who marries an alien to retain her Philippine citizenship . . ." because "such provision of the new Constitution does not apply to
Filipino women who had married aliens before said constitution took effect." Thus, P.D. 725 granted a new right to these women — the right to re-
acquire Filipino citizenship even during their marital coverture, which right did not exist prior to P.D. 725. On the other hand, said statute also
provide a new remedy and a new right in favor of other "natural born Filipinos who (had) lost their Philippine citizenship but now desire to re-acquire
Philippine citizenship," because prior to the promulgation of P.D. 725 such former Filipinos would have had to undergo the tedious and cumbersome
process of naturalization, but with the advent of P.D. 725 they could now re-acquire their Philippine citizenship under the simplified procedure of
repatriation.
4. ID.; ID.; REPATRIATION; REQUIREMENTS THEREOF ARE LESS TEDIOUS AND CUMBERSOME COMPARED TO NATURALIZATION. — The requirements
of repatriation under P.D. No. 725 are not difficult to comply with, nor are they tedious and cumbersome. In fact, P.D. 725 itself requires very little of
an applicant, and even the rules and regulations to implement the said decree were left to the Special Committee to promulgate. This is not unusual
since, unlike in naturalization where an alien covets a first-time entry into Philippine political life, in repatriation the applicant is a former natural-
born Filipino who is merely seeking to reacquire his previous citizenship.
5. ID.; ID.; ID.; WHEN MAY BE GIVEN A RETROACTIVE EFFECT; CASE AT BAR. — While it is true that the law was already in effect at the time
that Frivaldo became an American citizen, nevertheless, it is not only the law itself (P.D. 725) which is to be given retroactive effect, but even the
repatriation granted under said law to Frivaldo on June 30, 1995 is to be deemed to have retroactive to the date of his application therefor, August
17, 1994. The reason for this is simply that if, as in this case, it was the intent of the legislative authority that the law should apply to past events —
i.e., situations and transactions existing even before the law came into being — in order to benefit the greatest number of former Filipinos possible
thereby enabling them to enjoy and exercise the constitutionally guaranteed right of citizenship, and such legislative intention is to be given the
fullest effect and expression, then there is all the more reason to have the law apply in a retroactive or retrospective manner to situations, events and
transactions subsequent to the passage of such law. That is, the repatriation granted to Frivaldo on June 30, 1995 can and should be made to take
effect as of date of his application. As earlier mentioned, there is nothing in the law that would bar this or would show a contrary intention on the
part of the legislative authority; and there is no showing that damage or prejudice to anyone, or anything unjust or injurious would result from giving
retroactivity to his repatriation.
6. ID.; ID.; THE LOCAL GOVERNMENT CODE DOES NOT SPECIFY WHEN SUCH QUALIFICATION SHALL BE POSSESSED. — Under Sec. 39 of the Local
Government Code, it will be noted that the law does not specify any particular date or time when the candidate must possess citizenship, unlike that
for residence (which must consist of at least one year's residency immediately preceding the day of election) and age (at least twenty three years of
age on election day). Even from a literal (as distinguished from liberal) construction, it should be noted that Section 39 of the Local Government
Code speaks of "Qualifications" of 'ELECTIVE OFFICIALS', not of candidates. Literally, such qualifications unless otherwise expressly conditioned, as in
the case of age and residence — should thus be possessed when the "elective [or elected] official" begins to govern, i.e., at the time he is
proclaimed and at the start of his term. Section 39, par. (a) (thereof) speaks of "elective local official" while par. (b) to (f) refer to "candidates." The
citizenship requirement in the Local Government Code is to be possessed by an elective official at the latest as of the time he is proclaimed and at
the start of the term of office to which he has been elected.
7. STATUTORY CONSTRUCTION; REPEAL OF LAW; BY SUBSEQUENT ONES. — Laws are repealed only by subsequent ones [Art. 7, Civil Code of the
Philippines] and a repeal may be express or implied. It is obvious that no express repeal was made because then President Aquino in her
memorandum — based on the copy furnished us by Lee — did not categorically and/or impliedly state that PD 725 was being repealed or was being
rendered without any legal effect. In fact, she did not even mention it specifically by its number or text.
8. ID.; ID.; BY IMPLICATION IS NOT FAVORED; EXCEPTION. — It is a basic rule of statutory construction that repeals by implication are not favored. An
implied repeal will not be allowed "unless it is convincingly and unambiguously demonstrated that the two laws are clearly repugnant and patently
inconsistent that they cannot co-exist". The memorandum of then President Aquino cannot even be regarded as a legislative enactment, for not
every pronouncement of the Chief Executive even under the Transitory Provisions of the 1987 Constitution can nor should be regarded as an exercise
of her law-making powers. At best, it could be treated as an executive policy addressed to the Special Committee to hall the acceptance and
processing of applications for repatriation pending whatever "judgment the first Congress under the 1987 Constitution" might make. In other words,
the former President did not repeal P.D. 725 but left it to the first Congress — once created — to deal with the matter. If she had intended to repeal
such law, she should have unequivocally said so instead of referring the matter to Congress. The fact is she carefully couched her presidential
issuance in terms that clearly indicated the intention of "the present government, in the exercise of prudence and sound discretion" to leave the
matter of repeal to the new Congress. Any other interpretation of the said Presidential Memorandum, such as is now being proffered to the Court by
Lee, would visit unmitigated violence not only upon statutory construction but on common sense as well.
9. ID.; THE LAW PRESUMES THAT THE LAW MAKING-BODY INTENDED RIGHT AND JUSTICE TO PREVAIL. — In case of doubt in the interpretation or
application of laws, it is to be presumed that the lawmaking body intended right and justice to prevail. [Art. 10, Civil Code of the Philippines)
10. CIVIL LAW; RETROSPECTIVE OPERATIONS OF STATUTES; WHEN EFFECTIVE. — It is true that under Art. 4 of the Civil Code of the Philippines,
"(l)aws shall have no retroactive effect, unless the contrary is provided." But there are settled exceptions to this general rule, such as when the
statute is CURATIVE or REMEDIAL in nature or when it CREATES NEW RIGHTS. According to TOLENTINO, curative statutes are those which undertake
to cure errors and irregularities, thereby validating judicial or administrative proceedings, acts of public officers, or private deeds and contracts which
otherwise would not produce their intended consequences by reason of some statutory disability or failure to comply with some technical
requirement. They operate on conditions already existing, and are necessarily retroactive in operation. Agpalo, [Agpalo, Statutory Construction, 1990
ed., pp. 270-271] on the other hand, says that curative statutes are "healing acts . . . curing defects and adding to the means of enforcing existing
obligations . . . (and) are intended to supply defects, abridge superfluities in existing laws, and curb certain evils. . . . By their very nature, curative
statutes are retroactive . . . (and) reach back to past events to correct errors or irregularities and to render valid and effective attempted acts which
would be otherwise ineffective for the purpose the parties intended." On the other hand, remedial or procedural laws, i.e., those statutes relating to
remedies or modes of procedure, which do not create new or take away vested rights, but only operate in furtherance of the remedy or confirmation
of such rights, ordinarily do not come within the legal meaning of a retrospective law, nor within the general rule against the retrospective operation
of statutes.

11. POLITICAL LAW; OMNIBUS ELECTION CODE; PROVIDED REMEDY TO QUESTION INELIGIBILITY OF A CANDIDATE. — Section 253 of the Omnibus
Election Code gives any voter, presumably including the defeated candidate, the opportunity to question the ELIGIBILITY (or the disloyalty) of a
candidate. This is the only provision of the Code that authorizes a remedy on how to contest before the COMELEC an incumbent's ineligibility arising
from failure to meet the qualifications enumerated under Sec. 39 of the Local Government Code. Such remedy of Quo Warranto can be availed of
"within ten days after proclamation" of the winning candidate. Hence, it is only at such time that the issue of ineligibility may be taken cognizance of
by the Commission.
12. ID.; ID.; PETITION TO DENY DUE COURSE OR TO CANCEL A CERTIFICATE OF CANDIDACY UNDER SECTION 78 IS MERELY DIRECTORY. — Section 78
of the Omnibus Election Code is merely directory as Section 6 of R.A. No. 6646 authorizes the Commission to try and decide petitions for
disqualifications even after the elections.
13. ID.; ELECTIONS; THE RULE IS THE INELIGIBILITY OF A CANDIDATE RECEIVING MAJORITY VOTES DOES NOT ENTITLE THE ELIGIBLE CANDIDATE
RECEIVING THE NEXT HIGHEST NUMBER OF VOTES TO BE DECLARED ELECTED. — "The rule, therefore, is: the ineligibility of a candidate receiving
majority votes does not entitle the eligible candidate receiving the next highest number of votes to be declared elected. A minority or defeated
candidate cannot be deemed elected to the office."
14. ID.; ID.; ELECTORAL LAWS SHOULD BE LIBERALLY AND EQUITABLY CONSTRUED TO GIVE FULLEST EFFECT TO THE MANIFEST WILL OF OUR PEOPLE.
— This Court has time and again liberally and equitably construed the electoral laws of our country to give fullest effect to the manifest will of our
people, for in case of doubt, political laws must be interpreted to give life and spirit to the popular mandate freely expressed through the ballot.
Otherwise stated, legal niceties and technicalities cannot stand in the way of the sovereign will. Consistently, we have held: ". . . (L)aws governing
election contests must be liberally construed to the end that the will of the people in the choice of public officials may not be defeated by mere
technical objections." In any action involving the possibility of a reversal of the popular electoral choice, this Court must exert utmost effort to
resolve the issues in a manner that would give effect to the will of the majority, for it is merely sound public policy to cause elective offices to be
filled by those who are the choice of the majority. To successfully challenge a winning candidate's qualifications, the petitioner must clearly
demonstrate that the ineligibility is so patently antagonistic to constitutional and legal principles that overriding such ineligibility and thereby giving
effect to the apparent will of the people, would ultimately create greater prejudice to the very democratic institutions and juristic traditions that
our Constitution and laws so zealously protect and promote. The real essence of justice does not emanate from quibbling over patchwork legal
technicality. It proceeds from the spirit's gut consciousness of the dynamic role of law as a brick in the ultimate development of the social edifice.
Thus, the Court struggled against and eschewed the easy, legalistic, technical and sometimes harsh anachronisms of the law in order to evoke
substantial justice in the larger social context consistent with Frivaldo's unique situation approximating venerability in Philippine political life.
15. ID.; INTERNATIONAL LAW; A STATE DETERMINES ONLY THOSE WHO ARE ITS OWN CITIZENS — NOT WHO ARE THE CITIZENS OF OTHER
COUNTRIES. — Since our courts are charged only with the duty of the determining who are Philippine nationals, we cannot rule on the legal question
of who are or who are not Americans. It is basic in international law that a State determines ONLY those who are its own citizens — not who are the
citizens of other countries.
PUNO, J., Concurring Opinion:
1. POLITICAL LAW; DECLARATION OF PRINCIPLES AND STATE POLICIES; SOVEREIGNTY OF THE PEOPLE, CONSTRUED. — The sovereignty of our people
is the primary postulateof the 1987 Constitution. For this reason, it appears as the first in our declaration of principles and state policies. Thus,
Section 1 of Article II of our fundamental law proclaims that "[t]he Philippines is a democratic and republican State. Sovereignty resides in the people
and all government authority emanates from them." The same principle served as the bedrock of our 1973 and 1935 Constitutions. [The 1987
Constitution added the word "democratic" in the statement of the principle.] It is one of the few principles whose truth has been cherished by the
Americans as self-evident. Section 4, Article IV of the U.S. Constitution makes it a duty of the Federal government to guarantee to every state a
"republican form of government." With understandable fervor, the American authorities imposed republicanism as the cornerstone of our 1935
Constitution then being crafted by its Filipino farmers. Borne out of the 1986 people power EDSA revolution, our 1987 Constitution is more people-
oriented. Thus, Section 4 of Article II provides as a state policy that the prime duty of the Government is "to serve and protect the people." Section 1,
Article XI also provides that ". . . public officers . . . must at all times be accountable to the people . . . " Sections 15 and 16 of Article XIII define the
role and rights of people's organizations. Section 5(2) of Article XVI mandates that "[t]he state shall strengthen the patriotic spirit and nationalist
consciousness of the military, and respect for people's rights in the performance of their duty." And Section 2 of Article XVII provides that
"amendments to this Constitution may likewise be directly proposed by the people through initiative . . ." All these provisions and more are intended
to breathe more life to the sovereignty of our people. To be sure, the sovereignty of our people is not a kabalistic principle whose dimensions are
buried in mysticism. Its metes and bounds are familiar to the framers of our Constitutions. They knew that in its broadest sense, sovereignty is meant
to be supreme, the just summi imperu, the absolute right to govern. Former Dean Vicente Sinco states that an essential quality of sovereignty is legal
omnipotence, viz: "Legal theory establishes certain essential qualities inherent in the nature of sovereignty. The first is legal omnipotence. This
means that the sovereign is legally omnipotent and absolute in relation to other legal institutions. It has the power to determine exclusively its legal
competence. Its powers are original, not derivative. It is the sole judge of what it should do at any given time." Citing Barker, Principles of Social and
Political Theory, p. 59 (1952 ed.), he adds that a more amplified definition of sovereignty is that of "a final power of final legal adjustment of all legal
issues." The U.S. Supreme Court expressed the same thought in the landmark case of Yick Wo v. Hopkins, 118 U.S. 356, where it held that ". .
. sovereignty itself is, of course, not subject to law, for it is the author and source of law; but in our system, while sovereign powers are delegated to
the agencies of government, sovereignty itself remains with the people, by whom and for whom all government exists and acts."
2. ID.; ID.; REPRESENTATIVE DEMOCRACY DISTINGUISHED FROM PURE DEMOCRACY. — In our Constitution,the people established a representative
democracy as distinguished from a pure democracy. Justice Isagani Cruz explains ". . . A republic is a representative government, a government run
by and for the people. It is not a pure democracy where the people govern themselves directly. The essence of republicanism is representation and
renovation, the selection by the citizenry of a corps of public functionaries who derive their mandate for the people and act on their behalf, serving
for a limited period only, after which they are replaced or retained, at the option of their principal. Obviously, a republican government is a
responsible government whose officials hold and discharge their position as a public trust and shall, according to the Constitution, 'at all times be
accountable to the people' they are sworn to serve. The purpose of a republican government it is almost needless to state, is the promotion of the
common welfare according to the will of the people themselves."
3. ID.; ELECTION CASES; THE COURT SHOULD STRIVE TO ALIGN THE WILL OF THE LEGISLATURE WITH THE WILL OF THE SOVEREIGN PEOPLE. — In
election cases, we should strive to align the will of the legislature as expressed in its law with the will of the sovereign people as expressed in their
ballots. For law to reign, it must respect the will of the people. For in the eloquent prose of Mr. Justice Laurel, ". . . an enfranchised citizen is a particle
of popular sovereignty and is the ultimate source of established authority." (Moyav. del Fierro, 69 Phil. 199) The choice of the governed on who shall
be their governor merits the highest consideration by all agencies of government. In cases where the sovereignty of the people is at stake, we must
not only be legally right but also politically correct. We cannot fail by making the people succeed.
DAVIDE, JR., J., Dissenting Opinion"
1. POLITICAL LAW; LOCAL GOVERNMENT CODE OF 1991; ELECTION; WHEN THE PRESCRIBED QUALIFICATION SHALL BE POSSESSED. — Justice Davide
departs from the view in the ponencia is that Section 39 of the Local Government Code of 1991 does not specify the time when the citizenship
requirement must be met, and that being the case, then it suffices that citizenship be possessed upon commencement of the term of the office
involved. Section 39 actually prescribes the qualifications of elective local officials and not those of an elected local official. These adjectives are not
synonymous, as the ponencia seems to suggest, the first refers to the nature of the office, which requires the process of voting by the electorate
involved; while the second refers to a victorious candidate for an elective office. The section unquestionably refers to elective — not elected — local
officials. It falls under Title Two entitled ELECTIVE OFFICIALS; under Chapter 1 entitled Qualifications and Election; and paragraph (a) thereof begins
with the phrases "An elective local official," while paragraphs (b) to (f) thereof speak of candidates. It is thus obvious that Section 39 refers to no
other than the qualifications of candidates for elective local offices and their election. Hence, in no way may the section be construed to mean that
possession of qualifications should be reckoned from the commencement of the term of office of the elected candidate. It is not at all true that
Section 39 does not specify the time when the citizenship requirement must be possessed. I submit that the requirement must be satisfied, or that
Philippine citizenship must be possessed, not merely at the commencement of the term, but at an earlier time, the latest being election day itself.
Section 39 is not at all ambiguous nor uncertain that it meant this to be, as one basic qualification of an elective local official is that he be "A
REGISTERED VOTER IN THE BARANGAY, MUNICIPALITY, CITY OR PROVINCE . . . WHERE HE INTENDS TO VOTE." This simply means that he possesses all
the qualifications to exercise the right of suffrage. The fundamental qualification for the exercise of this sovereign right is the possession of Philippine
citizenship. No less than the Constitution makes it the first qualification.

2. ID.; ID.; ID.; AVAILABLE REMEDIES ON QUESTIONS OF INELIGIBILITY FOR PUBLIC OFFICE. — Quo warranto is not the sole remedy available to
question a candidate's ineligibility for public office. Section 78 of the Omnibus Election Code allows the filing of a petition to deny due course to or
cancel the certificate of candidacy on the ground that any material representation contained therein as required by Section 74, is false. Section 74, in
turn, requires that the person filing the certificate of candidacy must state inter alia, that he is eligible for the office, which means that he has all the
qualifications (including, of course, fulfilling the citizenship requirement) and none of the disqualifications as provided by law. The petitioner under
Section 78 may be filed at any time not later than 25 days from the filing of the certificate of candidacy. Rule 25 of the Revised COMELECrules of
procedure allows the filing of a petition for disqualification on the ground of failure to possess all the qualifications of a candidate as provided by the
Constitution or by existing laws, "any day after the last day for filing of certificates of candidacy but not later than the date of proclamation."
3. ID.; CITIZENSHIP; STEPS FOR THE REACQUISITION OF CITIZENSHIP BY REPATRIATION. — The steps to reacquire Philippine Citizenship by
repatriation under P.D. No. 725are: (1) filing the application; (2) action by the committee; and (3) taking of the oath of allegiance if the application is
approved. It is only UPON TAKING THE OATH OF ALLEGIANCE that the applicant is deemed ipso jure to have reacquired Philippine citizenship. If the
decree had intended the oath taking to retroact to the date of the filing of the application, then it should not have explicitly provided otherwise.
4. ID.; INTERNATIONAL LAW; STATELESSNESS, CONSTRUED. — Statelessness may be either de jure, which is the status of individuals stripped of their
nationality by their former government without having all opportunity to acquired another; or de facto, which is the status of individuals possessed
of a nationality whose country does not give them protection outside their own country, and who are commonly, albeit imprecisely, referred to as
refugees (JORGE R. COQUIA, et al., Conflict of Laws Cases, Materials and Comments, 1995, ed., 290). Under Chapter I, Article 1 of the United Nations
Convention Regarding the Status of Stateless Persons (Philippine Treaty Series, Compiled and Annotated by Haydee B. Yorac, vol. III, 363), a stateless
person is defined as "a person who is not considered as a national by any State under the operation of its law.
5. ID.; DECLARATION OF PRINCIPLES AND STATE POLICIES; DOCTRINE OF PEOPLE'S SOVEREIGNTY, CONSTRUED. — The doctrine of people's
sovereignty is founded on the principles of democracy and republicanism and refers exclusively to the sovereignty of the people of the Philippines.
Section 1 of Article II of the 1987 Constitution is quite clearon this. And the Preamble makes it clear when it solemnly opens it with a clause "We, the
sovereign Filipino people. . . " Thus, the sovereignty is an attribute of the Filipino peopleas one people, one body. That sovereign power of the Filipino
people cannot be fragmentized by looking at it as the supreme authority of the people of any of the political subdivisions to determine their own
destiny; neither can we convert and treat every fragment as the whole. In such a case, this Court would provide the formula for the division and
destruction of the State and render the Government ineffective and inutile.
DECISION
PANGANIBAN, J p:
The ultimate question posed before this Court in these twin cases is: Who should be declared the rightful governor of Sorsogon —
(i) Juan G. Frivaldo, who unquestionably obtained the highest number of votes in three successive elections but who was twice
declared by this Court to be disqualified to hold such office due to his alien citizenship, and who now claims to have re-assumed his lost
Philippine citizenship thru repatriation;
(ii) Raul R. Lee, who was the second placer in the canvass, but who claims that the votes cast in favor of Frivaldo should be considered
void; that the electorate should be deemed to have intentionally thrown away their ballots; and that legally, he secured the most number
of valid votes; or
(iii) The incumbent Vice-Governor, Oscar G. Deri, who obviously was not voted directly to the position of governor, but who according
to prevailing jurisprudence should take over the said post inasmuch as, by the ineligibility of Frivaldo, a "permanent vacancy in the contested
office has occurred"?
In ruling for Frivaldo, the Court lays down new doctrines on repatriation, clarifies/reiterates/amplifies existing
jurisprudence on citizenship and elections, and upholds the superiority of substantial justice over pure legalisms.
G.R. No. 123755
This is a special civil action under Rules 65 and 58 of the Rules of Court for certiorari and preliminary injunction to review and annul a
Resolution of the respondentCommission on Elections (Comelec), First Division, 1 promulgated on December 19, 1995 2 and another Resolution
of the Comelec en banc promulgated February 23, 1996 3 denying petitioner's motion for reconsideration.
The Facts
On March 20, 1995, private respondent Juan G. Frivaldo filed his Certificate of Candidacy for the office of Governor of Sorsogon in the
May 8, 1995 elections. On March 23, 1995, petitioner Raul R. Lee, another candidate, filed a petition 4 with the Comelec docketed as SPA No.
95-028 praying that Frivaldo "be disqualified from seeking or holding any public office or position by reason of not yet being a citizen of the
Philippines", and that his Certificate of Candidacy be cancelled. On May 1, 1995, the Second Division of the Comelec promulgated a Resolution 5
granting the petition with the following disposition: 6
"WHEREFORE, this Division resolves to GRANT the petition and declares that respondent is DISQUALIFIED to run for
the Office of Governor of Sorsogon on the ground that he is NOT a citizen of the Philippines. Accordingly, respondent's
certificate of candidacy is cancelled."
The Motion for Reconsideration filed by Frivaldo remained unacted upon until after the May 8, 1995 elections. So, his candidacy
continued and he was voted for during the elections held on said date. On May 11, 1995, the Comelec en banc 7 affirmed the aforementioned
Resolution of the Second Division.
The Provincial Board of Canvassers completed the canvass of the election returns and a Certificate of Votes 8 dated May 27, 1995 was
issued showing the following votes obtained by the candidates for the position of Governor of Sorsogon:
Antonio H. Escudero, Jr. 51,060
Juan G. Frivaldo 73,440
Raul R. Lee 53,304
Isagani P. Ocampo 1,925
On June 9, 1995, Lee filed in said SPA No. 95-028, a (supplemental) petition 9 praying for his proclamation as the duly-elected
Governor of Sorsogon.
In an order 10 dated June 21, 1995, but promulgated according to the petition "only on June 29, 1995," the Comelec en banc directed
"the Provincial Board of Canvassers of Sorsogon to reconvene for the purpose of proclaiming candidate Raul Lee as the winning gubernatorial
candidate in the province of Sorsogon on June 29, 1995 . . ." Accordingly, at 8:30 in the evening of June 30, 1995, Lee was proclaimed governor
of Sorsogon.
On July 6, 1995, Frivaldo filed with the Comelec a new petition, 11 docketed as SPC No. 95-317, praying for the annulment of the June
30, 1995 proclamation of Lee and for his own proclamation. He alleged that on June 30, 1995, at 2:00 in the afternoon, he took his oath of
allegiance as a citizen of the Philippines after "his petition for repatriation under P.D. 725 which he filed with the Special
Committee on Naturalization in September 1994 had been granted". As such, when "the said order (dated June 21, 1995) (of the Comelec) . . .
was released and received by Frivaldo on June 30, 1995 at 5:30 o'clock in the evening, there was no more legal impediment to the proclamation
(ofFrivaldo) as governor . . ." In the alternative, he averred that pursuant to the two cases of Labo vs. Comelec, 12 the Vice-Governor — not Lee
— should occupy said position of governor.
On December 19, 1995, the Comelec First Division promulgated the herein assailed Resolution 13 holding that Lee, "not having
garnered the highest number of votes," was not legally entitled to be proclaimed as duly-elected governor; and that Frivaldo, "having garnered
the highest number of votes, and . . . having reacquired his Filipino citizenship by repatriation on June 30, 1995 under the provisions
of Presidential Decree No. 725 . . . (is) qualified to hold the office of governor of Sorsogon"; thus:
"PREMISES CONSIDERED, the Commission (First Division), therefore RESOLVES to GRANT the Petition.
Consistent with the decisions of the Supreme Court, the proclamation of Raul R. Lee as Governor of Sorsogon is
hereby ordered annulled, being contrary to law, he not having garnered the highest number of votes to warrant his
proclamation.
Upon the finality of the annulment of the proclamation of Raul R. Lee, the Provincial Board of Canvassers is directed
to immediately reconvene and, on the basis of the completed canvass, proclaim petitioner Juan G. Frivaldo as the duly elected
Governor of Sorsogon having garnered the highest number of votes, and he having reacquired his Filipino citizenship by
repatriation on June 30, 1995 under the provisions of Presidential Decree No. 725 and, thus, qualified to hold the office of
Governor of Sorsogon.
Conformably with Section 260 of the Omnibus Election Code (B.P. Blg. 881), the Clerk of the Commission is directed
to notify His Excellency the President of the Philippines, and the Secretary of the Sangguniang Panlalawigan of the Province of
Sorsogon of this resolution immediately upon the due implementation thereof."
On December 26, 1995, Lee filed a motion for reconsideration which was denied by the Comelec en banc in its Resolution 14
promulgated on February 23, 1996. OnFebruary 26, 1996, the present petition was filed. Acting on the prayer for a temporary restraining order,
this Court issued on February 27, 1996 a Resolution which inter aliadirected the parties "to maintain the status quo prevailing prior to the filing
of this petition."

The Issues in G.R. No. 123755


Petitioner Lee's "position on the matter at hand briefly be capsulized in the following propositions" 15 :
"First — The initiatory petition below was so far insufficient in form and substance to warrant the exercise by
the COMELEC of its jurisdiction with the result that, in effect, the COMELEC acted without jurisdiction in taking cognizance of
and deciding said petition;
Second — The judicially declared disqualification of respondent was a continuing condition and rendered him
ineligible to run for, to be elected to and to hold the Office of Governor;
Third — The alleged repatriation of respondent was neither valid nor is the effect thereof retroactive as to cure his
ineligibility and qualify him to hold the Office of Governor; and
Fourth — Correctly read and applied, the Labo Doctrine fully supports the validity of petitioner's proclamation as duly
elected Governor of Sorsogon."
G.R. No. 120295
This is a petition to annul three Resolutions of the respondent Comelec, the first two of which are also at issue in G.R. No. 123755, as
follows:
1. Resolution 16 of the Second Division, promulgated on May 1, 1995, disqualifying Frivaldo from running for
governor of Sorsogon in the May 8, 1995elections "on the ground that he is not a citizen of the Philippines";
2. Resolution 17 of the Comelec en banc, promulgated on May 11, 1995; and
3. Resolution 18 of the Comelec en banc, promulgated also on May 11, 1995 suspending the proclamation of, among
others, Frivaldo.
The Facts and the Issue
The facts of this case are essentially the same as those in G.R. No. 123755. However, Frivaldo assails the above-mentioned
resolutions on a different ground: that under Section 78 of the Omnibus Election Code, which is reproduced hereinunder:
"Section 78. Petition to deny due course or to cancel a certificate of candidacy. — A verified petition seeking to deny
due course or to cancel a certificate of candidacy may be filed by any person exclusively on the ground that any material
representation contained therein as required under Section 74 hereof is false. The petition may be filed at any time not later
than twenty-five days from the time of the filing of the certificate of candidacy and shall be decided, after notice and
hearing, not later than fifteen days before the election." (Emphasis supplied.)
the Comelec had no jurisdiction to issue said Resolutions because they were not rendered "within the period allowed by law", i.e., "not later
than fifteen days before the election."
Otherwise stated, Frivaldo contends that the failure of the Comelec to act on the petition for disqualification within the period of
fifteen days prior to the election as provided by law is a jurisdictional defect which renders the said Resolutions null and void.
By Resolution on March 12, 1996, the Court consolidated G.R. Nos. 120295 and 123755 since they are intimately related in their
factual environment and are identical in the ultimate question raised, viz., who should occupy the position of governor of the province of
Sorsogon.
On March 19, 1995, the Court heard oral argument from the parties and required them thereafter to file simultaneously their
respective memoranda.
The Consolidated Issues
From the foregoing submissions, the consolidated issues may be restated as follows:
1. Was the repatriation of Frivaldo valid and legal? If so, did it seasonably cure his lack of citizenship as to qualify him to be proclaimed
and to hold the Office of Governor? If not, may it be given retroactive effect? If so, from when?
2. Is Frivaldo's "judicially declared" disqualification for lack of Filipino citizenship a continuing bar to his eligibility to run for, be elected
to or hold the governorship of Sorsogon?
3. Did the respondent Comelec have jurisdiction over the initiatory petition in SPC No. 95-317 considering that said petition is not "a
pre-proclamation case, an election protest or a quo warranto case"?
4. Was the proclamation of Lee, a runner-up in the election, valid and legal in light of existing jurisprudence?
5. Did the respondent Commission on Elections exceed its jurisdiction in promulgating the assailed Resolutions, all of which
prevented Frivaldo from assuming the governorship of Sorsogon, considering that they were not rendered within the period referred to in
Section 78 of the Omnibus Election Code, viz., "not later than fifteen days before the elections"?
The First Issue: Frivaldo's Repatriation
The validity and effectivity of Frivaldo's repatriation is the lis mota, the threshold legal issue in this case. All the other matters raised
are secondary to this.
The Local Government Code of 1991 19 expressly requires Philippine citizenship as a qualification for elective local officials, including
that of provincial governor, thus:
"Sec. 39. Qualifications. — (a) An elective local official must be a citizen of the Philippines; a registered voter in the
barangay, municipality, city, or province or, in the case of a member of the sangguniang panlalawigan, sangguniang
panlungsod, or sangguniang bayan, the district where he intends to be elected; a resident therein for at least one (1) year
immediately preceding the day of the election; and able to read and write Filipino or any other local language or dialect.
(b) Candidates for the position of governor, vice governor or member of the sangguniang panlalawigan, or mayor,
vice mayor or member of the sangguniang panlungsod of highly urbanized cities must be at least twenty-three (23) years of
age on election day.
xxx xxx xxx
Inasmuch as Frivaldo had been declared by this Court 20 as a non-citizen, it is therefore incumbent upon him to show that he has
reacquired citizenship; in fine, that he possesses the qualifications prescribed under the said statute (R.A. 7160).
Under Philippine law, 21 citizenship may be reacquired by direct act of Congress, by naturalization or by repatriation. Frivaldo told this
Court in G.R. No. 104654 22 and during the oral argument in this case that he tried to resume his citizenship by direct act of Congress, but that
the bill allowing him to do so "failed to materialize, notwithstanding the endorsement of several members of the House of Representatives"
due, according to him, to the "maneuvers of his political rivals." In the same case, his attempt at naturalization was rejected by this Court
because of jurisdictional, substantial and procedural defects.
Despite his lack of Philippine citizenship, Frivaldo was overwhelmingly elected governor by the electorate of Sorsogon, with a margin
of 27,000 votes in the 1988elections, 57,000 in 1992, and 20,000 in 1995 over the same opponent Raul Lee. Twice, he was judicially declared a
non-Filipino and thus twice disqualified from holding and discharging his popular mandate. Now, he comes to us a third time, with a fresh vote
from the people of Sorsogon and a favorable decision from the Commission on Electionsto boot. Moreover, he now boasts of having
successfully passed through the third and last mode of reacquiring citizenship: by repatriation under P.D. No. 725, with no less than the Solicitor
General himself, who was the prime opposing counsel in the previous cases he lost, this time, as counsel for co-respondent Comelec, arguing
the validity of his cause (in addition to his able private counsel Sixto S. Brillantes, Jr.). That he took his oath of allegiance under the provisions of
said Decree at 2:00 p.m. on June 30, 1995 is not disputed. Hence, he insists that he — not Lee — should have been proclaimed as the duly-
elected governor of Sorsogon when the Provincial Board of Canvassers met at 8:30 p.m. on the said date since, clearly and unquestionably, he
garnered the highest number of votes in the elections and since at that time, he already reacquired his citizenship.
En contrario, Lee argues that Frivaldo's repatriation is tainted with serious defects, which we shall now discuss in seriatim.
First, Lee tells us that P.D. No. 725 had "been effectively repealed", asserting that "then President Corazon Aquino exercising
legislative powers under the Transitory Provisions of the 1987 Constitution, forbade the grant of citizenship by Presidential Decree or Executive
Issuances as the same poses a serious and contentious issue of policy which the present government, in the exercise of prudence and sound
discretion, should best leave to the judgment of the first Congress under the 1987 Constitution", adding that in her memorandum dated March
27, 1987 to the members of the Special Committee on Naturalization constituted for purposes of Presidential Decree No. 725, President Aquino
directed them "to cease and desist from undertaking any and all proceedings within your functional area of responsibility as defined under
Letter of Instructions (LOI) No. 270 dated April 11, 1975, as amended." 23
This memorandum dated March 27, 1987 24 cannot by any stretch of legal hermeneutics be construed as a law sanctioning or
authorizing a repeal of P.D. No. 725. Laws are repealed only by subsequent ones 25 and a repeal may be express or implied. It is obvious that no
express repeal was made because then President Aquino in her memorandum — based on the copy furnished us by Lee — did not categorically
and/or impliedly state that P.D. 725 was being repealed or was being rendered without any legal effect. In fact, she did not even mention it
specifically by its number or text. On the other hand, it is a basic rule of statutory construction that repeals by implication are not favored. An
implied repeal will not be allowed "unless it is convincingly and unambiguously demonstrated that the two laws are clearly repugnant and
patently inconsistent that they cannot co-exist". 26
The memorandum of then President Aquino cannot even be regarded as a legislative enactment, for not every pronouncement of the
Chief Executive even under theTransitory Provisions of the 1987 Constitution can nor should be regarded as an exercise of her law-making
powers. At best, it could be treated as an executive policy addressed to the Special Committee to halt the acceptance and processing of
applications for repatriation pending whatever "judgment the first Congress under the 1987 Constitution" might make. In other words, the
former President did not repeal P.D. 725 but left it to the first Congress — once created — to deal with the matter. If she had intended to repeal
such law, she should have unequivocally said so instead of referring the matter to Congress. The fact is she carefully couched her presidential
issuance in terms that clearly indicated the intention of "the present government, in the exercise of prudence and sound discretion" to leave the
matter of repeal to the new Congress. Any other interpretation of the said Presidential Memorandum, such as is now being proffered to the
Court by Lee, would visit unmitigated violence not only upon statutory construction but on common sense as well.

Second. Lee also argues that "serious congenital irregularities flawed the repatriation proceedings," asserting that Frivaldo's
application therefor was "filed on June 29, 1995 . . . (and) was approved in just one day or on June 30, 1995 . . .", which "prevented a judicious
review and evaluation of the merits thereof." Frivaldo counters that he filed his application for repatriation with the Office of the President in
Malacañang Palace on August 17, 1994. This is confirmed by the Solicitor General. However, the Special Committee was reactivated
only on June 8, 1995, when presumably the said Committee started processing his application. On June 29, 1995, he filled up and re-submitted
the FORM that the Committee required. Under these circumstances, it could not be said that there was "indecent haste" in the processing of his
application.
Anent Lee's charge that the "sudden reconstitution of the Special Committee on Naturalization was intended solely for the personal
interest of respondent," 27 the Solicitor General explained during the oral argument on March 19, 1996 that such allegation is simply baseless as
there were many others who applied and were considered for repatriation, a list of whom was submitted by him to this Court, through a
Manifestation 28 filed on April 3, 1996.
On the basis of the parties' submissions, we are convinced that the presumption of regularity in the performance of official duty and
the presumption of legality in the repatriation of Frivaldo have not been successfully rebutted by Lee. The mere fact that the proceedings were
speeded up is by itself not a ground to conclude that such proceedings were necessarily tainted. After all, the requirements of repatriation
under P.D. No. 725 are not difficult to comply with, nor are they tedious and cumbersome. In fact, P.D. 725 29 itself requires very little of an
applicant, and even the rules and regulations to implement the said decree were left to the Special Committee to promulgate. This is not
unusual since, unlike in naturalization where an alien covets a first-time entry into Philippine political life, in repatriation the applicant is a
former natural-born Filipino who is merely seeking to reacquire his previous citizenship. In the case of Frivaldo, he was undoubtedly a natural-
born citizen who openly and faithfully served his country and his province prior to his naturalization in the United States — a naturalization he
insists was made necessary only to escape the iron clutches of a dictatorship he abhorred and could not in conscience embrace — and who,
after the fall of the dictator and the re-establishment of democratic space, wasted no time in returning to his country of birth to offer once more
his talent and services to his people.
So too, the fact that ten other persons, as certified to by the Solicitor General, were granted repatriation argues convincingly and
conclusively against the existence of favoritism vehemently posited by Raul Lee. At any rate, any contest on the legality of Frivaldo's repatriation
should have been pursued before the Committee itself, and, failing there, in the Office of the President, pursuant to the doctrine of exhaustion
of administrative remedies.
Third. Lee further contends that assuming the assailed repatriation to be valid, nevertheless it could only be effective as at 2:00 p.m.
of June 30, 1995 whereas the citizenship qualification prescribed by the Local Government Code "must exist on the date of his election, if not
when the certificate of candidacy is filed," citing our decision in G.R. 104654 30 which held that "both the Local Government Code and the
Constitution require that only Philippine citizens can run and be elected to Public office." Obviously, however, this was a mere obiter as the only
issue in said case was whether Frivaldo's naturalization was valid or not — and NOT the effective date thereof. Since the Court held his
naturalization to be invalid, then the issue of when an aspirant for public office should be a citizen was NOT resolved at all by the Court. Which
question we shall now directly rule on.
Under Sec. 39 of the Local Government Code, "(a)n elective local official must be:
* a citizen of the Philippines;
* a registered voter in the barangay, municipality, city, or province . . . where he intends to be elected;
* a resident therein for at least one (1) year immediately preceding the day of the election;
* able to read and write Filipino or any other local language or dialect."
* In addition, "candidates for the position of governor . . . must be at least twenty-three (23) years of age on election day."
From the above, it will be noted that the law does not specify any particular date or time when the candidate must possess citizenship,
unlike that for residence (which must consist of at least one year's residency immediately preceding the day of election) and age (at least twenty
three years of age on election day).
Philippine citizenship is an indispensable requirement for holding an elective public office, 31 and the purpose of the citizenship
qualification is none other than to ensure that no alien, i.e., no person owing allegiance to another nation, shall govern our people and our
country or a unit of territory thereof. Now, an official begins to govern or to discharge his functions only upon his proclamation and on the day
the law mandates his term of office to begin. Since Frivaldo re-assumed his citizenship on June 30, 1995 — the very day 32 the term of office of
governor (and other elective officials) began — he was therefore already qualified to be proclaimed, to hold such office and to discharge the
functions and responsibilities thereof as of said date. In short, at that time, he was already qualified to govern his native Sorsogon. This is the
liberal interpretation that should give spirit, life and meaning to our law on qualifications consistent with the purpose for which such law was
enacted. So too, even from a literal (as distinguished from liberal) construction, it should be noted that Section 39 of the Local Government
Code speaks of "Qualifications" of "ELECTIVE OFFICIALS", not of candidates. Why then should such qualification be required at the time of
election or at the time of the filing of the certificates of candidacies, as Lee insists? Literally, such qualifications — unless otherwise expressly
conditioned, as in the case of age and residence — should thus be possessed when the "elective [or elected] official" begins to govern, i.e., at
the time he is proclaimed and at the start of his term — in this case, on June 30, 1995. Paraphrasing this Court's ruling in Vasquez vs. Giap and Li
Seng Giap & Sons, 33 if the purpose of the citizenship requirement is to ensure that our people and country do not end up being governed by
aliens, i.e., persons owing allegiance to another nation, that aim or purpose would not be thwarted but instead achieved by construing the
citizenship qualification as applying to the time of proclamation of the elected official and at the start of his term.
But perhaps the more difficult objection was the one raised during the oral argument 34 to the effect that the citizenship qualification
should be possessed at the time the candidate (or for that matter the elected official) registered as a voter. After all, Section 39, apart from
requiring the official to be a citizen, also specifies as another item of qualification, that he be a "registered voter". And, under the law 35 a
"voter" must be a citizen of the Philippines. So therefore, Frivaldo could not have been a voter — much less a validly registered one — if he was
not a citizen at the time of such registration.
The answer to this problem again lies in discerning the purpose of the requirement. If the law intended the citizenship qualification to
be possessed prior to election consistent with the requirement of being a registered voter, then it would not have made citizenship a SEPARATE
qualification. The law abhors a redundancy. It therefore stands to reason that the law intended CITIZENSHIP to be a qualification distinct from
being a VOTER, even if being a voter presumes being a citizen first. It also stands to reason that the voter requirement was included as another
qualification (aside from "citizenship"), not to reiterate the need for nationality but to require that the official be registered as a voter IN THE
AREA OR TERRITORY he seeks to govern, i.e., the law states: "a registered voter in the barangay, municipality, city, or province . . . where he
intends to be elected." It should be emphasized that the Local Government Code requires an elective official to be a registered voter. It does not
require him to vote actually. Hence, registration — not the actual voting — is the core of this "qualification". In other words, the law's purpose
in this second requirement is to ensure that the prospective official is actually registered in the area he seeks to govern — and not anywhere
else.
Before this Court, Frivaldo has repeatedly emphasized — and Lee has not disputed — that he "was and is a registered voter of
Sorsogon, and his registration as a voter has been sustained as valid by judicial declaration . . . In fact, he cast his vote in his precinct on May 8,
1995." 36
So too, during the oral argument, his counsel stead- fastly maintained that "Mr. Frivaldo has always been a registered voter of
Sorsogon. He has voted in 1987, 1988, 1992, then he voted again in 1995. In fact, his eligibility as a voter was questioned, but the court
dismissed (sic) his eligibility as a voter and he was allowed to vote as in fact, he voted in all the previous elections including on May 8, 1995. 37
It is thus clear that Frivaldo is a registered voter in the province where he intended to be elected.
There is yet another reason why the prime issue of citizenship should be reckoned from the date of proclamation, not necessarily the
date of election or date of filing of the certificate of candidacy. Section 253 of the Omnibus Election Code 38 gives any voter, presumably
including the defeated candidate, the opportunity to question the ELIGIBILITY (or the disloyalty) of a candidate. This is the only provision of the
Code that authorizes a remedy on how to contest before the Comelec an incumbent's ineligibility arising from failure to meet the qualifications
enumerated under Sec. 39 of the Local Government Code. Such remedy of Quo Warranto can be availed of "within ten days after proclamation"
of the winning candidate. Hence, it is only at such time that the issue of ineligibility may be taken cognizance of by the Commission. And since,
at the very moment of Lee's proclamation (8:30 p.m., June 30, 1995), Juan G. Frivaldo was already and indubitably a citizen, having taken his
oath of allegiance earlier in the afternoon of the same day, then he should have been the candidate proclaimed as he unquestionably garnered
the highest number of votes in the immediately preceding elections and such oath had already cured his previous "judicially-declared" alienage.
Hence, at such time, he was no longer ineligible.

But to remove all doubts on this important issue, we also hold that the repatriation of Frivaldo RETROACTED to the date of the filing of
his application on August 17, 1994.
It is true that under the Civil Code of the Philippines, 39 "(l)aws shall have no retroactive effect, unless the contrary is provided." But
there are settled exceptions 40 to this general rule, such as when the statute is CURATIVE or REMEDIAL in nature or when it CREATES NEW
RIGHTS.
According to Tolentino, 41 curative statutes are those which undertake to cure errors and irregularities, thereby validating judicial or
administrative proceedings, acts of public officers, or private deeds and contracts which otherwise would not produce their intended
consequences by reason of some statutory disability or failure to comply with some technical requirement. They operate on conditions already
existing, and are necessarily retroactive in operation. Agpalo, 42 on the other hand, says that curative statutes are "healing acts . . . curing
defects and adding to the means of enforcing existing obligations . . . (and ) are intended to supply defects, abridge superfluities in existing laws,
and curb certain evils . . . By their very nature, curative statutes are retroactive . . . (and) reach back to past events to correct errors or
irregularities and to render valid and effective attempted acts which would be otherwise ineffective for the purpose the parties intended."
On the other hand, remedial or procedural laws, i.e., those statutes relating to remedies or modes of procedure, which do not create
new or take away vested rights, but only operate in furtherance of the remedy or confirmation of such rights, ordinarily do not come within the
legal meaning of a retrospective law, nor within the general rule against the retrospective operation of statutes. 43
A reading of P.D. 725 immediately shows that it creates a new right, and also provides for a new remedy, thereby filling certain voids
in our laws. Thus, in its preamble,P.D. 725 expressly recognizes the plight of "many Filipino women (who) had lost their Philippine citizenship by
marriage to aliens" and who could not, under the existing law (C.A. No. 63, as amended) avail of repatriation until "after the death of their
husbands or the termination of their marital status" and who could neither be benefited by the1973 Constitution's new provision allowing "a
Filipino woman who marries an alien to retain her Philippine citizenship . . ." because "such provision of the new Constitutiondoes not apply to
Filipino women who had married aliens before said constitution took effect." Thus, P.D. 725 granted a new right to these women — the right to
re-acquire Filipino citizenship even during their marital coverture, which right did not exist prior to P.D. 725. On the other hand, said statute also
provided a new remedy and a new right in favor of other "natural born Filipinos who (had) lost their Philippine citizenship but now desire to re-
acquire Philippine citizenship", because prior to the promulgation ofP.D. 725 such former Filipinos would have had to undergo the tedious and
cumbersome process of naturalization, but with the advent of P.D. 725 they could now re-acquire their Philippine citizenship under the
simplified procedure of repatriation.
The Solicitor General 44 argues:
"By their very nature, curative statutes are retroactive, (DBP vs. CA, 96 SCRA 342), since they are intended to supply
defects, abridge superfluities in existing laws (Del Castillo vs. Securities and Exchange Commission, 96 Phil. 119) and curb
certain evils (Santos vs. Duata, 14 SCRA 1041).
In this case, P.D. No. 725 was enacted to cure the defect in the existing naturalization law, specifically C.A. No.
63 wherein married Filipino women are allowed to repatriate only upon the death of their husbands, and natural-born Filipinos
who lost their citizenship by naturalization and other causes faced the difficulty of undergoing the rigid procedures of C.A.
63 for reacquisition of Filipino citizenship by naturalization.
Presidential Decree No. 725 provided a remedy for the aforementioned legal aberrations and thus its provisions are
considered essentially remedial and curative."
In light of the foregoing, and prescinding from the wording of the preamble, it is unarguable that the legislative intent was precisely to
give the statute retroactive operation. "(A) retrospective operation is given to a statute or amendment where the intent that it should so
operate clearly appears from a consideration of the act as a whole, or from the terms thereof." 45 It is obvious to the Court that the statute was
meant to "reach back" to those persons, events and transactions not otherwise covered by prevailing law and jurisprudence. And inasmuch as it
has been held that citizenship is a political and civil right equally as important as the freedom of speech, liberty of abode, the right against
unreasonable searches and seizures and other guarantees enshrined in the Bill of Rights, therefore the legislative intent to give retrospective
operation toP.D. 725 must be given the fullest effect possible. "(I)t has been said that a remedial statute must be so construed as to make it
effect the evident purpose for which it was enacted, so that if the reason of the statute extends to past transactions, as well as to those in the
future, then it will be so applied although the statute does not in terms so direct, unless to do so would impair some vested right or violate some
constitutional guaranty." 46 This is all the more true of P.D. 725, which did not specify any restrictionson or delimit or qualify the right of
repatriation granted therein.
At this point, a valid question may be raised: How can the retroactivity of P.D. 725 benefit Frivaldo considering that said law was
enacted on June 5, 1975, while Frivaldolost his Filipino citizenship much later, on January 20, 1983, and applied for repatriation even
later, on August 17, 1994?
While it is true that the law was already in effect at the time that Frivaldo became an American citizen, nevertheless, it is not only the
law itself (P.D. 725) which is to be given retroactive effect, but even the repatriation granted under said law to Frivaldo on June 30, 1995 is to be
deemed to have retroacted to the date of his application therefor, August 17, 1994. The reason for this is simply that if, as in this case, it was the
intent of the legislative authority that the law should apply to past events — i.e., situations and transactions existing even before the law came
into being — in order to benefit the greatest number of former Filipinos possible thereby enabling them to enjoy and exercise the
constitutionally guaranteed right of citizenship, and such legislative intention is to be given the fullest effect and expression, then there is all the
more reason to have the law apply in a retroactive or retrospective manner to situations, events and transactions subsequent to the passage of
such law. That is, the repatriation granted to Frivaldo on June 30, 1995 can and should be made to take effect as of date of his application. As
earlier mentioned, there is nothing in the law that would bar this or would show a contrary intention on the part of the legislative authority; and
there is no showing that damage or prejudice to anyone, or anything unjust or injurious would result from giving; retroactivity to his
repatriation. Neither has Lee shown that there will result the impairment of any contractual obligation, disturbance of any vested right or
breach of some constitutional guaranty.
Being a former Filipino who has served the people repeatedly, Frivaldo deserves a liberal interpretation of Philippine laws and
whatever defects there were in his nationality should now be deemed mooted by his repatriation.
Another argument for retroactivity to the date of filing is that it would prevent prejudice to applicants. If P.D. 725 were not to be given
retroactive effect, and the Special Committee decides not to act, i.e., to delay the processing of applications for any substantial length of time,
then the former Filipinos who may be stateless, as Frivaldo — having already renounced his American citizenship — was, may be prejudiced for
causes outside their control. This should not be. In case of doubt in the interpretation or application of laws, it is to be presumed that the law-
making body intended right and justice to prevail. 47
And as experience will show, the Special Committee was able to process, act upon and grant applications for repatriation within
relatively short spans of time after the same were filed. 48 The fact that such interregna were relatively insignificant minimizes the likelihood of
prejudice to the government as a result of giving retroactivity to repatriation. Besides, to the mind of the Court, direct prejudice to the
government is possible only where a person's repatriation has the effect of wiping out a liability of his to the government arising in connection
with or as a result of his being an alien, and accruing only during the interregnum between application and approval, a situation that is not
present in the instant case.
And it is but right and just that the mandate of the people, already twice frustrated, should now prevail. Under the circumstances,
there is nothing unjust or iniquitous in treating Frivaldo's repatriation as having become effective as of the date of his application, i.e., on August
17, 1994. This being so, all questions about his possession of the nationality qualification — whether at the date of proclamation (June 30, 1995)
or the date of election (May 8, 1995) or date of filing his certificate of candidacy (March 20, 1995) would become moot.
Based on the foregoing, any question regarding Frivaldo's status as a registered voter would also be deemed settled. Inasmuch as he is
considered as having been repatriated — i.e., his Filipino citizenship restored — as of August 17, 1994, his previous registration as a voter is
likewise deemed validated as of said date.
It is not disputed that on January 20, 1983 Frivaldo became an American. Would the retroactivity of his repatriation not effectively
give him dual citizenship, which underSec. 40 of the Local Government Code would disqualify him "from running for any elective local position?"
49 We answer this question in the negative, as there is cogent reason to hold that Frivaldo was really STATELESS at the time he took said oath of
allegiance and even before that, when he ran for governor in 1988. In his Comment, Frivaldowrote that he "had long renounced and had long
abandoned his American citizenship — long before May 8, 1995. At best, Frivaldo was stateless in the interim — when he abandoned and
renounced his US citizenship but before he was repatriated to his Filipino citizenship." 50
On this point, we quote from the assailed Resolution dated December 19, 1995: 51
"By the laws of the United States, petitioner Frivaldo lost his American citizenship when he took his oath of allegiance
to the Philippine Government when he ran for Governor in 1988, in 1992, and in 1995. Every certificate of candidacy contains
an oath of allegiance to the Philippine Government."
These factual findings that Frivaldo has lost his foreign nationality long before the elections of 1995 have not been effectively rebutted
by Lee. Furthermore, it is basic that such findings of the Commission are conclusive upon this Court, absent any showing of capriciousness or
arbitrariness or abuse. 52
The Second Issue: Is Lack of Citizenship a Continuing Disqualification?
Lee contends that the May 1, 1995 Resolution 53 of the Comelec Second Division in SPA No. 95-028 as affirmed in toto by Comelec En
Banc in its Resolution of May 11, 1995 "became final and executory after five (5) days or on May 17, 1995, no restraining order having been
issued by this Honorable Court." 54 Hence, before Lee "was proclaimed as the elected governor on June 30, 1995, there was already a final and
executory judgment disqualifying" Frivaldo. Lee adds that this Court's two rulings (whichFrivaldo now concedes were legally "correct")
declaring Frivaldo an alien have also become final and executory way before the 1995 elections, and these "judicial pronouncements of his
political status as an American citizen absolutely and for all time disqualified (him) from running for, and holding any public office in the
Philippines."
We do not agree.
It should be noted that our first ruling in G.R. No. 87193 disqualifying Frivaldo was rendered in connection with the
1988 elections while that in G.R. No. 104654 was in connection with the 1992 elections. That he was disqualified for such elections is final and
can no longer be changed. In the words of the respondent Commission (Second Division) in its assailed Resolution: 55
"The records show that the Honorable Supreme Court had decided that Frivaldo was not a Filipino citizen and thus
disqualified for the purpose of the 1988 and 1992elections. However, there is no record of any 'final judgment' of the
disqualification of Frivaldo as a candidate for the May 8, 1995 elections. What the Commission said in its Order of June 21,
1995 (implemented on June 30, 1995), directing the proclamation of Raul R. Lee, was that Frivaldo was not a Filipino citizen
'having been declared by the Supreme Court in its Order dated March 25, 1995, not a citizen of the Philippines.' This declaration
of the Supreme Court, however, was in connection with the 1992 elections."
Indeed, decisions declaring the acquisition or denial of citizenship cannot govern a person's future status with finality. This is because
a person may subsequently reacquire, or for that matter lose, his citizenship under any of the modes recognized by law for the purpose. Hence,
in Lee vs. Commissioner of Immigration, 56 we held:
"Everytime the citizenship of a person is material or indispensable in a judicial or administrative case, whatever the
corresponding court or administrative authority decides therein as to such citizenship is generally not considered res judicata,
hence it has to be threshed out again and again, as the occasion demands."
The Third Issue: Comelec's Jurisdiction Over The Petition in SPC No. 95-317
Lee also avers that respondent Comelec had no jurisdiction to entertain the petition in SPC No. 95-317 because the only "possible
types of proceedings that may be entertained by the Comelec are a pre-proclamation case, an election protest or a quo warranto case". Again,
Lee reminds us that he was proclaimed on June 30, 1995 but thatFrivaldo filed SPC No. 95-317 questioning his (Lee's) proclamation only on July
6, 1995 — "beyond the 5-day reglementary period." Hence, according to him, Frivaldo's "recourse was to file either an election protest or a quo
warranto action."
This argument is not meritorious. The Constitution 57 has given the Comelec ample power to "exercise exclusive original jurisdiction
over all contests relating to theelections, returns and qualifications of all elective. . . provincial . . . officials." Instead of dwelling at length on the
various petitions that Comelec, in the exercise of its constitutional prerogatives, may entertain, suffice it to say that this Court has invariably
recognized the Commission's authority to hear and decide petitions for annulment of proclamations — of which SPC No. 95-317 obviously is
one. 58 Thus, in Mentang vs. COMELEC, 59 we ruled:
"The petitioner argues that after proclamation and assumption of office, a pre-proclamation controversy is no longer
viable. Indeed, we are aware of cases holding that pre-proclamation controversies may no longer be entertained by
the COMELEC after the winning candidate has been proclaimed. (citing Gallardo vs. Rimando, 187 SCRA 463;
Salvacion vs. COMELEC, 170 SCRA 513; Casimiro vs. COMELEC, 171 SCRA 468.) This rule, however, is premised on an
assumption that the proclamation is no proclamation at all and the proclaimed candidate's assumption of office cannot deprive
the COMELEC of the power to make such declaration of nullity. (citing Aguam vs. COMELEC, 23 SCRA 883;
Agbayani vs. COMELEC, 186 SCRA 484.)"
The Court however cautioned that such power to annul a proclamation must "be done within ten (10) days following the
proclamation." Inasmuch as Frivaldo's petition was filed only six (6) days after Lee's proclamation, there is no question that
the Comelec correctly acquired jurisdiction over the same.
The Fourth Issue: Was Lee's Proclamation Valid?
Frivaldo assails the validity of the Lee proclamation. We uphold him for the following reasons:
First. To paraphrase this Court in Labo vs. COMELEC, 60 "the fact remains that he (Lee) was not the choice of the sovereign will," and
in Aquino vs. COMELEC, 61 Lee is "a second placer, . . . just that, a second placer."
In spite of this, Lee anchors his claim to the governorship on the pronouncement of this Court in the aforesaid Labo 62 case, as
follows:
"The rule would have been different if the electorate fully aware in fact and in law of a candidate's disqualification so
as to bring such awareness within the realm of notoriety, would nonetheless cast their votes in favor of the ineligible
candidate. In such case, the electorate may be said to have waived the validity and efficacy of their votes by notoriously
misapplying their franchise or throwing away their votes, in which case, the eligible candidate obtaining the next higher
number of votes may be deemed elected."
But such holding is qualified by the next paragraph, thus:
"But this is not the situation obtaining in the instant dispute. It has not been shown, and none was alleged, that
petitioner Labo was notoriously known as an ineligible candidate, much less the electorate as having known of such
fact. On the contrary, petitioner Labo was even allowed by no less than the Comelec itself in its resolution dated May 10, 1992
to be voted for the office of the city mayor as its resolution dated May 9, 1992 denying due course to petitioner Labo's
certificate of candidacy had not yet become final and subject to the final outcome of this case."
The last-quoted paragraph in Labo, unfortunately for Lee, is the ruling appropriate in this case because Frivaldo was in 1995 in an
identical situation as Labo was in 1992 when the Comelec's cancellation of his certificate of candidacy was not yet final on election day as there
was in both cases a pending motion for reconsideration, for which reason Comelec issued an (omnibus) resolution declaring that Frivaldo (like
Labo in 1992) and several others can still be voted for in the May 8, 1995 election, as in fact, he was.
Furthermore, there has been no sufficient evidence presented to show that the electorate of Sorsogon was "fully aware in fact and in
law" of Frivaldo's alleged disqualification as to "bring such awareness within the realm of notoriety;" in other words, that the voters
intentionally wasted their ballots knowing that, in spite of their voting for him, he was ineligible. If Labo has any relevance at all, it is that the
vice-governor — and not Lee — should be proclaimed, since in losing the election, Lee was, to paraphrase Labo again, "obviously not the choice
of the people" of Sorsogon. This is the emphatic teaching of Labo:
"The rule, therefore, is: the ineligibility of a candidate receiving majority votes does not entitle the eligible candidate
receiving the next highest number of votes to be declared elected. A minority or defeated candidate cannot be deemed
elected to the office."
Second. As we have earlier declared Frivaldo to have seasonably re-acquired his citizenship and inasmuch as he obtained the highest
number of votes in the 1995elections, he — not Lee — should be proclaimed. Hence, Lee's proclamation was patently erroneous and should
now be corrected.
The Fifth Issue: Is Section 78 of the Election Code Mandatory?
In G.R. No. 120295. Frivaldo claims that the assailed Resolution of the Comelec (Second Division) dated May 1, 1995 and the
confirmatory en banc Resolution of May 11, 1995 disqualifying him for want of citizenship should be annulled because they were rendered
beyond the fifteen (15) day period prescribed by Section 78 of the Omnibus Election Code which reads as follows:

"Section 78. Petition to deny due course or to cancel a certificate of candidacy. — A verified petition seeking to deny
due course or to cancel a certificate of candidacy may be filed by any person exclusively on the ground that any material
representation contained therein as required under Section 74 hereof is false. The petition may be filed at any time not later
than twenty-five days from the time of the filing of the certificate of candidacy and shall be decided after notice and
hearing, not later than fifteen days before the election." (emphasis supplied.)
This claim is now moot and academic inasmuch as these resolutions are deemed superseded by the subsequent ones issued by
the Commission (First Division) onDecember 19, 1995, affirmed en banc 63 on February 23, 1996, which both upheld his election. At any rate, it
is obvious that Section 78 is merely directory as Section 6 of R.A. No. 6646 authorizes the Commission to try and decide petitions for
disqualifications even after the elections, thus:
"SEC. 6. Effect of Disqualification Case. — Any candidate who has been declared by final judgment to be disqualified
shall not be voted for, and the votes cast for him shall not be counted. If for any reason a candidate is not declared by final
judgment before an election to be disqualified and he is voted for and receives the winning number of votes in such election, the
Court or Commission shall continue with the trial and hearing of the action, inquiry or protest and, upon motion of the
complainant or any intervenor, may during the pendency thereof order the suspension of the proclamation of such candidate
whenever the evidence of his guilt is strong." (emphasis supplied)
Refutation of
Mr. Justice Davide's Dissent
In his dissenting opinion, the esteemed Mr. Justice Hilario G. Davide, Jr. argues that President Aquino's memorandum dated March 27,
1987 should be viewed as a suspension (not a repeal, as urged by Lee) of P.D. 725. But whether it decrees a suspension or a repeal is a purely
academic distinction because the said issuance is not a statute that can amend or abrogate an existing law. The existence and subsistence
of P.D. 725 were recognized in the first Frivaldo case; 64 viz, "(u)nder CA No. 63 as amended by CA No. 473 and P.D. No. 725, Philippine
citizenship maybe reacquired by . . . repatriation". He also contends that by allowing Frivaldo to register and to remain as a registered voter,
the Comelec and in effect this Court abetted a "mockery" of our two previous judgments declaring him a non-citizen. We do not see such
abetting or mockery. The retroactivity of his repatriation, as discussed earlier, legally cured whatever defects there may have been in his
registration as a voter for the purpose of the 1995 elections. Such retroactivity did not change his disqualifications in 1988 and 1992, which were
the subjects of such previous rulings.
Mr. Justice Davide also believes that Quo Warranto is not the sole remedy to question the ineligibility of a candidate, citing
the Comelec's authority under Section 78 of the Omnibus Election Code allowing the denial of a certificate of candidacy on the ground of a false
material representation therein as required by Section 74. Citing Loong, he then states his disagreement with our holding that Section 78 is
merely directory. We really have no quarrel. Our point is that Frivaldo was in error in his claim in G.R. No. 120295 that the Comelec Resolutions
promulgated on May 1, 1995 and May 11, 1995 were invalid because they were issued "not later than fifteen days before the election" as
prescribed by Section 78. In dismissing the petition in G.R. No. 120295, we hold that the Comelec did not commit grave abuse of discretion
because "Section 6 of R.A. 6646authorizes the Comelec to try and decide disqualifications even after the elections." In spite of his disagreement
with us on this point, i.e., that Section 78 "is merely directory", we note that just like us, Mr. Justice Davide nonetheless votes to "DISMISS G.R.
No. 120295". One other point. Loong, as quoted in the dissent, teaches that a petition to deny due course under Section 78 must be filed within
the 25-day period prescribed therein. The present case however deals with the period during which the Comelec may decidesuch petition. And
we hold that it may be decided even after the fifteen day period mentioned in Section 78. Here, we rule that a decision promulgated by
the Comelec even after the elections is valid but Loong held that a petition filed beyond the 25-day period is out of time. There is no
inconsistency nor conflict.
Mr. Justice Davide also disagrees with the Court's holding that, given the unique factual circumstances of Frivaldo, repatriation may be
given retroactive effect. He argues that such retroactivity "dilutes" our holding in the first Frivaldo case. But the first (and even the
second Frivaldo) decision did not directly involve repatriation as a mode of acquiring citizenship. If we may repeat, there is no question
that Frivaldo was not a Filipino for purposes of determining his qualifications in the 1988 and 1992 elections. That is settled. But his supervening
repatriation has changed his political status — not in 1988 or 1992, but only in the 1995 elections.
Our learned colleague also disputes our holding that Frivaldo was stateless prior to his repatriation, saying that "informal renunciation
or abandonment is not a ground to lose American citizenship." Since our courts are charged only with the duty of the determining who are
Philippine nationals, we cannot rule on the legal question of who are or who are not Americans. It is basic in international law that a State
determines ONLY those who are its own citizens — not who are the citizens of other countries. 65 The issue here is: the Comelec made a finding
of fact that Frivaldo was stateless and such finding has not been shown by Lee to be arbitrary or whimsical. Thus, following settled case law,
such finding is binding and final.
The dissenting opinion also submits that Lee who lost by chasmic margins to Frivaldo in all three previous elections, should be
declared winner because "Frivaldo's ineligibility for being an American was publicly known." First, there is absolutely no empirical evidence for
such "public" knowledge. Second, even if there is, such knowledge can be true post facto only of the last two previous elections. Third, even
the Comelec and now this Court were/are still deliberating on his nationality before, during and after the 1995 elections. How then can there be
such "public" knowledge?
Mr. Justice Davide submits that Section 39 of the Local Government Code refers to the qualifications of elective local officials, i.e.,
candidates, and not elected officials, and that the citizenship qualification [under par. (a) of that section] must be possessed by candidates, not
merely at the commencement of the term, but by election day at the latest. We see it differently. Section 39, par. (a) thereof speaks of "elective
local official" while par. (b) to (f) refer to "candidates". If the qualifications under par. (a) were intended to apply to "candidates" and not elected
officials, the legislature would have said so, instead of differentiating par. (a) from the rest of the paragraphs. Secondly, if Congress had meant
that the citizenship qualification should be possessed at election day or prior thereto, it would have specifically stated such detail, the same way
it did in pars. (b) to (f) for other qualifications of candidates for governor, mayor, etc.
Mr. Justice Davide also questions the giving of retroactive effect to Frivaldo's repatriation on the ground, among others, that the law
specifically provides that it is only after taking the oath of allegiance that applicants shall be deemed to have reacquired Philippine citizenship.
We do not question what the provision states. We hold however that the provision should be understood thus: that after taking the oath of
allegiance the applicant is deemed to have reacquired Philippine citizenship, which reacquisition (or repatriation) is deemed for all purposes and
intents to have retroacted to the date of his application therefor.
In any event, our "so too" argument regarding the literal meaning of the word "elective" in reference to Section 39 of the Local
Government Code, as well as regarding Mr. Justice Davide's thesis that the very wordings of P.D. 725 suggest non-retroactivity, were already
taken up rather extensively earlier in this Decision.
Mr. Justice Davide caps his paper with a clarion call: "This Court must be the first to uphold the Rule of Law." We agree — we must all
follow the rule of law. But that is NOT the issue here. The issue is how should the law be interpreted and applied in this case so it can be
followed, so it can rule!
At balance, the question really boils down to a choice of philosophy and perception of how to interpret and apply laws relating
to elections: literal or liberal; the letter or the spirit; the naked provision or its ultimate purpose; legal syllogism or substantial justice; in isolation
or in the context of social conditions; harshly against or gently in favor of the voters' obvious choice. In applying election laws, it would be far
better to err in favor of popular sovereignty than to be right in complex but little understood legalisms. Indeed, to inflict a thrice rejected
candidate upon the electorate of Sorsogon would constitute unmitigated judicial tyranny and an unacceptable assault upon this Court's
conscience.
EPILOGUE
In sum, we rule that the citizenship requirement in the Local Government Code is to be possessed by an elective official at the latest as
of the time he is proclaimed andat the start of the term of office to which he has been elected. We further hold P.D. No. 725 to be in full force
and effect up to the present, not having been suspended or repealed expressly nor impliedly at any time, and Frivaldo's repatriation by virtue
thereof to have been properly granted and thus valid and effective. Moreover, by reason of the remedial or curative nature of the law granting
him a new right to resume his political status and the legislative intent behind it, as well as his unique situation of having been forced to give up
his citizenship and political aspiration as his means of escaping a regime he abhorred, his repatriation is to be given retroactive effect as of the
date of his application therefor, during the pendency of which he was stateless, he having given up his U.S. nationality. Thus, in contemplation
of law, he possessed the vital requirement of Filipino citizenship as of the start of the term of office of governor, and should have been
proclaimed instead of Lee. Furthermore, since his reacquisition of citizenship retroacted to August 17, 1994, his registration as a voter of
Sorsogon is deemed to have been validated as of said date as well. The foregoing, of course, are precisely consistent with our holding that lack
of the citizenship requirement is not a continuing disability or disqualification to run for and hold public office. And once again, we emphasize
herein our previous rulings recognizing the Comelec's authority and jurisdiction to hear and decide petitions for annulment of proclamations.

This Court has time and again liberally and equitably construed the electoral laws of our country to give fullest effect to the manifest
will of our people, 66 for in case of doubt, political laws must be interpreted to give life and spirit to the popular mandate freely expressed
through the ballot. Otherwise stated, legal niceties and technicalities cannot stand in the way of the sovereign will. Consistently, we have held:
". . . (L)aws governing election contests must be liberally construed to the end that the will of the people in the
choice of public officials may not be defeated by mere technical objections (citations omitted)." 67
The law and the courts must accord Frivaldo every possible protection, defense and refuge, in deference to the popular will. Indeed,
this Court has repeatedly stressed the importance of giving effect to the sovereign will in order to ensure the survival of our democracy. In any
action involving the possibility of a reversal of the popular electoral choice, this Court must exert utmost effort to resolve the issues in a manner
that would give effect to the will of the majority, for it is merely sound public policy to cause elective offices to be filled by those who are the
choice of the majority. To successfully challenge a winning candidate's qualifications, the petitioner must clearly demonstrate that the
ineligibility is so patently antagonistic 68 to constitutional and legal principles that overriding such ineligibility and thereby giving effect to the
apparent will of the people, would ultimately create greater prejudice to the very democratic institutions and juristic traditions that
our Constitution and laws so zealously protect and promote. In this undertaking, Lee has miserably failed.
In Frivaldo's case, it would have been technically easy to find fault with his cause. The Court could have refused to grant retroactivity
to the effects of his repatriation and hold him still ineligible due to his failure to show his citizenship at the time he registered as a voter before
the 1995 elections. Or, it could have disputed the factual findings of the Comelec that he was stateless at the time of repatriation and thus hold
his consequent dual citizenship as a disqualification "from running for any elective local position." But the real essence of justice does not
emanate from quibblings over patchwork legal technicality. It proceeds from the spirit's gut consciousness of the dynamic role of law as a brick
in the ultimate development of the social edifice. Thus, the Court struggled against and eschewed the easy, legalistic, technical and sometimes
harsh anachronisms of the law in order to evoke substantial justice in the larger social context consistent with Frivaldo's unique situation
approximating venerability in Philippine political life. Concededly, he sought American citizenship only to escape the clutches of the dictatorship.
At this stage, we cannot seriously entertain any doubt about his loyalty and dedication to this country. At the first opportunity, he returned to
this land, and sought to serve his people once more. The people of Sorsogon overwhelmingly voted for him three times. He took an oath of
allegiance to this Republic every time he filed his certificate of candidacy and during his failed naturalization bid. And let it not be overlooked,
his demonstrated tenacity and sheer determination to re-assume his nationality of birth despite several legal set-backs speak more loudly, in
spirit, in fact and in truth than any legal technicality, of his consuming intention and burning desire to re-embrace his native Philippines even
now at the ripe old age of 81 years. Such loyalty to and love of country as well as nobility of purpose cannot be lost on this Court of justice and
equity. Mortals of lesser mettle would have given up. After all, Frivaldo was assured of a life of ease and plenty as a citizen of the most powerful
country in the world. But he opted, nay, single-mindedly insisted on returning to and serving once more his struggling but beloved land of birth.
He therefore deserves every liberal interpretation of the law which can be applied in his favor. And in the final analysis, over and
above Frivaldo himself, the indomitable people of Sorsogon most certainly deserve to be governed by a leader of their overwhelming choice.
WHEREFORE, in consideration of the foregoing:
(1) The petition in G.R. No. 123755 is hereby DISMISSED. The assailed Resolutions of the respondent Commission are AFFIRMED.
(2) The petition in G.R. No. 120295 is also DISMISSED for being moot and academic. In any event, it has no merit.
No costs.
SO ORDERED.
Francisco, Hermosisima, Jr., Melo, Vitug, Kapunan and Torres, JJ., concur.
Padilla, Regalado, Romero and Bellosillo, JJ., pro hac vice.
Narvasa, C.J. and Mendoza, J., took no part.
||| (Frivaldo v. Commission on Elections, G.R. No. 120295, 123755, [June 28, 1996], 327 PHIL 521-598)
THIRD DIVISION
[G.R. No. 125793. August 29, 2006.]
JOEVANIE ARELLANO TABASA, petitioner, vs. HON. COURT OF APPEALS, BUREAU OF IMMIGRATION and DEPORTATION and
WILSON SOLUREN, respondents.
DECISION
VELASCO, JR., J p:
Citizenship is a priceless possession. Former U.S. Chief Justice Earl Warren fittingly emphasized its crowning value when he wrote that "it is man's
basic right for it is nothing less than to have rights." 1 When a person loses citizenship, therefore, the State sees to it that its reacquisition may only
be granted if the former citizen fully satisfies all conditions and complies with the applicable law. Without doubt, repatriation is not to be granted
simply based on the vagaries of the former Filipino citizen.
The Case
The instant petition for review 2 under Rule 45 of the 1997 Rules of Civil Procedure contests the denial by the Court of Appeals (CA) of the Petition
for Habeas Corpus interposed by petitioner Joevanie Arellano Tabasa from the Order of Summary Deportation issued by the Bureau of Immigration
and Deportation (BID) for his return to the United States.
The Facts
The facts as culled by the CA from the records show that petitioner Joevanie Arellano Tabasa was a natural-born citizen of the Philippines. In
1968, 3 when petitioner was seven years old, 4 his father, Rodolfo Tabasa, became a naturalized citizen 5 of the United States. By derivative
naturalization (citizenship derived from that of another as from a person who holds citizenship by virtue of naturalization 6 ), petitioner also acquired
American citizenship.
Petitioner arrived in the Philippines on August 3, 1995, and was admitted as a "balikbayan" for one year. Thereafter, petitioner was arrested and
detained by agent Wilson Soluren of the BID on May 23, 1996, pursuant to BID Mission Order No. LIV-96-72 in Baybay, Malay, Aklan; subsequently,
he was brought to the BID Detention Center in Manila. 7
Petitioner was investigated by Special Prosecutor Atty. Edy D. Donato at the Law and Investigation Division of the BID on May 28, 1996; and on the
same day, Tabasa was accused of violating Section 8, Chapter 3, Title 1, Book 3 of the 1987 Administrative Code, in a charge sheet which alleged:
1. That on 3 August 1995, respondent (petitioner herein [Tabasa]) arrived in the Philippines and was admitted as a balikbayan;
2. That in a letter dated 16 April 1996, Honorable Kevin Herbert, Consul General of [the] U.S. Embassy, informed the Bureau
that respondent's Passport No. 053854189 issued on June 10, 1994 in San Francisco, California, U.S.A., had been revoked by
the U.S. Department of State;
3. Hence, respondent [petitioner Tabasa] is now an undocumented and undesirable alien and may be summarily deported
pursuant to Law and Intelligence Instructions No. 53 issued by then Commissioner Miriam Defensor Santiago to effect his
deportation (Exhibit 3). 8
The pertinent portion of the Herbert letter is as follows:
The U.S. Department of State has revoked U.S. passport 053854189 issued on June 10, 1994 in San Francisco, California under
the name of Joevanie Arellano Tabasa, born on February 21, 1959 in the Philippines. Mr. Tabasa's passport has been revoked
because he is the subject of an outstanding federal warrant of arrest issued on January 25, 1996 by the U.S. District Court for
the Northern District of California, for violation of Section 1073, "Unlawful Flight to Avoid Prosecution," of Title 18 of the
United States Code. He is charged with one count of a felon in possession of a firearm, in violation of California Penal Code,
Section 12021(A)(1), and one count of sexual battery, in violation of California Penal Code, Section 243.4 (D). 9
The BID ordered petitioner's deportation to his country of origin, the United States, on May 29, 1996, in the following summary deportation order:
Records show that on 16 April 1996, Mr. Kevin F. Herbert, Consul General of the U.S. Embassy in Manila, filed a request with
the Bureau to apprehend and deport the abovenamed [sic] respondent [petitioner Tabasa] on the ground that a standing
warrant for several federal charges has been issued against him, and that the respondent's Passport No. 053854189 has been
revoked. AaSTIH
By reason thereof, and on the strength of Mission Order No. LIV-96-72, Intelligence operatives apprehended the respondent in
Aklan on 23 May 1996.
In Schonemann vs. Commissioner Santiago, et al., (G.R. No. 81461 [sic, '81461' should be '86461'], 30 May 1989), the Supreme
Court ruled that if a foreign embassy cancels the passport of an alien, or does not reissue a valid passport to him, the alien
loses the privilege to remain in the country. Further, under Office Memorandum Order No. 34 issued on 21 August 1989,
summary deportation proceedings lie where the passport of the alien has expired.
It is, thus, apparent that respondent has lost his privilege to remain in the country. 10
Petitioner filed before the CA a Petition for Habeas Corpus with Preliminary Injunction and/or Temporary Restraining Order 11 on May 29, 1996,
which was docketed as CA-G.R. SP No. 40771. Tabasa alleged that he was not afforded due process; that no warrant of arrest for deportation may be
issued by immigration authorities before a final order of deportation is made; that no notice of the cancellation of his passport was made by the U.S.
Embassy; that he is entitled to admission or to a change of his immigration status as a non-quota immigrant because he is married to a Filipino citizen
as provided in Section 13, paragraph (a) of the Philippine Immigration Act of 1940; and that he was a natural-born citizen of the Philippines prior to
his derivative naturalization when he was seven years old due to the naturalization of his father, Rodolfo Tabasa, in 1968.
At the time Tabasa filed said petition, he was already 35 years old. 12
On May 30, 1996, the CA ordered the respondent Bureau to produce the person of the petitioner on June 3, 1996 and show the cause of petitioner's
detention, and restrained the Bureau from summarily deporting him. On June 3, 1996, the BID presented Tabasa before the CA; and on June 6, 1996,
the CA granted both parties ten (10) days within which to file their memoranda, after which the case would be considered submitted for
decision. 13 Meanwhile, the Commissioner of Immigration granted the petitioner's temporary release on bail on a PhP20,000.00 cash bond. 14
However, on June 13, 1996, petitioner filed a Supplemental Petition alleging that he had acquired Filipino citizenship by repatriation in accordance
with Republic Act No. 8171 (RA 8171), and that because he is now a Filipino citizen, he cannot be deported or detained by the respondent Bureau. 15
The Ruling of the Court of Appeals
The CA, in its August 7, 1996 Decision, 16 denied Tabasa's petition on the ground that he had not legally and successfully acquired — by repatriation
— his Filipino citizenship as provided in RA 8171. The court said that although he became an American citizen by derivative naturalization when his
father was naturalized in 1968, there is no evidence to show that he lost his Philippine citizenship "on account of political or economic necessity," as
explicitly provided in Section 1, RA 8171 — the law governing the repatriation of natural-born Filipinos who have lost their citizenship. The affidavit
does not state that political or economic necessity was the compelling reason for petitioner's parents to give up their Filipino citizenship in 1968.
Moreover, the court a quo found that petitioner Tabasa did not dispute the truth of the April 16, 1996 letter of the United States Consul General
Kevin F. Herbert or the various warrants issued for his arrest by the United States court. The court a quo noted that after petitioner was ordered
deported by the BID on May 29, 1996, he successively executed an Affidavit of Repatriation on June 6, 1996 and took an oath of allegiance to the
Republic of the Philippines on June 13, 1996 — more than ten months after his arrival in the country on August 3, 1995. The appellate court
considered petitioner's "repatriation" as a last ditch effort to avoid deportation and prosecution in the United States. The appellate court concluded
that his only reason to want to reacquire Filipino citizenship is to avoid criminal prosecution in the United States of America. The court a quo,
therefore, ruled against Tabasa, whose petition is now before us.
The Issue
The only issue to be resolved is whether petitioner has validly reacquired Philippine citizenship under RA 8171. If there is no valid repatriation, then
he can be summarily deported for his being an undocumented alien.
The Court's Ruling
The Court finds no merit in this petition.
RA 8171, "An Act Providing for the Repatriation of Filipino Women Who Have Lost Their Philippine Citizenship by Marriage to Aliens and of Natural-
Born Filipinos," was enacted on October 23, 1995. It provides for the repatriation of only two (2) classes of persons, viz:
Filipino women who have lost their Philippine citizenship by marriage to aliens and natural-born Filipinos who have lost
their Philippine citizenship, including their minor children, on account of political or economic necessity, may reacquire
Philippine citizenship through repatriation in the manner provided in Section 4 of Commonwealth Act No. 63, as
amended: Provided, That the applicant is not a:
(1) Person opposed to organized government or affiliated with any association or group of persons who uphold and teach
doctrines opposing organized government;
(2) Person defending or teaching the necessity or propriety of violence, personal assault, or association for the predominance
of their ideas;
(3) Person convicted of crimes involving moral turpitude; or
(4) Person suffering from mental alienation or incurable contagious diseases. 17 (Emphasis supplied.)
Does petitioner Tabasa qualify as a natural-born Filipino who had lost his Philippine citizenship by reason of political or economic necessity under RA
8171?
He does not.
Persons qualified for repatriation under RA 8171
To reiterate, the only persons entitled to repatriation under RA 8171 are the following:
a. Filipino women who lost their Philippine citizenship by marriage to aliens; and
b. Natural-born Filipinos including their minor children who lost their Philippine citizenship on account of political or economic
necessity.
Petitioner theorizes that he could be repatriated under RA 8171 because he is a child of a natural-born Filipino, and that he lost his Philippine
citizenship by derivative naturalization when he was still a minor. TDAcCa
Petitioner overlooks the fact that the privilege of repatriation under RA 8171 is available only to natural-born Filipinos who lost their citizenship on
account of political or economic necessity, and to the minor children of said natural-born Filipinos. This means that if a parent who had renounced his
Philippine citizenship due to political or economic reasons later decides to repatriate under RA 8171, his repatriation will also benefit his minor
children according to the law. This includes a situation where a former Filipino subsequently had children while he was a naturalized citizen of a
foreign country. The repatriation of the former Filipino will allow him to recover his natural-born citizenship and automatically vest Philippine
citizenship on his children of jus sanguinis or blood relationship: 18 the children acquire the citizenship of their parent(s) who are natural-born
Filipinos. To claim the benefit of RA 8171, however, the children must be of minor age at the time the petition for repatriation is filed by the parent.
This is so because a child does not have the legal capacity for all acts of civil life much less the capacity to undertake a political act like the election of
citizenship. On their own, the minor children cannot apply for repatriation or naturalization separately from their parents.
In the case at bar, there is no dispute that petitioner was a Filipino at birth. In 1968, while he was still a minor, his father was naturalized as an
American citizen; and by derivative naturalization, petitioner acquired U.S. citizenship. Petitioner now wants us to believe that he is entitled to
automatic repatriation as a child of natural-born Filipinos who left the country due to political or economic necessity. This is absurd. Petitioner was
no longer a minor at the time of his "repatriation" on June 13, 1996. The privilege under RA 8171belongs to children who are of minor age at the
time of the filing of the petition for repatriation.
Neither can petitioner be a natural-born Filipino who left the country due to political or economic necessity. Clearly, he lost his Philippine citizenship
by operation of law and not due to political or economic exigencies. It was his father who could have been motivated by economic or political
reasons in deciding to apply for naturalization. The decision was his parent's and not his. The privilege of repatriation under RA 8171 is extended
directly to the natural-born Filipinos who could prove that they acquired citizenship of a foreign country due to political and economic reasons, and
extended indirectly to the minor children at the time of repatriation.
In sum, petitioner is not qualified to avail himself of repatriation under RA 8171. However, he can possibly reacquire Philippine citizenship by availing
of the Citizenship Retention and Re-acquisition Act of 2003 (Republic Act No. 9225) by simply taking an oath of allegiance to the Republic of the
Philippines.
Where to file a petition for repatriation pursuant to RA 8171
Even if we concede that petitioner Tabasa can avail of the benefit of RA 8171, still he failed to follow the procedure for reacquisition of Philippine
citizenship. He has to file his petition for repatriation with the Special Committee on Naturalization (SCN), which was designated to process petitions
for repatriation pursuant to Administrative Order No. 285 (A.O. No. 285) dated August 22, 1996, to wit:
SECTION 1. Composition. — The composition of the Special Committee on Naturalization, with the Solicitor General as
Chairman, the Undersecretary of Foreign Affairs and the Director-General of the National Intelligence Coordinating Agency, as
members, shall remain as constituted.
SECTION 2. Procedure. — Any person desirous of repatriating or reacquiring Filipino citizenship pursuant to R.A. No. 8171 shall
file a petition with the Special Committee on Naturalization which shall process the same. If their applications are
approved[,] they shall take the necessary oath of allegiance to the Republic of the Philippines, after which they shall be
deemed to have reacquired Philippine citizenship. The Commission on Immigration and Deportation shall thereupon cancel
their certificate of registration (emphasis supplied).
SECTION 3. Implementing Rules. — The Special Committee is hereby authorized to promulgate rules and regulations and
prescribe the appropriate forms and the required fees for the processing of petitions.
SECTION 4. Effectivity. — This Administrative Order shall take effect immediately.
In the Amended Rules and Regulations Implementing RA 8171 issued by the SCN on August 5, 1999, applicants for repatriation are required to submit
documents in support of their petition such as their birth certificate and other evidence proving their claim to Filipino citizenship. 19 These
requirements were imposed to enable the SCN to verify the qualifications of the applicant particularly in light of the reasons for the renunciation of
Philippine citizenship.
What petitioner simply did was that he took his oath of allegiance to the Republic of the Philippines; then, executed an affidavit of repatriation,
which he registered, together with the certificate of live birth, with the Office of the Local Civil Registrar of Manila. The said office subsequently
issued him a certificate of such registration. 20 At that time, the SCN was already in place and operational by virtue of the June 8, 1995
Memorandum issued by President Fidel V. Ramos. 21 Although A.O. No. 285 designating the SCN to process petitions filed pursuant to RA 8171 was
issued only on August 22, 1996, it is merely a confirmatory issuance according to the Court in Angat v. Republic. 22 Thus, petitioner should have
instead filed a petition for repatriation before the SCN.
Requirements for repatriation under RA 8171
Even if petitioner — now of legal age — can still apply for repatriation under RA 8171, he nevertheless failed to prove that his parents relinquished
their Philippine citizenship on account of political or economic necessity as provided for in the law. Nowhere in his affidavit of repatriation did he
mention that his parents lost their Philippine citizenship on account of political or economic reasons. It is notable that under the Amended Rules and
Regulations Implementing RA 8171, the SCN requires a petitioner for repatriation to set forth, among others, "the reason/s why petitioner lost
his/her Filipino citizenship, whether by marriage in case of Filipino woman, or whether by political or economic necessity in case of [a] natural-born
Filipino citizen who lost his/her Filipino citizenship. In case of the latter, such political or economic necessity should be specified." 23
Petitioner Tabasa asserts, however, that the CA erred in ruling that the applicant for repatriation must prove that he lost his Philippine citizenship on
account of political or economic necessity. He theorizes that the reference to 'political or economic reasons' is "merely descriptive, not restrictive, of
the widely accepted reasons for naturalization in [a] foreign country." 24
Petitioner's argument has no leg to stand on. AcaEDC
A reading of Section 1 of RA 8171 shows the manifest intent of the legislature to limit the benefit of repatriation only to natural-born Filipinos who
lost their Philippine citizenship on account of political or economic necessity, in addition to Filipino women who lost their Philippine citizenship by
marriage to aliens. The precursor of RA 8171, Presidential Decree No. 725 (P.D. 725), 25 which was enacted on June 5, 1975
amending Commonwealth Act No. 63, also gives to the same groups of former Filipinos the opportunity to repatriate but without the limiting phrase,
"on account of political or economic necessity" in relation to natural-born Filipinos. By adding the said phrase to RA 8171, the lawmakers clearly
intended to limit the application of the law only to political or economic migrants, aside from the Filipino women who lost their citizenship by
marriage to aliens. This intention is more evident in the following sponsorship speech of Rep. Andrea B. Domingo on House Bill No. 1248, the origin
of RA 8171, to wit:
Ms. Domingo: . . .
From my experience as the Commissioner of the Bureau of Immigration and Deportation, I observed that there are only four
types of Filipinos who leave the country.
The first is what we call the "economic refugees" who go abroad to work because there is no work to be found in the
country. Then we have the "political refugees" who leave the country for fear of their lives because they are not in
consonance with the prevailing policy of government. The third type is those who have committed crimes and would like to
escape from the punishment of said crimes. Lastly, we have those Filipinos who feel that they are not Filipinos, thereby seeking
other citizenship elsewhere.
Of these four types of Filipinos, Mr. Speaker, the first two have to leave the country not of choice, but rather out of sacrifice to
look for a better life, as well as for a safer abode for themselves and their families. It is for these two types of Filipinos that
this measure is being proposed for approval by this body. (Emphasis supplied.)
xxx xxx xxx
. . . [I]f the body would recall, I mentioned in my short sponsorship speech the four types of Filipinos who leave their country.
And the two types — the economic and political refugees — are the ones being addressed by this proposed law, and they are
not really Filipino women who lost their citizenship through marriage. We had a lot of problems with these people who left the
country because of political persecution or because of pressing economic reasons, and after feeling that they should come
back to the country and get back their citizenship and participate as they should in the affairs of the country, they find that it is
extremely difficult to get their citizenship back because they are treated no different from any other class of alien. 26
From these two sources, namely, P.D. 725 and the sponsorship speech on House Bill No. 1248, it is incontrovertible that the intent of our legislators
in crafting Section 1 of RA 8171, as it is precisely worded out, is to exclude those Filipinos who have abandoned their country for reasons other than
political or economic necessity.
Petitioner contends it is not necessary to prove his political or economic reasons since the act of renouncing allegiance to one's native country
constitutes a "necessary and unavoidable shifting of his political allegiance," and his father's loss of Philippine citizenship through naturalization
"cannot therefore be said to be for any reason other than political or economic necessity." 27
This argument has no merit.
While it is true that renunciation of allegiance to one's native country is necessarily a political act, it does not follow that the act is inevitably
politically or economically motivated as alleged by petitioner. To reiterate, there are other reasons why Filipinos relinquish their Philippine
citizenship. The sponsorship speech of former Congresswoman Andrea B. Domingo illustrates that aside from economic and political refugees, there
are Filipinos who leave the country because they have committed crimes and would like to escape from punishment, and those who really feel that
they are not Filipinos and that they deserve a better nationality, and therefore seek citizenship elsewhere.
Thus, assuming petitioner Tabasa is qualified under RA 8171, it is incumbent upon him to prove to the satisfaction of the SCN that the reason for his
loss of citizenship was the decision of his parents to forfeit their Philippine citizenship for political or economic exigencies. He failed to undertake this
crucial step, and thus, the sought relief is unsuccessful.
Repatriation is not a matter of right, but it is a privilege granted by the State. This is mandated by the 1987 Constitution under Section 3, Article IV,
which provides that citizenship may be lost or reacquired in the manner provided by law. The State has the power to prescribe by law the
qualifications, procedure, and requirements for repatriation. It has the power to determine if an applicant for repatriation meets the requirements of
the law for it is an inherent power of the State to choose who will be its citizens, and who can reacquire citizenship once it is lost. If the applicant, like
petitioner Tabasa, fails to comply with said requirements, the State is justified in rejecting the petition for repatriation.
Petitioner: an undocumented alien subject to summary deportation
Petitioner claims that because of his repatriation, he has reacquired his Philippine citizenship; therefore, he is not an undocumented alien subject to
deportation.
This theory is incorrect.
As previously explained, petitioner is not entitled to repatriation under RA 8171 for he has not shown that his case falls within the coverage of the
law.
Office Memorandum No. 34 dated August 21, 1989 of the BID is enlightening on summary deportation:
2. The Board of Special Inquiry and the Hearing Board IV shall observe summary deportation proceedings in cases where the
charge against the alien is overstaying, or the expiration or cancellation by his government of his passport. In cases involving
overstaying aliens, BSI and the Hearing Board IV shall merely require the presentation of the alien's valid passport and shall
decide the case on the basis thereof.
3. If a foreign embassy cancels the passport of the alien, or does not reissue a valid passport to him, the alien loses the
privilege to remain in the country, under the Immigration Act, Sections 10 and 15 (Schonemann v. Santiago, et al., G.R. No.
81461 [sic, should be '86461'], 30 May 1989). The automatic loss of the privilege obviates deportation proceedings. In such
instance, the Board of Commissioners may issue summary judgment of deportation which shall be immediately executory. 28
In addition, in the case of Schonemann v. Defensor Santiago, et al., this Court held:
It is elementary that if an alien wants to stay in the Philippines, he must possess the necessary documents. One of these
documents is a valid passport. There are, of course, exceptions where in the exercise of its sovereign prerogatives the
Philippines may grant refugee status, refuse to extradite an alien, or otherwise allow him or her to stay here even if he [the
alien] has no valid passport or Philippine visa. "Boat people" seeking residence elsewhere are examples. However, the grant of
the privilege of staying in the Philippines is discretionary on the part of the proper authorities. There is no showing of any grave
abuse of discretion, arbitrariness, or whimsicality in the questioned summary judgment. . . . 29
Petitioner Tabasa, whose passport was cancelled after his admission into the country, became an undocumented alien who can be summarily
deported. His subsequent "repatriation" cannot bar such deportation especially considering that he has no legal and valid reacquisition of Philippine
citizenship. TAacCE
WHEREFORE, this petition for review is DISMISSED, and the August 7, 1996 Decision of the Court of Appeals is AFFIRMED. No costs to the petitioner.
SO ORDERED. Quisumbing, Carpio, Carpio Morales and Tinga, JJ., concur.
||| (Tabasa v. Court of Appeals, G.R. No. 125793, [August 29, 2006], 531 PHIL 407-428)
THIRD DIVISION
[G.R. No. 132244. September 14, 1999.]
GERARDO ANGAT, petitioner, vs. REPUBLIC OF THE PHILIPPINES, respondent.
Romeo S. Gonzales for petitioner.
The Solicitor General for respondent.
SYNOPSIS
Petitioner Gerardo Angat was a natural born citizen of the Philippines. He lost his citizenship by naturalization in the United States of America. In
1991, he returned to the Philippines. On March 11, 1996, he filed before the Regional Trial Court (RTC) of Marikina City a petition to regain his status
as a citizen of the Philippines. On May 10, 1996, the Office of the Solicitor General (OSG) was notified of the initial hearing scheduled on January 27,
1997. However, on September 20, 1996, upon motion of the petitioner, he was allowed to take the Oath of Allegiance to the Republic of the
Philippines which was scheduled on October 3, 1996. On October 4, 1996, the trial judge issued an Order declaring the petitioner as repatriated and a
citizen of the Republic of the Philippines pursuant to Republic Act No. 8171. On March 19, 1997, the OSG filed a Motion asserting that the petition
itself should have been dismissed by the court a quo for lack of jurisdiction because the proper forum was the Special Committee on Naturalization
consistent with Administrative Order No. 285. On September 22, 1997, the trial court set aside its Orders dated September 20, 1996 and October 04,
1996 and dismissed the petition on the ground of lack of jurisdiction without prejudice to its refilling before the Special Committee on Naturalization.
Hence, this appeal by certiorari.
The Court ruled that when petitioner filed his petition on March 11, 1996, the Special Committee on Naturalization constituted pursuant to LOI No.
270 under P.D. No. 725 was in place. Administrative Order 285, promulgated on August 22, 1996 relative to R.A. No. 8171, in effect, was merely then
a confirmatory issuance. The Office of the Solicitor General was right in maintaining that Angat's petition should have been filed with the Committee,
aforesaid, and not with the RTC which had no jurisdiction thereover. The court's order of October 4, 1996 was thereby null and void, and it did not
acquire finality nor could be a source of right on the part of petitioner.
The petition for review was DENIED.
SYLLABUS
1. POLITICAL LAW; CONSTITUTIONAL LAW; CITIZENSHIP; REPUBLIC ACT NO. 8171; PURPOSE. — R.A. No. 8171, which has lapsed into law on 23
October 1995, is an act providing for the repatriation (a) of Filipino women who have lost their Philippine citizenship by marriage to aliens and (b) of
natural born Filipinos who have lost their Philippine citizenship on account of political or economic necessity.
2. ID.; ADMINISTRATIVE LAW; ADMINISTRATIVE AGENCY; SPECIAL COMMITTEE ON NATURALIZATION; NOT ABROGATED BY PRESIDENT AQUINO'S
MEMORANDUM. — Under Section 1 of the Presidential Decree ("P.D.") No. 725, dated 05 June 1975, amending Commonwealth Act No. 63, an
application for repatriation could be filed by Filipino women who lost their Philippine citizenship by marriage to aliens, as well as by natural born
Filipinos who lost their Philippine citizenship, with the Special Committee on Naturalization. The committee, chaired by the Solicitor General with the
Undersecretary of Foreign Affairs and the Director of the National Intelligence Coordinating Agency as the other members, was created pursuant to
Letter of Instruction ("LOI") No. 270, dated 11 April 1975, as amended by LOI No. 283 and LOI No. 491 issued, respectively, on 04 June 1975 and on
29 December 1976. Although the agency was deactivated by virtue of President Corazon C. Aquino's Memorandum of 27 March 1987, it was not,
however, abrogated. InFrivaldo vs. Commission on Elections, the Court observed that the aforedated memorandum of President Aquino had merely
directed the Special Committee on Naturalization "to cease and desist from undertaking any and all proceedings . . . under Letter of Instruction (`LOI')
270."
3. ID.; ID.; ID.; ID.; HAS JURISDICTION OVER CASE FILED ON MARCH 11, 1996; CASE AT BAR. — Indeed, the Committee was reactivated on 08 June
1995; hence, when petitioner filed his petition on 11 March 1996, the Special Committee on Naturalization constituted pursuant to LOI No.
270 under P.D. No. 725 was in place. Administrative Order 285, promulgated on 22 August 1996 relative to R.A. No. 8171, in effect, was merely then
a confirmatory issuance. The Office of the Solicitor General was right in maintaining that Angat's petition should have been filed with the Committee,
aforesaid, and not with the RTC which had no jurisdiction thereover. The court's order of 04 October 1996 was thereby null and void, and it did not
acquire finality nor could be a source of right on the part of petitioner.
4. ID.; CONSTITUTIONAL LAW; CITIZENSHIP; REPUBLIC ACT NO: 965 AND REPUBLIC ACT NO. 2630; NOT APPLICABLE IN CASE AT BAR. — It should also
be noteworthy that the petition in Case No. N-96-03-MK was one for repatriation, and it was thus incorrect for petitioner to initially invoke Republic
Act No. 965 and R.A. No. 2630 since these laws could only apply to persons who had lost their citizenship by rendering service to, or accepting
commission in, the armed forces of an allied foreign country or the armed forces of the United States of America, a factual matter not alleged in the
petition. Parenthetically, under these statutes, the person desiring to re-acquire Philippine citizenship would noteven be required to file a petition in
court, and all that he had to do was to take an oath of allegiance to the Republic of the Philippines and to register the fact with the civil registry in the
place of his residence or where he had last resided in the Philippines. EcASIC
DECISION
VITUG, J p:
The instant petition for review under Rule 45 assails the orders, dated 22 September 1997 and 29 December 1997, issued by the Regional Trial Court
("RTC") of Marikina City in Case No. N-96-03-MK, entitled "In the Matter of the Petition of Gerardo Angat y Legaspi to be Re-admitted as a Citizen of
the Philippines under Commonwealth Act No. 63, as amended, and Republic Act ("R.A.") No. 965 and 263[0]." cda
Petitioner Gerardo Angat was a natural born citizen of the Philippines until he lost his citizenship by naturalization in the United States of America.
Now residing at No. 69 New York Street, Provident Village, Marikina City, Angat filed on 11 March 1996 before the RTC of Marikina City, Branch 272,
a petition to regain his status as a citizen of the Philippines under Commonwealth Act No. 63, Republic Act No. 965 and Republic Act No. 2630
(docketed as N-96-03-MK). In his petition, "applying for naturalization," he averred that —
"FIRST. — His full name is GERARDO LEGASPI ANGAT. Copy of his latest picture is hereto attached and made an integral part of
this petition.
"SECOND. — His present place of residence is #69 New York St., Provident Village, Marikina, Metro Manila and his former
residence was in Las Vegas, U.S.
'THIRD. — His trade or profession is in buy and sell and managing the properties of his parents which he has been engaged
since his arrival here in the Philippines.
"FOURTH. — He was born on the 22nd day of June 1954 at Tondo, Manila. He was formerly a citizen of the Philippines. He lost
his Philippine citizenship by naturalization in a foreign country. He is at present a citizen or subject of the United States of
America. Copy of his birth certificate is hereto attached as Annex 'A.' cda
"FIFTH. — He is newly married to Zenaida Lim who was born in Tondo, Manila and now resides at petitioner's residence at
Marikina, Metro Manila. Copy of their marriage contract is hereto attached as Annex 'B.'
"SIXTH. — He returned to the Philippines from the United States of America in 1991. Copy of his alien registration is hereto
attached as Annex 'C.'
"SEVENTH. — He has the qualifications required by Commonwealth Act No. 63 as amended, and Republic Act Nos. 965 and
2639 to reacquire Philippine citizenship, and possesses none of the disqualification prescribed in Commonwealth Act No. 473.
He has resided in the Philippines at least six months immediately preceding the date of this petition, to wit: since 1991. He has
conducted himself in a proper and irreproachable manner during the entire period of his residence in the Philippines, in his
relations with the constituted government as well as with the community in which he is living.
"EIGHT. — He is not opposed to an organized government or affiliated with any association or group of persons who uphold
and teach doctrines opposing all organized government. He is not defending or teaching the necessity or propriety of violence,
personal assault or assassination for the success and predominance of men's ideas. He is not a polygamist or believer in the
practice of polygamy. He has not been convicted of any crime involving moral turpitude. He is not suffering from any mental
alienation or incurable contagious disease. The nation of which he is a citizen or subject is not at war with the Philippines.
"NINTH. — It is his intention to reacquire Philippine citizenship and to renounce absolutely and forever all allegiance and
fidelity to any foreign prince, potentate, state, or sovereignty, and particularly to the United States of America to which at this
time he is a citizen." 1
On 30 April 1996, the trial court, through the branch clerk of court, issued a notice setting the case for initial hearing on 27 January 1997 2 which,
along with the petition and its annexes, was received by the Office of the Solicitor General ("OSG") on 10 May 1996.
On 13 June 1996, petitioner sought to be allowed to take his oath of allegiance to the Republic of the Philippines pursuant to R.A. 8171. The motion
was denied by the trial judge in his order of 12 July 1996. Another motion filed by petitioner on 13 August 1996 to have the denial reconsidered was
found to be meritorious by the court a quo in an order, dated 20 September 1996, which stated, among other things, that —

"A close scrutiny of R.A. 8171 shows that petitioner is entitled to the benefits of the said law considering that herein petitioner
is a natural born Filipino citizen who lost his citizenship by naturalization in a foreign country. The petition and motion of the
petitioner to take his oath of allegiance to the Republic of the Philippines likewise show that the petitioner possesses all the
qualifications and none of the disqualifications under R.A. 8171." 3
Concluding, the court ruled:
"WHEREFORE, foregoing premises considered, the Order of the Court dated July 12, 1996 is hereby set aside. The petitioner is
ordered to take his oath of allegiance to the Republic of the Philippines pursuant to R.A. 8171 before the undersigned on
October 03, 1996 at 11:00 in the morning. prLL
"SO ORDERED." 4
After taking his Oath of Allegiance on 03 October 1996, another order was issued by the trial judge on 04 October 1996 to the following effect, viz:
"After the oath of allegiance to the Republic of the Philippines had been taken by the petitioner, Gerardo Angat y Legaspi
before the undersigned, the petitioner is hereby repatriated and declared as citizen of the Republic of the Philippines pursuant
to Republic Act No. 8171.
"The Bureau of Immigration is ordered to cancel the pertinent alien certificate of registration and issue the certificate of
identification as Filipino citizen to the petitioner upon the finality of this order.
"Likewise, let a copy of this Order be registered in the Local Civil Registry of the Municipality of Marikina, Metro Manila and the
General Civil Registrar, Sta. Mesa, Manila, after its finality.
"SO ORDERED." 5
On 19 March 1997, a Manifestation and Motion (virtually a motion for reconsideration) filed by the OSG asserted that the petition itself should have
been dismissed by the courta quo for lack of jurisdiction because the proper forum for it was the Special Committee on Naturalization consistently
with Administrative Order No. 285 ("AO 285"), dated 22 August 1996 issued by President Fidel V. Ramos. AO 285 had tasked the Special Committee
on Naturalization to be the implementing agency of R.A. 8171. The motion was found to be well taken by the trial court; thus, in an order, dated 22
September 1997, it adjudged:
"This resolves the Manifestation and Motion filed by the Office of the Solicitor General on March 19, 1997.
"The motion alleges that pursuant to Administrative Order No. 285 dated August 22, 1996 issued by President Fidel V. Ramos,
any person desirous of repatriating or reacquiring Filipino citizenship pursuant to R.A. 8171 shall file a petition with the Special
Committee on Naturalization, which is composed of the Solicitor General as Chairman, the Undersecretary of Foreign Affairs
and the Director-General of the National Intelligence Coordinating Agency, as members, which shall process the application;
that if their applications are approved they shall take the necessary oath of allegiance to the Republic of the Philippines, after
which they shall be deemed to have reacquired their Philippine citizenship and the Commission on Immigration and
Deportation shall thereupon cancel their certificate of registration.
"The motion prays that the herein petition be dismissed on the ground that the same should be filed with the Special
Committee on Naturalization. Cdpr
"The records show that on September 20, 1996, the Court granted the herein petition and as a consequence thereof, the
petitioner Gerardo Angat y Legaspi took his oath of allegiance to the Republic of the Philippines before the Presiding Judge of
this Court on October 03, 1996 and on October 04, 1996, the petitioner was ordered repatriated and declared as citizen of the
Philippines.
"On February 21, 1997, the Office of the Solicitor General entered its appearance as counsel of the State in the subject petition
and on March 19, 1997 filed the herein manifestation and motion.
"The allegations in the manifestation and motion of the Office of the Solicitor General clearly shows that this Court has no
jurisdiction over the herein petition as the same falls within the jurisdiction of the Special Committee on Naturalization.
Considering that this court has no jurisdiction over this case, the order granting the same is therefore null and void.
"WHEREFORE, foregoing premises considered, the motion to dismiss filed by the Office of the Solicitor General is hereby
granted. The orders of this Court dated September 20, 1996 and October 04, 1996 are hereby set aside and the herein petition
is ordered DISMISSED on the ground of lack of jurisdiction without prejudice to its re-filing before the Special Committee on
Naturalization.
"SO ORDERED." 6
A motion for reconsideration, filed by petitioner on 13 October 1997, questioned the aforequoted order asseverating that since his petition was filed
on 14 March 1996, or months before the Special Committee on Naturalization was constituted by the President under AO 285 on 22 August 1996,
the court a quo had the authority to take cognizance of the case.
In the Order, dated 29 December 1997, the trial judge denied the motion for reconsideration.
The instant appeal by certiorari under Rule 45 of the 1997 Rules of Procedure submits the lone assignment of error that —
"The Regional Trial Court (has) seriously erred in dismissing the petition by giving retroactive effect to Administrative Order No.
285, absent a provision on Retroactive Application."
Petitioner would insist that the trial court had jurisdiction over his petition for naturalization 7 filed on 11 March 1996, and that he had acquired a
vested right as a repatriated citizen of the Philippines when the court declared him repatriated following the order, dated 20 September 1996,
allowing him to take an oath of allegiance to the Republic of the Philippines which was, in fact, administered to him on 03 October 1996. LLphil
The contention is not meritorious.
R.A. No. 8171, which has lapsed into law on 23 October 1995, is an act providing for the repatriation (a) of Filipino women who have lost their
Philippine citizenship by marriage to aliens and (b) of natural-born Filipinos who have lost their Philippine citizenship on account of political or
economic necessity. The pertinent provisions of the law read:
"SECTION 1. Filipino women who have lost their Philippine citizenship by marriage to aliens and natural-born Filipinos who
have lost their Philippine citizenship, including their minor children, on account of political or economic necessity, may
reacquire Philippine citizenship through repatriation in the manner provided in Section 4 ofCommonwealth Act No. 63, as
amended: Provided, That the applicant is not a:
"(1) Person opposed to organized government or affiliated with any association or group of persons who uphold and teach
doctrines opposing organized government;
"(2) Person defending or teaching the necessity or propriety of violence, personal assault, or association for the predominance
of their ideas;
"(3) Person convicted of crimes involving moral turpitude; or
"(4) Person suffering from mental alienation or incurable contagious diseases. cdrep
"SECTION 2. Repatriation shall be effected by taking the necessary oath of allegiance to the Republic of the Philippines and
registration in the proper civil registry and in the Bureau of Immigration. The Bureau of Immigration shall thereupon cancel the
pertinent alien certificate of registration and issue the certificate of identification as Filipino citizen to the repatriated citizen."
Under Section 1 of Presidential Decree ("P.D.") No. 725, 8 dated 05 June 1975, amending Commonwealth Act No. 63, an application for repatriation
could be filed by Filipino women who lost their Philippine citizenship by marriage to aliens, as well as by natural born Filipinos who lost their
Philippine citizenship, with the Special Committee on Naturalization. The committee, chaired by the Solicitor General with the Undersecretary of
Foreign Affairs and the Director of the National Intelligence Coordinating Agency as the other members, was created pursuant to Letter of Instruction
("LOI") No. 270, dated 11 April 1975, as amended by LOI No. 283 and LOI No. 491 issued, respectively, on 04 June 1975 and on 29 December 1976.
Although the agency was deactivated by virtue of President Corazon C. Aquino's Memorandum of 27 March 1987, it was not,
however,abrogated. In Frivaldo vs. Commission on Elections, 9 the Court observed that the aforedated memorandum of President Aquino had merely
directed the Special Committee on Naturalization "to cease and desist from undertaking any and all proceedings . . . under Letter of Instruction ('LOI')
270." 10 The Court elaborated:
"This memorandum dated March 27, 1987 cannot by any stretch of legal hermeneutics be construed as a law sanctioning or
authorizing a repeal of P.D. No. 725. Laws are repealed only by subsequent ones and a repeal may be express or implied. It is
obvious that no express repeal was made because then President Aquino in her memorandum — based on the copy furnished
us by Lee — did not categorically and/or impliedly state that P.D. 725 was being repealed or was being rendered without any
legal effect. In fact, she did not even mention it specifically by its number or text. On the other hand, it is a basic rule of
statutory construction that repeals by implication are not favored. An implied repeal will not be allowed 'unless it is
convincingly and unambiguously demonstrated that the two laws are clear repugnant and patently inconsistent that they
cannot co-exist.' cdll
"The memorandum of then President Aquino cannot even be regarded as a legislative enactment, for not every
pronouncement of the Chief Executive even under the Transitory Provisions of the 1987 Constitution can nor should be
regarded as an exercise of her law-making powers. At best, it could be treated as an executive policy addressed to the Special
Committee to halt the acceptance and processing of applications for repatriation pending whatever 'judgment the first
Congress under the 1987 Constitution' might make. In other words, the former President did not repeal P.D. 725 but left it to
the first Congress — once created — to deal with the matter. If she had intended to repeal such law, she should have
unequivocally said so instead of referring the matter to Congress. The fact is she carefully couched her presidential issuance in
terms that clearly indicated the intention of 'the present government, in the exercise of prudence and sound discretion' to
leave the matter of repeal to the new Congress. Any other interpretation of the said Presidential Memorandum, such as is now
being proffered to the Court by Lee, would visit unmitigated violence not only upon statutory construction but on common
sense as well." 11

Indeed, the Committee was reactivated on 08 June 1995; 12 hence, when petitioner filed his petition on 11 March 1996, the Special Committee
on Naturalization constituted pursuant to LOI No. 270 under P.D. No. 725 was in place. Administrative Order 285, 13 promulgated on 22 August
1996 relative to R.A. No. 8171, in effect, was merely then a confirmatory issuance.
The Office of the Solicitor General was right in maintaining that Angat's petition should have been filed with the Committee, aforesaid, and not with
the RTC which had no jurisdiction thereover. The court's order of 04 October 1996 was thereby null and void, and it did not acquire finality 14 nor
could be a source of right on the part of petitioner. 15 It should also be noteworthy that the petition in Case No. N-96-03-MK was one for
repatriation, and it was thus incorrect for petitioner to initially invoke Republic Act No. 965 16 andR.A. No. 2630 17 since these laws could only apply
to persons who had lost their citizenship by rendering service to, or accepting commission in, the armed forces of an allied foreign country or the
armed forces of the United States of America, a factual matter not alleged in the petition. Parenthetically, under these statutes, the person desiring
to re-acquire Philippine citizenship would not even be required to file a petition in court, and all that he had to do was to take an oath of allegiance to
the Republic of the Philippines and to register that fact with the civil registry in the place of his residence or where he had last resided in the
Philippines. LLphil
WHEREFORE, the petition for review is DENIED, and the Order, dated 22 September 1996, issued by the court a quo, dismissing the petition of
petitioner in Civil Case No. N-96-03-MK for want of jurisdiction, is AFFIRMED. No costs.
SO ORDERED.
Melo, Panganiban, Purisima and Gonzaga-Reyes, JJ., concur.
||| (Angat v. Republic, G.R. No. 132244, [September 14, 1999], 373 PHIL 217-230)
EN BANC
[G.R. No. 163256. November 10, 2004.]
CICERON P. ALTAREJOS, petitioner, vs. COMMISSION ON ELECTIONS, JOSE ALMIÑE and VERNON VERSOZA, respondents.
DECISION
AZCUNA, J p:
This is a petition for certiorari, with prayer for the issuance of a temporary restraining order and/or a writ of prohibitory and mandatory injunction, to
set aside the Resolution promulgated by the Commission on Elections (COMELEC), First Division, on March 22, 2004 disqualifying petitioner Ciceron
P. Altarejos from running as mayor of San Jacinto, Masbate, and another resolution of the COMELEC en banc promulgated on May 7, 2004 denying
petitioner's motion for reconsideration.
The factual antecedents are as follows:
Petitioner Altarejos was a candidate for mayor in the Municipality of San Jacinto, Masbate in the May 10, 2004 national and local elections.
On January 15, 2004, private respondents Jose Almiñe Altiche and Vernon Versoza, registered voters of San Jacinto, Masbate, filed with the
COMELEC, a petition to disqualify and to deny due course or cancel the certificate of candidacy of petitioner on the ground that he is not a Filipino
citizen and that he made a false representation in his certificate of candidacy that "[he] was not a permanent resident of or immigrant to a foreign
country."
Private respondents alleged that based on a letter 1 from the Bureau of Immigration dated June 25, 2001, petitioner was a holder of a permanent
U.S. resident visa, an Alien Certificate of Registration No. E139507 issued on November 3, 1997, and an Immigration Certificate of Residence No.
320846 issued on November 3, 1997 by the Bureau of Immigration. 2
On January 26, 2004, petitioner filed an Answer 3 stating, among others, that he did not commit false representation in his application for candidacy
as mayor because as early as December 17, 1997, he was already issued a Certificate of Repatriation by the Special Committee on Naturalization,
after he filed a petition for repatriation pursuant to Republic Act No. 8171. Thus, petitioner claimed that his Filipino citizenship was already restored,
and he was qualified to run as mayor in the May 10, 2004 elections. Petitioner sought the dismissal of the petition. cEaTHD
On the date of the hearing, the parties were required to submit their Memoranda within three days. Private respondents filed their Memorandum,
while petitioner did not file one within the required period. 4 Petitioner, however, filed a Reply Memorandum 5 subsequently.
Atty. Zacarias C. Zaragoza, Jr., regional election director for Region V and hearing officer of this case, recommended that petitioner Altarejos be
disqualified from being a candidate for the position of mayor of San Jacinto, Masbate in the May 10, 2004 national and local elections. He found,
thus:
xxx xxx xxx
The provisions of law governing the qualifications and disqualifications of elective local officials are found in Sections 39 and 40
of Republic Act No. 7160 otherwise known as theLocal Government Code of 1991, which provide as follows:
SEC. 39. Qualifications. — (a) An elective local official must be a citizen of the Philippines; a registered voter in the
barangay, municipality, city or province or, in the case of member of the sangguniang panlalawigan, sangguniang
panlungsod, or sangguniang bayan, the district where he intends to be elected; a resident therein for at least one (1)
year immediately preceding the day of the election; and able to read and write Filipino or any other local language or
dialect.
xxx xxx xxx.
(c) Candidates for the position of mayor or vice-mayor of independent component cities, component cities or
municipalities must be at least twenty-one (21) years of age on election day.
[SEC. 40. Disqualifications. — The following persons are disqualified from running for any elective position:]
xxx xxx xxx.
(d) Those with dual citizenship.
xxx xxx xxx.
(f) Permanent residents in a foreign country or those who have acquired the right to reside abroad and continue to
avail of the same right after the effectivity of this Code; . . .
Under the terms of the above quoted statutory provisions, it is required that an elective local official must be a citizen of the
Philippines, and he must not have a dual citizenship; must not be a permanent resident in a foreign country or must not have
acquired the right to reside abroad.
In the present case, it has been established by clear and convincing evidence that respondent is a citizen of the United States of
America. Such fact is proven by his Alien Certificate of Registration (ACR) No. E139507 issued on 3 November 1997 and
Immigration Certificate of Residence (ICR) with No. 320846 issued on 3 November 1997 by the Alien Registration Division,
Bureau of Immigration and Deportation. This was further confirmed in a letter dated 25 June 2001 of then Commissioner
ANDREA D. DOMINGO of the Bureau of Immigration and Deportation.
Although respondent had petitioned for his repatriation as a Filipino citizen under Republic Act No. 8171 on 17 December
1997, this did not restore to respondent his Filipino citizenship, because Section 2 of the aforecited Republic Act No.
8171 specifically provides that "repatriation shall be effected by taking the necessary oath of allegiance to the Republic of the
Philippines and registration in the proper civil registry and in the Bureau of Immigration." SDAcaT
It appears from the records of this case that respondent failed to prove that he has fully complied with requirements of the
above-quoted Section 2 of Republic Act 8171 to perfect his repatriation and reacquire his Filipino citizenship. Respondent has
not submitted any document to prove that he has taken his oath of allegiance to the Republic of the Philippines and that he
has registered his fact of repatriation in the proper civil registry and in the Bureau of Immigration. In fact, in a letter date 25
June 2001, Commissioner ANDREA DOMINGO stated that RESPONDENT is still a holder of visa under Section 13 (g) of the
Philippine Immigration Act of 1940 as amended, with an indefinite authorized stay in the Philippines, implying that respondent
did not register his supposed Certificate of Repatriation with the Bureau of Immigration otherwise his Alien Visa would have
already been cancelled. The rule is that in case of doubt concerning the grant of citizenship, such doubt should be resolved in
favor of the State and against the applicant (Cheng vs.Republic, L-16999, 22 June 1965).
xxx xxx xxx
Not having been able to prove that he has fully reacquired his Filipino citizenship after being naturalized as a citizen of the
United States, it is clear that respondent is not qualified to be candidate for the position of Mayor of San Jacinto, Masbate, in
the 10 May 2004 National and Local Elections, pursuant to the aforequoted Sections 39 and 40 of the Local Government Code
of 1991.
As a further consequence of his not being a Filipino citizen, respondent has also committed false representation in his
certificate of candidacy by stating therein that he is a natural-born Filipino citizen, when in fact, he has not yet even perfected
the reacquisition of Filipino citizenship. Such false representation constitutes a material misrepresentation as it relates to his
qualification as a candidate for public office, which could be a valid ground for the cancellation of his certificate of candidacy
under Section 78 of the Omnibus Election Code . . . 6
In its Resolution promulgated on March 22, 2004, the COMELEC, First Division, adopted the findings and recommendation of Director Zaragoza. The
dispositive portion of said Resolution stated, thus:
WHEREFORE, premises considered, respondent CICERON PEREZ ALTAREJOS is hereby disqualified to run as Mayor of San
Jacinto, Masbate. Accordingly, his certificate of candidacy for the position of Municipal Mayor of San Jacinto, Masbate is
denied due course and cancelled and his name deleted from the certified list of candidates for the May 10, 2004 elections. 7
On March 25, 2004, petitioner filed a motion for reconsideration and attached the following documents to prove that he had completed all the
requirements for repatriation which thus entitled him to run for an elective office, viz:
(1) Oath of Allegiance dated December 17, 1997;
(2) Identification Certificate No. 116543 issued by the Bureau of Immigration on March 1, 2004;
(3) Certification from the City Civil Registration Office, Makati City, that the Certificate of Repatriation and Oath of Allegiance of
petitioner was received by said office and registered, with the corresponding fee paid, on February 18, 2004;
(4) A letter dated December 17, 1997 from the Special Committee on Naturalization to the Bureau on Immigration and
Deportation that it was furnishing said office with the Oath of Allegiance and Certificate of Repatriation of petitioner
for the cancellation of petitioner's registration in said office as an alien, and the issuance to him of the corresponding
Identification Card as Filipino citizen;
(5) A letter dated December 17, 1997 from the Special Committee on Naturalization to the Local Registrar of San Jacinto,
Masbate that it was sending petitioner's Oath of Allegiance and Certificate of Repatriation for registration in their
records and for petitioner's reacquisition of his former Philippine citizenship.
On May 7, 2004, the COMELEC en banc promulgated a resolution denying the motion for reconsideration, the dispositive portion of which reads:
WHEREFORE, premises considered, the Commission (En Banc) RESOLVED as it hereby RESOLVES to DENY the Motion for
Reconsideration for UTTER LACK OF MERIT and AFFIRMS the Resolution of the First Division. 8
The Comelec en banc held, thus:
The Comelec Rules of Procedure provides that insufficiency of evidence to justify the decision is a ground for a motion for
reconsideration (Rule 19, Section 1). The evidence referred to in the above provision and to be considered in the Motion for
Reconsideration are those which were submitted during the hearing and attached to the respective Memoranda of the parties
which are already part of the records of the case. In this regard, the evidence of the respondent were not able to overcome the
evidence of the petitioners. IcDHaT

When the entire records of the case was forwarded to the Commission (First Division) the respondent's only evidence was his
Certificate of Repatriation dated 17 December 1977 and marked as Annex 1 of his answer. This piece of evidence was not
enough to controvert the evidence of the petitioners which consist of the letter of the then Bureau of Immigration
Commissioner Andrea Domingo dated 25 June 2001 which stated that as of the even date respondent is a holder of permanent
resident visa (page 15 of the records) and the certification of Josephine C. Camata dated 28 January 2004 certifying, that the
name of the respondent could not be found in the records of repatriation. (page 42 of the records) The questioned resolution,
is therefore, in order as the evidence submitted by the respondent were insufficient to rebut the evidence of the petitioner.
Now, the respondent, in his Motion for Reconsideration, attempted to introduce to the record new pieces of evidence, which
introduction is not anymore allowed in a Motion for Reconsideration. These are the following a) Annex "2" — Oath of
Allegiance; b) Annex "3" — Bureau of Immigration Identification Certificate; c) Annex "4" — Certification of the City Civil
Registrar of Makati City; d) Annex "5" — Letter addressed to the Local Civil Registrar of San Jacinto, Masbate by Aurora
P. Cortes of Special Committee on Naturalization; and e) Annex "6" — Letter addressed to the Bureau of Immigration and
Deportation by Aurora P. Cortes of Special Committee on Naturalization.
Assuming that the new evidence of the respondent are admitted, with more reason should we cancel his certificate of
candidacy for his act of [misrepresenting] himself as a Filipino citizen when at the time he filed his certificate of candidacy, he
has not yet perfected the process of repatriation. He failed to comply with the requirements under Section 2 of [Republic Act
No.] 8171 which provides that repatriation shall be effected by taking the necessary oath of allegiance to the Republic of the
Philippines and registration in the proper civil registry and in the Bureau of Immigration.
The certification was issued by the same Ms. Josephine C. Camata, City Civil Registrar, dated February 18, 2004. This
time, she certifies that Ciceron Perez Altarejos was registered under Registry No. 1, Page 19, Book No. 1, Series of 2004 and
paid under OR nos. 88325/8833256 dated February 18, 2004. (page 65 of the records). Obviously, he was able to register in the
proper civil registry only on February 18, 2004.
The respondent was able to register with the Bureau of Immigration only on March 1, 2004 as evidenced by the Bureau of
Immigration Identification Certificate attached to the Motion as Annex "3."
This fact confirms the finding of the Commission (First Division) that at the time respondent filed his certificate of candidacy he
is yet to complete the requirement under section two (2) of RA 8171.
As a consequence of not being a Filipino citizen, he has committed false representation in his certificate of candidacy. Such
false representation constitutes a material misrepresentation as it relates to his qualification as a candidate. As such the
certificate of candidacy may be cancelled on such ground. (Ycain vs. Caneja, 18 Phil. 778) 9
On May 10, 2004, the election day itself, petitioner filed this petition praying that: (1) The petition be given due course and a temporary restraining
order and/or writ of preliminary injunction be issued ex parte restraining the respondents and all persons acting on their behalf, from fully
implementing the questioned COMELEC Resolutions promulgated on March 22, 2004 and May 7, 2004; (2) a writ of preliminary mandatory injunction
be issued ordering the COMELEC and all persons acting on its behalf to allow petitioner to run as Mayor of San Jacinto, Masbate in the May 10, 2004
elections, and to count and canvass the votes cast in his favor and to proclaim him as the winning mayor of San Jacinto, Masbate; and (3) after
proper proceedings, judgment be rendered declaring null and void and setting aside the COMELEC Resolutions promulgated on March 22, 2004 and
May 7, 2004 and other related Orders of the COMELEC or its representatives which have the effect of illegally preventing petitioner from running as
Mayor of San Jacinto, Masbate. cCAIaD
In its Comment, 10 the Office of the Solicitor General stated that, based on the information relayed to it by the COMELEC, petitioner's name, as a
mayoralty candidate in San Jacinto, Masbate, was retained in the list of candidates voted upon by the electorate in the said municipality. Hence, the
cancellation of petitioner's certificate of candidacy was never implemented. The COMELEC also informed the Office of the Solicitor General that
petitioner's opponent, Dr. Emilio Aris V. Espinosa, was already proclaimed duly elected Mayor of San Jacinto, Masbate.
The Office of the Solicitor General contends that said supervening event has rendered the instant petition moot and academic, and it prayed for the
dismissal of the petition.
In his Reply, 11 petitioner opposed the dismissal of his petition. He claims that the COMELEC resolutions disqualifying him from running as a
mayoralty candidate adversely affected his candidacy, since his supporters were made to believe that his votes would not be counted. Moreover, he
stated that said COMELEC resolutions cast a doubt on his Philippine citizenship.
Petitioner points out that he took his Oath of Allegiance to the Republic of the Philippines on December 17, 1997. In view thereof, he ran and was
even elected as Mayor of San Jacinto, Masbate during the 1998 elections. He argues that if there was delay in the registration of his Certificate of
Repatriation with the Bureau of Immigration and with the proper civil registry, the same was brought about by the inaction on the part of said offices
since the records of the Special Committee on Naturalization show that his Certificate of Repatriation and Oath of Allegiance have long been
transmitted to said offices.
Petitioner also asserts that the subsequent registration of his Certificate of Repatriation with the Bureau of Immigration and with the Civil Registry of
Makati City prior to the May 10, 2004 elections has the effect of curing the defect, if any, in the reacquisition of his Filipino citizenship as his
repatriation retroacted to the date of his application for repatriation as held in Frivaldo v. Comelec.
The pertinent issues raised are the following: (1) Is the registration of petitioner's repatriation with the proper civil registry and with the Bureau of
Immigration a prerequisite in effecting repatriation; and (2) whether or not the COMELEC en banc committed grave abuse of discretion amounting to
excess or lack of jurisdiction in affirming the Resolution of the COMELEC, First Division.
As stated by the Office of the Solicitor General, where the issues have become moot and academic, there is no justiciable controversy, thereby
rendering the resolution of the same of no practical use or value. 12 Nonetheless, courts will decide a question otherwise moot and academic if it is
capable of repetition, yet evading review. 13
First Issue: Is the registration of petitioner's repatriation
with the proper civil registry and with the Bureau of
Immigration a prerequisite in effecting repatriation?
The provision of law applicable in this case is Section 2 of Republic Act No. 8171, 14 thus:
SEC. 2. Repatriation shall be effected by taking the necessary oath of allegiance to the Republic of the Philippines and
registration in the proper civil registry and in the Bureau of Immigration. The Bureau of Immigration shall thereupon cancel the
pertinent alien certificate of registration and issue the certificate of identification as Filipino citizen to the repatriated citizen.
The law is clear that repatriation is effected "by taking the oath of allegiance to the Republic of the Philippines and registration in the proper civil
registry and in the Bureau of Immigration." Hence, in addition to taking the Oath of Allegiance to the Republic of the Philippines, the registration of
the Certificate of Repatriation in the proper civil registry and the Bureau of Immigration is a prerequisite in effecting the repatriation of a
citizen. aSTcCE
In this case, petitioner took his Oath of Allegiance on December 17, 1997, but his Certificate of Repatriation was registered with the Civil Registry of
Makati City only after six years or on February 18, 2004, and with the Bureau of Immigration on March 1, 2004. Petitioner, therefore, completed all
the requirements of repatriation only after he filed his certificate of candidacy for a mayoralty position, but before the elections.
When does the citizenship qualification of a candidate for an elective office apply?
In Frivaldo v. Commission on Elections, 15 the Court ruled that the citizenship qualification must be construed as "applying to the time of
proclamation of the elected official and at the start of his term." The Court, through Justice Artemio V. Panganiban, discussed, thus:
Under Sec. 39 of the Local Government Code, "(a)n elective local official must be:
* a citizen of the Philippines;
* a registered voter in the barangay, municipality, city, or province . . . where he intends to be elected;
* a resident therein for at least one (1) year immediately preceding the day of the election;
* able to read and write Filipino or any other local language or dialect."
* In addition, "candidates for the position of governor . . . must be at least twenty-three (23) years of age on election
day."
From the above, it will be noted that the law does not specify any particular date or time when the candidate must possess
citizenship, unlike that for residence (which must consist of at least one year's residency immediately preceding the day of
election) and age (at least twenty three years of age on election day).

Philippine citizenship is an indispensable requirement for holding an elective public office, and the purpose of the citizenship
qualification is none other than to ensure that no alien, i.e., no person owing allegiance to another nation, shall govern our
people and our country or a unit of territory thereof. Now, an official begins to govern or to discharge his functions only upon
his proclamation and on the day the law mandates his term of office to begin. Since Frivaldo re-assumed his citizenship on June
30, 1995 — the very day the term of office of governor (and other elective officials) began — he was therefore already
qualified to be proclaimed, to hold such office and to discharge the functions and responsibilities thereof as of said date. In
short, at that time, he was already qualified to govern his native Sorsogon. This is the liberal interpretation that should give
spirit, life and meaning to our law on qualifications consistent with the purpose for which such law was enacted. . . .
Paraphrasing this Court's ruling in Vasquez v. Giap and Li Seng Giap & Sons, if the purpose of the citizenship requirement is to
ensure that our people and country do not end up being governed by aliens, i.e., persons owing allegiance to another nation,
that aim or purpose would not be thwarted but instead achieved by construing the citizenship qualification as applying to the
time of proclamation of the elected official and at the start of his term. 16 (Emphasis supplied.)
Moreover, in the case of Frivaldo v. Commission on Elections, the Court ruled that "the repatriation of Frivaldo RETROACTED to the date of the filing
of his application." In said case, the repatriation of Frivaldo was by virtue of Presidential Decree No. 725, which took effect on June 5, 1975. The
Court therein declared that Presidential Decree No. 725 was a curative statute, which is retroactive in nature. The retroactivity of Frivaldo's
repatriation to the date of filing of his application was justified by the Court, thus:
xxx xxx xxx
. . . The reason for this is simply that if, as in this case, it was the intent of the legislative authority that the law should apply
to past events — i.e., situations and transactionsexisting even before the law came into being — in order to benefit the
greatest number of former Filipinos possible thereby enabling them to enjoy and exercise the constitutionally guaranteed right
of citizenship, and such legislative intention is to be given the fullest effect and expression, then there is all the more reason to
have the law apply in a retroactive or retrospective manner to situations, events and transactions subsequent to the passage of
such law. That is, the repatriation granted to Frivaldo . . . can and should be made to take effect as of date of his application. As
earlier mentioned, there is nothing in the law that would bar this or would show a contrary intention on the part of the
legislative authority; and there is no showing that damage or prejudice to anyone, or anything unjust or injurious would result
from giving retroactivity to his repatriation. Neither has Lee shown that there will result the impairment of any contractual
obligation, disturbance of any vested right or breach of some constitutional guaranty.IHCacT
xxx xxx xxx
Another argument for retroactivity to the date of filing is that it would prevent prejudice to applicants. If P.D. 725 were not to
be given retroactive effect, and the Special Committee decides not to act, i.e., to delay the processing of applications for any
substantial length of time, then the former Filipinos who may be stateless, as Frivaldo — having already renounced his
American citizenship — was, may be prejudiced for causes outside their control. This should not be. In case of doubt in the
interpretation or application of laws, it is to be presumed that the law-making body intended right and justice to prevail. 17
Republic Act No. 8171 18 has impliedly repealed Presidential Decree No. 725. They cover the same subject matter: Providing for the repatriation of
Filipino women who have lost their Philippine citizenship by marriage to aliens and of natural-born Filipinos. The Court's ruling in Frivaldo
v. Commission on Elections that repatriation retroacts to the date of filing of one's application for repatriation subsists for the same reasons quoted
above.
Accordingly, petitioner's repatriation retroacted to the date he filed his application in 1997. Petitioner was, therefore, qualified to run for a mayoralty
position in the government in the May 10, 2004 elections. Apparently, the COMELEC was cognizant of this fact since it did not implement the assailed
Resolutions disqualifying petitioner to run as mayor of San Jacinto, Masbate.
Second Issue: Whether or not the COMELEC en banc
gravely abused its discretion in affirming the
Resolution of the COMELEC, First Division?
The Court cannot fault the COMELEC en banc for affirming the decision of the COMELEC, First Division, considering that petitioner failed to prove
before the COMELEC that he had complied with the requirements of repatriation. Petitioner submitted the necessary documents proving compliance
with the requirements of repatriation only during his motion for reconsideration, when the COMELEC en banc could no longer consider said
evidence. As the COMELEC en banc correctly stated:
The Comelec Rules of Procedure provides that insufficiency of evidence to justify the decision is a ground for a motion for
reconsideration (Rule 19, Section 1). The evidence referred to in the above provision and to be considered in the Motion for
Reconsideration are those which were submitted during the hearing and attached to the respective Memoranda of the parties
which are already part of the records of the case. In this regard, the evidence of the respondent were not able to overcome the
evidence of the petitioners. 19
It is, therefore, incumbent upon candidates for an elective office, who are repatriated citizens, to be ready with sufficient evidence of their
repatriation in case their Filipino citizenship is questioned to prevent a repetition of this case.
WHEREFORE, the petition seeking the nullification of the Resolution of the COMELEC en banc of May 7, 2004, affirming the Resolution of its First
Division dated March 22, 2004, is hereby DENIED. No costs.
SO ORDERED.
Davide, Jr., C .J ., Panganiban, Quisumbing, Ynares-Santiago, Sandoval-Gutierrez, Carpio, Austria-Martinez, Carpio Morales, Callejo, Sr., Chico-
Nazario and Garcia, JJ ., concur.
Puno and Tinga, JJ ., are on official leave.
Corona, J ., is on leave.

||| (Altarejos v. Commission on Elections, G.R. No. 163256, [November 10, 2004], 484 PHIL 609-625)
EN BANC
[G.R. No. 162759. August 4, 2006.]
LOIDA NICOLAS-LEWIS, GREGORIO B. MACABENTA, ALEJANDRO A. ESCLAMADO, ARMANDO B. HEREDIA, REUBEN S.
SEGURITAN, ERIC LACHICA FURBEYRE, TERESITA A. CRUZ, JOSEFINA OPENA DISTERHOFT, MERCEDES V. OPENA, CORNELIO R.
NATIVIDAD, EVELYN D. NATIVIDAD, petitioners, vs. COMMISSION ONELECTIONS, respondent.
DECISION
GARCIA, J p:
In this petition for certiorari and mandamus, petitioners, referring to themselves as "duals" or dual citizens, pray that they and others who retained
or reacquired Philippine citizenship under Republic Act (R.A.) No. 9225, the Citizenship Retention and Re-Acquisition Act of 2003, be allowed to avail
themselves of the mechanism provided under theOverseas Absentee Voting Act of 2003 1 (R.A. 9189) and that
the Commission on Elections (COMELEC) accordingly be ordered to allow them to vote and register as absentee voters under the aegis of R.A. 9189.
The facts:
Petitioners are successful applicants for recognition of Philippine citizenship under R.A. 9225 which accords to such applicants the right of suffrage,
among others. Long before the May 2004 national and local elections, petitioners sought registration and certification as "overseas absentee voter"
only to be advised by the Philippine Embassy in the United States that, per a COMELEC letter to the Department of Foreign Affairs dated September
23, 2003 2 , they have yet no right to vote in such elections owing to their lack of the one-year residence requirement prescribed by the Constitution.
The same letter, however, urged the different Philippine posts abroad not to discontinue their campaign for voter's registration, as the residence
restriction adverted to would contextually affect merely certain individuals who would likely be eligible to vote in future elections.
Prodded for clarification by petitioner Loida Nicolas-Lewis in the light of the ruling in Macalintal vs. COMELEC 3 on the residency requirement,
the COMELEC wrote in response:
Although R.A. 9225 enjoys the presumption of constitutionality . . ., it is the Commission's position that those who have availed
of the law cannot exercise the right of suffrage given under the OAVL for the reason that the OAVL was not enacted for them.
Hence, as Filipinos who have merely re-acquired their citizenship on 18 September 2003 at the earliest, and as law and
jurisprudence now stand, they are considered regular voters who have to meet the requirements of residency, among others
under Section 1, Article 5 ofthe Constitution. 4
Faced with the prospect of not being able to vote in the May 2004 elections owing to the COMELEC's refusal to include them in the National Registry
of Absentee Voters, petitioner Nicolas-Lewis et al., 5 filed on April 1, 2004 this petition for certiorari and mandamus. DHACES
A little over a week before the May 10, 2004 elections, or on April 30, 2004, the COMELEC filed a Comment, 6 therein praying for the denial of the
petition. As may be expected, petitioners were not able to register let alone vote in said elections.
On May 20, 2004, the Office of the Solicitor General (OSG) filed a Manifestation (in Lieu of Comment), therein stating that "all qualified overseas
Filipinos, including dual citizens who care to exercise the right of suffrage, may do so", observing, however, that the conclusion of the
2004 elections had rendered the petition moot and academic. 7
The holding of the 2004 elections had, as the OSG pointed out, indeed rendered the petition moot and academic, but insofar only as petitioners'
participation in such political exercise is concerned. The broader and transcendental issue tendered or subsumed in the petition, i.e., the propriety of
allowing "duals" to participate and vote as absentee voter in future elections, however, remains unresolved.
Observing the petitioners' and the COMELEC's respective formulations of the issues, the same may be reduced into the question of whether or not
petitioners and others who might have meanwhile retained and/or reacquired Philippine citizenship pursuant to R.A. 9225 may vote as absentee
voter under R.A. 9189.
The Court resolves the poser in the affirmative, and thereby accords merit to the petition.
In esse, this case is all about suffrage. A quick look at the governing provisions on the right of suffrage is, therefore, indicated.
We start off with Sections 1 and 2 of Article V of the Constitution, respectively reading as follows:
SECTION 1. Suffrage may be exercised by all citizens of the Philippines not otherwise disqualified by law, who are at least
eighteen years of age, and who shall have resided in the Philippines for at least one year and in the place wherein they propose
to vote for at least six months immediately preceding the election. . . . .
SEC 2. The Congress shall provide . . . a system for absentee voting by qualified Filipinos abroad.
In a nutshell, the aforequoted Section 1 prescribes residency requirement as a general eligibility factor for the right to vote. On the other hand,
Section 2 authorizes Congress to devise a system wherein an absentee may vote, implying that a non-resident may, as an exception to the residency
prescription in the preceding section, be allowed to vote.
In response to its above mandate, Congress enacted R.A. 9189 — the OAVL 8 — identifying in its Section 4 who can vote under it and in the following
section who cannot, as follows:
Section 4. Coverage. — All citizens of the Philippines abroad, who are not otherwise disqualified by law, at least eighteen (18)
years of age on the day of elections, may vote for president, vice-president, senators and party-list representatives.
Section 5. Disqualifications. — The following shall be disqualified from voting under this Act:
(a) Those who have lost their Filipino citizenship in accordance with Philippine laws;
(b) Those who have expressly renounced their Philippine citizenship and who have pledged allegiance to a foreign country;
(c) Those who have . . . [been] convicted in a final judgment by a court or tribunal of an offense punishable by imprisonment of
not less than one (1) year, including those who have . . . been found guilty of Disloyalty as defined under Article 137 of the
Revised Penal Code, . . . .;
(d) An immigrant or a permanent resident who is recognized as such in the host country, unless he/she executes, upon
registration, an affidavit prepared for the purpose by the Commission declaring that he/she shall resume actual physical
permanent residence in the Philippines not later than three (3) years from approval of his/her registration under this Act. Such
affidavit shall also state that he/she has not applied for citizenship in another country. Failure to return shall be the cause for
the removal of the name of the immigrant or permanent resident from the National Registry of Absentee Voters and his/her
permanent disqualification to vote in absentia.
(e) Any citizen of the Philippines abroad previously declared insane or incompetent by competent authority . . . . (Words in
bracket added.)
Notably, Section 5 lists those who cannot avail themselves of the absentee voting mechanism. However, Section 5(d) of the enumeration respecting
Filipino immigrants and permanent residents in another country opens an exception and qualifies the disqualification rule. Section 5(d) would,
however, face a constitutional challenge on the ground that, as narrated in Macalintal, it —
. . . violates Section 1, Article V of the 1987 Constitution which requires that the voter must be a resident in the Philippines for
at least one year and in the place where he proposes to vote for at least six months immediately preceding an election. [The
challenger] cites . . . Caasi vs. Court of Appeals 9 to support his claim [where] the Court held that a "green card" holder
immigrant to the [US] is deemed to have abandoned his domicile and residence in the Philippines.
[The challenger] further argues that Section 1, Article V of the Constitution does not allow provisional registration or a promise
by a voter to perform a condition to be qualified to vote in a political exercise; that the legislature should not be allowed to
circumvent the requirement of the Constitution on the right of suffrage by providing a condition thereon which in effect
amends or alters the aforesaid residence requirement to qualify a Filipino abroad to vote. He claims that the right of suffrage
should not be granted to anyone who, on the date of the election, does not possess the qualifications provided for by Section
1, Article V of the Constitution. 10 (Words in bracket added.)
As may be recalled, the Court upheld the constitutionality of Section 5(d) of R.A. 9189 mainly on the strength of the following premises:
As finally approved into law, Section 5(d) of R.A. No. 9189 specifically disqualifies an immigrant or permanent resident who is
"recognized as such in the host country" because immigration or permanent residence in another country implies renunciation
of one's residence in his country of origin. However, same Section allows an immigrant and permanent resident abroad to
register as voter for as long as he/she executes an affidavit to show that he/she has not abandoned his domicile in pursuance
of the constitutional intent expressed in Sections 1 and 2 of Article V that "all citizens of the Philippines not otherwise
disqualified by law" must be entitled to exercise the right of suffrage and, that Congress must establish a system for absentee
voting; for otherwise, if actual, physical residence in the Philippines is required, there is no sense for the framers of the
Constitution to mandate Congress to establish a system for absentee voting. aEHIDT
Contrary to the claim of [the challenger], the execution of the affidavit itself is not the enabling or enfranchising act. The
affidavit required in Section 5(d) is not only proof of the intention of the immigrant or permanent resident to go back and
resume residency in the Philippines, but more significantly, it serves as an explicit expression that he had not in fact abandoned
his domicile of origin. Thus, it is not correct to say that the execution of the affidavit under Section 5(d) violates the
Constitution that proscribes "provisional registration or a promise by a voter to perform a condition to be qualified to vote in a
political exercise." 11

Soon after Section 5(d) of R.A. 9189 passed the test of constitutionality, Congress enacted R.A. 9225 the relevant portion of which reads:
SEC. 2. Declaration of Policy. — It is hereby declared the policy of the State that all Philippine citizens who become citizens of
another country shall be deemed not to have lost their Philippine citizenship under the conditions of this Act.
SEC. 3. Retention of Philippine Citizenship. — Any provision of law to the contrary notwithstanding, natural-born citizens of the
Philippines who have lost their Philippine citizenship by reason of their naturalization as citizens of a foreign country are
hereby deemed to have re-acquired Philippine citizenship upon taking the following oath of allegiance to the Republic:
xxx xxx xxx
Natural-born citizens of the Philippines who, after the effectivity of this Act, become citizens of a foreign country shall retain
their Philippine citizenship upon taking the aforesaid oath.
SEC. 4. Derivative Citizenship. — The unmarried child, whether legitimate, illegitimate or adopted, below eighteen (18) years of
age, of those who re-acquire Philippine citizenship upon effectivity of this Act shall be deemed citizens of the Philippines.
SEC. 5. Civil and Political Rights and Liabilities. — Those who retain or re-acquire Philippine citizenship under this Act shall
enjoy full civil and political rights and be subject to all attendant liabilities and responsibilities under existing laws of the
Philippines and the following conditions:
(1) Those intending to exercise their right of suffrage must meet the requirements under Section 1, Article V of the
Constitution, Republic Act No. 9189, otherwise known as "The Overseas Absentee Voting Act of 2003" and other
existing laws;
(2) Those seeking elective public office in the Philippines shall meet the qualifications for holding such public office as
required by the Constitution and existing laws and, at the time of the filing of the certificate of candidacy, make a
personal and sworn renunciation of any and all foreign citizenship . . .;
3) . . . ;
(4) . . . ;
(5) That right to vote or be elected or appointed to any public office in the Philippines cannot be exercised by, or
extended to, those who:
(a) are candidates for or are occupying any public office in the country of which they are naturalized
citizens; and/or
(b) are in active service as commissioned or non-commissioned officers in the armed forces of the country
which they are naturalized citizens.
After what appears to be a successful application for recognition of Philippine citizenship under R.A. 9189, petitioners now invoke their right to enjoy
. . . political rights, specifically the right of suffrage, pursuant to Section 5 thereof. caHCSD
Opposing the petitioners' bid, however, respondent COMELEC invites attention to the same Section 5 (1) providing that "duals" can enjoy their right
to vote, as an adjunct to political rights, only if they meet the requirements of Section 1, Article V of the Constitution, R.A. 9189 and other existing
laws. Capitalizing on what at first blush is the clashing provisions of the aforecited provision of the Constitution, which, to repeat, requires residency
in the Philippines for a certain period, and R.A. 9189 which grants a Filipino non-resident absentee voting rights, 12 COMELEC argues:
4. 'DUALS' MUST FIRST ESTABLISH THEIR DOMICILE/RESIDENCE IN THE PHILIPPINES
4.01. The inclusion of such additional and specific requirements in RA 9225 is logical. The 'duals,' upon renouncement
of their Filipino citizenship and acquisition of foreign citizenship, have practically and legally abandoned
their domicile and severed their legal ties to the homeland as a consequence. Having subsequently
acquired a second citizenship (i.e., Filipino) then, 'duals' must, for purposes of voting, first of all, decisively
and definitely establish their domicile through positive acts; 13
The Court disagrees.
As may be noted, there is no provision in the dual citizenship law — R.A. 9225 — requiring "duals" to actually establish residence and physically stay
in the Philippines first before they can exercise their right to vote. On the contrary, R.A. 9225, in implicit acknowledgment that "duals" are most likely
non-residents, grants under its Section 5(1) the same right of suffrage as that granted an absentee voter under R.A. 9189. It cannot be
overemphasized that R.A. 9189 aims, in essence, to enfranchise as much as possible all overseas Filipinos who, save for the residency requirements
exacted of an ordinary voter under ordinary conditions, are qualified to vote. Thus, wrote the Court in Macalintal:
It is clear from these discussions of the . . . Constitutional Commission that [it] intended to enfranchise as much as
possible all Filipino citizens abroad who have not abandoned their domicile of origin. The Commission even intended to extend
to young Filipinos who reach voting age abroad whose parents' domicile of origin is in the Philippines, and consider them
qualified as voters for the first time. DICSaH
It is in pursuance of that intention that the Commission provided for Section 2 [Article V] immediately after the residency
requirement of Section 1. By the doctrine of necessary implication in statutory construction, . . ., the strategic location of
Section 2 indicates that the Constitutional Commission provided for an exception to the actual residency requirement of Section
1 with respect to qualified Filipinos abroad. The same Commission has in effect declared that qualified Filipinos who are not in
the Philippines may be allowed to vote even though they do not satisfy the residency requirement in Section 1, Article V of the
Constitution.
That Section 2 of Article V of the Constitution is an exception to the residency requirement found in Section 1 of the same
Article was in fact the subject of debate when Senate Bill No. 2104, which became R.A. No. 9189, was deliberated upon on the
Senate floor, thus:
Senator Arroyo. Mr. President, this bill should be looked into in relation to the constitutional provisions. I think the
sponsor and I would agree that the Constitution is supreme in any statute that we may enact.
Let me read Section 1, Article V, of the Constitution . . . .
xxx xxx xxx
Now, Mr. President, the Constitution says, "who shall have resided in the Philippines." They are permanent
immigrants. They have changed residence so they are barred under the Constitution. This is why I asked whether this
committee amendment which in fact does not alter the original text of the bill will have any effect on this?
Senator Angara. Good question, Mr. President. And this has been asked in various fora. This is in compliance with the
Constitution. One, the interpretation here of "residence" is synonymous with "domicile."
As the gentleman and I know, Mr. President, "domicile" is the intent to return to one's home. And the fact that a
Filipino may have been physically absent from the Philippines and may be physically a resident of the United
States, for example, but has a clear intent to return to the Philippines, will make him qualified as a resident of the
Philippines under this law.
This is consistent, Mr. President, with the constitutional mandate that we — that Congress — must provide a
franchise to overseas Filipinos.
If we read the Constitution and the suffrage principle literally as demanding physical presence, then there is no
way we can provide for offshore voting to our offshore kababayan, Mr. President.
Senator Arroyo. Mr. President, when the Constitution says, in Section 2 of Article V, it reads: "The Congress shall
provide a system for securing the secrecy and sanctity of the ballot as well as a system for absentee voting by
qualified Filipinos abroad."
The key to this whole exercise, Mr. President, is "qualified." In other words, anything that we may do or say in
granting our compatriots abroad must be anchored on the proposition that they are qualified. Absent the
qualification, they cannot vote. And "residents" (sic) is a qualification.
xxx xxx xxx
Look at what the Constitution says — "In the place wherein they propose to vote for at least six months immediately
preceding the election." acHDTA
Mr. President, all of us here have run (sic) for office.
I live in Makati. My neighbor is Pateros . . . . We are separated only by a creek. But one who votes in Makati cannot
vote in Pateros unless he resides in Pateros for six months. That is how restrictive our Constitution is. . . . .
As I have said, if a voter in Makati would want to vote in Pateros, yes, he may do so. But he must do so, make the
transfer six months before the election, otherwise, he is not qualified to vote.
xxx xxx xxx
Senator Angara. It is a good point to raise, Mr. President. But it is a point already well-debated even in the
constitutional commission of 1986. And the reason Section 2 of Article V was placed immediately after the six-
month/one-year residency requirement is to demonstrate unmistakably that Section 2 which authorizes absentee
voting is an exception to the six-month/one-year residency requirement. That is the first principle, Mr. President,
that one must remember.
The second reason, Mr. President, is that under our jurisprudence . . . — "residency" has been interpreted as
synonymous with "domicile."
But the third more practical reason, . . . is, if we follow the interpretation of the gentleman, then it is legally and
constitutionally impossible to give a franchise to vote to overseas Filipinos who do not physically live in the
country, which is quite ridiculous because that is exactly the whole point of this exercise — to enfranchise them
and empower them to vote. 14 (Emphasis and words in bracket added; citations omitted)
Lest it be overlooked, no less than the COMELEC itself admits that the Citizenship Retention and Re-Acquisition Act expanded the coverage of
overseas absentee voting. According to the poll body:

1.05 With the passage of RA 9225 the scope of overseas absentee voting has been consequently expanded so as to include
Filipinos who are also citizens of other countries, subject, however, to the strict prerequisites indicated in the pertinent
provisions of RA 9225; 15
Considering the unison intent of the Constitution and R.A. 9189 and the expansion of the scope of that law with the passage of R.A. 9225, the
irresistible conclusion is that "duals" may now exercise the right of suffrage thru the absentee voting scheme and as overseas absentee voters. R.A.
9189 defines the terms adverted to in the following wise:
"Absentee Voting" refers to the process by which qualified citizens of the Philippines abroad exercise their right to vote;
"Overseas Absentee Voter" refers to a citizen of the Philippines who is qualified to register and vote under this Act, not
otherwise disqualified by law, who is abroad on the day ofelections;
While perhaps not determinative of the issue tendered herein, we note that the expanded thrust of R.A. 9189 extends also to what might be tag as
the next generation of "duals". This may be deduced from the inclusion of the provision on derivative citizenship in R.A. 9225 which reads:
SEC. 4. Derivative Citizenship. — The unmarried child, whether legitimate, illegitimate or adopted, below eighteen (18) years of
age, of those who re-acquire Philippine citizenship upon effectivity of this Act shall be deemed citizens of the Philippines.
It is very likely that a considerable number of those unmarried children below eighteen (18) years of age had never set foot in the Philippines. Now
then, if the next generation of "duals" may nonetheless avail themselves the right to enjoy full civil and political rights under Section 5 of the Act,
then there is neither no rhyme nor reason why the petitioners and other present day "duals," provided they meet the requirements under Section 1,
Article V of the Constitution in relation to R.A. 9189, be denied the right of suffrage as an overseas absentee voter. Congress could not have plausibly
intended such absurd situation. cEaTHD
WHEREFORE, the instant petition is GRANTED. Accordingly, the Court rules and so holds that those who retain or re-acquire Philippine citizenship
under Republic Act No. 9225, the Citizenship Retention and Re-Acquisition Act of 2003, may exercise the right to vote under the system of absentee
voting in Republic Act No. 9189, the Overseas Absentee Voting Act of 2003.
SO ORDERED.
Panganiban, C.J., Puno, Quisumbing, Ynares-Santiago, Sandoval-Gutierrez, Carpio, Austria-Martinez, Corona, Carpio-Morales, Callejo, Sr., Azcuna,
Tinga, Chico-Nazario andVelasco, Jr., JJ., concur.

||| (Nicolas-Lewis v. Commission on Elections, G.R. No. 162759, [August 4, 2006], 529 PHIL 642-659)
EN BANC
[G.R. No. 160869. May 11, 2007.]
AASJS (ADVOCATES AND ADHERENTS OF SOCIAL JUSTICE FOR SCHOOL TEACHERS AND ALLIED WORKERS) MEMBER —
HECTOR GUMANGAN CALILUNG,petitioner, vs. THE HONORABLE SIMEON DATUMANONG, in his official capacity as the
Secretary of Justice, respondent.
DECISION
QUISUMBING, J p:
This is an original action for prohibition under Rule 65 of the 1997 Revised Rules of Civil Procedure.
Petitioner filed the instant petition against respondent, then Secretary of Justice Simeon Datumanong, the official tasked to implement laws
governing citizenship. 1 Petitioner prays that a writ of prohibition be issued to stop respondent from implementing Republic Act No. 9225,
entitled "An Act Making the Citizenship of Philippine Citizens Who Acquire Foreign Citizenship Permanent, Amending for the Purpose Commonwealth
Act No. 63, As Amended, and for Other Purposes." Petitioner avers that Rep. Act No. 9225 is unconstitutional as it violates Section 5, Article IV of
the 1987 Constitution that states, "Dual allegiance of citizens is inimical to the national interest and shall be dealt with by law."
Rep. Act No. 9225, signed into law by President Gloria M. Arroyo on August 29, 2003, reads:
SECTION 1. Short Title. — This Act shall be known as the "Citizenship Retention and Reacquisition Act of 2003."
SEC. 2. Declaration of Policy. — It is hereby declared the policy of the State that all Philippine citizens who become citizens of
another country shall be deemed not to have lost their Philippine citizenship under the conditions of this Act. TIDaCE
SEC. 3. Retention of Philippine Citizenship. — Any provision of law to the contrary notwithstanding, natural-born citizens of the
Philippines who have lost their Philippine citizenship by reason of their naturalization as citizens of a foreign country are
hereby deemed to have reacquired Philippine citizenship upon taking the following oath of allegiance to the Republic:
"I ___________________________, solemnly swear (or affirm) that I will support and defend the Constitution of the
Republic of the Philippines and obey the laws and legal orders promulgated by the duly constituted authorities of the
Philippines; and I hereby declare that I recognize and accept the supreme authority of the Philippines and will
maintain true faith and allegiance thereto; and that I impose this obligation upon myself voluntarily without mental
reservation or purpose of evasion."
Natural-born citizens of the Philippines who, after the effectivity of this Act, become citizens of a foreign country shall retain
their Philippine citizenship upon taking the aforesaid oath.
SEC. 4. Derivative Citizenship. — The unmarried child, whether legitimate, illegitimate or adopted, below eighteen (18) years of
age, of those who reacquire Philippine citizenship upon effectivity of this Act shall be deemed citizens of the Philippines.
SEC. 5. Civil and Political Rights and Liabilities. — Those who retain or reacquire Philippine citizenship under this Act shall enjoy
full civil and political rights and be subject to all attendant liabilities and responsibilities under existing laws of the Philippines
and the following conditions: DcITaC
(1) Those intending to exercise their right of suffrage must meet the requirements under Section 1, Article V of the
Constitution, Republic Act No. 9189, otherwise known as "The Overseas Absentee Voting Act of 2003" and other
existing laws;
(2) Those seeking elective public office in the Philippines shall meet the qualifications for holding such public office as
required by the Constitution and existing laws and, at the time of the filing of the certificate of candidacy, make a
personal and sworn renunciation of any and all foreign citizenship before any public officer authorized to administer
an oath;
(3) Those appointed to any public office shall subscribe and swear to an oath of allegiance to the Republic of the
Philippines and its duly constituted authorities prior to their assumption of office: Provided, That they renounce their
oath of allegiance to the country where they took that oath;
(4) Those intending to practice their profession in the Philippines shall apply with the proper authority for a license or
permit to engage in such practice; and
(5) That right to vote or be elected or appointed to any public office in the Philippines cannot be exercised by, or
extended to, those who:
(a) are candidates for or are occupying any public office in the country of which they are naturalized
citizens; and/or TSHEIc
(b) are in the active service as commissioned or noncommissioned officers in the armed forces of the
country which they are naturalized citizens.
SEC. 6. Separability Clause. — If any section or provision of this Act is held unconstitutional or invalid, any other section or
provision not affected thereby shall remain valid and effective.
SEC. 7. Repealing Clause. — All laws, decrees, orders, rules and regulations inconsistent with the provisions of this Act are
hereby repealed or modified accordingly.
SEC. 8. Effectivity Clause. — This Act shall take effect after fifteen (15) days following its publication in the Official Gazette or
two (2) newspapers of general circulation.
In this petition for prohibition, the following issues have been raised: (1) Is Rep. Act No. 9225 unconstitutional? (2) Does this Court have jurisdiction
to pass upon the issue of dual allegiance?
We shall discuss these issues jointly.
Petitioner contends that Rep. Act No. 9225 cheapens Philippine citizenship. He avers that Sections 2 and 3 of Rep. Act No. 9225, together, allow dual
allegiance and not dual citizenship. Petitioner maintains that Section 2 allows all Filipinos, either natural-born or naturalized, who become foreign
citizens, to retain their Philippine citizenship without losing their foreign citizenship. Section 3 permits dual allegiance because said law allows
natural-born citizens of the Philippines to regain their Philippine citizenship by simply taking an oath of allegiance without forfeiting their foreign
allegiance. 2 The Constitution, however, is categorical that dual allegiance is inimical to the national interest. ASEcHI
The Office of the Solicitor General (OSG) claims that Section 2 merely declares as a state policy that "Philippine citizens who become citizens of
another country shall be deemed not to have lost their Philippine citizenship." The OSG further claims that the oath in Section 3 does not allow dual
allegiance since the oath taken by the former Filipino citizen is an effective renunciation and repudiation of his foreign citizenship. The fact that the
applicant taking the oath recognizes and accepts the supreme authority of the Philippines is an unmistakable and categorical affirmation of his
undivided loyalty to the Republic. 3
In resolving the aforecited issues in this case, resort to the deliberations of Congress is necessary to determine the intent of the legislative branch in
drafting the assailed law. During the deliberations, the issue of whether Rep. Act No. 9225 would allow dual allegiance had in fact been the subject of
debate. The record of the legislative deliberations reveals the following:
xxx xxx xxx
Pursuing his point, Rep. Dilangalen noted that under the measure, two situations exist — the retention of foreign citizenship,
and the reacquisition of Philippine citizenship. In this case, he observed that there are two citizenships and therefore, two
allegiances. He pointed out that under the Constitution, dual allegiance is inimical to public interest. He thereafter asked
whether with the creation of dual allegiance by reason of retention of foreign citizenship and the reacquisition of Philippine
citizenship, there will now be a violation of the Constitution. IEAacT
Rep. Locsin underscored that the measure does not seek to address the constitutional injunction on dual allegiance as inimical
to public interest. He said that the proposed law aims to facilitate the reacquisition of Philippine citizenship by speedy
means. However, he said that in one sense, it addresses the problem of dual citizenship by requiring the taking of an oath.
He explained that the problem of dual citizenship is transferred from the Philippines to the foreign country because the
latest oath that will be taken by the former Filipino is one of allegiance to the Philippines and not to the United States, as
the case may be. He added that this is a matter which the Philippine government will have no concern and competence over.
Rep. Dilangalen asked why this will no longer be the country's concern, when dual allegiance is involved.
Rep. Locsin clarified that this was precisely his objection to the original version of the bill, which did not require an oath of
allegiance. Since the measure now requires this oath, the problem of dual allegiance is transferred from the Philippines to
the foreign country concerned, he explained.
xxx xxx xxx
Rep. Dilangalen asked whether in the particular case, the person did not denounce his foreign citizenship and therefore still
owes allegiance to the foreign government, and at the same time, owes his allegiance to the Philippine government, such that
there is now a case of dual citizenship and dual allegiance.
Rep. Locsin clarified that by swearing to the supreme authority of the Republic, the person implicitly renounces his foreign
citizenship. However, he said that this is not a matter that he wishes to address in Congress because he is not a member of a
foreign parliament but a Member of the House.
xxx xxx xxx
Rep. Locsin replied that it is imperative that those who have dual allegiance contrary to national interest should be dealt with
by law. However, he said that the dual allegiance problem is not addressed in the bill. He then cited the Declaration of Policy in
the bill which states that "It is hereby declared the policy of the State that all citizens who become citizens of another country
shall be deemed not to have lost their Philippine citizenship under the conditions of this Act." He stressed that what the bill
does is recognize Philippine citizenship but says nothing about the other citizenship. cHCSDa

Rep. Locsin further pointed out that the problem of dual allegiance is created wherein a natural-born citizen of the Philippines
takes an oath of allegiance to another country and in that oath says that he abjures and absolutely renounces all allegiance to
his country of origin and swears allegiance to that foreign country. The original Bill had left it at this stage, he explained. In the
present measure, he clarified, a person is required to take an oath and the last he utters is one of allegiance to the country.
He then said that the problem of dual allegiance is no longer the problem of the Philippines but of the other foreign
country. 4 (Emphasis supplied.)
From the above excerpts of the legislative record, it is clear that the intent of the legislature in drafting Rep. Act No. 9225 is to do away with the
provision in Commonwealth Act No. 63 5 which takes away Philippine citizenship from natural-born Filipinos who become naturalized citizens of
other countries. What Rep. Act No. 9225 does is allow dual citizenship to natural-born Filipino citizens who have lost Philippine citizenship by reason
of their naturalization as citizens of a foreign country. On its face, it does not recognize dual allegiance. By swearing to the supreme authority of the
Republic, the person implicitly renounces his foreign citizenship. Plainly, from Section 3, Rep. Act No. 9225 stayed clear out of the problem of dual
allegiance and shifted the burden of confronting the issue of whether or not there is dual allegiance to the concerned foreign country. What happens
to the other citizenship was not made a concern of Rep. Act No. 9225.
Petitioner likewise advances the proposition that although Congress has not yet passed any law on the matter of dual allegiance, such absence of a
law should not be justification why this Court could not rule on the issue. He further contends that while it is true that there is no enabling law yet on
dual allegiance, the Supreme Court, throughMercado v. Manzano, 6 already had drawn up the guidelines on how to distinguish dual allegiance from
dual citizenship. 7
For its part, the OSG counters that pursuant to Section 5, Article IV of the 1987 Constitution, dual allegiance shall be dealt with by law. Thus, until a
law on dual allegiance is enacted by Congress, the Supreme Court is without any jurisdiction to entertain issues regarding dual allegiance. 8
To begin with, Section 5, Article IV of the Constitution is a declaration of a policy and it is not a self-executing provision. The legislature still has to
enact the law on dual allegiance. In Sections 2 and 3 of Rep. Act No. 9225, the framers were not concerned with dual citizenship per se, but with the
status of naturalized citizens who maintain their allegiance to their countries of origin even after their naturalization. 9 Congress was given a
mandate to draft a law that would set specific parameters of what really constitutes dual allegiance. 10 Until this is done, it would be premature for
the judicial department, including this Court, to rule on issues pertaining to dual allegiance. cADTSH
Neither can we subscribe to the proposition of petitioner that a law is not needed since the case of Mercado had already set the guidelines for
determining dual allegiance. Petitioner misreads Mercado. That case did not set the parameters of what constitutes dual allegiance but merely made
a distinction between dual allegiance and dual citizenship.
Moreover, in Estrada v. Sandiganbayan, 11 we said that the courts must assume that the legislature is ever conscious of the borders and edges of its
plenary powers, and passed laws with full knowledge of the facts and for the purpose of promoting what is right and advancing the welfare of the
majority. Hence, in determining whether the acts of the legislature are in tune with the fundamental law, we must proceed with judicial restraint and
act with caution and forbearance. 12 The doctrine of separation of powers demands no less. We cannot arrogate the duty of setting the parameters
of what constitutes dual allegiance when the Constitution itself has clearly delegated the duty of determining what acts constitute dual allegiance for
study and legislation by Congress.
WHEREFORE, the petition is hereby DISMISSED for lack of merit.
SO ORDERED.
Puno, C.J., Ynares-Santiago, Sandoval-Gutierrez, Carpio, Carpio-Morales, Azcuna, Tinga, Chico-Nazario, Garcia, Velasco, Jr. and Nachura, JJ., concur.
Austria-Martinez and Corona, JJ., are on leave.
||| (Calilung v. Datumanong, G.R. No. 160869, [May 11, 2007], 551 PHIL 110-119)
EN BANC
[G.R. No. 221697. March 8, 2016.]
MARY GRACE NATIVIDAD S. POE-LLAMANZARES, petitioner, vs. COMMISSION ON ELECTIONS AND ESTRELLA C.
ELAMPARO, respondents.
[G.R. Nos. 221698-700. March 8, 2016.]
MARY GRACE NATIVIDAD S. POE-LLAMANZARES, petitioner, vs. COMMISSION ON ELECTIONS, FRANCISCO S. TATAD,
ANTONIO P. CONTRERAS AND AMADO D. VALDEZ, respondents.
DECISION
PEREZ, J p:
Before the Court are two consolidated petitions under Rule 64 in relation to Rule 65 of the Rules of Court with extremely urgent
application for an ex parte issuance of temporary restraining order/status quo ante order and/or writ of preliminary injunction assailing the
following: (1) 1 December 2015 Resolution of the Commission on Elections (COMELEC) Second Division; (2) 23 December 2015 Resolution of the
COMELEC En Banc, in SPA No. 15-001 (DC); (3) 11 December 2015 Resolution of the COMELEC First Division; and (4) 23 December 2015
Resolution of the COMELEC En Banc, in SPA No. 15-002 (DC), SPA No. 15-007 (DC) and SPA No. 15-139 (DC) for having been issued without
jurisdiction or with grave abuse of discretion amounting to lack or excess of jurisdiction.
The Facts
Mary Grace Natividad S. Poe-Llamanzares (petitioner) was found abandoned as a newborn infant in the Parish Church of Jaro, Iloilo by
a certain Edgardo Militar (Edgardo) on 3 September 1968. Parental care and custody over petitioner was passed on by Edgardo to his relatives,
Emiliano Militar (Emiliano) and his wife. Three days after, 6 September 1968, Emiliano reported and registered petitioner as a foundling with the
Office of the Civil Registrar of Iloilo City (OCR-Iloilo). In her Foundling Certificate and Certificate of Live Birth, the petitioner was given the name
"Mary Grace Natividad Contreras Militar." 1
When petitioner was five (5) years old, celebrity spouses Ronald Allan Kelley Poe (a.k.a. Fernando Poe, Jr.) and Jesusa Sonora Poe
(a.k.a. Susan Roces) filed a petition for her adoption with the Municipal Trial Court (MTC) of San Juan City. On 13 May 1974, the trial court
granted their petition and ordered that petitioner's name be changed from "Mary Grace Natividad Contreras Militar" to "Mary Grace Natividad
Sonora Poe." Although necessary notations were made by OCR-Iloilo on petitioner's foundling certificate reflecting the court decreed
adoption, 2 the petitioner's adoptive mother discovered only sometime in the second half of 2005 that the lawyer who handled petitioner's
adoption failed to secure from the OCR-Iloilo a new Certificate of Live Birth indicating petitioner's new name and the name of her adoptive
parents. 3 Without delay, petitioner's mother executed an affidavit attesting to the lawyer's omission which she submitted to the OCR-Iloilo. On
4 May 2006, OCR-Iloilo issued a new Certificate of Live Birth in the name of Mary Grace Natividad Sonora Poe. 4
Having reached the age of eighteen (18) years in 1986, petitioner registered as a voter with the local COMELEC Office in San Juan City.
On 13 December 1986, she received her COMELEC Voter's Identification Card for Precinct No. 196 in Greenhills, San Juan, Metro Manila. 5
On 4 April 1988, petitioner applied for and was issued Philippine Passport No. F927287 6 by the Department of Foreign Affairs (DFA).
Subsequently, on 5 April 1993 and 19 May 1998, she renewed her Philippine passport and respectively secured Philippine Passport Nos.
L881511 and DD156616. 7
Initially, the petitioner enrolled and pursued a degree in Development Studies at the University of the Philippines 8 but she opted to
continue her studies abroad and left for the United States of America (U.S.) in 1988. Petitioner graduated in 1991 from Boston College in
Chestnuts Hill, Massachusetts where she earned her Bachelor of Arts degree in Political Studies. 9
On 27 July 1991, petitioner married Teodoro Misael Daniel V. Llamanzares (Llamanzares), a citizen of both the Philippines and the U.S.,
at Sanctuario de San Jose Parish in San Juan City. 10 Desirous of being with her husband who was then based in the U.S., the couple flew back to
the U.S. two days after the wedding ceremony or on 29 July 1991. 11
While in the U.S., the petitioner gave birth to her eldest child Brian Daniel (Brian) on 16 April 1992. 12 Her two daughters Hanna
MacKenzie (Hanna) and Jesusa Anika (Anika) were both born in the Philippines on 10 July 1998 and 5 June 2004, respectively. 13
On 18 October 2001, petitioner became a naturalized American citizen. 14 She obtained U.S. Passport No. 017037793 on 19 December
2001. 15
On 8 April 2004, the petitioner came back to the Philippines together with Hanna to support her father's candidacy for President in the
May 2004 elections. It was during this time that she gave birth to her youngest daughter Anika. She returned to the U.S. with her two daughters
on 8 July 2004. 16
After a few months, specifically on 13 December 2004, petitioner rushed back to the Philippines upon learning of her father's
deteriorating medical condition. 17 Her father slipped into a coma and eventually expired. The petitioner stayed in the country until 3 February
2005 to take care of her father's funeral arrangements as well as to assist in the settlement of his estate. 18
According to the petitioner, the untimely demise of her father was a severe blow to her entire family. In her earnest desire to be with
her grieving mother, the petitioner and her husband decided to move and reside permanently in the Philippines sometime in the first quarter of
2005. 19 The couple began preparing for their resettlement including notification of their children's schools that they will be transferring to
Philippine schools for the next semester; 20 coordination with property movers for the relocation of their household goods, furniture and cars
from the U.S. to the Philippines; 21 and inquiry with Philippine authorities as to the proper procedure to be followed in bringing their pet dog
into the country. 22 As early as 2004, the petitioner already quit her job in the U.S. 23
Finally, petitioner came home to the Philippines on 24 May 2005 24 and without delay, secured a Tax Identification Number from the
Bureau of Internal Revenue. Her three (3) children immediately followed 25 while her husband was forced to stay in the U.S. to complete
pending projects as well as to arrange the sale of their family home there. 26
The petitioner and her children briefly stayed at her mother's place until she and her husband purchased a condominium unit with a
parking slot at One Wilson Place Condominium in San Juan City in the second half of 2005. 27 The corresponding Condominium Certificates of
Title covering the unit and parking slot were issued by the Register of Deeds of San Juan City to petitioner and her husband on 20 February
2006. 28 Meanwhile, her children of school age began attending Philippine private schools.
On 14 February 2006, the petitioner made a quick trip to the U.S. to supervise the disposal of some of the family's remaining
household belongings. 29 She travelled back to the Philippines on 11 March 2006. 30
In late March 2006, petitioner's husband officially informed the U.S. Postal Service of the family's change and abandonment of their
address in the U.S. 31 The family home was eventually sold on 27 April 2006. 32 Petitioner's husband resigned from his job in the U.S. in April
2006, arrived in the country on 4 May 2006 and started working for a major Philippine company in July 2006. 33
In early 2006, petitioner and her husband acquired a 509-square meter lot in Corinthian Hills, Quezon City where they built their
family home 34 and to this day, is where the couple and their children have been residing. 35 A Transfer Certificate of Title covering said
property was issued in the couple's name by the Register of Deeds of Quezon City on 1 June 2006.
On 7 July 2006, petitioner took her Oath of Allegiance to the Republic of the Philippines pursuant to Republic Act (R.A.) No. 9225 or
the Citizenship Retention and Re-acquisition Act of 2003. 36 Under the same Act, she filed with the Bureau of Immigration (BI) a sworn petition
to reacquire Philippine citizenship together with petitions for derivative citizenship on behalf of her three minor children on 10 July 2006. 37 As
can be gathered from its 18 July 2006 Order, the BI acted favorably on petitioner's petitions and declared that she is deemed to have reacquired
her Philippine citizenship while her children are considered as citizens of the Philippines. 38 Consequently, the BI issued Identification
Certificates (ICs) in petitioner's name and in the names of her three (3) children. 39
Again, petitioner registered as a voter of Barangay Santa Lucia, San Juan City on 31 August 2006. 40 She also secured from the DFA a
new Philippine Passport bearing the No. XX4731999. 41 This passport was renewed on 18 March 2014 and she was issued Philippine Passport
No. EC0588861 by the DFA. 42
On 6 October 2010, President Benigno S. Aquino III appointed petitioner as Chairperson of the Movie and Television Review and
Classification Board (MTRCB). 43 Before assuming her post, petitioner executed an "Affidavit of Renunciation of Allegiance to the United States
of America and Renunciation of American Citizenship" before a notary public in Pasig City on 20 October 2010, 44 in satisfaction of the legal
requisites stated in Section 5 of R.A. No. 9225. 45 The following day, 21 October 2010 petitioner submitted the said affidavit to the BI 46 and
took her oath of office as Chairperson of the MTRCB. 47 From then on, petitioner stopped using her American passport. 48
On 12 July 2011, the petitioner executed before the Vice Consul of the U.S. Embassy in Manila an "Oath/Affirmation of Renunciation
of Nationality of the United States."49 On that day, she accomplished a sworn questionnaire before the U.S. Vice Consul wherein she stated that
she had taken her oath as MTRCB Chairperson on 21 October 2010 with the intent, among others, of relinquishing her American
citizenship. 50 In the same questionnaire, the petitioner stated that she had resided outside of the U.S., specifically in the Philippines, from 3
September 1968 to 29 July 1991 and from May 2005 to present. 51

On 9 December 2011, the U.S. Vice Consul issued to petitioner a "Certificate of Loss of Nationality of the United States" effective 21
October 2010. 52
On 2 October 2012, the petitioner filed with the COMELEC her Certificate of Candidacy (COC) for Senator for the 2013 Elections
wherein she answered "6 years and 6 months" to the question "Period of residence in the Philippines before May 13, 2013." 53 Petitioner
obtained the highest number of votes and was proclaimed Senator on 16 May 2013. 54
On 19 December 2013, petitioner obtained Philippine Diplomatic Passport No. DE0004530. 55
On 15 October 2015, petitioner filed her COC for the Presidency for the May 2016 Elections. 56 In her COC, the petitioner declared
that she is a natural-born citizen and that her residence in the Philippines up to the day before 9 May 2016 would be ten (10) years and eleven
(11) months counted from 24 May 2005. 57 The petitioner attached to her COC an "Affidavit Affirming Renunciation of U.S.A. Citizenship"
subscribed and sworn to before a notary public in Quezon City on 14 October 2015. 58
Petitioner's filing of her COC for President in the upcoming elections triggered the filing of several COMELEC cases against her which
were the subject of these consolidated cases.
Origin of Petition for Certiorari in G.R. No. 221697
A day after petitioner filed her COC for President, Estrella Elamparo (Elamparo) filed a petition to deny due course or cancel said COC
which was docketed as SPA No. 15-001 (DC) and raffled to the COMELEC Second Division. 59 She is convinced that the COMELEC has jurisdiction
over her petition. 60 Essentially, Elamparo's contention is that petitioner committed material misrepresentation when she stated in her COC
that she is a natural-born Filipino citizen and that she is a resident of the Philippines for at least ten (10) years and eleven (11) months up to the
day before the 9 May 2016 Elections. 61
On the issue of citizenship, Elamparo argued that petitioner cannot be considered as a natural-born Filipino on account of the fact that
she was a foundling. 62 Elamparo claimed that international law does not confer natural-born status and Filipino citizenship on
foundlings. 63 Following this line of reasoning, petitioner is not qualified to apply for reacquisition of Filipino citizenship under R.A. No. 9225 for
she is not a natural-born Filipino citizen to begin with. 64 Even assuming arguendo that petitioner was a natural-born Filipino, she is deemed to
have lost that status when she became a naturalized American citizen. 65 According to Elamparo, natural-born citizenship must be continuous
from birth. 66
On the matter of petitioner's residency, Elamparo pointed out that petitioner was bound by the sworn declaration she made in her
2012 COC for Senator wherein she indicated that she had resided in the country for only six (6) years and six (6) months as of May 2013
Elections. Elamparo likewise insisted that assuming arguendo that petitioner is qualified to regain her natural-born status under R.A. No. 9225,
she still fell short of the ten-year residency requirement of the Constitution as her residence could only be counted at the earliest from July
2006, when she reacquired Philippine citizenship under the said Act. Also on the assumption that petitioner is qualified to reacquire lost
Philippine Citizenship, Elamparo is of the belief that she failed to reestablish her domicile in the Philippines. 67
Petitioner seasonably filed her Answer wherein she countered that:
(1) the COMELEC did not have jurisdiction over Elamparo's petition as it was actually a petition for quo warranto which could
only be filed if Grace Poe wins in the Presidential elections, and that the Department of Justice (DOJ) has primary
jurisdiction to revoke the BI's July 18, 2006 Order;
(2) the petition failed to state a cause of action because it did not contain allegations which, if hypothetically admitted, would
make false the statement in her COC that she is a natural-born Filipino citizen nor was there any allegation that there
was a willful or deliberate intent to misrepresent on her part;
(3) she did not make any material misrepresentation in the COC regarding her citizenship and residency qualifications for:
a. the 1934 Constitutional Convention deliberations show that foundlings were considered
citizens;
b. foundlings are presumed under international law to have been born of citizens of the place
where they are found;
c. she reacquired her natural-born Philippine citizenship under the provisions of R.A. No. 9225;
d. she executed a sworn renunciation of her American citizenship prior to the filing of her COC
for President in the May 9, 2016 Elections and that the same is in full force and effect and has not been
withdrawn or recanted;
e. the burden was on Elamparo in proving that she did not possess natural-born status;
f. residence is a matter of evidence and that she reestablished her domicile in the Philippines
as early as May 24, 2005;
g. she could reestablish residence even before she reacquired natural-born citizenship
under R.A. No. 9225;
h. statement regarding the period of residence in her 2012 COC for Senator was an honest
mistake, not binding and should give way to evidence on her true date of reacquisition of domicile;
i. Elamparo's petition is merely an action to usurp the sovereign right of the Filipino people to
decide a purely political question, that is, should she serve as the country's next leader. 68
After the parties submitted their respective Memoranda, the petition was deemed submitted for resolution.
On 1 December 2015, the COMELEC Second Division promulgated a Resolution finding that petitioner's COC, filed for the purpose of
running for the President of the Republic of the Philippines in the 9 May 2016 National and Local Elections, contained material representations
which are false. The fallo of the aforesaid Resolution reads:
WHEREFORE, in view of all the foregoing considerations, the instant Petition to Deny Due Course to or Cancel
Certificate of Candidacy is hereby GRANTED. Accordingly, the Certificate of Candidacy for President of the Republic of the
Philippines in the May 9, 2016 National and Local Elections filed by respondent Mary Grace Natividad Sonora Poe
Llamanzares is hereby CANCELLED. 69
Motion for Reconsideration of the 1 December 2015 Resolution was filed by petitioner which the COMELEC En Banc resolved in its 23
December 2015 Resolution by denying the same. 70
Origin of Petition for Certiorari in G.R. Nos. 221698-700
This case stemmed from three (3) separate petitions filed by Francisco S. Tatad (Tatad), Antonio P. Contreras (Contreras) and Amado
D. Valdez (Valdez) against petitioner before the COMELEC which were consolidated and raffled to its First Division.
In his petition to disqualify petitioner under Rule 25 of the COMELEC Rules of Procedure, 71 docketed as SPA No. 15-002 (DC), Tatad
alleged that petitioner lacks the requisite residency and citizenship to qualify her for the Presidency. 72
Tatad theorized that since the Philippines adheres to the principle of jus sanguinis, persons of unknown parentage, particularly
foundlings, cannot be considered natural-born Filipino citizens since blood relationship is determinative of natural-born status. 73 Tatad invoked
the rule of statutory construction that what is not included is excluded. He averred that the fact that foundlings were not expressly included in
the categories of citizens in the 1935 Constitution is indicative of the framers' intent to exclude them. 74 Therefore, the burden lies on
petitioner to prove that she is a natural-born citizen. 75
Neither can petitioner seek refuge under international conventions or treaties to support her claim that foundlings have a
nationality. 76 According to Tatad, international conventions and treaties are not self-executory and that local legislations are necessary in order
to give effect to treaty obligations assumed by the Philippines. 77He also stressed that there is no standard state practice that automatically
confers natural-born status to foundlings. 78
Similar to Elamparo's argument, Tatad claimed that petitioner cannot avail of the option to reacquire Philippine citizenship under R.A.
No. 9225 because it only applies to former natural-born citizens and petitioner was not as she was a foundling. 79
Referring to petitioner's COC for Senator, Tatad concluded that she did not comply with the ten (10)-year residency
requirement. 80 Tatad opined that petitioner acquired her domicile in Quezon City only from the time she renounced her American citizenship
which was sometime in 2010 or 2011. 81 Additionally, Tatad questioned petitioner's lack of intention to abandon her U.S. domicile as evinced by
the fact that her husband stayed thereat and her frequent trips to the U.S. 82
In support of his petition to deny due course or cancel the COC of petitioner, docketed as SPA No. 15-139 (DC), Valdez alleged that her
repatriation under R.A. No. 9225did not bestow upon her the status of a natural-born citizen. 83 He advanced the view that former natural-born
citizens who are repatriated under the said Act reacquires only their Philippine citizenship and will not revert to their original status as natural-
born citizens. 84
He further argued that petitioner's own admission in her COC for Senator that she had only been a resident of the Philippines for at
least six (6) years and six (6) months prior to the 13 May 2013 Elections operates against her. Valdez rejected petitioner's claim that she could
have validly reestablished her domicile in the Philippines prior to her reacquisition of Philippine citizenship. In effect, his position was that
petitioner did not meet the ten (10)-year residency requirement for President.
Unlike the previous COMELEC cases filed against petitioner, Contreras' petition, 85 docketed as SPA No. 15-007 (DC), limited the attack
to the residency issue. He claimed that petitioner's 2015 COC for President should be cancelled on the ground that she did not possess the ten-
year period of residency required for said candidacy and that she made false entry in her COC when she stated that she is a legal resident of the
Philippines for ten (10) years and eleven (11) months by 9 May 2016. 86 Contreras contended that the reckoning period for computing
petitioner's residency in the Philippines should be from 18 July 2006, the date when her petition to reacquire Philippine citizenship was
approved by the BI. 87 He asserted that petitioner's physical presence in the country before 18 July 2006 could not be valid evidence of
reacquisition of her Philippine domicile since she was then living here as an American citizen and as such, she was governed by the Philippine
immigration laws. 88

In her defense, petitioner raised the following arguments:


First, Tatad's petition should be dismissed outright for failure to state a cause of action. His petition did not invoke grounds proper for
a disqualification case as enumerated under Sections 12 and 68 of the Omnibus Election Code. 89 Instead, Tatad completely relied on the
alleged lack of residency and natural-born status of petitioner which are not among the recognized grounds for the disqualification of a
candidate to an elective office. 90
Second, the petitions filed against her are basically petitions for quo warranto as they focus on establishing her ineligibility for the
Presidency. 91 A petition for quo warranto falls within the exclusive jurisdiction of the Presidential Electoral Tribunal (PET) and not the
COMELEC. 92
Third, the burden to prove that she is not a natural-born Filipino citizen is on the respondents. 93 Otherwise stated, she has a
presumption in her favor that she is a natural-born citizen of this country.
Fourth, customary international law dictates that foundlings are entitled to a nationality and are presumed to be citizens of the
country where they are found. 94Consequently, the petitioner is considered as a natural-born citizen of the Philippines. 95
Fifth, she claimed that as a natural-born citizen, she has every right to be repatriated under R.A. No. 9225 or the right to reacquire her
natural-born status. 96 Moreover, the official acts of the Philippine Government enjoy the presumption of regularity, to wit: the issuance of the
18 July 2006 Order of the BI declaring her as natural-born citizen, her appointment as MTRCB Chair and the issuance of the decree of adoption
of San Juan RTC. 97 She believed that all these acts reinforced her position that she is a natural-born citizen of the Philippines. 98
Sixth, she maintained that as early as the first quarter of 2005, she started reestablishing her domicile of choice in the Philippines as
demonstrated by her children's resettlement and schooling in the country, purchase of a condominium unit in San Juan City and the
construction of their family home in Corinthian Hills. 99
Seventh, she insisted that she could legally reestablish her domicile of choice in the Philippines even before she renounced her
American citizenship as long as the three determinants for a change of domicile are complied with. 100 She reasoned out that there was no
requirement that renunciation of foreign citizenship is a prerequisite for the acquisition of a new domicile of choice. 101
Eighth, she reiterated that the period appearing in the residency portion of her COC for Senator was a mistake made in good faith. 102
In a Resolution 103 promulgated on 11 December 2015, the COMELEC First Division ruled that petitioner is not a natural-born citizen,
that she failed to complete the ten (10)-year residency requirement, and that she committed material misrepresentation in her COC when she
declared therein that she has been a resident of the Philippines for a period of ten (10) years and eleven (11) months as of the day of the
elections on 9 May 2016. The COMELEC First Division concluded that she is not qualified for the elective position of President of the Republic of
the Philippines. The dispositive portion of said Resolution reads:
WHEREFORE, premises considered, the Commission RESOLVED, as it hereby RESOLVES, to GRANT the Petitions
and cancel the Certificate of Candidacy of MARY GRACE NATIVIDAD SONORA POE-LLAMANZARES for the elective position
of President of the Republic of the Philippines in connection with the 9 May 2016 Synchronized Local and National Elections.
Petitioner filed a motion for reconsideration seeking a reversal of the COMELEC First Division's Resolution. On 23 December 2015, the
COMELEC En Banc issued a Resolution denying petitioner's motion for reconsideration.
Alarmed by the adverse rulings of the COMELEC, petitioner instituted the present petitions for certiorari with urgent prayer for the
issuance of an ex parte temporary restraining order/status quo ante order and/or writ of preliminary injunction. On 28 December 2015,
temporary restraining orders were issued by the Court enjoining the COMELEC and its representatives from implementing the assailed
COMELEC Resolutions until further orders from the Court. The Court also ordered the consolidation of the two petitions filed by petitioner in its
Resolution of 12 January 2016. Thereafter, oral arguments were held in these cases.
The Court GRANTS the petition of Mary Grace Natividad S. Poe-Llamanzares and to ANNUL and SET ASIDE the:
1. Resolution dated 1 December 2015 rendered through its Second Division, in SPA No. 15-001 (DC), entitled Estrella C. Elamparo,
petitioner, vs. Mary Grace Natividad Sonora Poe-Llamanzares.
2. Resolution dated 11 December 2015, rendered through its First Division, in the consolidated cases SPA No. 15-002 (DC)
entitled Francisco S. Tatad, petitioner, vs. Mary Grace Natividad Sonora Poe-Llamanzares, respondent; SPA No. 15-007 (DC)
entitled Antonio P. Contreras, petitioner, vs. Mary Grace Natividad Sonora Poe-Llamanzares, respondent; and SPA No. 15-139
(DC) entitled Amado D. Valdez, petitioner, v. Mary Grace Natividad Sonora Poe-Llamanzares, respondent.
3. Resolution dated 23 December 2015 of the Commission En Banc, upholding the 1 December 2015 Resolution of the Second Division.
4. Resolution dated 23 December 2015 of the Commission En Banc, upholding the 11 December 2015 Resolution of the First Division.
The procedure and the conclusions from which the questioned Resolutions emanated are tainted with grave abuse of discretion
amounting to lack of jurisdiction. The petitioner is a QUALIFIED CANDIDATE for President in the 9 May 2016 National Elections.
The issue before the COMELEC is whether or not the COC of petitioner should be denied due course or cancelled "on the exclusive
ground" that she made in the certificate a false material representation. The exclusivity of the ground should hedge in the discretion of the
COMELEC and restrain it from going into the issue of the qualifications of the candidate for the position, if, as in this case, such issue is yet
undecided or undetermined by the proper authority. The COMELEC cannot itself, in the same cancellation case, decide the qualification or lack
thereof of the candidate.
We rely, first of all, on the Constitution of our Republic, particularly its provisions in Article IX, C, Section 2:
Section 2. The Commission on Elections shall exercise the following powers and functions:
(1) Enforce and administer all laws and regulations relative to the conduct of an election, plebiscite, initiative,
referendum, and recall.

(2) Exercise exclusive original jurisdiction over all contests relating to the elections, returns, and qualifications of all
elective regional, provincial, and city officials, and appellate jurisdiction over all contests involving elective
municipal officials decided by trial courts of general jurisdiction, or involving elective barangay officials
decided by trial courts of limited jurisdiction.
Decisions, final orders, or rulings of the Commission on election contests involving elective municipal and barangay
offices shall be final, executory, and not appealable.
(3) Decide, except those involving the right to vote, all questions affecting elections, including determination of the
number and location of polling places, appointment of election officials and inspectors, and registration of
voters.
(4) Deputize, with the concurrence of the President, law enforcement agencies and instrumentalities of the
Government, including the Armed Forces of the Philippines, for the exclusive purpose of ensuring free,
orderly, honest, peaceful, and credible elections.
(5) Register, after sufficient publication, political parties, organizations, or coalitions which, in addition to other
requirements, must present their platform or program of government; and accredit citizens' arms of the
Commission on Elections. Religious denominations and sects shall not be registered. Those which seek to
achieve their goals through violence or unlawful means, or refuse to uphold and adhere to
this Constitution, or which are supported by any foreign government shall likewise be refused registration.
Financial contributions from foreign governments and their agencies to political parties, organizations, coalitions, or
candidates related to elections constitute interference in national affairs, and, when accepted, shall be an
additional ground for the cancellation of their registration with the Commission, in addition to other
penalties that may be prescribed by law.
(6) File, upon a verified complaint, or on its own initiative, petitions in court for inclusion or exclusion of voters;
investigate and, where appropriate, prosecute cases of violations of election laws, including acts or
omissions constituting election frauds, offenses, and malpractices.
(7) Recommend to the Congress effective measures to minimize election spending, including limitation of places
where propaganda materials shall be posted, and to prevent and penalize all forms of election frauds,
offenses, malpractices, and nuisance candidacies.
(8) Recommend to the President the removal of any officer or employee it has deputized, or the imposition of any
other disciplinary action, for violation or disregard of, or disobedience to its directive, order, or decision.
(9) Submit to the President and the Congress a comprehensive report on the conduct of each election, plebiscite,
initiative, referendum, or recall.
Not any one of the enumerated powers approximate the exactitude of the provisions of Article VI, Section 17 of the same basic law
stating that:
The Senate and the House of Representatives shall each have an Electoral Tribunal which shall be the sole judge of
all contests relating to the election, returns, and qualifications of their respective Members. Each Electoral Tribunal shall be
composed of nine Members, three of whom shall be Justices of the Supreme Court to be designated by the Chief Justice, and
the remaining six shall be Members of the Senate or the House of Representatives, as the case may be, who shall be chosen
on the basis of proportional representation from the political parties and the parties or organizations registered under the
party-list system represented therein. The senior Justice in the Electoral Tribunal shall be its Chairman.

or of the last paragraph of Article VII, Section 4 which provides that:


The Supreme Court, sitting en banc, shall be the sole judge of all contests relating to the election, returns, and
qualifications of the President or Vice-President, and may promulgate its rules for the purpose.
The tribunals which have jurisdiction over the question of the qualifications of the President, the Vice-President, Senators and the
Members of the House of Representatives was made clear by the Constitution. There is no such provision for candidates for these positions.
Can the COMELEC be such judge?
The opinion of Justice Vicente V. Mendoza in Romualdez-Marcos v. Commission on Elections, 104 which was affirmatively cited in
the En Banc decision in Fermin v. COMELEC 105 is our guide. The citation in Fermin reads:
Apparently realizing the lack of an authorized proceeding for declaring the ineligibility of candidates, the COMELEC
amended its rules on February 15, 1993 so as to provide in Rule 25 §1, the following:
Grounds for disqualification. — Any candidate who does not possess all the qualifications of a
candidate as provided for by the Constitution or by existing law or who commits any act declared by law
to be grounds for disqualification may be disqualified from continuing as a candidate.
The lack of provision for declaring the ineligibility of candidates, however, cannot be supplied by a mere rule. Such
an act is equivalent to the creation of a cause of action which is a substantive matter which the COMELEC, in the exercise of
its rule-making power under Art. IX, A, §6 of the Constitution, cannot do it. It is noteworthy that the Constitution withholds
from the COMELEC even the power to decide cases involving the right to vote, which essentially involves an inquiry
into qualifications based on age, residence and citizenship of voters. [Art. IX, C, §2(3)]
The assimilation in Rule 25 of the COMELEC rules of grounds for ineligibility into grounds for disqualification is
contrary to the evident intention of the law. For not only in their grounds but also in their consequences are proceedings for
"disqualification" different from those for a declaration of "ineligibility." "Disqualification" proceedings, as already stated,
are based on grounds specified in §12 and §68 of the Omnibus Election Code and in §40 of the Local Government Code and
are for the purpose of barring an individual from becoming a candidate or from continuing as a candidate for public office. In
a word, their purpose is to eliminate a candidate from the race either from the start or during its progress. "Ineligibility," on
the other hand, refers to the lack of the qualifications prescribed in the Constitution or the statutes for holding public
office and the purpose of the proceedings for declaration of ineligibility is to remove the incumbent from office.
Consequently, that an individual possesses the qualifications for a public office does not imply that he is not
disqualified from becoming a candidate or continuing as a candidate for a public office and vice versa. We have this sort of
dichotomy in our Naturalization Law. (C.A. No. 473) That an alien has the qualifications prescribed in §2 of the Law does not
imply that he does not suffer from any of [the] disqualifications provided in §4.
Before we get derailed by the distinction as to grounds and the consequences of the respective proceedings, the importance of the
opinion is in its statement that "the lack of provision for declaring the ineligibility of candidates, however, cannot be supplied by a mere rule".
Justice Mendoza lectured in Romualdez-Marcos that:
Three reasons may be cited to explain the absence of an authorized proceeding for determining before election the
qualifications of a candidate.
First is the fact that unless a candidate wins and is proclaimed elected, there is no necessity for determining his
eligibility for the office. In contrast, whether an individual should be disqualified as a candidate for acts constituting election
offenses (e.g., vote buying, over spending, commission of prohibited acts) is a prejudicial question which should be
determined lest he wins because of the very acts for which his disqualification is being sought. That is why it is provided that
if the grounds for disqualification are established, a candidate will not be voted for; if he has been voted for, the votes in his
favor will not be counted; and if for some reason he has been voted for and he has won, either he will not be proclaimed or
his proclamation will be set aside.
Second is the fact that the determination of a candidates' eligibility, e.g., his citizenship or, as in this case, his
domicile, may take a long time to make, extending beyond the beginning of the term of the office. This is amply
demonstrated in the companion case (G.R. No. 120265, Agapito A. Aquino v. COMELEC) where the determination of Aquino's
residence was still pending in the COMELEC even after the elections of May 8, 1995. This is contrary to the summary
character proceedings relating to certificates of candidacy. That is why the law makes the receipt of certificates of candidacy
a ministerial duty of the COMELEC and its officers. The law is satisfied if candidates state in their certificates of candidacy
that they are eligible for the position which they seek to fill, leaving the determination of their qualifications to be made
after the election and only in the event they are elected. Only in cases involving charges of false representations made in
certificates of candidacy is the COMELEC given jurisdiction.
Third is the policy underlying the prohibition against pre-proclamation cases in elections for President, Vice
President, Senators and members of the House of Representatives. (R.A. No. 7166, § 15) The purpose is to preserve the
prerogatives of the House of Representatives Electoral Tribunal and the other Tribunals as "sole judges" under the
Constitution of the election, returns and qualifications of members of Congress of the President and Vice President, as the
case may be. 106
To be sure, the authoritativeness of the Romualdez pronouncements as reiterated in Fermin, led to the amendment through COMELEC
Resolution No. 9523, on 25 September 2012 of its Rule 25. This, the 15 February 1993 version of Rule 25, which states that:
Grounds for disqualification. — Any candidate who does not possess all the qualifications of a candidate as provided for
by the Constitution or by existing law or who commits any act declared by law to be grounds for disqualification may be
disqualified from continuing as a candidate. 107
was in the 2012 rendition, drastically changed to:
Grounds. — Any candidate who, in action or protest in which he is a party, is declared by final decision of a competent court,
guilty of, or found by the Commission to be suffering from any disqualification provided by law or the Constitution.
A Petition to Disqualify a Candidate invoking grounds for a Petition to Deny to or Cancel a Certificate of Candidacy or Petition
to Declare a Candidate as a Nuisance Candidate, or a combination thereof, shall be summarily dismissed.
Clearly, the amendment done in 2012 is an acceptance of the reality of absence of an authorized proceeding for determining before
election the qualifications of candidate. Such that, as presently required, to disqualify a candidate there must be a declaration by a final
judgment of a competent court that the candidate sought to be disqualified "is guilty of or found by the Commission to be suffering from any
disqualification provided by law or the Constitution."
Insofar as the qualification of a candidate is concerned, Rule 25 and Rule 23 are flipsides of one to the other. Both do not allow, are
not authorizations, are not vestment of jurisdiction, for the COMELEC to determine the qualification of a candidate. The facts of qualification
must beforehand be established in a prior proceeding before an authority properly vested with jurisdiction. The prior determination of
qualification may be by statute, by executive order or by a judgment of a competent court or tribunal.
If a candidate cannot be disqualified without a prior finding that he or she is suffering from a disqualification "provided by law or the
Constitution," neither can the certificate of candidacy be cancelled or denied due course on grounds of false representations regarding his or
her qualifications, without a prior authoritative finding that he or she is not qualified, such prior authority being the necessary measure by which
the falsity of the representation can be found. The only exception that can be conceded are self-evident facts of unquestioned or
unquestionable veracity and judicial confessions. Such are, anyway, bases equivalent to prior decisions against which the falsity of
representation can be determined.
The need for a predicate finding or final pronouncement in a proceeding under Rule 23 that deals with, as in this case, alleged false
representations regarding the candidate's citizenship and residence, forced the COMELEC to rule essentially that since foundlings 108 are not
mentioned in the enumeration of citizens under the 1935 Constitution, 109 they then cannot be citizens. As the COMELEC stated in oral
arguments, when petitioner admitted that she is a foundling, she said it all. This borders on bigotry. Oddly, in an effort at tolerance, the
COMELEC, after saying that it cannot rule that herein petitioner possesses blood relationship with a Filipino citizen when "it is certain that such
relationship is indemonstrable," proceeded to say that "she now has the burden to present evidence to prove her natural filiation with a Filipino
parent."
The fact is that petitioner's blood relationship with a Filipino citizen is DEMONSTRABLE.
At the outset, it must be noted that presumptions regarding paternity is neither unknown nor unaccepted in Philippine Law. The
Family Code of the Philippines has a whole chapter on Paternity and Filiation. 110 That said, there is more than sufficient evidence that
petitioner has Filipino parents and is therefore a natural-born Filipino. Parenthetically, the burden of proof was on private respondents to show
that petitioner is not a Filipino citizen. The private respondents should have shown that both of petitioner's parents were aliens. Her admission
that she is a foundling did not shift the burden to her because such status did not exclude the possibility that her parents were Filipinos,
especially as in this case where there is a high probability, if not certainty, that her parents are Filipinos.

The factual issue is not who the parents of petitioner are, as their identities are unknown, but whether such parents are Filipinos.
Under Section 4, Rule 128:
Sec. 4. Relevancy, collateral matters — Evidence must have such a relation to the fact in issue as to induce belief in
its existence or non-existence. Evidence on collateral matters shall not be allowed, except when it tends in any reasonable
degree to establish the probability of improbability of the fact in issue.
The Solicitor General offered official statistics from the Philippine Statistics Authority (PSA) 111 that from 1965 to 1975, the total
number of foreigners born in the Philippines was 15,986 while the total number of Filipinos born in the country was 10,558,278. The statistical
probability that any child born in the Philippines in that decade is natural-born Filipino was 99.83%. For her part, petitioner presented census
statistics for Iloilo Province for 1960 and 1970, also from the PSA. In 1960, there were 962,532 Filipinos and 4,734 foreigners in the
province; 99.62% of the population were Filipinos. In 1970, the figures were 1,162,669 Filipinos and 5,304 foreigners, or 99.55%. Also presented
were figures for the child producing ages (15-49). In 1960, there were 230,528 female Filipinos as against 730 female foreigners or 99.68%. In
the same year, there were 210,349 Filipino males and 886 male aliens, or 99.58%. In 1970, there were 270,299 Filipino females versus 1,190
female aliens, or 99.56%. That same year, there were 245,740 Filipino males as against only 1,165 male aliens or 99.53%. COMELEC did not
dispute these figures. Notably, Commissioner Arthur Lim admitted, during the oral arguments, that at the time petitioner was found in 1968, the
majority of the population in Iloilo was Filipino. 112
Other circumstantial evidence of the nationality of petitioner's parents are the fact that she was abandoned as an infant in a Roman
Catholic Church in Iloilo City. She also has typical Filipino features: height, flat nasal bridge, straight black hair, almond shaped eyes and an oval
face.
There is a disputable presumption that things have happened according to the ordinary course of nature and the ordinary habits of
life. 113 All of the foregoing evidence, that a person with typical Filipino features is abandoned in Catholic Church in a municipality where the
population of the Philippines is overwhelmingly Filipinos such that there would be more than a 99% chance that a child born in the province
would be a Filipino, would indicate more than ample probability if not statistical certainty, that petitioner's parents are Filipinos. That probability
and the evidence on which it is based are admissible under Rule 128, Section 4 of the Revised Rules on Evidence.
To assume otherwise is to accept the absurd, if not the virtually impossible, as the norm. In the words of the Solicitor General:
Second. It is contrary to common sense because foreigners do not come to the Philippines so they can get
pregnant and leave their newborn babies behind. We do not face a situation where the probability is such that every
foundling would have a 50% chance of being a Filipino and a 50% chance of being a foreigner. We need to frame our
questions properly. What are the chances that the parents of anyone born in the Philippines would be foreigners? Almost
zero. What are the chances that the parents of anyone born in the Philippines would be Filipinos? 99.9%.
According to the Philippine Statistics Authority, from 2010 to 2014, on a yearly average, there were 1,766,046
children born in the Philippines to Filipino parents, as opposed to 1,301 children in the Philippines of foreign parents. Thus,
for that sample period, the ratio of non-Filipino children to natural born Filipino children is 1:1357. This means that the
statistical probability that any child born in the Philippines would be a natural born Filipino is 99.93%.
From 1965 to 1975, the total number of foreigners born in the Philippines is 15,986 while the total number of
Filipinos born in the Philippines is 15,558,278. For this period, the ratio of non-Filipino children is 1:661. This means that the
statistical probability that any child born in the Philippines on that decade would be a natural born Filipino is 99.83%.
We can invite statisticians and social anthropologists to crunch the numbers for us, but I am confident that the
statistical probability that a child born in the Philippines would be a natural born Filipino will not be affected by whether or
not the parents are known. If at all, the likelihood that a foundling would have a Filipino parent might even be higher than
99.9%. Filipinos abandon their children out of poverty or perhaps, shame. We do not imagine foreigners abandoning their
children here in the Philippines thinking those infants would have better economic opportunities or believing that this
country is a tropical paradise suitable for raising abandoned children. I certainly doubt whether a foreign couple has ever
considered their child excess baggage that is best left behind.
To deny full Filipino citizenship to all foundlings and render them stateless just because there may be a theoretical
chance that one among the thousands of these foundlings might be the child of not just one, but two, foreigners is
downright discriminatory, irrational, and unjust. It just doesn't make any sense. Given the statistical certainty — 99.9% —
that any child born in the Philippines would be a natural born citizen, a decision denying foundlings such status is effectively
a denial of their birthright. There is no reason why this Honorable Court should use an improbable hypothetical to sacrifice
the fundamental political rights of an entire class of human beings. Your Honor, constitutional interpretation and the use of
common sense are not separate disciplines.
As a matter of law, foundlings are as a class, natural-born citizens. While the 1935 Constitution's enumeration is silent as to
foundlings, there is no restrictive language which would definitely exclude foundlings either. Because of silence and ambiguity in the
enumeration with respect to foundlings, there is a need to examine the intent of the framers. In Nitafan v. Commissioner of Internal
Revenue, 114 this Court held that:
The ascertainment of that intent is but in keeping with the fundamental principle of constitutional construction
that the intent of the framers of the organic law and of the people adopting it should be given effect. The primary task in
constitutional construction is to ascertain and thereafter assure the realization of the purpose of the framers and of the
people in the adoption of the Constitution. It may also be safely assumed that the people in ratifying the Constitution were
guided mainly by the explanation offered by the framers. 115
As pointed out by petitioner as well as the Solicitor General, the deliberations of the 1934 Constitutional Convention show that the
framers intended foundlings to be covered by the enumeration. The following exchange is recorded:
Sr. Rafols:
For an amendment. I propose that after subsection 2, the following is inserted: "The natural children of a foreign father and a
Filipino mother not recognized by the father.
xxx xxx xxx
President:
[We] would like to request a clarification from the proponent of the amendment. The gentleman refers to natural children or
to any kind of illegitimate children?
Sr. Rafols:
To all kinds of illegitimate children. It also includes natural children of unknown parentage, natural or illegitimate children of
unknown parents.
Sr. Montinola:
For clarification. The gentleman said "of unknown parents." Current codes consider them Filipino, that is, I refer to the Spanish
Code wherein all children of unknown parentage born in Spanish territory are considered Spaniards, because the
presumption is that a child of unknown parentage is the son of a Spaniard. This may be applied in the Philippines in
that a child of unknown parentage born in the Philippines is deemed to be Filipino, and there is no need. . . .
Sr. Rafols:
There is a need, because we are relating the conditions that are [required] to be Filipino.
Sr. Montinola:
But that is the interpretation of the law, therefore, there is no [more] need for amendment.
Sr. Rafols:
The amendment should read thus: "Natural or illegitimate of a foreign father and a Filipino mother recognized by one, or the
children of unknown parentage."
Sr. Briones:
The amendment [should] mean children born in the Philippines of unknown parentage.
Sr. Rafols:
The son of a Filipina to a Foreigner, although this [person] does not recognize the child, is not unknown.
President:
Does the gentleman accept the amendment or not?
Sr. Rafols:
I do not accept the amendment because the amendment would exclude the children of a Filipina with a foreigner who does
not recognize the child. Their parentage is not unknown and I think those of overseas Filipino mother and father
[whom the latter] does not recognize, should also be considered as Filipinos.
President:
The question in order is the amendment to the amendment from the Gentleman from Cebu, Mr. Briones.
Sr. Busion:
Mr. President, don't you think it would be better to leave this matter in the hands of the Legislature?
Sr. Roxas:
Mr. President, my humble opinion is that these cases are few and far in between, that the constitution need [not] refer to
them. By international law the principle that children or people born in a country of unknown parents are citizens in
this nation is recognized, and it is not necessary to include a provision on the subject exhaustively. 116
Though the Rafols amendment was not carried out, it was not because there was any objection to the notion that persons of
"unknown parentage" are not citizens but only because their number was not enough to merit specific mention. Such was the
account, 117 cited by petitioner, of delegate and constitution law author Jose Aruego who said:

During the debates on this provision, Delegate Rafols presented an amendment to include as Filipino citizens the
illegitimate children with a foreign father of a mother who was a citizen of the Philippines, and also foundlings; but this
amendment was defeated primarily because the Convention believed that the cases, being too few to warrant the inclusion
of a provision in the Constitution to apply to them, should be governed by statutory legislation. Moreover, it was believed
that the rules of international law were already clear to the effect that illegitimate children followed the citizenship of the
mother, and that foundlings followed the nationality of the place where they were found, thereby making unnecessary the
inclusion in the Constitution of the proposed amendment.
This explanation was likewise the position of the Solicitor General during the 16 February 2016 Oral Arguments:
We all know that the Rafols proposal was rejected. But note that what was declined was the proposal for a textual
and explicit recognition of foundlings as Filipinos. And so, the way to explain the constitutional silence is by saying that it was
the view of Montinola and Roxas which prevailed that there is no more need to expressly declare foundlings as Filipinos.
Obviously, it doesn't matter whether Montinola's or Roxas' views were legally correct. Framers of
a constitution can constitutionalize rules based on assumptions that are imperfect or even wrong. They can even overturn
existing rules. This is basic. What matters here is that Montinola and Roxas were able to convince their colleagues in the
convention that there is no more need to expressly declare foundlings as Filipinos because they are already impliedly so
recognized.
In other words, the constitutional silence is fully explained in terms of linguistic efficiency and the avoidance of
redundancy. The policy is clear: it is to recognize foundlings, as a class, as Filipinos under Art. IV, Section 1 (3) of the 1935
Constitution. This inclusive policy is carried over into the 1973 and 1987 Constitution. It is appropriate to invoke a famous
scholar as he was paraphrased by Chief Justice Fernando: the constitution is not silently silent, it is silently vocal. 118
The Solicitor General makes the further point that the framers "worked to create a just and humane society," that "they were
reasonable patriots and that it would be unfair to impute upon them a discriminatory intent against foundlings." He exhorts that, given the
grave implications of the argument that foundlings are not natural-born Filipinos, the Court must search the records of the 1935,1973 and 1987
Constitutions "for an express intention to deny foundlings the status of Filipinos. The burden is on those who wish to use the constitution to
discriminate against foundlings to show that the constitution really intended to take this path to the dark side and inflict this across the board
marginalization."
We find no such intent or language permitting discrimination against foundlings. On the contrary, all three Constitutions guarantee
the basic right to equal protection of the laws. All exhort the State to render social justice. Of special consideration are several provisions in the
present charter: Article II, Section 11 which provides that the "State values the dignity of every human person and guarantees full respect for
human rights," Article XIII, Section 1 which mandates Congress to "give highest priority to the enactment of measures that protect and enhance
the right of all the people to human dignity, reduce social, economic, and political inequalities . . ." and Article XV, Section 3 which requires the
State to defend the "right of children to assistance, including proper care and nutrition, and special protection from all forms of neglect, abuse,
cruelty, exploitation, and other conditions prejudicial to their development." Certainly, these provisions contradict an intent to discriminate
against foundlings on account of their unfortunate status.
Domestic laws on adoption also support the principle that foundlings are Filipinos. These laws do not provide that adoption confers
citizenship upon the adoptee. Rather, the adoptee must be a Filipino in the first place to be adopted. The most basic of such laws is Article 15 of
the Civil Code which provides that "[l]aws relating to family rights, duties, status, conditions, legal capacity of persons are binding on citizens of
the Philippines even though living abroad." Adoption deals with status, and a Philippine adoption court will have jurisdiction only if the adoptee
is a Filipino. In Ellis and Ellis v. Republic, 119 a child left by an unidentified mother was sought to be adopted by aliens. This Court said:
In this connection, it should be noted that this is a proceedings in rem, which no court may entertain unless it has
jurisdiction, not only over the subject matter of the case and over the parties, but also over the res, which is the personal
status of Baby Rose as well as that of petitioners herein. Our Civil Code (Art. 15) adheres to the theory that jurisdiction over
the status of a natural person is determined by the latter's nationality. Pursuant to this theory, we have jurisdiction over the
status of Baby Rose, she being a citizen of the Philippines, but not over the status of the petitioners, who are
foreigners. 120 (Underlining supplied)
Recent legislation is more direct. R.A. No. 8043 entitled "An Act Establishing the Rules to Govern the Inter-Country Adoption of Filipino
Children and for Other Purposes" (otherwise known as the "Inter-Country Adoption Act of 1995"), R.A. No. 8552, entitled "An Act Establishing
the Rules and Policies on the Adoption of Filipino Children and For Other Purposes" (otherwise known as the Domestic Adoption Act of 1998)
and this Court's A.M. No. 02-6-02-SC or the "Rule on Adoption," all expressly refer to "Filipino children" and include foundlings as among Filipino
children who may be adopted.
It has been argued that the process to determine that the child is a foundling leading to the issuance of a foundling certificate under
these laws and the issuance of said certificate are acts to acquire or perfect Philippine citizenship which make the foundling a naturalized
Filipino at best. This is erroneous. Under Article IV, Section 2 "Natural-born citizens are those who are citizens of the Philippines from birth
without having to perform any act to acquire or perfect their Philippine citizenship." In the first place, "having to perform an act" means that the
act must be personally done by the citizen. In this instance, the determination of foundling status is done not by the child but by the
authorities. 121 Secondly, the object of the process is the determination of the whereabouts of the parents, not the citizenship of the child.
Lastly, the process is certainly not analogous to naturalization proceedings to acquire Philippine citizenship, or the election of such citizenship by
one born of an alien father and a Filipino mother under the1935 Constitution, which is an act to perfect it.
In this instance, such issue is moot because there is no dispute that petitioner is a foundling, as evidenced by a Foundling Certificate
issued in her favor. 122 The Decree of Adoption issued on 13 May 1974, which approved petitioner's adoption by Jesusa Sonora Poe and Ronald
Allan Kelley Poe, expressly refers to Emiliano and his wife, Rosario Militar, as her "foundling parents," hence effectively affirming petitioner's
status as a foundling. 123
Foundlings are likewise citizens under international law. Under the 1987 Constitution, an international law can become part of the
sphere of domestic law either by transformation or incorporation. The transformation method requires that an international law be
transformed into a domestic law through a constitutional mechanism such as local legislation. 124 On the other hand, generally accepted
principles of international law, by virtue of the incorporation clause of the Constitution, form part of the laws of the land even if they do not
derive from treaty obligations. Generally accepted principles of international law include international custom as evidence of a general practice
accepted as law, and general principles of law recognized by civilized nations. 125 International customary rules are accepted as binding as a
result from the combination of two elements: the established, widespread, and consistent practice on the part of States; and a psychological
element known as the opinion juris sive necessitates (opinion as to law or necessity). Implicit in the latter element is a belief that the practice in
question is rendered obligatory by the existence of a rule of law requiring it. 126 "General principles of law recognized by civilized nations" are
principles "established by a process of reasoning" or judicial logic, based on principles which are "basic to legal systems generally," 127 such as
"general principles of equity, i.e., the general principles of fairness and justice," and the "general principle against discrimination" which is
embodied in the "Universal Declaration of Human Rights, the International Covenant on Economic, Social and Cultural Rights, the International
Convention on the Elimination of All Forms of Racial Discrimination, the Convention Against Discrimination in Education, the Convention (No.
111) Concerning Discrimination in Respect of Employment and Occupation."128 These are the same core principles which underlie
the Philippine Constitution itself, as embodied in the due process and equal protection clauses of the Bill of Rights. 129
Universal Declaration of Human Rights ("UDHR") has been interpreted by this Court as part of the generally accepted principles of
international law and binding on the State. 130 Article 15 thereof states:
1. Everyone has the right to a nationality.
2. No one shall be arbitrarily deprived of his nationality nor denied the right to change his nationality.

The Philippines has also ratified the UN Convention on the Rights of the Child (UNCRC). Article 7 of the UNCRC imposes the following
obligations on our country:
Article 7
1. The child shall be registered immediately after birth and shall have the right from birth to a name, the right to acquire a
nationality and as far as possible, the right to know and be cared for by his or her parents.
2. States Parties shall ensure the implementation of these rights in accordance with their national law and their obligations
under the relevant international instruments in this field, in particular where the child would otherwise be stateless.
In 1986, the country also ratified the 1966 International Covenant on Civil and Political Rights (ICCPR). Article 24 thereof provide for
the right of every child "to acquire a nationality:"
Article 24
1. Every child shall have, without any discrimination as to race, colour, sex, language, religion, national or social origin, property
or birth, the right, to such measures of protection as are required by his status as a minor, on the part of his family,
society and the State.
2. Every child shall be registered immediately after birth and shall have a name.
3. Every child has the right to acquire a nationality.
The common thread of the UDHR, UNCRC and ICCPR is to obligate the Philippines to grant nationality from birth and ensure that no
child is stateless. This grant of nationality must be at the time of birth, and it cannot be accomplished by the application of our present
naturalization laws, Commonwealth Act No. 473, as amended, andR.A. No. 9139, both of which require the applicant to be at least eighteen (18)
years old.
The principles found in two conventions, while yet unratified by the Philippines, are generally accepted principles of international law.
The first is Article 14 of the 1930 Hague Convention on Certain Questions Relating to the Conflict of Nationality Laws under which a foundling is
presumed to have the "nationality of the country of birth," to wit:
Article 14
A child whose parents are both unknown shall have the nationality of the country of birth. If the child's parentage
is established, its nationality shall be determined by the rules applicable in cases where the parentage is known.
A foundling is, until the contrary is proved, presumed to have been born on the territory of the State in which it
was found. (Underlining supplied)
The second is the principle that a foundling is presumed born of citizens of the country where he is found, contained in Article 2 of the
1961 United Nations Convention on the Reduction of Statelessness:
Article 2
A foundling found in the territory of a Contracting State shall, in the absence of proof to the contrary, be
considered to have been born within the territory of parents possessing the nationality of that State.
That the Philippines is not a party to the 1930 Hague Convention nor to the 1961 Convention on the Reduction of Statelessness does
not mean that their principles are not binding. While the Philippines is not a party to the 1930 Hague Convention, it is a signatory to the
Universal Declaration on Human Rights, Article 15 (1) of which 131effectively affirms Article 14 of the 1930 Hague Convention. Article 2 of the
1961 "United Nations Convention on the Reduction of Statelessness" merely "gives effect" to Article 15 (1) of the UDHR. 132 In Razon v.
Tagitis, 133 this Court noted that the Philippines had not signed or ratified the "International Convention for the Protection of All Persons from
Enforced Disappearance." Yet, we ruled that the proscription against enforced disappearances in the said convention was nonetheless binding
as a "generally accepted principle of international law." Razon v. Tagitis is likewise notable for declaring the ban as a generally accepted
principle of international law although the convention had been ratified by only sixteen states and had not even come into force and which
needed the ratification of a minimum of twenty states. Additionally, as petitioner points out, the Court was content with the practice of
international and regional state organs, regional state practice in Latin America, and State Practice in the United States.
Another case where the number of ratifying countries was not determinative is Mijares v. Rañada, 134 where only four countries had
"either ratified or acceded to" 135 the 1966 "Convention on the Recognition and Enforcement of Foreign Judgments in Civil and Commercial
Matters" when the case was decided in 2005. The Court also pointed out that that nine member countries of the European Common Market
had acceded to the Judgments Convention. The Court also cited U.S. laws and jurisprudence on recognition of foreign judgments. In all, only the
practices of fourteen countries were considered and yet, there was pronouncement that recognition of foreign judgments was widespread
practice.
Our approach in Razon and Mijares effectively takes into account the fact that "generally accepted principles of international law" are
based not only on international custom, but also on "general principles of law recognized by civilized nations," as the phrase is understood in
Article 38.1 paragraph (c) of the ICJ Statute. Justice, fairness, equity and the policy against discrimination, which are fundamental principles
underlying the Bill of Rights and which are "basic to legal systems generally," 136 support the notion that the right against enforced
disappearances and the recognition of foreign judgments, were correctly considered as "generally accepted principles of international law"
under the incorporation clause.
Petitioner's evidence 137 shows that at least sixty countries in Asia, North and South America, and Europe have passed legislation
recognizing foundlings as its citizen. Forty-two (42) of those countries follow the jus sanguinis regime. Of the sixty, only thirty-three (33) are
parties to the 1961 Convention on Statelessness; twenty-six (26) are not signatories to the Convention. Also, the Chief Justice, at the 2 February
2016 Oral Arguments pointed out that in 166 out of 189 countries surveyed (or 87.83%), foundlings are recognized as citizens. These
circumstances, including the practice of jus sanguinis countries, show that it is a generally accepted principle of international law to presume
foundlings as having been born of nationals of the country in which the foundling is found.
Current legislation reveals the adherence of the Philippines to this generally accepted principle of international law. In particular, R.A.
No. 8552, R.A. No. 8042 and this Court's Rules on Adoption, expressly refer to "Filipino children." In all of them, foundlings are among the
Filipino children who could be adopted. Likewise, it has been pointed that the DFA issues passports to foundlings. Passports are by law, issued
only to citizens. This shows that even the executive department, acting through the DFA, considers foundlings as Philippine citizens.
Adopting these legal principles from the 1930 Hague Convention and the 1961 Convention on Statelessness is rational and reasonable
and consistent with the jus sanguinis regime in our Constitution. The presumption of natural-born citizenship of foundlings stems from the
presumption that their parents are nationals of the Philippines. As the empirical data provided by the PSA show, that presumption is at more
than 99% and is a virtual certainty.
In sum, all of the international law conventions and instruments on the matter of nationality of foundlings were designed to address
the plight of a defenseless class which suffers from a misfortune not of their own making. We cannot be restrictive as to their application if we
are a country which calls itself civilized and a member of the community of nations. The Solicitor General's warning in his opening statement is
relevant:
. . . the total effect of those documents is to signify to this Honorable Court that those treaties and conventions were drafted
because the world community is concerned that the situation of foundlings renders them legally invisible. It would be
tragically ironic if this Honorable Court ended up using the international instruments which seek to protect and uplift
foundlings a tool to deny them political status or to accord them second-class citizenship. 138
The COMELEC also ruled 139 that petitioner's repatriation in July 2006 under the provisions of R.A. No. 9225 did not result in the
reacquisition of natural-born citizenship. The COMELEC reasoned that since the applicant must perform an act, what is reacquired is not
"natural-born" citizenship but only plain "Philippine citizenship."
The COMELEC's rule arrogantly disregards consistent jurisprudence on the matter of repatriation statutes in general and of R.A. No.
9225 in particular.
In the seminal case of Bengson III v. HRET, 140 repatriation was explained as follows:
Moreover, repatriation results in the recovery of the original nationality. This means that a naturalized Filipino
who lost his citizenship will be restored to his prior status as a naturalized Filipino citizen. On the other hand, if he was
originally a natural-born citizen before he lost his Philippine citizenship, he will be restored to his former status as a natural-
born Filipino.
R.A. No. 9225 is a repatriation statute and has been described as such in several cases. They include Sobejana-Condon v.
COMELEC 141 where we described it as an "abbreviated repatriation process that restores one's Filipino citizenship . . . ." Also included
is Parreño v. Commission on Audit, 142 which cited Tabasa v. Court of Appeals, 143where we said that "[t]he repatriation of the former Filipino
will allow him to recover his natural-born citizenship. Parreño v. Commission on Audit 141 is categorical that "if petitioner reacquires his Filipino
citizenship (under R.A. No. 9225), he will . . . recover his natural-born citizenship."
The COMELEC construed the phrase "from birth" in the definition of natural citizens as implying "that natural-born citizenship must
begin at birth and remain uninterrupted and continuous from birth." R.A. No. 9225 was obviously passed in line with Congress' sole prerogative
to determine how citizenship may be lost or reacquired. Congress saw it fit to decree that natural-born citizenship may be reacquired even if it
had been once lost. It is not for the COMELEC to disagree with the Congress' determination.

More importantly, COMELEC's position that natural-born status must be continuous was already rejected in Bengson III v.
HRET 145 where the phrase "from birth" was clarified to mean at the time of birth: "A person who at the time of his birth, is a citizen of a
particular country, is a natural-born citizen thereof." Neither is "repatriation" an act to "acquire or perfect" one's citizenship. In Bengson III v.
HRET, this Court pointed out that there are only two types of citizens under the 1987 Constitution: natural-born citizen and naturalized, and that
there is no third category for repatriated citizens:
It is apparent from the enumeration of who are citizens under the present Constitution that there are only two
classes of citizens: (1) those who are natural-born and (2) those who are naturalized in accordance with law. A citizen who is
not a naturalized Filipino, i.e., did not have to undergo the process of naturalization to obtain Philippine citizenship,
necessarily is a natural-born Filipino. Noteworthy is the absence in said enumeration of a separate category for persons who,
after losing Philippine citizenship, subsequently reacquire it. The reason therefor is clear: as to such persons, they would
either be natural-born or naturalized depending on the reasons for the loss of their citizenship and the mode prescribed by
the applicable law for the reacquisition thereof. As respondent Cruz was not required by law to go through naturalization
proceedings in order to reacquire his citizenship, he is perforce a natural-born Filipino. As such, he possessed all the
necessary qualifications to be elected as member of the House of Representatives. 146
The COMELEC cannot reverse a judicial precedent. That is reserved to this Court. And while we may always revisit a doctrine, a new
rule reversing standing doctrine cannot be retroactively applied. In Morales v. Court of Appeals and Jejomar Erwin S. Binay, Jr., 147 where we
decreed reversed the condonation doctrine, we cautioned that it "should be prospective in application for the reason that judicial decisions
applying or interpreting the laws of the Constitution, until reversed, shall form part of the legal system of the Philippines." This Court also said
that "while the future may ultimately uncover a doctrine's error, it should be, as a general rule, recognized as good law prior to its
abandonment. Consequently, the people's reliance thereupon should be respected." 148
Lastly, it was repeatedly pointed out during the oral arguments that petitioner committed a falsehood when she put in the spaces for
"born to" in her application for repatriation under R.A. No. 9225 the names of her adoptive parents, and this misled the BI to presume that she
was a natural-born Filipino. It has been contended that the data required were the names of her biological parents which are precisely
unknown.
This position disregards one important fact — petitioner was legally adopted. One of the effects of adoption is "to sever all legal ties
between the biological parents and the adoptee, except when the biological parent is the spouse of the adoptee." 149 Under R.A. No. 8552,
petitioner was also entitled to an amended birth certificate "attesting to the fact that the adoptee is the child of the adopter(s)" and which
certificate "shall not bear any notation that it is an amended issue." 150 That law also requires that "[a]ll records, books, and papers relating to
the adoption cases in the files of the court, the Department [of Social Welfare and Development], or any other agency or institution
participating in the adoption proceedings shall be kept strictly confidential." 151 The law therefore allows petitioner to state that her adoptive
parents were her birth parents as that was what would be stated in her birth certificate anyway. And given the policy of strict confidentiality of
adoption records, petitioner was not obligated to disclose that she was an adoptee.
Clearly, to avoid a direct ruling on the qualifications of petitioner, which it cannot make in the same case for cancellation of COC, it
resorted to opinionatedness which is, moreover, erroneous. The whole process undertaken by COMELEC is wrapped in grave abuse of
discretion.
On Residence
The tainted process was repeated in disposing of the issue of whether or not petitioner committed false material representation when
she stated in her COC that she has before and until 9 May 2016 been a resident of the Philippines for ten (10) years and eleven (11) months.
Petitioner's claim that she will have been a resident for ten (10) years and eleven (11) months on the day before the 2016 elections, is
true.
The Constitution requires presidential candidates to have ten (10) years' residence in the Philippines before the day of the elections.
Since the forthcoming elections will be held on 9 May 2016, petitioner must have been a resident of the Philippines prior to 9 May 2016 for ten
(10) years. In answer to the requested information of "Period of Residence in the Philippines up to the day before May 09, 2016," she put in "10
years 11 months" which according to her pleadings in these cases corresponds to a beginning date of 25 May 2005 when she returned for good
from the U.S.
When petitioner immigrated to the U.S. in 1991, she lost her original domicile, which is the Philippines. There are three requisites to
acquire a new domicile: 1. Residence or bodily presence in a new locality; 2. an intention to remain there; and 3. an intention to abandon the
old domicile. 152 To successfully effect a change of domicile, one must demonstrate an actual removal or an actual change of domicile; a bona
fide intention of abandoning the former place of residence and establishing a new one and definite acts which correspond with the purpose. In
other words, there must basically be animus manendi coupled with animus non revertendi. The purpose to remain in or at the domicile of choice
must be for an indefinite period of time; the change of residence must be voluntary; and the residence at the place chosen for the new domicile
must be actual. 153
Petitioner presented voluminous evidence showing that she and her family abandoned their U.S. domicile and relocated to the
Philippines for good. These evidence include petitioner's former U.S. passport showing her arrival on 24 May 2005 and her return to the
Philippines every time she travelled abroad; e-mail correspondences starting in March 2005 to September 2006 with a freight company to
arrange for the shipment of their household items weighing about 28,000 pounds to the Philippines; e-mail with the Philippine Bureau of Animal
Industry inquiring how to ship their dog to the Philippines; school records of her children showing enrollment in Philippine schools starting June
2005 and for succeeding years; tax identification card for petitioner issued on July 2005; titles for condominium and parking slot issued in
February 2006 and their corresponding tax declarations issued in April 2006; receipts dated 23 February 2005 from the Salvation Army in the
U.S. acknowledging donation of items from petitioner's family; March 2006 e-mail to the U.S. Postal Service confirming request for change of
address; final statement from the First American Title Insurance Company showing sale of their U.S. home on 27 April 2006; 12 July 2011 filled-
up questionnaire submitted to the U.S. Embassy where petitioner indicated that she had been a Philippine resident since May 2005; affidavit
from Jesusa Sonora Poe (attesting to the return of petitioner on 24 May 2005 and that she and her family stayed with affiant until the
condominium was purchased); and Affidavit from petitioner's husband (confirming that the spouses jointly decided to relocate to the
Philippines in 2005 and that he stayed behind in the U.S. only to finish some work and to sell the family home).
The foregoing evidence were undisputed and the facts were even listed by the COMELEC, particularly in its Resolution in the Tatad,
Contreras and Valdez cases.
However, the COMELEC refused to consider that petitioner's domicile had been timely changed as of 24 May 2005. At the oral
arguments, COMELEC Commissioner Arthur Lim conceded the presence of the first two requisites, namely, physical presence and animus
manendi, but maintained there was no animus non-revertendi. 154 The COMELEC disregarded the import of all the evidence presented by
petitioner on the basis of the position that the earliest date that petitioner could have started residence in the Philippines was in July 2006 when
her application under R.A. No. 9225 was approved by the BI. In this regard, COMELEC relied on Coquilla v. COMELEC, 155 Japzon v.
COMELEC 156 and Caballero v. COMELEC. 157 During the oral arguments, the private respondents also added Reyes v.
COMELEC. 158 Respondents contend that these cases decree that the stay of an alien former Filipino cannot be counted until he/she obtains a
permanent resident visa or reacquires Philippine citizenship, a visa-free entry under abalikbayan stamp being insufficient. Since petitioner was
still an American (without any resident visa) until her reacquisition of citizenship under R.A. No. 9225, her stay from 24 May 2005 to 7 July 2006
cannot be counted.
But as the petitioner pointed out, the facts in these four cases are very different from her situation. In Coquilla v. COMELEC, 159 the
only evidence presented was a community tax certificate secured by the candidate and his declaration that he would be running in the
elections. Japzon v. COMELEC 160 did not involve a candidate who wanted to count residence prior to his reacquisition of Philippine citizenship.
With the Court decreeing that residence is distinct from citizenship, the issue there was whether the candidate's acts after reacquisition sufficed
to establish residence. In Caballero v. COMELEC, 161 the candidate admitted that his place of work was abroad and that he only visited during
his frequent vacations. In Reyes v. COMELEC, 162 the candidate was found to be an American citizen who had not even reacquired Philippine
citizenship underR.A. No. 9225 or had renounced her U.S. citizenship. She was disqualified on the citizenship issue. On residence, the only proof
she offered was a seven-month stint as provincial officer. The COMELEC, quoted with approval by this Court, said that "such fact alone is not
sufficient to prove her one-year residency."

It is obvious that because of the sparse evidence on residence in the four cases cited by the respondents, the Court had no choice but
to hold that residence could be counted only from acquisition of a permanent resident visa or from reacquisition of Philippine citizenship. In
contrast, the evidence of petitioner is overwhelming and taken together leads to no other conclusion that she decided to permanently abandon
her U.S. residence (selling the house, taking the children from U.S. schools, getting quotes from the freight company, notifying the U.S. Post
Office of the abandonment of their address in the U.S., donating excess items to the Salvation Army, her husband resigning from U.S.
employment right after selling the U.S. house) and permanently relocate to the Philippines and actually re-established her residence here on 24
May 2005 (securing T.I.N., enrolling her children in Philippine schools, buying property here, constructing a residence here, returning to the
Philippines after all trips abroad, her husband getting employed here). Indeed, coupled with her eventual application to reacquire Philippine
citizenship and her family's actual continuous stay in the Philippines over the years, it is clear that when petitioner returned on 24 May 2005 it
was for good.
In this connection, the COMELEC also took it against petitioner that she had entered the Philippines visa-free as a balikbayan. A closer
look at R.A. No. 6768 as amended, otherwise known as the "An Act Instituting a Balikbayan Program," shows that there is no overriding intent to
treat balikbayans as temporary visitors who must leave after one year. Included in the law is a former Filipino who has been naturalized abroad
and "comes or returns to the Philippines." 163 The law institutes a balikbayan program "providing the opportunity to avail of the necessary
training to enable the balikbayan to become economically self-reliant members of society upon their return to the country" 164 in line with the
government's "reintegration program." 165 Obviously, balikbayans are not ordinary transients.
Given the law's express policy to facilitate the return of a balikbayan and help him reintegrate into society, it would be an unduly
harsh conclusion to say in absolute terms that the balikbayan must leave after one year. That visa-free period is obviously granted him to allow
him to re-establish his life and reintegrate himself into the community before he attends to the necessary formal and legal requirements of
repatriation. And that is exactly what petitioner did — she reestablished life here by enrolling her children and buying property while awaiting
the return of her husband and then applying for repatriation shortly thereafter.
No case similar to petitioner's, where the former Filipino's evidence of change in domicile is extensive and overwhelming, has as yet
been decided by the Court. Petitioner's evidence of residence is unprecedented. There is no judicial precedent that comes close to the facts of
residence of petitioner. There is no indication in Coquilla v. COMELEC, 166 and the other cases cited by the respondents that the Court intended
to have its rulings there apply to a situation where the facts are different. Surely, the issue of residence has been decided particularly on the
facts-of-the case basis.
To avoid the logical conclusion pointed out by the evidence of residence of petitioner, the COMELEC ruled that petitioner's claim of
residence of ten (10) years and eleven (11) months by 9 May 2016 in her 2015 COC was false because she put six (6) years and six (6) months as
"period of residence before May 13, 2013" in her 2012 COC for Senator. Thus, according to the COMELEC, she started being a Philippine resident
only in November 2006. In doing so, the COMELEC automatically assumed as true the statement in the 2012 COC and the 2015 COC as false.
As explained by petitioner in her verified pleadings, she misunderstood the date required in the 2013 COC as the period of residence
as of the day she submitted that COC in 2012. She said that she reckoned residency from April-May 2006 which was the period when the U.S.
house was sold and her husband returned to the Philippines. In that regard, she was advised by her lawyers in 2015 that residence could be
counted from 25 May 2005.
Petitioner's explanation that she misunderstood the query in 2012 (period of residence before 13 May 2013) as inquiring about
residence as of the time she submitted the COC, is bolstered by the change which the COMELEC itself introduced in the 2015 COC which is now
"period of residence in the Philippines up to the day before May 09, 2016." The COMELEC would not have revised the query if it did not
acknowledge that the first version was vague.
That petitioner could have reckoned residence from a date earlier than the sale of her U.S. house and the return of her husband is
plausible given the evidence that she had returned a year before. Such evidence, to repeat, would include her passport and the school records
of her children.
It was grave abuse of discretion for the COMELEC to treat the 2012 COC as a binding and conclusive admission against petitioner. It
could be given in evidence against her, yes, but it was by no means conclusive. There is precedent after all where a candidate's mistake as to
period of residence made in a COC was overcome by evidence. InRomualdez-Marcos v. COMELEC, 167 the candidate mistakenly put seven (7)
months as her period of residence where the required period was a minimum of one year. We said that "[i]t is the fact of residence, not a
statement in a certificate of candidacy which ought to be decisive in determining whether or not an individual has satisfied the constitution's
residency qualification requirement." The COMELEC ought to have looked at the evidence presented and see if petitioner was telling the truth
that she was in the Philippines from 24 May 2005. Had the COMELEC done its duty, it would have seen that the 2012 COC and the 2015
COC both correctly stated the pertinent period of residency.
The COMELEC, by its own admission, disregarded the evidence that petitioner actually and physically returned here on 24 May 2005
not because it was false, but only because COMELEC took the position that domicile could be established only from petitioner's repatriation
under R.A. No. 9225 in July 2006. However, it does not take away the fact that in reality, petitioner had returned from the U.S. and was here to
stay permanently, on 24 May 2005. When she claimed to have been a resident for ten (10) years and eleven (11) months, she could do so in
good faith.
For another, it could not be said that petitioner was attempting to hide anything. As already stated, a petition for quo warranto had
been filed against her with the SET as early as August 2015. The event from which the COMELEC pegged the commencement of residence,
petitioner's repatriation in July 2006 under R.A. No. 9225, was an established fact to repeat, for purposes of her senatorial candidacy.
Notably, on the statement of residence of six (6) years and six (6) months in the 2012 COC, petitioner recounted that this was first
brought up in the media on 2 June 2015 by Rep. Tobias Tiangco of the United Nationalist Alliance. Petitioner appears to have answered the issue
immediately, also in the press. Respondents have not disputed petitioner's evidence on this point. From that time therefore when Rep. Tiangco
discussed it in the media, the stated period of residence in the 2012 COC and the circumstances that surrounded the statement were already
matters of public record and were not hidden.
Petitioner likewise proved that the 2012 COC was also brought up in the SET petition for quo warranto. Her Verified Answer, which
was filed on 1 September 2015, admitted that she made a mistake in the 2012 COC when she put in six (6) years and six (6) months as she
misunderstood the question and could have truthfully indicated a longer period. Her answer in the SET case was a matter of public record.
Therefore, when petitioner accomplished her COC for President on 15 October 2015, she could not be said to have been attempting to hide her
erroneous statement in her 2012 COC for Senator which was expressly mentioned in her Verified Answer.
The facts now, if not stretched to distortion, do not show or even hint at an intention to hide the 2012 statement and have it covered
by the 2015 representation. Petitioner, moreover, has on her side this Court's pronouncement that:
Concededly, a candidate's disqualification to run for public office does not necessarily constitute material
misrepresentation which is the sole ground for denying due course to, and for the cancellation of, a COC. Further, as already
discussed, the candidate's misrepresentation in his COC must not only refer to a material fact (eligibility and qualifications
for elective office), but should evince a deliberate intent to mislead, misinform or hide a fact which would otherwise render
a candidate ineligible. It must be made with an intention to deceive the electorate as to one's qualifications to run for public
office. 168
In sum, the COMELEC, with the same posture of infallibilism, virtually ignored a good number of evidenced dates all of which can
evince animus manendi to the Philippines and animus non revertendi to the United States of America. The veracity of the events of coming and
staying home was as much as dismissed as inconsequential, the focus having been fixed at the petitioner's "sworn declaration in her COC for
Senator" which the COMELEC said "amounts to a declaration and therefore an admission that her residence in the Philippines only commence
sometime in November 2006"; such that "based on this declaration, [petitioner] fails to meet the residency requirement for President." This
conclusion, as already shown, ignores the standing jurisprudence that it is the fact of residence, not the statement of the person that
determines residence for purposes of compliance with the constitutional requirement of residency for election as President. It ignores the easily
researched matter that cases on questions of residency have been decided favorably for the candidate on the basis of facts of residence far less
in number, weight and substance than that presented by petitioner. 169 It ignores, above all else, what we consider as a primary reason why
petitioner cannot be bound by her declaration in her COC for Senator which declaration was not even considered by the SET as an issue against
her eligibility for Senator. When petitioner made the declaration in her COC for Senator that she has been a resident for a period of six (6) years
and six (6) months counted up to the 13 May 2013 Elections, she naturally had as reference the residency requirements for election as Senator
which was satisfied by her declared years of residence. It was uncontested during the oral arguments before us that at the time the declaration
for Senator was made, petitioner did not have as yet any intention to vie for the Presidency in 2016 and that the general public was never made
aware by petitioner, by word or action, that she would run for President in 2016. Presidential candidacy has a length-of-residence different from
that of a senatorial candidacy. There are facts of residence other than that which was mentioned in the COC for Senator. Such other facts of
residence have never been proven to be false, and these, to repeat include:

[Petitioner] returned to the Philippines on 24 May 2005. [petitioner's] husband however stayed in the USA to
finish pending projects and arrange the sale of their family home.

Meanwhile [petitioner] and her children lived with her mother in San Juan City. [Petitioner] enrolled Brian in
Beacon School in Taguig City in 2005 and Hanna in Assumption College in Makati City in 2005. Anika was enrolled in Learning
Connection in San Juan in 2007, when she was already old enough to go to school.
In the second half of 2005, [petitioner] and her husband acquired Unit 7F of One Wilson Place Condominium in San
Juan. [Petitioner] and her family lived in Unit 7F until the construction of their family home in Corinthian Hills was
completed.
Sometime in the second half of 2005, [petitioner's] mother discovered that her former lawyer who handled
[petitioner's] adoption in 1974 failed to secure from the Office of the Civil Registrar of Iloilo a new Certificate of Live Birth
indicating [petitioner's] new name and stating that her parents are "Ronald Allan K. Poe" and "Jesusa L. Sonora."
In February 2006, [petitioner] travelled briefly to the US in order to supervise the disposal of some of the family's
remaining household belongings. [Petitioner] returned to the Philippines on 11 March 2006.
In late March 2006, [petitioner's] husband informed the United States Postal Service of the family's abandonment
of their address in the US.
The family home in the US was sole n on 27 April 2006.
In April 2006, [petitioner's] husband resigned from his work in the US. He returned to the Philippines on 4 May
2006 and began working for a Philippine company in July 2006.
In early 2006, [petitioner] and her husband acquired a vacant lot in Corinthian Hills, where they eventually built
their family home. 170
In light of all these, it was arbitrary for the COMELEC to satisfy its intention to let the case fall under the exclusive ground of false
representation, to consider no other date than that mentioned by petitioner in her COC for Senator.
All put together, in the matter of the citizenship and residence of petitioner for her candidacy as President of the Republic, the
questioned Resolutions of the COMELEC in Division and En Banc are, one and all, deadly diseased with grave abuse of discretion from root to
fruits.
WHEREFORE, the petition is GRANTED. The Resolutions, to wit:
1. dated 1 December 2015 rendered through the COMELEC Second Division, in SPA No. 15-001 (DC), entitled Estrella C. Elamparo,
petitioner, vs. Mary Grace Natividad Sonora Poe-Llamanzares, respondent, stating that:
[T]he Certificate of Candidacy for President of the Republic of the Philippines in the May 9, 2016 National and Local Elections
filed by respondent Mary Grace Natividad Sonora Poe-Llamanzares is hereby GRANTED.
2. dated 11 December 2015, rendered through the COMELEC First Division, in the consolidated cases SPA No. 15-002 (DC)
entitled Francisco S. Tatad, petitioner, vs. Mary Grace Natividad Sonora Poe-Llamanzares, respondent; SPA No. 15-007 (DC) entitled Antonio P.
Contreras, petitioner, vs. Mary Grace Natividad Sonora Poe-Llamanzares, respondent; and SPA No. 15-139 (DC) entitled Amado D. Valdez,
petitioner, v. Mary Grace Natividad Sonora Poe-Llamanzares, respondent; stating that:
WHEREFORE, premises considered, the Commission RESOLVED, as it hereby RESOLVES, to GRANT the petitions and
cancel the Certificate of Candidacy of MARY GRACE NATIVIDAD SONORA POE-LLAMANZARES for the elective position of
President of the Republic of the Philippines in connection with the 9 May 2016 Synchronized Local and National Elections.
3. dated 23 December 2015 of the COMELEC En Banc, upholding the 1 December 2015 Resolution of the Second Division stating that:
WHEREFORE, premises considered, the Commission RESOLVED, as it hereby RESOLVES, to DENY the Verified
Motion for Reconsideration of SENATOR MARY GRACE NATIVIDAD SONORA POE-LLAMANZARES. The Resolution dated 11
December 2015 of the Commission First Division is AFFIRMED.
4. dated 23 December 2015 of the COMELEC En Banc, upholding the 11 December 2015 Resolution of the First Division.
are hereby ANNULED and SET ASIDE. Petitioner MARY GRACE NATIVIDAD SONORA POE-LLAMANZARES is DECLARED QUALIFIED to be a
candidate for President in the National and Local Elections of 9 May 2016.
SO ORDERED.
Bersamin and Mendoza, JJ., concur.
Sereno, C.J., Velasco, Jr., Leonen, Jardeleza and Caguioa, JJ., see Concurring Opinion.
Carpio, Brion, Del Castillo and Perlas-Bernabe, JJ., see dissenting opinion.
Leonardo-de Castro, J., please see Separate Dissenting Opinion.
Peralta, J., I join J. Caguioa's opinion.
Reyes, J., I concur with the Dissenting Opinion of Justice Perlas-Bernabe.
||| (Poe-Llamanzares v. Commission on Elections, G.R. Nos. 221697 & 221698-700, [March 8, 2016])

You might also like